Vous êtes sur la page 1sur 549

1

ORTHO BULLETS

Volume
One
Trauma
2017
Collected By : Dr AbdulRahman AbdulNasser
drxabdulrahman@gmail.com
Preface
Orthobullets.com is an educational resource for
orthopaedic surgeons designed to improve
training through the communal efforts of those
who use it as a learning resource. It is a simple but
powerful concept. All of our topics, technique
guides, cases, and user-generated videos are
free, and will stay that way.

The site was collected to PDF files, to make it


easy to navigate through topics, a well-organized
index is included in table of contents at the
beginning of each volume, another way for e-book
users is bookmarks function of your favorite PDF
viewer, it easily accessed through PC or any
smart device, and easily can reach to any topic in
the e-book.

To be easy to study, all trauma topics collected


in one volume , in volume one you find adult trauma
topics including spine trauma, hand trauma, foot
and ankle trauma, and pediatric trauma, also
chapter of infections (adult osteomyelitis, septic
arthritis , wound & hardware infections, necrotizing
fasciitis and Gas gangrene) all these topics moved
from trauma to pathology volume eight.
In other volumes you will find a note about any
topics that moved to trauma volume.
Also any text that copied from another source than orthobullets.com formatted in a red box like this.

Dr, AbdulRahman AbdulNasser


Table of Contents 2. Occipitocervical Instability &
Dislocation ................................. 64
I. Genaral Trauma ........................... 0
3. Atlantoaxial Instability ............. 68
A. Evaluation ................................ 1
4. Atlas Fracture & Transverse
1. Evaluation, Resuscitation & DCO
Ligament Injuries ........................ 71
.................................................... 1
5. Odontoid Fracture (Adult and
2. Gustilo Classification ................ 4
Pediatric) ................................... 74
3. Tscherne Classification ............ 7
6. Traumatic Spondylolisthesis of
4. Trauma Scoring Systems .......... 8 Axis (Hangman's Fracture) ......... 80
5. Open Fractures Management .. 14 C. SubAxial Cervical Trauma ....... 82
B. Specific Presentations ............ 17 1. Cervical Facet Dislocations &
1. Gun Shot Wounds ................... 17 Fractures ................................... 82

2. Amputations ........................... 21 2. Cervical Lateral Mass Fracture


Separation ................................. 87
3. Domestic and Elder Abuse ...... 26
3. Subaxial Cervical Vertebral Body
4. Adult Respiratory Distress
Fractures ................................... 90
Syndrome .................................. 27
4. Clay-shoveler Fracture (Cervical
C. Compartment Syndrome ......... 30
Spinous Process Fracture) ......... 92
1. Leg Compartment Syndrome .. 30
D. Cervical Trauma Procedures .. 94
2. Thigh Compartment Syndrome 35
1. Closed Cervical Traction ......... 94
3. Hand & Forearm Compartment
2. Halo Orthosis Immobilization... 96
Syndrome .................................. 37
E. Thoracolumbar Trauma ........ 100
4. Foot Compartment Syndrome . 41
1. Thoracic & Lumbar Trauma
II. Spine Trauma ............................. 44
Introduction ............................. 100
A. Spinal Cord Injury ................... 45
2. Thoracolumbar Burst Fractures
1. Cervical Spine Trauma ................................................ 102
Evaluation .................................. 45
3. Chance Fracture (flexion-
2. Spinal Cord Injuries ................ 48 distraction injury) ..................... 106
3. Incomplete Spinal Cord Injuries 4. Thoracolumbar Fracture-
.................................................. 57 Dislocation ............................... 108
B. AtlantoAxial Trauma ............... 60 5. Osteoporotic Vertebral
1. Occipital Condyle Fractures ... 60 Compression Fracture .............. 110
6. Cauda Equina Syndrome ....... 114
III. Upper Extremity ...................... 117 A. Tendon Injuries..................... 211
A. Shoulder .............................. 118 1. Flexor Tendon Injuries .......... 211
1. Brachial Plexus Injuries ........ 118 2. Jersey Finger........................ 218
2. Sternoclavicular Dislocation . 124 3. Extensor Tendon Injuries ...... 220
3. Clavicle Fractures ................ 127 4. Mallet Finger ......................... 224
4. Scapula Fractures ................ 134 5. Sagittal Band Rupture (traumatic
5. Scapulothoracic Dissociation 137 extensor tendon dislocation) ..... 227

6. Flail Chest ............................ 139 B. Wrist Trauma ........................ 231

B. Humerus .............................. 141 1. Scaphoid Fracture ................ 231

1. Proximal Humerus Fractures 141 2. Lunate Dislocation (Perilunate


dissociation)............................. 235
2. Proximal Humerus Fracture
Malunion .................................. 148 3. Hook of Hamate Fracture ...... 238

3. Humeral Shaft Fractures ....... 152 4. Hamate Body Fracture .......... 241

4. Distal Humerus Fractures ..... 157 5. Pisiform Fracture .................. 242

C. Elbow ................................... 164 6. Seymour Fracture ................. 244

1. Elbow Dislocation ................. 164 7. TFCC Injury .......................... 246

2. Radial Head Fractures .......... 168 C. Finger Trauma ...................... 250

3. Coronoid Fractures .............. 175 1. Metacarpal Fractures ........... 250

4. Terrible Triad Injury of Elbow 178 2. MCP Dislocations .................. 254

5. Olecranon Fractures............. 181 3. Phalanx Fractures................. 256

6. Capitellum Fractures ............ 185 4. Phalanx Dislocations ............. 258

D. Forearm ............................... 188 5. Digital Collateral Ligament Injury


................................................ 262
1. Monteggia Fractures............. 188
D. Thumb Trauma ..................... 263
2. Radius and Ulnar Shaft Fractures
................................................ 191 1. Base of Thumb Fractures ...... 263

3. Radioulnar Synostosis .......... 194 2. Thumb CMC dislocation ........ 265

4. Distal Radius Fractures ........ 196 3. Thumb Collateral Ligament Injury


................................................ 267
5. Distal Radial Ulnar Joint (DRUJ)
Injuries..................................... 203 E. Other Traumatic Injuries ....... 269

6. Galeazzi Fractures ............... 207 1. Human Bite ........................... 269

IV. Hand Trauma .......................... 210 2. Dog and Cat Bites ................. 271
3. Nail Bed Injury ...................... 274 1. Ankle Fractures .................... 377
4. High-Pressure Injection Injuries 2. Talar Neck Fractures ............ 387
................................................ 276 3. Talus Fracture (other than neck)
5. Frostbite............................... 278 ................................................ 390
V. Pelvis Trauma .......................... 284 4. Subtalar Dislocations ............ 394
A. Pelvis ................................... 285 5. Calcaneus Fractures............. 396
1. Pelvic Ring Fractures............ 285 VII. Foot & Ankle Trauma ............ 405
2. SI Dislocation & Crescent A. Ankle Sprains ....................... 406
Fractures ................................. 294 1. High Ankle Sprain &
3. Sacral Fractures ................... 297 Syndesmosis Injury .................. 406
4. Ilium Fractures ..................... 301 2. Low Ankle Sprain .................. 410
B. Acetabulum .......................... 303 B. Mid & Forefoot Trauma ......... 414
1. Acetabular Fractures............ 303 1. Lisfranc Injury (Tarsometatarsal
2. Hip Dislocation ..................... 311 fracture-dislocation) ................. 414

VI. Lower Extremity ...................... 314 2. 5th Metatarsal Base Fracture 420

A. Femur .................................. 315 3. Metatarsal Fractures ............ 422

1. Femoral Head Fractures ....... 315 4. Tarsal Navicular Fractures.... 425

2. Femoral Neck Fractures ....... 320 C. Tendon Injuries..................... 427

3. Intertrochanteric Fractures .. 326 1. Achilles Tendon Rupture ....... 427

4. Subtrochanteric Fractures.... 330 2. Peroneal Tendon Subluxation &


Dislocation ............................... 431
5. Femoral Shaft Fractures ....... 334
3. Anterior Tibialis Tendon Rupture
6. Distal Femur Fractures ......... 342 ................................................ 435
B. Knee .................................... 347 VIII. Pediatric trauma ................... 439
1. Patella Fracture .................... 347 A. Introduction.......................... 440
2. Knee Dislocation .................. 350 1. Pediatric Abuse .................... 440
C. Leg ...................................... 354 2. Pediatric Evaluation &
1. Tibial Plateau Fractures ........ 354 Resuscitation ........................... 442
2. Proximal Third Tibia Fracture 359 3. Physeal Considerations ........ 445
3. Tibia Shaft Fractures ............ 363 B. Shoulder & Humerus Fractures
4. Tibial Plafond Fractures ........ 370 448

D. Ankle and Hindfoot ............... 377


1. Medial Clavicle Physeal 2. Traumatic Hip Dislocation -
Fractures ................................. 448 Pediatric .................................. 503
2. Distal Clavicle Physeal Fractures 3. Proximal Femur Fractures -
................................................ 450 Pediatric .................................. 506
3. Proximal Humerus Fracture - 4. Femoral Shaft Fractures -
Pediatric .................................. 450 Pediatric .................................. 511
4. Humerus Shaft Fracture - 5. Distal Femoral Physeal Fractures
Pediatric .................................. 454 - Pediatric ................................ 516
5. Distal Humerus Physeal F. Knee & Proximal Tibia ........... 519
Separation - Pediatric ............... 457 1. Tibial Eminence Fracture ...... 519
C. Elbow Fractures ................... 460 2. Tibial Tubercle Fracture ........ 521
1. Supracondylar Fracture - 3. Patella Sleeve Fracture ......... 526
Pediatric .................................. 460
4. Proximal Tibia Epiphyseal
2. Medial Epicondylar Fractures - Fractures - Pediatric ................. 528
Pediatric .................................. 468
5. Proximal Tibia Metaphyseal
3. Lateral Condyle Fracture - Fractures - Pediatric ................. 530
Pediatric .................................. 470
G. Leg & Ankle Fractures .......... 532
4. Olecranon Fractures - Pediatric
................................................ 474 1. Tibia Shaft Fracture - Pediatric
................................................ 532
5. Radial Head and Neck Fractures
- Pediatric ................................ 477 2. Ankle Fractures - Pediatric ... 534

6. Nursemaid's Elbow ............... 481 3. Tillaux Fractures ................... 536

7. Elbow Dislocation - Pediatric 483 4. Triplane Fractures ................ 538

D. Forearm Fractures ............... 486


1. Both Bone Forearm Fracture -
Pediatric .................................. 486
2. Distal Radius Fractures -
Pediatric .................................. 488
3. Monteggia Fracture - Pediatric
................................................ 493
4. Galeazzi Fracture - Pediatric . 495
E. Hip & Femur Fractures......... 498
1. Pelvis Fractures - Pediatric ... 498
OrthoBullets2017 | Evaluation

ORTHO BULLETS

I.Genaral Trauma

- 0 -
By Dr, AbdulRahman AbdulNasser Genaral Trauma | Evaluation

A. Evaluation

1. Evaluation, Resuscitation & DCO


Introduction
 Three peak times of death after trauma
o 50% within the first minutes of sustaining the injury
 caused by massive blood loss or neurologic injury
o 30% within the first few days
 most commonly from neurologic injury
o 20% within days to weeks following injury
 multi system organ failure and infection are leading causes
 Golden Hour
o period of time when life threating and limb threatening injuries should be treated in order
to decrease mortality
o estimated 60% of preventable deaths can occur during this time ranging from minutes to
hours
 Use of an airbag in a head-on collision significantly decreases the rate of
o closed head injuries
o facial fractures
o thoracoabdominal injuries
o need for extraction

Primary Survey
 Airway
o includes cervical spine control
 Breathing
 Circulation
o includes hemorrhage control and resuscitation (below)
o pregnant women should be placed in the left lateral decubitus position to limit positional
hypotension
Hemorrhagic Shock Classification & Fluid Resuscitation

Class % Blood Loss HR BP Urine pH MS Treatment


< 15% > 30
I normal normal normal anxious Fluid
(<750ml) mL/hr
confused
15% to 30% > 100 20-30
II normal normal irritable Fluid
(750-1500ml) bpm mL/hr
combative
30% to 40% > 120 5-15 lethargic Fluid &
III decreased decreased
(1500-2000ml) bpm mL/hr irritable Blood
> 40% (life
> 140 lethargic Fluid &
IV threatening) decreased negligible decreased
bpm coma Blood
(>2000ml)

- 1 -
OrthoBullets2017 Genaral Trauma | Evaluation
 Introduction
o average adult (70 kg male) has an estimated 4.7 - 5 L of circulating blood
o average child (2-10 years old) has an estimated 75 - 80 ml/kg of circulating blood
 Methods of Resuscitation
o fluids
 crystalloid isotonic solution
o blood options
 O negative blood (universal donor)
 Type specific blood
 Cross-matched blood
 transfuse in 1:1:1 ratio (red blood cells: platelets: plasma)
 Indicators of adequate resuscitation
o MAP > 60
o HR < 100
o urine output 0.5-1.0 ml/kg/hr (30 cc/hr)
o serum lactate levels
 most sensitive indicator as to whether some circulatory beds remain inadequately
perfused (normal < 2.5 mmol/L)
o gastric mucosal ph
o base deficit
 normal -2 to +2
 Risk of transfusion
o risk of viral transmission following allogenic blood transfusion
 hepatitis B (HBV) has highest risk: 1 in 205,000 donations
 hepatitis C (HCV): 1 in 1.8 million donations
 human immunodeficiency virus (HIV): 1 in 1.9 million
 transfused blood is screened for
 HIV-1 (cause of AIDS)
 HIV-2
 hepatitis B
 hepatitis C
 West Nile virus
 syphilis
o clerical error leading to transfusion reaction (1:12,000 to 1:50,000)
o bacterial contamination leading to sepsis (1:1million)
o anaphylactic reaction (1:150,000)

Septic Shock
 Septic shock vs. hypovolemic shock
o the key variable to differentiate septic shock and hypovolemic shock is that systemic
vascular resistance is decreased with septic shock and increased with hypovolemic shock

- 2 -
By Dr, AbdulRahman AbdulNasser Genaral Trauma | Evaluation

Hypovolemic Shock Septic Shock


Systemic Vascular Resistance increased decreased
Cardiac Output decreased decreased
Pulmonary Capillary Wedge Pressure decreased decreased
Central Venous Pressure decreased decreased
Mixed Venous Oxygen decreased increased

Imaging
 Delay of fracture diagnosis is most commonly caused by failure to image extremity
 AP Chest
o mediastinal widening
o pneumothorax
 Lateral C-spine
o must visualize C7 on T1
o not commonly utilized in lieu of increased sensitivity with cervical spine CT
 AP Pelvis
o pelvic ring
 further CT imaging should be delayed until preliminary pelvic stabilization has
been accomplished
o acetabulum
o proximal femur
 CT Scan
o C spine, chest, abdomen, pelvis
o often used in initial evaluation of trauma patient to rule out life threatening injuries

Damage Control Orthopaedics


 Involves staging definitive management to avoid adding trauma to patient during vulnerable
period
o the decision to operate and surgical timing on multiple injured trauma patients remains
controversial
o intra-operative hypotension increases mortality rate in patients with head injury
 Parameters that help decide who should be treated with DCO
o ISS >40 (without thoracic trauma)
o ISS >20 with thoracic trauma
o GCS of 8 or below
o multiple injuries with severe pelvic/abdominal trauma and hemorrhagic shock
o bilateral femoral fractures
o pulmonary contusion noted on radiographs
o hypothermia <35 degrees C
o head injury with AIS of 3 or greater
o IL-6 values above 500pg/dL
 Optimal time of surgery
o patient are at increased risk of ARDS and multisystem failure during acute inflammatory
window (period from 2 to 5 days characterized by a surge in inflammatory markers)

- 3 -
OrthoBullets2017 Genaral Trauma | Evaluation
 therefore only potentially life-threatening injuries should be treated in this period
including
 compartment syndrome
 fractures with vascular injuries
 unreduced dislocations
 long bone fractures
 unstable spine fractures
 open fractures
 Stabilization followed by staged definitive management
o to minimize trauma, initial stabilization should be performed and followed by staged
definitive management
 includes initial pelvic volume reduction via sheet, pelvic packing, skeletal traction,
binder, or external fixation
 if hemodynamically stable
 proceed with further imaging including CT chest, abdomen, pelvis
 if not hemodynamically stable
 consider pelvic angiography and embolization
o definitive treatment delayed for
 7-10 days for pelvic fractures
 within 3 weeks for femur fractures (conversion from exfix to IMN)
 7-10 days for tibia fractures (conversion from external fixation to IMN)

2. Gustilo Classification
Abridged version
 Type I
o wound < 1 cm
 Type II
o 1-10cm
 Type III A
o > 10 cm, high energy
o adequate tissue for coverage
o includes segmental / comminuted fractures even if wound <10cm
o farm injuries are automatically Gustillo III
 Type IIIB
o extensive periosteal stripping and requires free soft tissue transfer
 Type IIIC
o vascular injury requiring vascular repair

- 4 -
By Dr, AbdulRahman AbdulNasser Genaral Trauma | Evaluation
Complete version

Gustilo Type I II IIIA IIIB IIIC


Energy Low energy Moderate High High High
Wound Size < 1 cm > 1cm >10cm >10cm >10cm
Soft Tissue Minimal Moderate Extensive Extensive Extensive
Moderate
Contamination Clean Extensive Extensive Extensive
contamination
Simple fx Severe Severe Severe
pattern with Moderate comminution or comminution or comminution or
Fracture Pattern
minimal comminution segmental segmental segmental
comminution fractures fractures fractures
Periosteal
No No Yes Yes Yes
Stripping
Requires free
Local coverage tissue flap or Typically requires
Skin Coverage Local coverage Local coverage
including rotational flap flap coverage
coverage
Exposed fracture
Neurovascular with arterial
Normal Normal Normal Normal
Injury damage that
requires repair
 1st generation  1st generation cephalosporin for gram
cephalosporin for 24 positive coverage.
hours after closure  Aminoglycoside (such as gentamicin) for
gram negative coverage in type III injuries
o the cephalosporin/aminoglycoside
should be continued for 24-72 hours
after the last debridement procedure
 Penicillin should be added if concern for
Antibiotics
anaerobic organism (farm injury)

aminoglycoside (such as gentamicin) for gram negative


coverage
1st cephalosporin (ancef) for gram positive coverage.
the cephalosporin/aminoglycoside should be continued for
72 hours after the last debridment procedure
penicillin should be added if concern for anaerobic
organism (farm injury
 Flouroquinolones
o should be used for fresh water wounds or salt water wounds
Antibiotics (other o can be used if allergic to cephalosporins or clindamycin
considerations)  Doxycycline and ceftazidime
o can be used for salt water wounds

- 5 -
OrthoBullets2017 Genaral Trauma | Evaluation

Figure ‎I:1 Gustillo type one Figure ‎I:2 Gustillo type two

Figure ‎I:3 Gustillo Type3 A

Figure ‎I:4 Gustillo Type 3 B Figure ‎I:5 Gustillo Type 3 C

- 6 -
By Dr, AbdulRahman AbdulNasser Genaral Trauma | Evaluation
Antibiotic Indications for Open Fractures
 Gustillo Grade I and II
o 1st generation cephalosporin
 Gustillo Grade III
o 1st generation cephalosporin + aminoglycoside
 traditionally recommended, but there is no evidence in the literature to support its
use
 With farm injury / bowel contamination
o 1st generation cephalosporin + aminoglycoside + PCN
o add PCN for clostridia
 Duration
o initiate as soon as possible
 increased infection rate when antibiotics are delayed > 3 hours from time of injury
o continue for 72 hours after I&D
o 48 hours after each procedure
 Tetanus booster if not up to date

3. Tscherne Classification

Closed Fractures
 Oestern and Tscherne classification of soft tissue injury in closed fractures

Grade 0  Minimal soft tissue damage


 indirect injury to limb (torsion)
 simple fracture pattern

Grade 1  Superficial abrasion or contusion


 mild fracture pattern

Grade 2  Deep abrasion


 skin or muscle contusion
 severe fracture pattern
 direct trauma to limb

Grade 3  Extensive skin contusion or crush injury


 severe damage to underlying muscle
 compartment syndrome
 subcutaneous avulsion

- 7 -
OrthoBullets2017 Genaral Trauma | Evaluation
Open Fractures
 The Oestern and Tscherne classification for open fractures uses wound size, level of
contamination, and fracture pattern to grade open fractures

Grade I  Open fractures with a small puncture wound without skin


contusion
 negligible bacterial contamination
 low-energy fracture pattern

Grade II  Open injuries with small skin and soft tissue contusions
 moderate contamination
 variable fracture patterns

Grade III  Open fractures with heavy contamination


 extensive soft tissue damage
 often, associated arterial or neural injuries

Grade IV  Open fractures with incomplete or complete amputations

4. Trauma Scoring Systems


Introduction
 Purpose of scoring systems
o appropriate triage and classification of trauma patients
o predict outcomes for patient and family counseling
o quality assurance
o research
 extremely useful for the study of outcomes
o reimbursement purposes
 Classifications
o physiologic
 Revised Trauma Score (RTS)
 Acute Physiology and Chronic Health Evaluation (APACHE)
 Sequential Organ Failure Assessment Score (SOFA)
 Systemic Inflammatory Response Syndrome Score (SIRS)
 Emergency Trauma Score
o anatomic
 Abbreviated Injury Score (AIS)
 Injury Severity Score (ISS)
 New Injury Severity Score (NISS)
 Anatomic Profile (AP)
 Penetrating Abdominal Trauma Index (PATI)
 ICD-based Injury Severity Score (ICISS)
 Trauma Mortality Prediction Model (TMPM-ICD9)
- 8 -
By Dr, AbdulRahman AbdulNasser Genaral Trauma | Evaluation
o combined
 Trauma Score - Injury Severity Score (TRISS)
 A Severity Characterization of Trauma (ASCOT)
 International Classification of Diseases Injury Severity Score (ICISS)

Glasgow Coma Scale


 Introduction
o quantifies severity of head injury by measuring CNS function
o used as
 initial assessment tool
 continual re-evaluation of head injured patients
 Variables
o Best Motor Response
 6 - Obeys command
 5 - Localizes pain
 4 - Normal withdrawal (flexion)
 3 - Abnormal withdrawal (flexion): decorticate
 2 - Abnormal withdrawal (extension): decerebrate
 1 - None (flaccid)
o Best Verbal Response
 5 - Oriented
 4 - Confused conversation
 3 - Inappropriate words
 2 - Incomprehensible sounds
 1 - None
o Eye Opening
 4 - Spontaneous
 3 - To speech
 2 - To pain
 1 - None
 Calculation
o motor response + verbal response + eye opening
 Interpretation
o brain injury
 severe <9
 moderate 9-12
 minor 13 and above
- 9 -
OrthoBullets2017 Genaral Trauma | Evaluation
 Pros
o reliably predicts outcomes for diffuse and focal lesions
 Cons
o does not take into account
 focal or lateralizing signs
 diffuse metabolic processes
 intoxication

Revised Trauma Score (RTS)


 Introduction
o most widely used prehospital field triage tool
 Variables
o Glasgow Coma Scale (GCS)
 score
 4: 13-15
 3: 9-12
 2: 6-8
 1: 4-5
 0: 3
o systolic blood pressure
 score
 4: >90
 3: 76-89
 2: 50-75
 1: 1-49
 0: 0
o respiratory rate
 score
 4: 10-29

- 10 -
By Dr, AbdulRahman AbdulNasser Genaral Trauma | Evaluation
 3: >30
 2: 6-9
 1: 1-5
 0: 0
 Calculation
o Glasgow coma scale score + systolic blood pressure score + respiratory rate score
 Interpretation
o lower score indicates higher severity
o RTS <4 proposed for transfer to trauma center
 Pros
o useful during triage to determine which patients need to be transported to a trauma center
 Cons
o can underestimate injury severity in patients injured in one system

Injury Severity Scale (ISS)


 Introduction
o first scoring system to be based on anatomic criteria
o defines injury severity for comparative purposes
 Variables
o based on scores of 9 anatomic regions
1. head
2. face
3. neck
4. thorax
5. abdominal and pelvic contents
6. spine
7. upper extremity
8. lower extremity
9. external
 Calculation
o Abbreviated Injury Scale (AIS) grades
 0 - no injury
 1 - minor
 2 - moderate
 3 - severe (not life-threatening)
 4 - severe (life-threatening, survival probable)

- 11 -
OrthoBullets2017 Genaral Trauma | Evaluation
 5 - severe (critical, survival uncertain)
 6 - maximal, possibly fatal
ISS
 ISS = sum of squares for the highest AIS grades in the three most severely injured
ISS body regions
2 2 2
 ISS = A + B + C
 where A, B, C are the AIS scores of the three most severely injured
ISS body regions
 scores range from 1 to 75
 single score of 6 on any AIS region results in automatic score of 75
Interpretation
ISS > 15 associated with mortality of 10%
Pros
integrates anatomic areas of injury in formulating a prediction of outcomes
Cons
difficult to calculate during initial evaluation and resuscitation in emergency room
difficult to predict outcomes for patients with severe single body area injury
 New Injury Severity Score (NISS) overcomes this deficit
Modifications
Modified Injury Severity Score (MISS)
 similar to ISS but for pediatric trauma
 categorizes body into 5 areas, instead of 9
 sum of the squares for the highest injury score grades in the three most severely
injured body regions
New Injury Severity Score (NISS)
 takes three highest scores regardless of anatomic area
 more predictive of complications and mortality than ISS

Mangled Extremity Severity Score (MESS)


 Introduction
o used to predict necessity of amputation after lower extremity trauma
 Variables
o skeletal and soft tissue injury (graded 1-4)
o limb ischemia (graded 1-3)
o shock (graded 0-2)
o age (graded 0-2)
 Calculation
o score determined by adding scores of components in four categories
 Interpretation
o score of 7 or more is highly predictive of amputation
 Pros
o high specificity for predicting amputation
 Cons
o low sensitivity for predicting amputation

- 12 -
By Dr, AbdulRahman AbdulNasser Genaral Trauma | Evaluation
Sickness Impact Profile

 Introduction
o evaluates the impact of disease on physical and emotional functioning
 Variables
o 12 categories
 sleep
 eating
 work
 home management
 recreation
 physical dimension
 ambulation
 body care
 movement
 psychosocial dimension
 social interaction
 alertness behavior
 emotional behavior
 communication
 Relevance to trauma
o lower extremity injuries
 psychosocial subscale does not improve with time
o polytrauma
 at 10 year follow-up after a major polytrauma, females have
 decreased quality-of-life scores
 increased PTSD rates
 increased absentee sick days when compared to males

Systemic Inflammatory Response Syndrome (SIRS)


 Introduction
o a generalized response to trauma characterized by
 an increase in cytokines
 an increase in complement
 an increase in hormones
o a marker for an individual's generalized response to trauma that likely has a genetic
predisposition
o associated with conditions such as
 disseminated intravascular coagulopathy (DIC)
 acute respiratory distress syndrome (ARDS)
 renal failure
 multisystem organ failure
 shock
 Variables
o heart rate > 90 beats/min
o WBC count <4000cells/mm³ OR >12,000 cells/mm³

- 13 -
OrthoBullets2017 Genaral Trauma | Evaluation
o respiratory rate > 20 or PaCO2 < 32mm (4.3kPa)
o temperature less than 36 degrees or greater than 38 degrees
 Calculation
o each component (heart rate, WBC count, respiratory rate, temperature) is given 1 point if
it meets the above criteria
 Interpretation
o score of 2 or more meets criteria for SIRS

5. Open Fractures Management


Introduction
 Open fracture definition
o a fracture with direct communication to the external environment

Basic Principles of Open Fracture Management in the Emergency Room


 Fracture management begins after initial trauma survey and resuscitation is complete
 Antibiotics
o initiate early IV antibiotics and update tetanus prophylaxis as indicated
 Control bleeding
o direct pressure will control active bleeding
o do not blindly clamp or place tourniquets on damaged extremities
 Assessment
o soft-tissue damage
o neurovascular exam
 Dressing
o remove gross debris from wound
o place sterile saline-soaked dressing on the wound

- 14 -
By Dr, AbdulRahman AbdulNasser Genaral Trauma | Evaluation
 Stabilize
o splint fracture for temporary stabilization
 decreases pain, further injury from bone ends, and disruption of clots

Basic Principles of Open Fracture Management in the Operating Room


 Aggressive debridement and irrigation
o thorough debridement is critical to prevention of deep infection
o low and high pressure lavage are equally effective in reducing bacterial counts
o saline shown to be most effective irrigating agent
 on average, 3L of saline are used for each successive Gustilo type
 Type I: 3L
 Type II: 6L
 Type III: 9L
o bony fragments without soft tissue attachment can be removed
 Fracture stabilization
o can be with internal or external fixation, as indicated
 Staged debridement and irrigation
o perform every 24 to 48 hours as needed
 Early soft tissue coverage or wound closure is ideal
o timing of flap coverage for open tibial fractures remains controversial
o increased risk of infection beyond 7 days
 Can place antibiotic bead-pouch in open dirty wounds

o beads made by mixing methylmethacrylate with heat-stable antibiotic powder


Antibiotic Treatment
 Gustilo Type I and II

o 1st generation cephalosporin


o clindamycin or vancomycin can also be used if allergies exist
 Gustilo Type III
o 1st generation cephalosporin and aminoglycoside
 Farm injuries or possible bowel contamination
o add penicillin for anaerobic coverage (clostridium)
 Duration
o initiate as soon as possible
 studies show increased infection rate when antibiotics are delayed for more than 3
hours from time of injury
o continue for 24 hours after initial injury if wound is able to be closed primarily
o continue until 24 hours after final closure if wound is not closed during initial surgical
debridement
Bone Gap Reconstruction
 Reconstruction options
o Masquelet technique
o distraction osteogenesis
o vascularized bone flap

- 15 -
OrthoBullets2017 Genaral Trauma | Evaluation
Tetanus Prophylaxis
 Initiate in emergency room or trauma bay
 Two forms of prophylaxis
o toxoid dose 0.5 mL, regardless of age
o immune globulin dosing
 <5-years-old receives 75U
 5-10-years-old receives 125U
 >10-years-old receives 250U
o toxoid and immunoglobulin should be given intramuscularly with two different syringes in
two different locations
 Guidelines for tetanus prophylaxis depend on 3 factors
o complete or incomplete vaccination history (3 doses)
o date of most recent vaccination
o severity of wound

- 16 -
By Dr, AbdulRahman AbdulNasser Genaral Trauma | Specific Presentations

B. Specific Presentations

1. Gun Shot Wounds


Introduction
 Epidemiologyrepresent second-leading cause of death
for youth in United States
 Pathoanatomy
o wounding capability of a bullet directly related to
its kinetic energy
o damage caused by
 passage of missile
 secondary shock wave
 cavitation
o exponential increase in injury with increasing
velocity and efficient energy transfer
o fractures may be caused even without direct impact ‎I:6 This image demonstrates advanced
 Associated conditions destruction of the hip joint from a
retained bullet
o lead intoxication (plumbism)
 may be caused by intra-articular missile
 systemic effects include
 neurotoxicity
 anemia
 emesis
 abdominal colic
o GSW to hip and acetabulum are most commonly associated with bowel perforation > vascular
injury > urogenital injuries
Classification
 Low velocity
o muzzle velocity <350 meters per second or < 2,000 feet per second
o most handguns except for magnums
o wounds comparable to Gustillo-Anderson Type I or II
 Intermediate velocity
o muzzle velocity 350-500 meters per second
o shotgun blasts
 highly variable depending on distance from target
 can reflect wounding potential of high velocity firearms from close range (less than 21 feet)
or multiple low velocity weapons
 wound contamination/infection with close range injuries due to shotgun wadding
 wounding potential depends on 3 factors
 shot pattern
 load (size of individual pellet)
 distance from target
 High velocity
o muzzle velocity >600 meters per second or >2,000 feet per second
- 17 -
OrthoBullets2017 Genaral Trauma | Specific Presentations
o military (assault) and hunting rifles
o wounds comparable to Gustillo-Anderson Type III regardless of size
o high risk of infection
 secondary to wide zone of injury and devitalized tissue

Presentation
 Symptoms
o pain, deformity
 Physical exam
o perform careful neurovascular exam
o clinical suspicion for compartment syndrome
 secondary to increased muscle edema from higher velocity
wounds
o examine and document all associated wounds
 massive bone and soft tissue injuries occur even with low
velocity weapons
‎I:7 This clinical photo demonstrates a
 Evaluation large soft tissue wound with exposed
 Radiographs tibialis anterior and bone at the level of
the ankle joint
o obtain to identify bone involvement and/or fracture pattern
 CT scan
o identify potential intra-articular missile
o detect hollow viscus injury that may communicate with
fracture
 high index of suspicion for pelvis or spine fractures given
increased risk of associated bowel injury
Treatment General
 Nonoperative
o local wound care
 indications
 low velocity GSW with no bone involvement and clean
wound edges ‎I:8 This sagittal CT image
o local wound care, tetanus +/- short course of oral antibiotics demonstrates an intra-articular
bullet in the ankle joint
 indications
 low-velocity injury with no bone involvement or non-
operative fractures
 technique
 primary closure contraindicated
 antibiotic use controversial but currently recommended if wound appears contaminated
 Operative
o treatment of other non-orthopedic injuries
 for trans-abdominal trajectories, laparotomy takes precedence over arthrotomy
o ORIF/external fixation
 indications
 unstable/operative fracture pattern in low-velocity gunshot injury
 technique

- 18 -
By Dr, AbdulRahman AbdulNasser Genaral Trauma | Specific Presentations
 treatment dictated by fracture characteristics similar to closed fracture without gunshot
wound
 stabilize extremity with associated vascular or nerve injuries
 stabilize soft tissues in high velocity/high energy gunshot injuries
 grossly contaminated/devitalized wounds managed with aggressive debridement per
open fracture protocol
o arthrotomy
 indications
 intra-articular missile
 may lead to local inflammation, arthritis and lead intoxication (plumbism)
 transabdominal GSW

This image demonstrates


This image demonstrates a open reduction and internal
comminuted humeral shaft This angiogram demonstrates fixation of the humeral shaft
fracture following a gun shot abrupt cutoff of the brachial fracture with dual anterior and
injury with an associated artery at the level of the medial plating through the
brachial artery injury humeral shaft fracture vascular exposure

This image demonstrates a This image demonstrates This image demonstrates


severely comminuted humeral external fixation of the advanced destruction of the
shaft fracture associated with previously seen humeral shaft hip joint from a retained bullet
a significant soft tissue injury fracture with associated soft
from a gun shot wound tissue injury

- 19 -
OrthoBullets2017 Genaral Trauma | Specific Presentations
GSW to Hand/Foot
 Nonoperative
o antibiotics
 indications
 gross contamination
 joint penetration ‎I:9 This image
demonstrates a
 extent of contamination unclear comminuted femoral
 Operative shaft fracture
secondary to a
o surgical debridement +/- ORIF/external fixation ballistic injury
 indications
 articular involvement
 unstable fractures
 presentation 8 or more hours after injury
 tendon involvement
 superficial fragments in the palm or sole
I‎:10 This image
GSW to Femur demonstrates immediate
 Operative intramedullary nailing
of a femur fracture from
o intramedullary nailing a gun shot wound
 indications
 diaphyseal femur fracture secondary to low-velocity gunshot wound
 superficial wound debridement and immediate reamed nailing
 similar union and infection rates to closed injuries
o external fixation
 indications
 high-velocity gunshot wounds or close range shotgun blasts
 stabilize soft tissues and debride aggressively
 associated vascular injury
 temporize extremity until amenable to intramedullary nailing

GSW to Spine
 Nonoperative
o broad spectrum IV antibiotics for 7-14 days
 indications
 gunshot wounds to the spine with associated perforated viscus
 bullets which pass through the alimentary canal and cause spinal cord injuries do not
require surgical removal of the bullet
 Operative
o surgical decompression and bullet fragment removal
 indications
 when a neurologic deficit is present that correlates with
radiographic findings of neurologic compression
 a retained bullet fragment within the spinal canal in patients
with incomplete motor deficits is a relative indication for
surgical excision of the fragment ‎I:11 This axial CT image
demonstrates a retained
bullet in the spinal canal

- 20 -
By Dr, AbdulRahman AbdulNasser Genaral Trauma | Specific Presentations

2. Amputations
Introduction
 May be used to treat trauma
o infection
o tumor
o vascular disease
o congenital anomalies
 Prognosis
o outcomes are improved with involvement of psychological counseling for coping mechanisms
o amputation vs. reconstruction
 LEAP study
 impact on decision to amputate limb
 severe soft tissue injury
 highest impact on decision-making process
 absence of plantar sensation
 2nd highest impact on surgeon's decision making process
 not an absolute contraindication to reconstruction
 plantar sensation can recover by long-term follow-up
 outcome measure
 SIP (sickness impact profile) and return to work not significantly different between
amputation and reconstruction at 2 years in limb-threatening injuries
 most important factor to determine patient-reported outcome is the ability to return to
work
 Complications
o wound healing
o neuroma
o phantom limb pain : mirror therapy is a noninvasive treatment modality

Metabolic Demand
 Metabolic cost of walking
o increases with more proximal amputations
 perform amputations at lowest possible level to preserve function
 exception
 Syme amputation is more efficient than midfoot amputation
o inversely proportional to length of remaining limb
 Ranking of metabolic demand (% represents amount of increase compared to baseline)
o Syme - 15%
o transtibial
 traumatic - 25% average
 short BKA - 40%
 long BKA - 10%
 vascular - 40%
o transfemoral
 traumatic - 68%
 vascular - 100%

- 21 -
OrthoBullets2017 Genaral Trauma | Specific Presentations
o thru-knee amputation
 varies based on patient habitus but is somewhere between transtibial and transfemoral
 most proximal amputation level available in children to maintain walking speeds without
increased energy expenditure compared to normal children
o bilateral amputations

 BKA + BKA - 40%


 AKA + BKA - 118%
 AKA + AKA - >200%

Wound Healing
 Dependent on
o vascular supply
o nutritional status
o immune status
 Improved with
o albumin > 3.0 g/dL
o ischemic index > .5
 measurement of doppler pressure at level being tested compared to brachial systolic pressure
o transcutaneous oxygen tension > 30 mm Hg (ideally 45 mm Hg)
o toe pressure > 40 mm Hg (will not heal if < 20 mm Hg)
o ankle-brachial index (ABI) > 0.45
o total lymphocyte count (TLC) > 1500/mm3
 Hyperbaric oxygen therapy
o contraindications include
 chemo or radiation therapy
 pressure-sensitive implanted medical device (automatic implantable cardiac defibrillator,
pacemaker, dorsal column stimulator, insulin pump)
 undrained pneumothorax

Upper Extremity Amputation


 Indications
o irreparable loss of blood supply
o severe soft tissue compromise
o malignant tumors
o smoldering infection
o congenital anomalies
 Levels of amputation
o wrist disarticulation versus transradial amputation
 wrist disarticulation advantages
 improved pronation and supination
 recommended in children for preservation of distal radial and ulnar physes
 longer lever arm
 transradial advantages
 more aesthetically pleasing
 easier to fit prosthesis

- 22 -
By Dr, AbdulRahman AbdulNasser Genaral Trauma | Specific Presentations
o transhumeral versus elbow disarticulation
 elbow disarticulation advantages
 indicated in children to prevent bony overgrowth seen in transhumeral amputations
 Techniques
o transcarpal
 transect finger flexor/extensor tendons
 anchor wrist flexor/extensor tendons to carpus
o wrist disarticulation : preserve radial styloid flare to improve prosthetic suspension
o transradial amputation : middle third of forearm amputation maintains length and is ideal
o transhumeral amputation : maintain as much length as possible
o shoulder disarticulation : retain humeral head to maintain shoulder contour

Figure ‎I:14 shoulder disarticulation with


preservation of humeral head to maintain
Figure ‎I:12 wrist disarticulation Figure ‎I:13transradial amputation shoulder girdle.

Transfemoral Amputation
 Maintain as much length as possible
o however, ideal cut is 12 cm above knee joint to allow for prosthetic
fitting
 Technique
o 5-10 degrees of adduction is ideal for improved prosthesis function
o adductor myodesis
 improves clinical outcomes
 creates dynamic muscle balance
 provides soft tissue envelope that enhances prosthetic fitting
‎I:15 Illustration showing
Through-Knee-Amputation adductor myodesis technique.

 Indications
o ambulatory patients who cannot have a transtibial amputation
o non-ambulatory patients
 Technique
o suture patellar tendon to cruciate ligaments in notch
o use gastrocnemius muscles for padding at end of amputation
 Outcomes (based on LEAP data)
o slower self-selected walking speeds than BKA
o similar amounts of pain compared to AKA and BKA
o worse performance on the Sickness Impact Profile (SIP) than BKA and
AKA
- 23 -
OrthoBullets2017 Genaral Trauma | Specific Presentations
o physicians were less satisfied with the clinical, cosmetic, and functional recovery
o require more dependence with patient transfers than BKA
Below-Knee-Amputation (BKA)
 Long posterior flap
o 12-15 cm below knee joint is ideal
 ensures adequate lever arm
o need approximately 8-12 cm from ground to fit most modern high-impact prostheses
o osteomyoplastic transtibial amputation (Ertl) technique
 create a strut from the tibia to fibula from a piece of fibula or osteoperiosteal flap
o "dog ears"
 left in place to preserve blood supply to the flap
 Modified Ertl
o designed to enhance prosthetic end-bearing
o technique
 the original Ertl amputation required a corticoperiosteal flap bridge
 the modified Ertl uses a fibular strut graft
 requires longer operative and touniquet times than standard BKA transtibial amputation
 fibula is fixed in place with cortical screws, fiberwire suture with end buttons, or heavy
nonabsorbable sutures.

Example of a below-knee The Ertl Example showing maintained dog-ears


amputation for a high-voltage technique for preservation of blood supply.
electrical injury. creates a
strut from the
tibia to fibula.

Ankle/Foot Amputation
 Syme amputation (ankle disarticulation)
o patent tibialis posterior artery is required
o more energy efficient than midfoot even though it is more proximal
o stable heel pad is most important factor
o used successfully to treat forefoot gangrene in diabetics
 Pirogoff amputation (hindfoot amputation)
o removal of the forefoot and talus followed by calcaneotibial arthrodesis
o calcaneus is osteotomized and rotated 50-90 degrees to keep posterior aspect of calcaneus distal
o allows patient to mobilize independently without use of prosthetic
 Chopart amputation (hindfoot amputation)
o a partial foot amputation through the talonavicular and calcaneocuboid joints

- 24 -
By Dr, AbdulRahman AbdulNasser Genaral Trauma | Specific Presentations
o primary complication is equinus deformity
 avoid by lengthening of the Achilles tendon and transfer of the tibialis anterior to the talar
neck
 Lisfranc amputation
o equinovarus deformity is common
 caused by unopposed pull of tibialis posterior and gastroc/soleus
 prevent by maintaining insertion of peroneus brevis

Figure ‎I:16 Syme Amputation Figure ‎I:17 Chopart amputation


Figure ‎I:18 Chopart amputation
radiology

‎I:19 Lisfranc amputation ‎I:20 Transmetatarsal amputation


 Transmetatarsal amputation
o more appealing to patients who refuse transtibial amputations
o almost all require achilles lengthening to prevent equinus
 Great toe amputations
o preserve 1cm at base of proximal phalanx
 preserves insertion of plantar fascia, sesamoids, and flexor hallucis brevis
 reduces amount of weight transfer to remaining toes
 lessens risk of ulceration

Pediatric Amputation
 Most common complication is bone overgrowth
o prevent by performing disarticulation or using epihphyseal cap to cover medullary canal

- 25 -
OrthoBullets2017 Genaral Trauma | Specific Presentations

3. Domestic and Elder Abuse


Elder abuse
 Introduction
o can be in the form of mental or physical abuse, neglect, financial wrongdoings or abandonment
o epidemiology
 affects around 2 million Americans per year
 approximately 1 in 25 individuals over 65 y/o experiences abuse or neglect
o risk factors of victims for elder abuse
 increasing age
 functional disability
 child abuse within the regional population
 cognitive impairment
 gender is NOT a risk factor
o caretaker risk factors for elder abuse
 financial dependence on the elder person
 substance abuse
 perceiving the caretaking duty as burdensome
o associated injuries
 long bone fractures
 rib fractures
 bruises caused from abuse are
 commonly over 5cm in size
 located on the face, neck or back
 Clinical signs of elder abuse
o unexplained injuries
o delays in seeking care
o repeated fractures, burns, lacerations, etc
o change in behavior
o poor hygiene
 Treatment
o duty to report
 physicians are mandated reporters of abuse on elders in almost all states
 elder abuse is very commonly under reported
o admit to hospital
 if the patient is in immediate danger, the patient should be admitted to the hospital

Domestic, spousal or intimate partner abuse


 Introduction
o epidemiology
 as many as 35% of women presenting to the ER with injuries are a result of domestic
violence
 approximately 25% of women experience domestic violence
 15% of males are victims of domestic violence
 over 8.5% reported prior history of abuse
 children abused in close to 50% of homes where domestic violence occurs

- 26 -
By Dr, AbdulRahman AbdulNasser Genaral Trauma | Specific Presentations
o risks for domestic abuse
 female
 19-29 years
 pregnant
 low-income families/low socioeconomic status
o characteristic injuries or patterns
 injuries inconsistent with history
 long delay between injury and treatment
 repeat injuries
o characteristics of abused patient
 change in affect
 constantly seeking partner approval
 finding excuses to stay in treatment facility for prolonged period of time
 repeated visits to the emergency department
 significant time missed at work or decreased productivity at work
o characteristics of the abuser
 refuses to leave patient alone
 overly attentive
 aggressive or hostile
 refuses to let the patient answer their own questions
o barrier to reporting
 fear of retaliation
 shame
 difficulty reporting to male physicians
 fear of custody conflicts
 Treatment
o duty to act
 health care workers should inquire into the safety environment at home in cases of suspected
abuse
 emotional abuse is more difficult to discern than physical violence
 reporting requirements for adult abuse is not standardized among states
 a physician does not have authority to provide protection to abused spouses in most states
 should encourage victim to seek protection and report case to law enforcement
 physician should document encounter completely and be familiar with their state laws

4. Adult Respiratory Distress Syndrome


Introduction
 Acute Respiratory Distress Syndrome (ARDS) results from acute lung injury that leads to
o non-cardiogenic pulmonary edema
o respiratory distress
o refractory hypoxemia
o decreased lung compliance
 Etiology
o acute endothelial damage resulting from
 aspiration

- 27 -
OrthoBullets2017 Genaral Trauma | Specific Presentations
 infection
 pancreatitis
 multiple blood transfusions
 lung injury
 sepsis or shock
 major trauma
 large surface area burns
 fat emboli
 thromboembolism
 multi-system organ failure
 Prognosis : high mortality rate (50% overall) is associated with ARDS even in setting of ICU

Classification
 ARDS is represented by three phases

Exudative phase initially hyaline membrane comprised of fibrin form


Proliferative phase 3 days alveolar exudate resolves or organizes
Fibrotic phase 3-4 weeks alveolar ducts and spaces undergo fibrosis

Presentation
 Symptoms
o acute onset (12-48 hours) of
 dyspnea
 fever
 mottled or cyanotic skin
 Physical exam
o resistant hypoxia
o intercostal retractions
o rales/crackles and ronchi
o tachypnea

Evaluation
 Hypoxemia is refractory to O2
o 3 different categories of ARDS based on degree of hypoxemia
o PaO2 / FIO2 ratio < 300 mm Hg= mild
o PaO2 / FIO2 ratio < 200 mm Hg= moderate
o PaO2 / FIO2 ratio < 100 mm Hg= severe
 Chest xray
o shows patchy pulmonary edema (air space disease)
o diffuse bilateral pulmonary infiltrates
 normal sized heart
 makes CHF less likely
 Respiratory compliance (<40 mL/cm H20)
 Positive end-expiratory pressure (>10cm H20)
 Corrected expired volume per minute (>10L/min)

- 28 -
By Dr, AbdulRahman AbdulNasser Genaral Trauma | Specific Presentations
Differential
 Cardiogenic pulmonary edema (i.e. CHF or MI), bilateral pneumonia, SARS

Treatment
 Nonoperative
o PEEP ventilation and steroids
o treat the underlying pathology/disease
 Operative
o early stabilization of long bone fractures (femur)
 Prevention
o closely monitor PEEP in patients at-risk of ARDS
o serial X-rays in concerning patients can assist in early identification and intervention

Complications
 Pneumothorax
o secondary to ventilator with high PEEP

- 29 -
OrthoBullets2017 Genaral Trauma | Compartment Syndrome

C. Compartment Syndrome
Introduction
 Devastating condition where an osseofascial compartment pressure rises to a level that decreases
perfusion , may lead to irreversible muscle and nerve damage
 Epidemiology
o location : compartment syndrome may occur anywhere that skeletal muscle is surrounded by
fascia, but most commonly
 leg
 forearm
 hand
 foot
 thigh
 buttock
 shoulder
 paraspinous muscles
 Pathophysiology
o etiology
 trauma
 fractures (69% of cases)
 crush injuries
 contusions
 gunshot wounds
 tight casts, dressings, or external wrappings
 extravasation of IV infusion
 burns
 postischemic swelling
 bleeding disorders
 arterial injury
o pathoanatomy
 cascade of events includes
 local trauma and soft tissue destruction>
 bleeding and edema >
 increased interstitial pressure >
 vascular occlusion >
 myoneural ischemia

1. Leg Compartment Syndrome


Anatomy
o 4 compartments of the leg
o anterior compartment
 function
 dorsiflexion of foot and ankle
 muscles
 tibialis anterior

- 30 -
By Dr, AbdulRahman AbdulNasser Genaral Trauma | Compartment Syndrome
 extensor hallucis longus
 extensor digitorum longus
 peroneus tertius
o lateral compartment
 function
 plantarflexion and eversion of
foot
 muscles
 peroneus longus
 peroneus brevis
 isolated lateral compartment
syndrome would only affect
superficial peroneal nerve
o deep posterior compartment
 function

 plantarflexion and inversion of Figure ‎I:21 Compartments of the leg


foot
 muscles
 tibialis posterior
 flexor digitorum longus
 flexor hallucis longus
o superficial posterior compartment
 function
 mainly plantarflexion of foot and ankle
 muscles
 gastrocnemius
 soleus
 plantaris

Presentation
 Symptoms
o pain out of proportion to clinical situation is usually first symptom
 may be absent in cases of nerve damage
 pain is difficult to assess in a polytrauma patient and impossible to assess in a sedated patient
 difficult to assess in children (unable to verbalize)
 Physical exam
o pain w/ passive stretch : is most sensitive finding prior to onset of ischemia
o paresthesia and hypoesthesia
 indicative of nerve ischemia in affected compartment
o paralysis
 late finding
 full recovery is rare in this case
o palpable swelling
o peripheral pulses absent
 late finding
 amputation usually inevitable in this case

- 31 -
OrthoBullets2017 Genaral Trauma | Compartment Syndrome
Imaging
 Radiographs
o obtain to rule-out fracture

Studies
 Compartment pressure measurements
o indications
 polytrauma patients
 patient not alert/unreliable
 inconclusive physical exam findings
o relative contraindication
 unequivocally positive clinical findings should prompt emergent operative
intervention without need for compartment measurements
o technique
 should be performed within 5cm of fracture site
 anterior compartment
 entry point
 1cm lateral to anterior border of tibia within 5cm of fracture site if possible
 needle should be perpendicular to skin
 deep posterior compartment
 entry point
 just posterior to the medial border of tibia
 advance needle perpendicular to skin towards fibula
 lateral compartment
 entry point
 just anterior to the posterior border of fibula
 superficial posterior
 entry point
 middle of calf within 5 cm of fracture site if possible
 Diagnosis
o based primarily on physical exam in patient with intact mental status

Figure ‎I:22 Compartment pressure measurement

- 32 -
By Dr, AbdulRahman AbdulNasser Genaral Trauma | Compartment Syndrome

Figure ‎I:23 dual medial-lateral incision


Figure ‎I:25 anterolateral incision

Figure ‎I:24 single lateral incision Figure ‎I:26 posteromedial incision

Treatment
 Nonoperative
o observation
 indications
 diastolic differential pressure (delta p) is > 30
 presentation not consistent with compartment syndrome

o bi-valving the cast and loosening circumferential dressings


indications : initial treatment for swelling or pain that is NOT compartment syndrome

 splinting the ankle between neutral and resting plantar flexion (37 deg) can also
decrease intra compartmental pressures
o hyperbaric oxygen therapy
 works by increasing the oxygen diffusion gradient
 Operative
o emergent fasciotomy of all four compartments
 indications
 clinical presentation consistent with compartment syndrome
 compartment pressures within 30 mm Hg of diastolic blood pressure (delta p)
 intraoperatively, diastolic blood pressure may be decreased from anesthesia
 must compare intra-operative measurement to pre-operative diastolic pressure
 attempt to restore systemic blood pressure prior to measurement
 contraindications
 missed compartment syndrome

- 33 -
OrthoBullets2017 Genaral Trauma | Compartment Syndrome
 Special considerations
o pediatrics
 children are unable to verbalize feelings
 if suspicion, then perform compartment pressure measurement under sedation
o hemophiliacs : give Factor VIII replacement before measuring compartment pressures

Techniques
 Emergent fasciotomy of all four compartments
o dual medial-lateral incision
 approach
 two 15-18cm vertical incisions separated by 8cm skin bridge
 anterolateral incision
 posteromedial incision
 technique
 anterolateral incision
 identify and protect the superficial peroneal nerve
 fasciotomy of anterior compartment performed 1cm in front of intermuscular
septum
 fasciotomy of lateral compartment performed 1cm behind intermuscular septum
 posteromedial incision
 protect saphenous vein and nerve
 incise superficial posterior compartment
 detach soleal bridge from back of tibia to adequately decompress deep posterior
compartment
 post-operative
 dressing changes followed by delayed primary closure or skin grafting at 3-7 days post
decompression

 pros
 easy to perform
 excellent exposure
 cons : requires two incisions
o single lateral incision
 approach: single lateral incision from head of fibula to ankle along line of fibula
 technique
 identify superficial peroneal nerve
 perform anterior compartment fasciotomy 1cm anterior to the intermuscular septum
 perform lateral compartment fasciotomy 1cm posterior to the intermuscular septum
 identify and perform fasciotomy on superficial posterior compartment
 enter interval between superficial posterior and lateral compartment
 reach deep posterior compartment by following interosseous membrane from the
posterior aspect of fibula and releasing compartment from this membrane
 common peroneal nerve at risk with proximal dissection
 pros : single incision
 cons : decreased exposure

- 34 -
By Dr, AbdulRahman AbdulNasser Genaral Trauma | Compartment Syndrome

2. Thigh Compartment Syndrome

Anatomy
 3 thigh compartments
o anterior compartment
 muscles
 quadriceps
 sartorious
 nerves
 femoral nerve
o posterior compartment
 muscles
 hamstrings
 nerves
 sciatic nerve
o adductor compartment
 muscles
 adductors
 nerves Figure ‎I:27 Thigh Compartments
 obturator nerve

Presentation
 Symptoms
o pain out of proportion to clinical situation is usually first symptom
 may be absent in cases of nerve damage
 pain is difficult to assess in a polytrauma patient and impossible to assess in a sedated
patient
 difficult to assess in children (unable to verbalize)
 Physical exam
o pain with passive stretch : is most sensitive finding prior to onset of ischemia
 must test each compartment separately
 anterior compartment
 pain with passive flexion of knee
 posterior compartment
 pain with passive extension of knee
 medial compartment
 pain with passive abduction of hip
o paraesthesia and hypoesthesia : indicative of nerve ischemia in affected compartment
o paralysis
 late finding
 full recovery is rare in this case
o palpable swelling
o peripheral pulses absent
 late finding
 amputation usually inevitable in this case

- 35 -
OrthoBullets2017 Genaral Trauma | Compartment Syndrome
Evaluation
 Diagnosis
o based primarily on physical exam in patient with intact mental status
 Radiographs
o obtain to rule-out fracture
 Compartment pressure measurements
o indications
 polytrauma patients
 patient not alert/unreliable
 inconclusive physical exam findings
o relative contraindication
 unequivocally positive clinical findings should prompt emergent operative
intervention without need for compartment measurements
o technique
 should be performed within 5cm of fracture site or area of maximal swelling
 must test each compartment separately

Treatment
 Nonoperative
o observation
 indications
 delta p > 30, and
 presentation not consistent with compartment syndrome
 Operative
o emergent fasciotomy of all affected compartments
 indications
 clinical presentation consistent with compartment syndrome
 compartment pressures with absolute value of 30-45 mm Hg
 compartment pressures within 30 mm Hg of diastolic blood pressure (delta p)
 intraoperatively, diastolic blood pressure may be decreased from anesthesia
 must compare intra-operative measurement to pre-operative diastolic pressure
 contraindications
 missed compartment syndrome

Surgical Techniques
 Thigh fasciotomies
o approach
 anterolateral incision over length of thigh
o technique
 single incision technique for anterior and posterior compartments
 incise fascia lata
 expose and decompress anterior compartment
 retract vastus lateralis medially to expose lateral intermuscular septum
 incise lateral intermuscular septum to decompress posterior compartment
 may add medial incision for decompression of adductor compartment

- 36 -
By Dr, AbdulRahman AbdulNasser Genaral Trauma | Compartment Syndrome
Complications
 Associated with significant long-term morbidity
o over 50% will experience functional deficits
including
 pain
 decreased knee flexion
 myositis ossificans
 sensory deficits
 decreased strength

Figure ‎I:28 thigh compartment release

3. Hand & Forearm Compartment Syndrome


Anatomy
 Forearm compartments
o 3 in total
 volar : most commonly affected
 dorsal
 mobile wad (lateral)
 rarely involved
 muscles
 brachioradialis
 extensor carpi radialis longus
 extensor carpi radialis brevis
 Hand compartments
o 10 in total
 hypothenar
 thenar
 adductor pollicis
 dorsal interosseous (x4)
 volar (palmar) interosseous (x3)

Figure ‎I:29 Forearm compartments Figure ‎I:30 Hand compartments

- 37 -
OrthoBullets2017 Genaral Trauma | Compartment Syndrome
Presentation
 Symptoms
o pain out of proportion to clinical situation is usually first symptom
 may be absent in cases of nerve damage
 difficult to assess in
 polytrauma
 sedated patients
 children
 Physical exam
o pain w/ passive stretch of fingers
 most sensitive finding
o paraesthesia and hypoesthesia
 indicative of nerve ischemia in affected compartment
o paralysis
 late finding
 full recovery is rare in this case
o palpable swelling
 tense hand in intrinsic minus position
 most consistent clinical finding
o peripheral pulses absent
 late finding
 amputation usually inevitable in this case

Evaluation
 Radiographs : obtain to rule-out fracture
 Compartment pressure measurements
o indications
 polytrauma patients
 patient not alert/unreliable
 inconclusive physical exam findings
o relative contraindication
 unequivocally positive clinical findings should prompt emergent operative intervention
without need for compartment measurements
o threshold for decompression
 controversial, but generally considered to be
 absolute value of 30-45 mm Hg
 within 30 mm Hg of diastolic blood pressure (delta p)
 intraoperatively, diastolic blood pressure may be decreased from anesthesia
 if delta p is less than 30 mmHg intraoperatively, check preoperative diastolic
pressure and follow postoperatively as intraoperative pressures may be low and
misleading
Treatment
 Nonoperative
o indications
 exam not consistent with compartment syndrome
 delta p > 30

- 38 -
By Dr, AbdulRahman AbdulNasser Genaral Trauma | Compartment Syndrome
 Operative
o emergent forearm fasciotomies
 indications
 clinical presentation consistent with compartment syndrome
 compartment measurements with absolute value of 30-45 mm Hg
 compartment measurements within 30 mm Hg of diastolic blood pressure (delta p)
 intraoperatively, diastolic blood pressure may be decreased from anesthesia
 must compare intra-operative measurement to pre-operative diastolic pressure
o emergent hand fasciotomies
 indications
 clinical presentation consistent with compartment syndrome
 compartment measurements with absolute value of 30-45 mm Hg
 compartment measurements within 30 mm Hg of diastolic blood pressure (delta p)
 intraoperatively, diastolic blood pressure may be decreased from anesthesia
 must compare intra-operative measurement to pre-operative diastolic pressure

Surgical Techniques
 Forearm
o emergent fasciotomies of all involved compartments
 approach
 volar incision
 decompresses volar compartment, dorsal compartment,
carpal tunnel
 incision starts just radial to FCU at wrist and extends
proximally to medial epicondyle
 may extend distally to release carpal tunnel
 dorsal incision
 decompresses mobile wad
 dorsal longitudinal incision 2cm distal to lateral
epicondyle toward midline of wrist
 technique
 volar incision
 open lacertus fibrosus and fascia over FCU
 retract FCU ulnarly, retract FDS radially
 open fascia over deep muscles of forearm Figure ‎I:31 Volar incision
 dorsal incision
 dissect interval between EDC and ECRB
 decompress mobile wad and dorsal
compartment
 post-operative
 leave wounds open
 wound VAC Figure ‎I:32 Dorsal incision

 sterile wet-to-dry dressings


 repeat irrigation and debridement 48-72 hours later
 debride all dead muscle

- 39 -
OrthoBullets2017 Genaral Trauma | Compartment Syndrome
 possible delayed primary wound closure
 VAC dressing when closure cannot be obtained
 follow with split-thickness skin grafting at a
later time
 Hand
Figure ‎I:33 Leave the wound open
o emergent fasciotomies of all involved
compartments
 approach
 two longitudinal incisions over 2nd and 4th
metacarpals
 decompresses volar/dorsal interossei and
adductor compartment
 longitudinal incision radial side of 1st metacarpal
 decompresses thenar compartment
 longitudinal incision over ulnar side of 5th
metacarpal
 decompresses hypothenar compartment
 technique Figure ‎I:34 Hand incisions
 first volar interosseous and adductor pollicis muscles are
decompressed through blunt dissection along ulnar side of 2nd metacarpal
 post-operative
 wounds left open until primary closure is possible
 if primary closure not possible, split-thickness skin grafting is used

Complications
 Volkman's ischemic contracture
o irreversible muscle contractures in the forearm, wrist and hand that result from muscle
necrosis
o contracture positioning
 elbow flexion
 forearm pronation
 wrist flexion
 thumb adduction
 MCP joints in extension
 IP joints in flexion
o classification : Tsuge Classification (see table below)

Stages & Treatment of Volkman's Ischemic Contracture of Hand


Stage Affected muscle Treatment
Mild Finger flexors Dynamic splinting, tendon lengthening
Wrist and finger Excision of necrotic tissue, median and ulnar neurolysis, BR to FPL
Moderate
flexors and ECRL to FDP tendon transfers, distal slide of viable flexors
Wrist/finger flexors
Severe Same as above (moderate) with possible free muscle transfer
and extensors

- 40 -
By Dr, AbdulRahman AbdulNasser Genaral Trauma | Compartment Syndrome

4. Foot Compartment Syndrome


Anatomy
 9 main compartments (controversial)
o medial
 abductor hallucis
 flexor hallucis brevis
o lateral
 abductor digiti minimi
 flexor digiti minimi brevis
o interosseous (x4)
o central (x3)
 superficial
 flexor digitorum brevis
 central
 quadratus plantae
 deep
 adductor hallucis Figure ‎I:35 Compartments of the foot
 posterior tibial neurovascular bundle

Presentation
 Symptoms
o pain out of proportion to injury
 Physical exam
o pain with dorsiflexion of toes (MTPJ)
 places intrinsic muscles on stretch
o tense swollen foot
o loss of two-point discrimination
o pulses
 presence of pulses does not exclude diagnosis

Evaluation
 Radiographs
o obtain to rule-out fracture
 Compartment pressure measurements
o indications
 polytrauma patients
 patient not alert/unreliable
 inconclusive physical exam findings
o relative contraindication
 unequivocally positive clinical findings should prompt emergent operative intervention
without need for compartment measurements
o technique
 central compartment
 base of first metatarsal
 direct needle lateral and plantar through abductor hallucis
 medial compartment
- 41 -
OrthoBullets2017 Genaral Trauma | Compartment Syndrome
 base of first metatarsal advancing 2cm into abductor hallucis
 interosseous
 second, third, and fourth webspaces
 advance plantar 2cm to puncture extensor fascia
 lateral
 midshaft of fifth metatarsal
 advance 1cm medial and plantar
o threshold for decompression
 controversial, but generally considered to be
 absolute value of 30-45 mm Hg
 within 30 mm Hg of diastolic blood pressure (delta p)
 intraoperatively, diastolic blood pressure may be decreased from anesthesia
 must compare intra-operative measurement to pre-operative diastolic pressure

Treatment
 Nonoperative
o observation
 indications
 delta p > 30
 exam not consistent with compartment syndrome
 Operative
o emergent foot fasciotomies
 indications
 clinical presentation consistent with compartment syndrome
 compartment measurements with absolute value of 30-45 mm Hg
 compartment measurements within 30 mm Hg of diastolic blood pressure (delta p)
 intraoperatively, diastolic blood pressure may be decreased from anesthesia
 must compare intra-operative measurement to pre-operative diastolic pressure

Surgical Technique
 Emergent fasciotomies of all compartments
o dual dorsal incisions (gold standard)
 approach
 dorsal medial incision
 medial to 2nd metatarsal
 releases 1st and 2nd interosseous, medial, and deep central compartment
 dorsal lateral incision
 lateral to 4th metatarsal
 releases 3rd and 4th interosseous, lateral, superficial and middle
central compartments
 technique
 dorsal fascia of each interosseous compartment opened longitudinally
 strip muscle from medial fascia in first interosseous compartment
 split adductor compartment
 may add medial incision for decompression of calcaneal compartment
 post-operative
 delayed wound closure with possible skin grafting

- 42 -
By Dr, AbdulRahman AbdulNasser Genaral Trauma | Compartment Syndrome
 pros
 direct access to all compartments
 provides exposure for Chopart, Lisfranc, or tarsometatarsal fractures
 cons
 does not provide access for fixation of calcaneus fractures
o single medial incision
 technique
 single medial incision used to release all nine compartments
 cons
 technically challenging

Complications
 Chronic pain and hypersensitivity
o difficult to manage
 Fixed flexion deformity of digits (claw toes)
o release flexor digitorum brevis and longus at level of digits

Figure ‎I:36 medial incision Figure ‎I:37 dual dorsal incisions

Figure ‎I:38 surgical approach for dual dorsal incisions Figure ‎I:39 claw toes

Chapter of infections (adult osteomyelitis, septic arthritis, wound & hardware infections, necrotizing
fasciitis and Gas gangrene) all these topics moved from trauma to pathology volume eight.

- 43 -
OrthoBullets2017 Spine Trauma | Compartment Syndrome

ORTHO BULLETS

II. Spine Trauma

- 44 -
By Dr, AbdulRahman AbdulNasser Spine Trauma | Spinal Cord Injury

A. Spinal Cord Injury

1. Cervical Spine Trauma Evaluation


Introduction
 All trauma patients have a cervical spine injury until proven otherwise
 Cervical spine clearance defined as confirming the absence of cervical spine injury
o important to clear cervical spine and remove collar in an efficient manner
 delayed clearance associated with increased complication rate
o cervical clearance can be performed with
 physical exam
 radiographically
 Missed cervical spine injuries
o may lead to permanent disability
o careful clinical and radiographic evaluation is paramount
 high rate of missed cervical spine injuries due to
 inadequate imaging of affected level
 loss of consciousness
 multisystem trauma
o cervical spine injury necessitates careful examination of entire spine
 noncontiguous spinal column injuries reported in 10-15% of patients

History
 Details of accident
o energy of accident
 higher level of concern when there is a history of high energy trauma as indicated by
 MVA at > 35 MPH
 fall from > 10 feet
 closed head injuries
 neurologic deficits referable to cervical spine
 pelvis and extremity fractures
o mechanism of accident
 e.g., elderly person falls and hits forehead (hyperextension injury)
 e.g., patient rear-ended at high speed (hyperextension injury)
o condition of patient at scene of accident
 general condition
 degree of consciousness
 presence or absence of neurologic deficits
 Identify associated conditions and comorbidities
o ankylosing spondylitis (AS)
o diffuse idiopathic skeletal hyperostosis (DISH)
o previous cervical spine fusion (congenital or acquired)
o connective tissue disorders leading to ligamentous laxity

Physical Exam
 Useful for detecting major injuries

- 45 -
OrthoBullets2017 Spine Trauma | Spinal Cord Injury
 Primary survey
o airway
o breathing
o circulation
o visual and manual inspection of entire spine should be performed
 manual inline traction should be applied whenever cervical immobilization is removed for
securing airway
 seat belt sign (abdominal ecchymosis) should raise suspicion for flexion distraction injuries
of thoracolumbar spine
 Secondary survey
o cervical spine exam
 remove immobilization collar
 examine face and scalp for evidence of direct trauma
 inspect for angular or rotational deformities in the holding position of the patient's head
 rotational deformity may indicate a unilateral facet dislocation
 palpate posterior cervical spine looking for tenderness along the midline or paraspinal tissues
 absence of posterior midline tenderness in the awake, alert patient predicts low
probability of significant cervical injury7,
 log roll patient to inspect and palpate entire spinal axis
 perform careful neurologic exam

Clinical Cervical Clearance


 Removal of cervical collar WITHOUT radiographic studies allowed if
o patient is awake, alert, and not intoxicated AND
o has no neck pain, tenderness, or neurologic deficits AND
o has no distracting injuries

Radiographic Cervical Clearance


 Indications for obtaining radiographic clearance
o intoxicated patients OR
o patients with altered mental status OR
o neck pain or tenderness present OR
o distracting injury present
 Mandatory radiographic clearance with either
o cervical spine radiographic series
 must include top of T1 vertebra
 includes : AP, lateral, open-mouth odontoid view
 inadequate radiographs are the most common reason for missed injury to the cervical spine
 assess alignment by looking at the four parallel lines on the lateral radiograph
 look for subtle abnormalities such as
 soft-tissue swelling
 hypolordosis
 disk-space narrowing or widening

- 46 -
By Dr, AbdulRahman AbdulNasser Spine Trauma | Spinal Cord Injury

‎II:2 standard Lat crvical spine ‎II:3 standard open-mouth odontoid view
‎II:1 standard AP crvical spine radiograph radiograph radiograph

‎II:5 widening of the interspinous


‎II:4 Flexion and extesion views distances ‎II:6 four parallel lines on the lateral radiograph
 widening of the interspinous distances
o CT to bottom of first thoracic vertebra
 replacing conventional radiographs as initial imaging in most trauma centers
 pros
 more sensitive in detecting injury than plain radiographs
 some studies show faster to obtain than plain radiographs
 cons : increased radiation exposure
 Supplementary radiographic studies include
o flexion-extension radiographs
 pros : effective at ruling-out instability
 cons : can only be performed in awake and alert patient
o MRI
 pros
 highly sensitive for detection of soft tissue injuries
 disc herniations
 posterior ligament injuries
 spinal cord changes I‎I:7 MRI showing injury of posterior
ligamintous complex

- 47 -
OrthoBullets2017 Spine Trauma | Spinal Cord Injury
 cons
 high rate of false positives
 only effective if done within 48 hours of injury
 can be difficult to obtain in obtunded or intoxicated patients
o MR and CT angiography
 pros : effective for evaluating vertebral artery

Treatment
 Nonoperative
o cervical collar
 indications : initiated at scene of injury until directed examination performed
o early active range of motion
 indications
 "whiplash-like" symptoms and
 cleared from a serious cervical injury by exam or imaging

Complications
 Delayed clearance associated with increased complication rate including
o increased risk of aspiration
o inhibition of respiratory function
o decubitus ulcers in occipital and submandibular areas
o possible increase in intracranial pressure

2. Spinal Cord Injuries


Introduction
 Epidemiology
o incidence
 11,000 new cases/year in US
 34% incomplete tetraplegia
 central cord syndrome most common
 25% complete paraplegia
 22% complete tetraplegia
 17% incomplete paraplegia
o demographics
 bimodal distribution
 young individuals with significant trauma
 older individuals that have minor trauma compounded by degenerative spinal canal
narrowing
o location : 50% in cervical spine
 Mechanism
o MVA causes 50%
o falls
o GSW
o iatrogenic
 it is estimated that 3-25% of all spinal cord injuries occur after initial traumatic episode due
to improper immobilization and transport.
- 48 -
By Dr, AbdulRahman AbdulNasser Spine Trauma | Spinal Cord Injury
 Pathophysiology
o primary injury
 damage to neural tissue due to direct trauma
 irreversible
o secondary injury
 injury to adjacent tissue due to
 decreased perfusion
 lipid peroxidation
 free radical / cytokines
 cell apoptosis
 methylprednisone used to prevent secondary injury by improving perfusion, inhibiting lipid
peroxidation, and decreasing the release of free radicals
 Associated conditions
o acute phase conditions (see below)
 spinal shock
 neurogenic shock
o associated injuries
 closed head injuries
 noncontiguous spinal fractures
 vertebral artery injury
 risk factors for vertebral artery injury include
 atlas fractures
 facet dislocations
 most people with unilateral injury remain asymptomatic
 imaging
 magnetic resonance angiography is least invasive method
 treatment
 stenting only if patient is symptomatic from basilar arterial insufficiency
 Prognosis
o only 1% have complete recovery at time of hospital diagnosis
 conus medullaris syndrome has a better prognosis for recovery than more proximal lesions

Relevant Anatomy
 See Spinal Cord Anatomy

Classification
 Descriptive
o atetrplegia
 injury to the cervical spinal cord leading to impairment of function in the arms, trunk, legs,
and pelvic organs
o paraplegia
 injury to the thoracic, lumbar or sacral segments leading to impairment of function in the
trunk, legs, and pelvic organs depending on the level of injury. Arm function is preserved

- 49 -
OrthoBullets2017 Spine Trauma | Spinal Cord Injury
o complete injury
 an injury with no spared motor or sensory function below the affected level.
 patients must have recovered from spinal shock (bulbocavernosus reflex is intact) before an
injury can be determined as complete
 classified as an ASIA A
o incomplete injury
 an injury with some preserved motor or sensory function below the injury level
 incomplete spinal cord injuries include
 anterior cord syndrome
 Brown-Sequard syndrome
 central cord syndrome
 posterior cord syndrome
 conus medullaris syndromes
 cauda equina syndrome

ASIA Classification
 Determine if patient is in spinal shock
o check bulbocavernosus reflex
 Determine neurologic level of injury
o lowest segment with intact sensation and antigravity (3 or more) muscle function strength
o in regions where there is no myotome to test, the motor level is presumed to be the same as the
sensory level.
 Determine whether the injury is COMPLETE or INCOMPLETE
o COMPLETE defined as (ASIA A)
 no voluntary anal contraction (sacral sparing) AND
 0/5 distal motor AND
 0/2 distal sensory scores (no perianal sensation) AND
 bulbocavernosus reflex present (patient not in spinal shock)
o INCOMPLETE defined as
 voluntary anal contraction (sacral sparing)
 sacral sparing critical to determine complete vs. incomplete
 OR palpable or visible muscle contraction below injury level OR
 perianal sensation present
 Determine ASIA Impairment Scale (AIS) Grade:

ASIA Impairment Scale


A Complete No motor or sensory function is preserved in the sacral segments S4-S5.
B Incomplete Sensory function preserved but not motor function is preserved below the
neurological level and includes the sacral segments S4-S5.
C Incomplete Motor function is preserved below the neurological level, and more than half of
key muscles below the neurological level have a muscle grade less than 3.
D Incomplete Motor function is preserved below the neurological level, and at least half of key
muscles below the neurological level have a muscle grade of 3 or more.
E Normal Motor and sensory function are normal.

- 50 -
By Dr, AbdulRahman AbdulNasser Spine Trauma | Spinal Cord Injury
Acute Phase Conditions
 Neurogenic shock
o characterized by hypotension & relative bradycardia in patient with an acute spinal cord injury
 potentially fatal
o mechanism
 circulatory collapse from loss of sympathetic tone
 disruption of autonomic pathway within the spinal cord leads to
 lack of sympathetic tone
 decreased systemic vascular resistance
 pooling of blood in extremities
 hypotension
o treatment
 Swan-Ganz monitoring for careful fluid management
 pressors to treat hypotension
 Spinal shock
o defined as temporary loss of spinal cord function and reflex activity below the level of a spinal
cord injury.
o characterized by
 flaccid areflexic paralysis
 bradycardia & hypotension (due to loss of sympathetic tone)
 absent bulbocavernosus reflex
 reflex characterized by anal sphincter contraction in response to squeezing the glans
penis or tugging on an indwelling Foley catheter
o timing
 variable but usually resolves within 48 hours
 at its conclusion spasticity, hyperreflexia, and clonus slowly progress over days to weeks
o mechanism
 neurophysiologic in nature
 neurons become hyperpolarized and unresponsive to stimuli from brain
o evaluation
 important because one cannot evaluate neurologic deficit until spinal shock phase has
resolved
 end of spinal shock indicated by return of the bulbocavernous reflex
 conus or cauda equina injuries may lead to permanent loss of the bulbocavernous reflex

Spinal Shock Neurogenic Shock Hypovolemic Shock


BP Hypotension Hypotension Hypotension
Pulse Bradycardia Bradycardia Tachycardia
Reflexes /
Bulbocavernosus Absent Variable/independent Variable/independent
Reflex
Motor Flaccid Paralysis Variable/independent Variable/independent
Time ~48-72 hours immediately ~48-72 hours immediately after spinal cord Following excessive blood
after spinal cord injury injury loss
Mechanism Peripheral neurons become Disruption of autonomic pathway leads to
Decreased preload leads to
temporarily unresponsive to loss of sympathetic tone and decreased
decreased cardiac output.
brain stimuli. systemic vascular resistance.

- 51 -
OrthoBullets2017 Spine Trauma | Spinal Cord Injury
Evaluation
 Field treatment
o treatment of potential spinal cord injuries begins at the accident scene with proper spinal
immobilization
o immobilization
 immobilization should include rigid cervical collar and transport on firm spine board with
lateral support devices
 patient should be rolled with standard log roll techniques with control of cervical spine
o athletes
 in the setting of sports-related injuries helmets and shoulder pads should be left on until
arrival at hospital or until experienced personnel can perform simultaneous removal of
helmet and shoulder pads in a controlled situation
 Initial evaluation
o primary survey
 airway
 breathing
 SCI above C5 likely to require intubation
 circulation
 initial survey to inspect for obvious injuries of head and spine
 visual and manual inspection of entire spine should be performed
 seat belt sign (abdominal ecchymoses) should raise suspicion for flexion distraction
injuries of thoracolumbar spine
o secondary survey
 cervical spine exam
 remove immobilization collar
 examine face and scalp for evidence of direct trauma
 inspect for angular or rotational deformities in
the holding position of the patient's head
 rotational deformity may indicate a unilateral
facet dislocation
 palpate posterior cervical spine looking for
tenderness along the midline or paraspinal
tissues
 absence of posterior midline tenderness in
the awake, alert patient predicts low
probability of significant cervical injury7,
 log roll patient to inspect and palpate entire
spinal axis
 perform careful neurologic exam
o cervical spine clearance

- 52 -
By Dr, AbdulRahman AbdulNasser Spine Trauma | Spinal Cord Injury

Acute Treatment
 Nonoperative
o high dose methylprednisone
 indications
 nonpenetrating SCI within 8 hours of injury
 recommended by NASCIS III
 contraindications include
 GSW
 pregnancy
 under 13 years
 > 8 hours after injury
 brachial plexus injuries
 technique
 load 30 mg/kg over 1st hour (2 grams for 70kg man)
 drip 5.4 mg/kg/hr drip
 for 23 hours if started < 3 hrs after injury
 for 47 hours if started 3-8 hours after injury
 outcomes
 leads to improved root function at level of injury
 may or may not lead to spinal cord function improvement
o monosialotetrahexosylganglioside (GM-1)
 indications
 remains controversial
- 53 -
OrthoBullets2017 Spine Trauma | Spinal Cord Injury
 large multicenter RCT did not show long term benefit
 some evidence of faster recovery
o acute closed reduction with axial traction
 indications
 alert and oriented patient with neurologic deficits and compression due to
fracture/dislocation
 bilateral facet dislocation with spinal cord injury in alert and oriented patient is most
common reason to perform acute reduction with axial traction
 technique
 reasons to abort
 overdistraction
 worsening neurologic exam
 failure to obtain reduction
o DVT prophlaxis
 indications
 most patients
 contraindications include
 coagulopathy
 hemorrhage
 modalities
 low-molecular weight heparin
 rotating bed
 pneumatic compression stocking
o cardiopulmonary management
 careful hemodynamic monitoring and stabilization is critical in early treatment
 hypotension should be avoided
 implement immediate aggressive pulmonary protocols
 Operative : rarely indicated in acute setting

Definitive Treatment
 Nonoperative
o bracing and observation
 indications
 most GSWs
 exceptions listed below
 metastatic CA patients with < 6 mos life expectancy
 presence of six variables below correspond to short life expectancy
 multiple spinal mets
 multiple extraspinal mets
 unresectable lesions in major organs
 SCI (complete or incomplete)
 aggressive CA: lung, osteosarcoma, pancreas
 critically ill
 Operative
o surgical decompression and stabilization
 indications
 most incomplete SCI (except GSW)
- 54 -
By Dr, AbdulRahman AbdulNasser Spine Trauma | Spinal Cord Injury
 decompress when patient hits neurologic plateau or if worsening neurologically
 decompression may facilitate nerve root function return at level of injury (may
recover 1-2 levels)
 most complete SCI (except GSW)
 stabilize spine to facilitate rehab and minimize need for halo or orthosis
 decompression may facilitate nerve root function return at level of injury (may
recover 1-2 levels)
 consider for tendon transfers
 e.g. Deltoid to triceps transfer for C5 or C6 SCI
 metastatic CA patients with > 6 mos life expectancy
 ~ no for six question above
 GSW with
 progressive neurological deterioration with retained bullet within the spinal canal
 cauda equina syndrome (considered a peripheral nerve)
 retained bullet fragment within the thecal sac
 CSF leads to the breakdown of lead products that may lead to lead poisoning

Complications
 Skin problems
o treatment is prevention
o start in ER
 do not leave on back board
 start log rolling early
 proper bedding
 Venous Thromboembolism
o prevent with immediate DVT prophylaxis
 Urosepsis
o common cause of death
o strict aseptic technique when placing catheter
o don't let bladder become overly distended
 Sinus bradycardia
o most common cardiac arrhythmia in acute stage following SCI
 Orthostatic hypotension
o occurs as a result of lack of sympathetic tone
 Autonomic dysreflexia
o potentially fatal
o presents with headache, agitation, hypertension
o caused by unchecked visceral stimulation
 check foley
 disimpact patient
 Major depressive disorder
o ~11% of patients with spinal cord injuries suffer from MDD
o MDD in spinal cord injury patients is highly associated with suicidal ideation in both the acute
and chronic phase.

- 55 -
OrthoBullets2017 Spine Trauma | Spinal Cord Injury
Rehabilitation
 Goals
o goal is to assess and identify mechanisms for reintegration into community based on functional
level and daily needs
o patients learn transfer techniques, self care retraining, mobility skills
 Restoring hand function
o hand function is often limiting factor for many patients
o tendon transfers can be used to restore function to paralyzed arms and hands by giving working
muscles different jobs
 Modalities
o functional electrical stimulation is a technique that uses electrical currents to stimulate and
activate muscles affected by paralysis

Level Patient Function


C1-C3 - Ventilator dependent with limited talking.
- Electric wheelchair with head or chin control
C3-C4 - Initially ventilator dependent, but can become independent
- Electric wheelchair with head or chin control
C5 - Ventilator independent
- Has biceps, deltoid, and can flex elbow, but lacks wrist extension and supination needed to
feed oneself
- Independent ADL’s; electric wheelchair with hand control, minimal manual wheelchair function
C6 - C6 has much better function than C5 due to ability to bring hand to mouth and feed oneself
(wrist extension and supination intact)
- Independent living; manual wheelchair with sliding board transfers, can drive a car with
manual controls
C7 - Improved triceps strength
- Daily use of a manual wheelchair with independent transfers
C8-T1 - Improved hand and finger strength and dexterity
- Fully independent transfers
T2-T6 - Normal UE function
- Improved trunk control
- Wheelchair-dependent
T7-T12 - Increased abdominal muscle control
- Able to perform unsupported seated activities; with extensive bracing walking may be possible
L1-L5 - Variable LE and B/B function
- Assist devices and bracing may be needed
S1-S5 - Various return of B/B and sexual function
- Walking with minimal or no assistance

- 56 -
By Dr, AbdulRahman AbdulNasser Spine Trauma | Spinal Cord Injury

3. Incomplete Spinal Cord Injuries


Introduction
 Defined as spinal cord injury with some preserved motor or sensory function below the injury level
including
o voluntary anal contraction (sacral sparing)
 sacral sparing critical to separate complete vs. incomplete injury
o OR palpable or visible muscle contraction below injury level
o OR perianal sensation present
 Epidemiology
o 11,000 new cases/year in US
 34% incomplete tetraplegia
 central cord syndrome most common
 17% incomplete paraplegia
 remaining 47% are complete
 Prognosis
o most important prognostic variable relating to neurologic recovery is completeness of the
lesion (severity of neurologic deficit)
Anatomy
 Descending Tracts (motor)
o lateral corticospinal tract (LCT)
o ventral corticospinal tract
 Ascending tracts (sensory)
o dorsal columns
 deep touch
 vibration
 proprioception
o lateral spinothalamic tract (LST)
 pain
 temperature
o ventral spinothalamic tract (VST)
 light touch

Classification
 Clinical classification
o anterior cord syndrome (see below)
o Brown-Sequard syndrome
o central cord syndrome
o posterior cord syndrome
 ASIA classification
o method to scale

- 57 -
OrthoBullets2017 Spine Trauma | Spinal Cord Injury
ASIA Impairment Scale
A Complete No motor or sensory function is preserved in the sacral segments S4-S5.
B Incomplete Sensory function preserved but not motor function is preserved below the
neurological level and includes the sacral segments S4-S5.
C Incomplete Motor function is preserved below the neurological level, and more than half of
key muscles below the neurological level have a muscle grade less than 3.
D Incomplete Motor function is preserved below the neurological level, and at least half of key
muscles below the neurological level have a muscle grade of 3 or more.
E Normal Motor and sensory function are normal.

Central Cord Syndrome


 Epidemiology
o incidence : most common incomplete cord injury
o demographics
 often in elderly with minor extension injury mechanisms
 due to anterior osteophytes and posterior infolded ligamentum flavum
 Pathophysiology
o believed to be caused by spinal cord compression and central cord edema with selective
destruction of lateral corticospinal tract white matter
o anatomy of spinal cord explains why upper extremities and hand preferentially affected
 hands and upper extremities are located "centrally" in corticospinal tract
 Presentation
o symptoms
 weakness with hand dexterity most affected
 hyperpathia
 burning in distal upper extremity
o physical exam
 loss
 motor deficit worse in UE than LE (some preserved motor function)
 hands have more pronounced motor deficit than arms
 preserved
 sacral sparing
o late clinical presentation
 UE have LMN signs (clumsy)
 LE has UMN signs (spastic)
 Treatment
o nonoperative vs. operative
 extremely controversial
 Prognosis
o final outcome
 good prognosis although full functional recovery rare
 usually ambulatory at final follow up
 usually regain bladder control
 upper extremity and hand recovery is unpredictable and patients often have permanent
clumsy hands
- 58 -
By Dr, AbdulRahman AbdulNasser Spine Trauma | Spinal Cord Injury
o recovery occurs in typical pattern
 lower extremity recovers first
 bowel and bladder function next
 proximal upper extremity next
 hand function last to recover

Anterior Cord Syndrome


 A condition characterized by
o motor dysfunction
o dissociated sensory deficit below level of SCI
 Pathophysiology
o injury to anterior spinal cord caused by
 direct compression (osseous) of the anterior spinal cord
 anterior spinal artery injury
 anterior 2/3 spinal cord supplied by anterior spinal artery
 Mechanism : usually result of flexion/ compression injury
 Exam
o lower extremity affected more than upper extremity
o loss
 LCT (motor)
 LST (pain, temperature)
o preserved
 DC (proprioception, vibratory sense)
 Prognosis
o worst prognosis of incomplete SCI
o most likely to mimic complete cord syndrome
o 10-20% chance of motor recovery

Brown-Sequard Syndrome
 Caused by complete cord hemitransection
o usually seen with penetrating trauma
 Exam
o ipsilateral deficit
 LCS tract
 motor function
 dorsal columns
 proprioception
 vibratory sense
o contralateral deficit
 LST
 pain
 temperature
 spinothalamic tracts cross at spinal cord level (classically 2-levels below)
 Prognosis
o excellent prognosis
o 99% ambulatory at final follow up
o best prognosis for function motor activity

- 59 -
OrthoBullets2017 Spine Trauma | AtlantoAxial Trauma
Posterior Cord Syndrome
 Introduction : very rare
 Exam
o loss : proprioception
o preserved : motor, pain, light touch

B. AtlantoAxial Trauma

1. Occipital Condyle Fractures


Introduction
 Occipital condyle fractures are traumatic injuries that involve the craniocervicaljunction (CCJ)
 Epidemiology
o incidence
 approximately 1-3% of population with blunt craniocervical trauma
 often missed due to low diagnosis sensitivity of plain radiographs
 increased rate of diagnosis use due to increase in CT scan use
 Pathophysiology
o mechanism of injury
 high energy, non-penetrating trauma to the head/neck
 fracture patterns are dependent on the directional forces applied to the craniocervical junction
 Anderson and Montesano classification
 Type 1 = compression
 Type 2 = direct blow
 Type 3 = rotational/lateral bending
 Associated injuries
o orthopaedic manifestations
 cervical spinal cord injuries (31%)
 polytrauma
o medical manifestations
 intracranial bleeding
 brainstem and vascular lesions I‎I:8 Anderson and Montesano classification
 elevated ICP
 Prognosis : High mortality rate (11%) due to associated injuries

Anatomy
 Osteology
o occipital condyles are paired prominences of the occipital bone
o oval or bean shaped structures forming lateral aspects of the foramen magnum
 Joint articulations
o intrinsic relationship between occiput, atlas and axis to form the occipitoatlantoaxial complex or
CCJ
o 6 main synovial articulations
 anterior and posterior median atlanto-odontoid joints
 paired occipitoatloid joints
 paired atlantoaxial joints

- 60 -
By Dr, AbdulRahman AbdulNasser Spine Trauma | AtlantoAxial Trauma
 Ligamentous structures
o intrinsic ligaments are located within the spinal canal, provide most of the ligamentous stability.
They include
 transverse ligament
 primary stabilizer of atlantoaxial junction
 connects the posterior odontoid to the anterior atlas arch, inserting laterally on bony
tubercles.
 paired alar ligaments
 connect the odontoid to the occipital condyles
 relatively strong and contributes to occipitalcervical stability
 apical ligament
 relatively weak midline structure
 runs vertically between the odontoid and foramen magnum.
 tectorial membrane
 connects the posterior body of the axis to the anterior foramen magnum and is the
cephalad continuation of the PLL
 Neurovascular considerations
o proximity of the occipital condyles to:
 medulla oblongata
 vertebral arteries
 lower cranial nerves (CN IX - CN XII)

- 61 -
OrthoBullets2017 Spine Trauma | AtlantoAxial Trauma
Classification

Anderson and Montesano classification of occipital condyle fractures


Type I • Impaction-type fracture with comminution of the occipital condyle
• Due to compression between the atlanto-odontoid joint
• Stable injury due to minimal fragment displacement into the foremen magnum
Type II • Basilar skull fracture that extends into one- or both occipital condyles
• Due to a direct blow to skull
• Stable injury as the alar ligament and tectorial membrane are usually preserved
Type III • Avulsion fracture of condyle in region of the alar ligament attachment.
• Due to forced rotation with combined lateral bending.
• Has the potential to be unstable due to craniocervical disruption
Type I • MRI shows craniocervical ligament injury
• Craniocervical alignment is within 2mm of normal
• <2mm of cervical distraction with traction
Type II • MRI shows craniocervical ligament injury.
• Craniocervical alignment is within 2mm of normal.
• >2mm of cervical distraction with traction
Type III • Craniocervical malalignment is greater than 2mm
• >2mm of cervical distraction with traction

Harborview Classification of Craniocervical Injuries


Type I • MRI shows craniocervical ligament injury
• Craniocervical alignment is within 2mm of normal
• <2mm of cervical distraction with traction
Type II • MRI shows craniocervical ligament injury.
• Craniocervical alignment is within 2mm of normal.
• >2mm of cervical distraction with traction
Type III • Craniocervical malalignment is greater than 2mm
• >2mm of cervical distraction with traction

- 62 -
By Dr, AbdulRahman AbdulNasser Spine Trauma | AtlantoAxial Trauma
Presentation
 History
o clinical presentation is highly variable
o presentation is largely dependent on associated injury (eg, head injury, brainstem injury, vascular
injury)
o neurological deficits may be acute (63% of cases) or delayed (37% of cases)
 Symptoms
o high cervical pain
o reduced head/neck ROM
o torticollis
o lower cranial nerve deficits
o motor paresis
 Physical Examination
o lower cranial nerve deficits most commonly affect CN IX, X, and XI

Imaging
 Radiographs
o recommended views
 AP, lateral, open-mouth AP view
o alternative views
 traction is generally not recommended
o findings
 diagnosis rarely made on plain radiographs due to superimposition of structures (maxilla,
occiput) blocking view of occipital condyles
 open-mouth AP view may depict occiptal condyle injuries
 CT
o indications
 method of choice
 routine CT imaging in high-energy trauma patients
 clinical criteria:
 altered consciousness
 occipital pain and tenderness
 impaired CCJ motion
 lower cranial nerve paresis
 motor paresis
o views : must include cranial-cervical junction with thin-section technique
o findings : occiput fracture or CCJ instability
 MRI
o indications
 evaluation of soft-tissue craniocervical trauma
 fractured fragment located in the vertebral canal
 spinal cord or brain stem ischemia
o views
 MR angiogram may be considered with suspected vascular injury
o findings

- 63 -
OrthoBullets2017 Spine Trauma | AtlantoAxial Trauma
 MRI better than CT for the assessment of associated brain and brain-stem injuries, although
CT still considered standard for evaluating acute subarachnoid hemorrhage
Treatment
 Nonoperative
o analgesics, cervical orthosis
 indications
 Type 1 and 2
 Type 3 without overt instability
 modalities
 semi-rigid or rigid cervical collar
 Operative
o occipitocervical fusion
 indications
 Type 3 with overt instability
 neural compression from displaced fracture fragment
 associated occipital-atlantal or atlanto-axial injuries
 technique
 C0-C2/C3 occipitocervical arthrodesis using rigid segmental fixation or posterior
decompression and instrumented fusion
 may require bone grafting or removal of boney fragments compressing neurovascular
structures.

2. Occipitocervical Instability & Dislocation


Introduction
 Occipitocervical instability may be
o traumatic occipitocervical dislocation
 severe injury and patients rarely survive
 most patients die of brainstem destruction
o acquired occipitocervical instability
 may be seen in patients with Down's syndrome
 occipital condyle hypoplasia
 results in limited AOJ motion and basilar invagination
 Epidemiology
o traumatic
 incidence : ~15-30% of cervical spine injuries occur at the occipitocervical junction
 prevalence : identified in 19% fatal cervical injuries
o acquired
 most frequently seen in Down syndrome population
 usually asymptomatic and identified in screen for surgery or special olympic participation
 Pathophysiology
o traumatic
 mechanism of injury
 high-energy trauma
 translation or distraction injuries that destabilize the occipitocervical junction
 pathoanatomy : head most often displaces anteriorly

- 64 -
By Dr, AbdulRahman AbdulNasser Spine Trauma | AtlantoAxial Trauma
o acquired
 pathoanatomy
 dueto bony dysplasia or ligament and soft-tissue laxity
 Associated conditions
o atlantoaxial instability
 also seen in Down syndrome patients
o neurologic deficits
o vertebral or carotid artery injuries
o Down Syndrome

Classification

Traynelis Classification (direction of displacement)


Type I Anterior occiput dislocation
Type II Longitudinal dislocation
Type III Posterior occiput dislocation

Harbourview Classification System (degree of instability)

Stage I Minimal or non-displaced, unilateral injury to craniocervical


Stable
ligaments

Stage II Minimally displaced, but MRI demonstrates significant soft-tissue Stable or


injuries. Stability may be based on traction test Unstable

Stage III Gross craniocervical misaligment (BAI or BDI > 2mm beyond normal
Unstable
limits)

Imaging
 Radiographs
o recommended views
 AP, lateral and odontoid views
o findings
 low sensitivity in detecting injury (57%)
o measurements
 used to diagnosis occipitocervical dislocation
 Powers ratio = C-D/A-B
 C-D: distance from basion to posterior arch
 A-B: distance from anterior arch to opisthion
 significance

- 65 -
OrthoBullets2017 Spine Trauma | AtlantoAxial Trauma
ratio ~ 1 is normal

 if > 1.0 concern for
 anterior dislocation
 ratio < 1.0 raises concern for
 posterior atlanto-occipital dislocation
 odontoid fractures
 ring of atlas fractures
 Harris rule of 12
 basion-dens interval or basion-posterior axial interval
 >12mm suggest occipitocervical dissociation
 CT
o indications
‎II:9 Harris rule of 12: >12mm suggests
occipitocervical dissociation
 considered gold standard for osseous injuries of the spine
o views
 midsaggital CT reconstruction
 MRI
o indications
 suspected ligamentous injury with preserved alignment or occult injury
 neurological deficits

- 66 -
By Dr, AbdulRahman AbdulNasser Spine Trauma | AtlantoAxial Trauma
Treatment
 Nonoperative
o provisional stabilization while avoiding traction
 indications
 traumatic instability with distraction of the occipitoatlantal joint
 techniques
 halo vest
 tongs
 prolonged cervical orthosis is not recommended due to poor stabilization of the AOJ
 outcomes
 use of traction should be avoided in most cases
 traction may be considered in stage 2 injuries when MRI demonstates soft-tissue injury
with perserved aligment
 Operative
o posterior occipitocervical fusion (C0 - C2 or lower)
 indications
 most traumatic cases require stabilization
 acquired cases when evidence of myelpathy or significant symptomatic neck pain
 invagination and atlanto-axial impaction secondary to inflammatory arthropathy (e.g.,
rheumatoid arthritis)
 tumor

Technique
 Posterior occipitocervical fusion
o approach
 midline posterior approach to base of skull
o instrumentation
 rigid occipitocervical screw-rod or plate construct
 aim for 3 bicortical occipital screws on each side of the midline (total 6 screws in occiput)
 extend to C2 or lower with polyaxial
pedical screws to achieve fixation
 the safe zone for occipital screws is located
within an area measuring 20mm lateral to
the external occipital protuberance along the
superior nuchal line
 the major dural venous sinuses are located
just below the external occipital protuberance
and are at risk of penetrative injury during
occipitocervical fusion
 autogenous bone graft

Complications
 Nonunion
 Bleeding

‎II:10 Safe zone for occipital screws

- 67 -
OrthoBullets2017 Spine Trauma | AtlantoAxial Trauma

3. Atlantoaxial Instability
Introduction
 The atlantoaxial joint is an important "transitional zone" in the cervical spine
o prone to instability by both degenerative and traumatic processes.
 Pathophysiology
o adult causes
 degenerative
 Down's syndrome
 Rheumatoid Arthritis
 Os odontoideum
 traumatic
 Type I odontoid fracture (very rare)
 Atlas fractures
 Transverse ligament injuries
o pediatric causes
 degenerative
 JRA
 Morquio's Syndrome
 lysosomal storage disorder
 trauma/infection
 rotatory atlantoaxial subluxation

Anatomy
 Osteology
o bony articulations
 C1-C2 facet joints
 Ligaments
o transverse apical alar ligament complex
 transverse ligament
 most important stabilizer
 apical ligament
 single midline structure
 alar ligaments
 paired parasagittal ligament
 Biomechanics
o the atlantoaxial joint provides ~50% of rotation in the cervical spine
 this is enabled by the peg (C2)-ring(C1)
anatomy
Physical Exam
 Symptoms
o symptomatic
o neck pain
o neurologic symptoms
 Physical exam
o neurologic deficits

- 68 -
By Dr, AbdulRahman AbdulNasser Spine Trauma | AtlantoAxial Trauma
 often appear late in disease process due to capacious nature of spinal canal at the C1 level
 myelopathic symptoms
 hyperreflexia (patellar tendon reflex)
 muscles weakness
 broad based gait
 decreased hand dexterity
 loss of motor milestones
 bladder problems

Imaging
 Radiographs
o flexion-extension xrays
 atlanto-dens interval (ADI)
 measurement
 distance between odontoid process and the posterior border of the anterior arch of the
atlas
 adult parameters
 > 3.5mm considered unstable
 > 10mm indicates surgery in RA
 other
 must get preoperative flexion-extension radiographs to clear all high-risk patients for
any type of surgery
 space-available-cord (SAC) = posterior atlanto-dens-interval (PADI)
 measurement
 distance from posterior surface of dens to anterior surface of posterior arch of atlas
 adult parameters
 in adults with RA < 14 mm associated with increased risk of neurologic injury and is
an indication for surgery
o open mouth odontoid
 sum of lateral mass displacement
 measurement
 lateral mass are connect by ring of C1, and therefore can only be displaced relative to
each other if
 there is a bony fracture (disruption of the ring)
 the transverse ligament is ruptured
 transverse ligaments binds them together
 adult parameters
 if > 8.1 mm, then a transverse ligament rupture is assured and the injury pattern is
considered unstable

- 69 -
OrthoBullets2017 Spine Trauma | AtlantoAxial Trauma

‎II:11 space-available-cord (SAC) = posterior atlanto-dens- ‎II:12 atlanto-dens interval (ADI)


interval (PADI)

‎II:13 sum of lateral mass displacement

Treatment
 Determined by specific condition
o adult atlantoaxial instability
 Down's syndrome
 Rheumatoid Arthritis
 Os odontoideum
 Odontoid fracture
 Atlas fractures
 Transverse ligament injuries
o pediatric atlantoaxial instability
 JRA
 Morquio's Syndrome
 Rotatory atlantoaxial subluxation

- 70 -
By Dr, AbdulRahman AbdulNasser Spine Trauma | AtlantoAxial Trauma

4. Atlas Fracture & Transverse Ligament Injuries


Introduction
 Epidemiology
o make up ~7% of cervical spine fractures
o risk of neurologic injury is low
o commonly missed due to inadequate imaging of occipitocervical junction
 Pathophysiology
o mechanism
 includes hyperextension, lateral compression, and axial compression
 Associated conditions
o spine fracture
 50% have an associated spine injury
 40% associated with axis fx
 Prognosis
o stability dependent on degree of injury and healing potential of
transverse ligament
Anatomy
 Bony anatomy
 Atlas osteology
o atlas (C1) is a ring containing two articular lateral masses

 it lacks a vertebral body or a spinous process ‎II:14 Embyology of atlas


 embryology
 forms from 3 ossification centers
 anatomic variation
 incomplete formation of the posterior arch is a relatively
common anatomic variant and does not represent a
traumatic injury
 Ligamentous anatomy
o occipital-cervical junction and atlantoaxial junction are coupled
o intrinsic ligaments are located within the spinal canal, provide
most of the ligamentous stability. They include ‎II:15 incomplete C1
 transverse ligament
 primary stabilizer of atlantoaxial junction
 connects the posterior odontoid to the anterior atlas arch, inserting laterally on bony
tubercles.
 paired alar ligaments
 connect the odontoid to the occipital condyles
 relatively strong and contributes to occipitalcervical stability
 apical ligament
 relatively weak midline structure
 runs vertically between the odontoid and foramen magnum.
 tectorial membrane
 connects the posterior body of the axis to the anterior foramen magnum and is the
cephalad continuation of the PLL
- 71 -
OrthoBullets2017 Spine Trauma | AtlantoAxial Trauma
Classification
Landells Atlas Fractures Classification
Type I Isolated anterior or posterior arch fracture. A "plough fracture is an
isolated anterior arch fracture caused by a force driving the odontoid
through the anterior arch. Stable. Treat with hard collar.
Type II Jefferson burst fracture with bilateral fractures of anterior and posterior
arch resulting from axial load. Stability determined by integrity of
transverse ligament. If intact, hard collar. If disrupted, halo vest (for bony
avulsion) or C1-2 fusion (for intrasubstance tear)(see Dickman
classification below).
Type III Unilateral lateral mass fx. Stability determined by integrity of transverse
ligament. If stable, treat with hard collar. If unstable, halo vest.

Dickman Transverse Ligament Injuries Classification


Type I Intrasubstance tear. Treat with C1-2 fusion.
Type II Bony avulsion at tubercle on C1 lateral mass. Treat with halo vest (successful in
75%)

plough fracture

CT Jefferson fracture type 3 atlas fracture

- 72 -
By Dr, AbdulRahman AbdulNasser Spine Trauma | AtlantoAxial Trauma
Imaging
 Radiographs
o lateral radiographs
 atlantodens interval (ADI)
 < 3 mm = normal in adult (< 5mm normal in child)
 3-5 mm = injury to transverse ligament with intact alar and apical ligaments
 > 5 mm = injury to transverse, alar ligament, and tectorial membrane
o open-mouth odontoid
 open-mouth odontoid view important to identify atlas fractures
 sum of lateral mass displacement
 if sum of lateral mass displacement is > 7 mm (8.1mm with radiographic magnification)
then a transverse ligament rupture is assured and the injury pattern is considered unstable
 CT
o study of choice to delineate fracture pattern and identify associated injuries in the cervical spine
 MRI
o more sensitive at detecting injury to transverse ligament

Lateral radiograph showing


atlantodens interval (ADI) > 5

Open mouth view with sum of lateral mass


displacement is > 7 mm MRI

- 73 -
OrthoBullets2017 Spine Trauma | AtlantoAxial Trauma
Treatment
 Nonoperative
o hard collar vs. halo immobilization for 6-12 weeks
 indications
 stable Type I fx (intact transverse ligament)
 stable Jefferson fx (Type II) (intact transverse ligament)
 stable Type III (intact transverse ligament)
 technique : controversy exists around optimal form of immobilization
 Operative
o posterior C1-C2 fusion vs. occipitocervical fusion
 indications
 unstable Type II (controversial)
 unstable Type III (controversial)
 technique : may consider preoperative traction to reduce displaced lateral masses

Techniques
 Posterior C1-C2 fusion
o preserves motion compared to occipitocervical fusion
o fixation
 C1 lateral mass / C2 pedicle screw construct
 may be sufficient if adequate purchase with C1 lateral mass screws
 C1-2 transarticular screw placement
 Occipitocervical fusion (C0-C2)
o uses when unable to obtain adequate purchase of C1 (comminuted C1 fracture)
o leads to significant loss of motion

Complications
 Delayed C-spine clearance
o higher rate of complications in patients with delayed C-spine clearance so it is important to clear
expeditiously

5. Odontoid Fracture (Adult and Pediatric)


Introduction
 A fracture of the dens of the axis (C2)
 Epidemiology
o incidence
 most common fracture of the axis
 account for 10-15% of all cervical fractures
o demographics
 occur in bimodal fashion in elderly and young patients
 elderly
 common, often missed, and caused by simple falls
 associated with increased morbidity and mortality compared to younger patients with
this injury
 young patients
 result from blunt trauma to head leading to cervical hyperflexion or hyperextension

- 74 -
By Dr, AbdulRahman AbdulNasser Spine Trauma | AtlantoAxial Trauma
 Pathophysiology
o mechanism
 displacement may be anterior (hyperflexion) or posterior (hyperextension)
 anterior displacement
 is associated with transverse ligament failure and atlanto-axial instability
 posterior displacement
 caused by direct impact from the anterior arch of atlas during hyperextension
o biomechanics
 a fracture through the base of the odontoid process severely compromises the stability of the
upper cervical spine.
 Associated conditions
o Os odontoideum
 appears like a type II odontoid fx on
xray
 previously thought to be due to
failure of fusion at the base of the
odontoid
 evidence now suggests it may
represent the residuals of an old traumatic process
 treatment is observation

Anatomy
 Axis Osteology
o axis has odontoid process (dens) and body
o embryology
 develops from five ossification centers
 subdental (basilar) synchondrosis is an initial
cartilagenous junction between the dens and vertebral
body that does not fuse until ~6 years of age
 the secondary ossification center appears at ~ age 3 and fuses to the dens at ~ age 12
 Axis Kinematics
o CI-C2 (atlantoaxial) articulation
 is a diarthrodal joint that provides
 50 (of 100) degrees of cervical rotation
 10 (of 110) degrees of flexion/extension
 0 (of 68) degrees of lateral bend
o C2-3 joint
 participates in subaxial (C2-C7) cervical motion which
provides
 50 (of 100) degrees of rotation
 50 (of 110) degrees of flexion/extension
 60 (of 68) degrees of lateral bend
 Occipital-C1-C2 ligamentous stability
o provided by the odontoid process and its supporting ligaments
 transverse ligament: limits anterior translation of the atlas
 apical ligaments : limit rotation of the upper cervical spine
 alar ligaments : limit rotation of the upper cervical spine
- 75 -
OrthoBullets2017 Spine Trauma | AtlantoAxial Trauma
 Blood Supply
oa vascular watershed exists between the apex and the base of the odontoid
 apex is supplied by branches of internal carotid artery
 base is supplied from branches of vertebral artery
 the limited blood supply in this watershed area is thought to affect healing of type II odontoid
fractures.

Classification
 Anderson and D'Alonzo Classification
Anderson and D'Alonzo Classification
Type I Oblique avulsion fx of tip of odontoid. Due to avulsion of alar ligament. Although
rare, atlantooccipital instability should be ruled out with flexion and extension
films.
Type II Fx through waist (high nonunion rate due to interruption of blood supply).
Type III Fx extends into cancellous body of C2 and involves a variable portion of the C1-C2
joint.

Grauer Classification of Type II Odontoid fractures


Type IIA Nondisplaced/minimally displaced with no comminution. Treatment is external
immobilization
Type IIB Displaced fracture with fracture line from anterosuperior to posteroinferior.
Treatment is with anterior odontoid screw (if adequate bone density).
Type IIC Fracture from anteroinferior to posterosuperior, or with significant comminution.
Treatment is with posterior stabilization.

- 76 -
By Dr, AbdulRahman AbdulNasser Spine Trauma | AtlantoAxial Trauma

CT sagital - Type II odointoid fx CT sagital - Type II odointoid fx

Presentation
 Symptoms
o neck pain worse with motion
o dysphagia may be present when associated with a large retropharyngeal hematoma
 Physical exam
o myelopathy
 very rare due to large cross section area of spinal canal at this level

Imaging
 Radiographs
o required views
 AP, lateral, open-mouth odontoid view of cervical spine
o optional views

- 77 -
OrthoBullets2017 Spine Trauma | AtlantoAxial Trauma
 flexion-extension radiographs are important to diagnose occipitocervical instability in Type I
fractures and Os odontoideum
 instability defined as
 atlanto-dens-interval (ADI) > 10mm
 < 13mm space available for cord (SAC)
o findings
 fx pattern best seen on open-mouth odontoid
 CT
o study of choice for fracture delineation and to assess stability of fracture pattern
 CT angiogram
o required to determine location of vertebral artery prior to posterior instrumentation procedures
 MRI
o indicated if neurologic symptoms present

Treatment
Treatment Overview
Os Odontoideum Observation
Type I Cervical Orthosis
Type II Young Halo if no risk factors for nonunion
Surgery if risk factors for nonunion
Type II Elderly Cervical Orthosis if not surgical candidates
Surgery if surgical candidates
Type III Cervical Orthosis
 Nonoperative
o observation alone
 indications
 Os odontoideum
 assuming no neurologic symptoms or instability
o hard cervical orthosis for 6-12 weeks
 indications
 Type I
 Type II in elderly who are not surgical candidates
 union is unlikely, however a fibrous union should provide sufficient stability except in
the case of major trauma
 Type III fractures
 no evidence to support Halo over hard collar
o halo vest immobilization for 6-12 weeks
 indications
 Type II young patient with no risk factors for nonunion
 contraindications
 elderly patients
 do not tolerate halo (may lead to aspiration, pneumonia, and death)
 Operative
o posterior C1-C2 fusion
 indications
- 78 -
By Dr, AbdulRahman AbdulNasser Spine Trauma | AtlantoAxial Trauma
 Type II fractures with risk factors for nonunion
 Type II/III fracture nonunions
 Os odontoideum with neurologic deficits or instability
o anterior odontoid osteosynthesis
 indications
 Type II fractures with risk factors for nonunion AND
 acceptable alignment and minimal displacement
 oblique fracture pattern perpendicular to screw trajectory
 patient body habitus must allow proper screw trajectory
 outcomes
 associated with higher failure rates than posterior C1-2 fusion
o transoral odontoidectomy
 indications
 severe posterior displacement of dens with spinal cord compression and neurologic
deficits
Surgical Techniques
 Halo immobilization

o in children and adults


 C1-C2 posterior fusion techniques
o approach
 posterior midline cervical approach
o stabilization technique
 sublaminar wiring techniques (Gallie or Brooks)
 require postoperative halo immobilization and rarely used
 posterior C1-C2 transarticular screws construct
 contraindicated in patients with an aberrant vertebral artery
 posterior C1 lateral mass screw and C2 pedicle screw construct
 modern screw constructs do not require postoperative halo immobilization
o outcomes
 C1-C2 fusion will lead to 50% loss of neck motion
 Anterior odontoid screw osteosynthesis
o approach
 anterior approach to cervical spine
o technique
 single screw adequate
o pros & cons
 associated with higher failure rate than posterior C1-2 fusion
 advantage is preservation of atlantoaxial motion
 Transoral odontoidectomy
o technique
 usually combined with posterior stabilization procedure

- 79 -
OrthoBullets2017 Spine Trauma | AtlantoAxial Trauma
Complications
 Nonunion
o increased risk in Type II fractures due to poor
blood supply
 average nonunion rate 33% (up to as high as
88%)
o risk factors for nonunion include
 ≥ 6 mm displacement (>50% nonunion rate)
 strongest reason to opt for surgery
 age > 50 years
 fx comminution
 angulations > 10° ‎II:16 Anterior odontoid screw osteosynthesis
 delay in treatment
 smoker

‎II:17 posterior C1 lateral mass screw and C2 pedicle screw construct ‎II:18 posterior C1-C2 transarticular screws construct

6. Traumatic Spondylolisthesis of Axis (Hangman's Fracture)


Introduction
 Traumatic anterior spondylolithesis of the axis due to bilateral fracture of pars interarticularis
o MVA is most common cause
 Mechanism is
o hyperextension : leads to fracture of pars
o secondary flexion : tears PLL and disc allowing subluxation
 Associated injuries
o 30% have concomitant c-spine fx

Presentation
 Symptoms
o neck pain
 Physical exam
o patients are usually neurologically intact

- 80 -
By Dr, AbdulRahman AbdulNasser Spine Trauma | AtlantoAxial Trauma
Imaging
 Radiographs

o flexion and extension radiographs show subluxation


 CT
o study of choice to delineate fracture pattern
 MRA
o consider if suspicious of a vascular injury to the vertebral artery
Classification & Treatment

Levine and Edwards Classification (based on mechanism of injury)


Type I  < 3mm horizontal displacement C2/3  Rigid collar x 4-6 weeks
 No angulation
 C2/3 disc remains intact
 stable fx pattern

Type II  > 3mm of horizontal displacement  If < 5mm displacement, reduction


 Significant angulation with traction then halo
 Vertical fracture line immobilization x 6-12 weeks
 C2/3 disc and PLL are disrupted  If > 5mm displacement, surgery or
 unstable fracture pattern prolonged traction
 Usually heal despite displacement
(autofuse C2 on C3)

Type IIA  No horizontal displacement  Avoid Traction in Type IIA.


 Horizontal fracture line  Reduction with gentle axial load +
 Significant angulation hyperextension, then compression
halo immobilization for 6-12 weeks.

 Type I fracture with associated bilateral  Surgical reduction of facet


C2-3 facet dislocation dislocation followed by stabilization
Type III
 Rare injury pattern required.

- 81 -
OrthoBullets2017 Spine Trauma | SubAxial Cervical Trauma
Treatment
 Nonoperative
o rigid cervical collar x 4-6 weeks
 indications
 Type I fractures (< 3mm horizontal displacement)
o closed reduction followed by halo immobilization for 8-12 weeks
 indications
 Type II with 3-5 mm displacement
 Type IIA
 reduction technique
 Type II use axial traction combined + extension
 Type IIA use hyperextension (avoid axial traction in Type IIA)
 Operative
o reduction with surgical stabilization
 indications
 Type II with > 5 mm displacement and severe angulation
 Type III (facet dislocations)
 technique
 anterior C2-3 interbody fusion
 posterior C1-3 fusion
 bilateral C2 pars screw osteosynthesis

C. SubAxial Cervical Trauma

1. Cervical Facet Dislocations & Fractures


Introduction
 Represent spectrum of osteoligamentous pathology that includes
o unilateral facet dislocation
 most frequently missed cervical spine injury on plain xrays
 leads to ~25% subluxation on xray
 associated with monoradiculopathy that improves with traction
o bilateral facet dislocation
 leads to ~50% subluxation on xray
 often associated with significant spinal cord injury
o facet fractures
 more frequently involves superior facet
 may be unilateral or bilateral
 Epidemiology
o location
 ~75% of all facet dislocations occur within the subaxial spine (C3 to C7)
 17% of all injuries are fractures of C7 or dislocation at the C7-T1 junction
 this reinforces the need to obtain radiographic visualization of the cervicothoracic
junction
 Pathophysiology
o mechanism : flexion and distraction forces +/- an element of rotation

- 82 -
By Dr, AbdulRahman AbdulNasser Spine Trauma | SubAxial Cervical Trauma
Classification
 Descriptive classification (subaxial cervical spine injuries)
o includes
 compression fracture
 burst fraction
 flexion-distraction injury
 facet dislocation (unilateral or bilateral)
 facet fracture
o more commonly used in clinical setting
 Allen and Ferguson classification (of subaxial cervical spine injuries)
o typically used for research and not in clinical setting
o based solely on static radiographs and mechanisms of injury

Allen and Ferguson Classification (of subaxial spine injuries)


1. Flexion-compression
2. Vertical compression
3. Flexion-distraction Stage 1: Facet subluxation
Stage 2: Unilateral facet dislocation

Stage 3: Bilateral facet dislocation with 50% displacement

Stage 4: Complete dislocation (100% displacement)


4. Extension-
compression
5. Extension-distraction
6. Lateral flexion

Presentation
 Physical exam
o monoradiculopathy
 seen in patients with unilateral dislocations
 C5/6 unilateral dislocation
 usually presents with a C6 radiculopathy
 weakness to wrist extension
 numbness and tingling in the thumb
 C6/7 unilateral dislocation
 usually presents with a C7 radiculopathy
 weakness to triceps and wrist flexion
 numbness in index and middle finger
o spinal cord injury symptoms
 seen with bilateral dislocations
 symptoms worsen with increasing subluxation

- 83 -
OrthoBullets2017 Spine Trauma | SubAxial Cervical Trauma
Imaging
 Radiographs
o lateral shows subluxation of vertebral bodies
o unilateral dislocations lead to ~ 25% subluxation
o bilateral facet dislocation leads to ~ 50% subluxation on xray
o loss of disc height might indicated retropulsed disc in canal
 CT scan
o essential to demonstrate
 bony anatomy of the injury
 malalignment or subtle subluxation of facet
 facet fracture
 associated fractures of the pedicle or lamina
 MRI
o indications are controversial but include
 acute facet dislocation in patient with altered mental status
 failed closed reduction and before open reduction to look for disc herniation
 any neurologic deterioration is seen during closed reduction
o timing
 timing of MRI depends on severity and progression of neurologic injury
 an MRI should always be performed prior to open reduction or surgical stabilization
 if a disc herniation is present with compression on the spinal cord, then you must go
anterior to perform a anterior cervical diskectomy
o valuable in demonstrating
 disc herniations
 extent of posterior ligamentous injury
 spinal cord compression or myelomalacia

MRI bilateral facet dislocation


Lat Xray - Unilateral lat xray bilateral facet CT bilateral facet C5-6 with spinal cord
facet dislocation dislocation C4-5 dislocation combined compression

- 84 -
By Dr, AbdulRahman AbdulNasser Spine Trauma | SubAxial Cervical Trauma

Treatment
 Nonoperative
o cervical orthosis or external immobilization (6-12 weeks)
 indications
 facet fractures without significant subluxation, dislocation, or kyphosis
 Operative
o immediate closed reduction, then MRI, then surgical stabilization
 indications

 bilateral facet dislocation with deficits in awake and cooperative patient


 unilateral facet dislocation with deficits in awake and cooperative patient
 technique
 never perform closed reduction in patient with mental status changes
 surgical stabilization following successful closed reduction
 unilateral dislocations are more difficult to reduce but more stable after reduction
 bilateral dislocation are easier to reduce (PLL torn) but less stable following reduction
 always obtain MRI prior to surgical stabilization
 PSF or ACDF can be performed in the absence of significant disc herniation
 ACDF performed if significant disc herniation present
 outcomes
 26% of patients will fail closed reduction and require open reduction
o immediate MRI then open reduction surgical stabilization
 indications
 facet dislocations (unilateral or bilateral) in patient with mental status changes
 patients who fail closed reduction
 technique

- 85 -
OrthoBullets2017 Spine Trauma | SubAxial Cervical Trauma
 always obtain MRI prior to open reduction and stabilization
 ifdisc herniation with presence of spinal cord compression then you must use an
anterior approach and do a discectomy
Techniques
 Closed reduction
o requirements
 adequate anesthesia
 sedation
 supervision of respiratory function
 serial cross table laterals
o technique
 gradually increase axial traction with the addition of weights
 a component of cervical flexion can facilitate reduction
 perform serial neurologic exams and plain radiographs after addition of each weight
 abort if neurologic exam worsens and obtain immediate MRI
 Anterior open reduction & ACDF
o indications
 facet dislocations reduced through closed methods with a MRI showing cervical disc
herniation with significant compression on the spinal cord
 unilateral facet dislocations that fail closed reduction with a disc herniation with significant
compression on the spinal cord
o anterior open reduction techniques
 can be used to reduce a unilateral facet dislocation
 reduction technique involves distracting vertebral bodies with caspar pins and then rotating
the proximal pin towards the side of the dislocation
 not effective for reducing bilateral facet dislocations
 Posterior reduction & instrumented stabilization
o indications
 when unable to reduce by closed or anterior approach
 no anterior compression of spinal cord(no disc herniation)
o technique
 performed with lateral mass screws
 usually have to fuse two levels due to inadequate lateral mass purchase at level of dislocation
 Combined anterior decompression and posterior reduction / stabilization
o indications
 when disc herniation present that requires decompression in patient that can not be reduced
through closed or open anterior technique
o technique
 go anterior first, perform discectomy, position plate but only fix plate to superior vertebral
body
 this way the plate will prevent graft kick-out but still allows rotation during the posterior
reduction
 this technique eliminates the need for a second anterior procedure

- 86 -
By Dr, AbdulRahman AbdulNasser Spine Trauma | SubAxial Cervical Trauma

2. Cervical Lateral Mass Fracture Separation


Introduction
 Fracture separations of the lateral mass-facet are uncommon injuries characterized by
o high degree of instability
o neurological deficit
o affect 2 levels (2 adjacent motion segments)
 because of involvement of the superior facet and inferior facet on either side of the fractured
articular mass
 Epidemiology
o demographics
 male : female ratio = 2:1
 mean age 35 yrs (20-70yrs)
o location
 C6 > C5 > C7 > C4 > C3
 Pathophysiology
o mechanism of injury
 traffic accident, falls, heavy object landing on head
 hyperextension, lateral compression and rotation of the cervical spine
 Associated conditions
o anterior translation (listhesis)
 fractured vertebrae (77%)
 superior adjacent vertebrae (24%)
 inferior adjacent vertebrae (10%)
o coronal translation (33%)
o vertebral body collapse (33%)
 lower in Type A Separation fracture subtypes

Classification
 Kotani Classification

Kotani Classification
Fracture Type Fracture Description Rates of Anterior Rates of Anterior
Translation (same Translation
level) (adjacent level)
Type A -
2 fracture lines of unilateral lamina and
Separation 91% 20%
pedicle
fracture
Type B - Multiple fracture lines with lateral wedging
- 50%
Comminution type in coronal plane
Type C - Split type Vertical fracture line in the coronal plane,
with invagination of the superior articular 80% 0%
process of the caudal vertebra
Type D - Bilateral horizontal fracture lines of the
Traumatic pars interarticularis, leading to separation 100% 50%
spondylolysis of the anterior-posterior spinal elements

- 87 -
OrthoBullets2017 Spine Trauma | SubAxial Cervical Trauma

Presentation
 History
o commonest mechanisms (Allen and Ferguson classification)
 extension-compression
 lateral flexion : results in Type B Comminuted subtype
 flexion-distraction
 Symptoms
o neurologic symptoms common (up to 66%)
 radicular pain, radiculopathy or spinal cord injury/myelopathy
 can be classified by Frankel grade or ASIA impairment scale
 Physical exam
o inspection
 torticollis, paravertebral muscle spasm
o neurovascular
 radicular pain and numbness
 myelopathy

Imaging
 Radiographs
o recommended views
 AP, lateral, oblique views
o findings
 disc space narrowing
 often difficult to detect on plain radiographs
 instability
 >3.5mm displacement
 >10deg kyphosis
 >10deg rotation difference compared with adjacent vertebra
o sensitivity and specificity
 low sensitivity
 38% pickup rate on plain radiographs
 CT
o indications
 to further evaluate fracture morphology
 fracture line extends
 rostrally/caudally into adjacent superior/inferior facets
 ventrally into foramen transversarium, transverse process and pedicle
 dorsally into lamina

- 88 -
By Dr, AbdulRahman AbdulNasser Spine Trauma | SubAxial Cervical Trauma
o findings
 translationof fractured/adjacent vertebrae in sagittal and coronal planes
 uncovertebral joint subluxation
 degree of vertebral body destruction
 MRI
o findings
 disruption of ligaments
 50-75% rupture of anterior longitudinal ligament (ALL)
 30-35% disruption of posterior longitudinal ligament (PLL)
 10-75% disruption interspinous and supraspinous ligaments (ISL and SSL)
 disruption of intervertebral disc
 bone bruising

Treatment
 Nonoperative
o NSAIDS, rest, immobilization
 indications
 stable injuries without neurological deficit
 hyperextension/rotation is poorly immobilized in a halo
 techniques
 Miami J collar
 halo vest
 outcomes
 long term results of non-operative treatment are less desirable
 may be successful in the absence of instability
 surveillance is necessary to detect late instability and persistent pain
 spontaneous fusion rate is only 20%
 Operative
o posterior decompression and two-level instrumented fusion
 indications
 most cases require surgery
 main injured structures are posterior, thus preferred approach is posterior
 also indicated for nonoperatively managed cases with late instability and persistent pain
 techniques
 two-level lateral mass or pedicle screw and rod fixation
 lateral mass plating
 outcomes
 risk of anterior disc space collapse and late kyphotic deformity
 midline fusion does not control rotation
o anterior plating and interbody fusion
 indications
 controls anterior collapse and rotation
 techniques
 using iliac crest bone graft
o single posterior pedicle screw
 indications
 Type A Separation fracture without instability
- 89 -
OrthoBullets2017 Spine Trauma | SubAxial Cervical Trauma
o anterior and posterior decompression and fusion
 indications
 if additional anterior column support is needed
 if anterior approach is attempted initially, with unsuccessful reduction because of
complicated fracture morphology or late presentation

Lateral mass plating


Complications
 Vertebral artery injury : from pedicle screw placement
 Late kyphotic deformity
 Late instability (anterior translation)
 Chronic neck pain and radiculopathy

3. Subaxial Cervical Vertebral Body Fractures


Introduction
 Fracture patterns vary by mechanism and include
o compression fracture
 characterized by
 compressive failure of anterior vertebral body without disruption of posterior body cortex
and without retropulsion into canal
 often associated with posterior ligamentous injury
o burst fracture
 characterized by
 fracture extension through posterior cortex with retropulsion into the spinal canal
 often associated with posterior ligamentous injury
 prognosis : often associated with complete and incompete spinal cord injury
 treatment : unstable and usually requires surgery
o flexion teardrop fracture
 characterized by
 anterior column failure in flexion/compression
 posterior portion of vertebra retropulsed posteriorly
 posterior column failure in tension
 larger anterior lip fragments may be called 'quadrangular fractures'

- 90 -
By Dr, AbdulRahman AbdulNasser Spine Trauma | SubAxial Cervical Trauma
 prognosis
 associated with SCI
 treatment
 unstable and usually requires surgery
o extension teardrop avulsion fracture
 characterized by
 small fleck of bone is avulsed of anterior endplate
 usually occur at C2
 must differentiate from a true teardrop fracture
 mechanism
 extension
 prognosis
 stable injury pattern and not associated with SCI
 treatment
 cervical collar

extension teardrop avulsion


burst fracture flexion teardrop fracture fracture
Subaxial Spine Injury Classification
 Allen and Ferguson classification (of subaxial spine injuries)
o typically used for research and not in clinical setting
o based solely on static radiographs appearance and mechanisms of injury
o six groups represent a spectrum of anatomic disruption and include
1. flexion-compression
2. vertical compression
3. flexion-distraction
4. extension-compression
5. extension-distraction
6. lateral flexion
 Radiographic description classification (of subaxial spine injuries)
o more commonly used in clinical setting
o includes
 compression fracture
 burst fraction
 flexion-distraction injury
 facet dislocation (unilateral or bilateral)
 facet fracture

- 91 -
OrthoBullets2017 Spine Trauma | SubAxial Cervical Trauma
Presentation
 Symtoms : incomplete vs. complete cord injury

Imaging
 Must determine if there is a posterior ligamentous injury so MRI often important

Treatment
 Nonoperative
o collar immobilization for 6 to 12 weeks
 indications
 stable mild compression fractures (intact posterior ligaments & no significant kyphosis)
 anterior teardrop avulsion fracture
o external halo immobilization
 indications
 only if stable fracture pattern (intact posterior ligaments & no significant kyphosis)
 Operative
o anterior decompression, corpectomy, strut graft, & fusion with instrumentation
 indications
 compression fracture with 11 degrees of angulation or 25% loss of vertebral body height
 unstable burst fracture with cord compression
 unstable tear-drop fracture with cord compression
 minimal injury to posterior elements
o posterior decompression, & fusion with instrumentation
 indications
 significant injury to posterior elements
 anterior decompression not required

4. Clay-shoveler Fracture (Cervical Spinous Process Fracture)


Introduction
 Avulsion-type spinous process fracture in the lower cervical or upper thoracic spine
 Epidemiology
o incidence : rare
o demographics
 direct trauma to posterior spinous process
 indirect trauma : sudden muscular/ligamentous pull in flexion or extension
o body location
 most commonly C7, but can affect C6 to T3
 usually occurs midway between the spinous tip and lamina
o risk factors
 labourers
 racket or contact sports
 motor vehicle accidents
 Associated conditions
o usually occurs in isolation
 other orthopaedic injuries to consider
 lamina fracture , facet dislocations

- 92 -
By Dr, AbdulRahman AbdulNasser Spine Trauma | SubAxial Cervical Trauma
 Prognosis
o stable injury in isolation
o very rarely assoicated with neurological injury
o high union rate

Presentation
 Symptoms
o sudden onset of pain between the shoulder blades or base of neck
o reduced head/neck ROM
 Physical exam
o inspection
 localized swelling and tenderness
 crepitus
o motion
 document flexion-extension of cervical spine
o neurovascular examination

Imaging
 Radiographs
o recommended views
 cervical +/- throacic xrays that should always be obtained on evaluation
o alternative views : flexion and extension views
o findings
 lateral view
 fracture line is usually obliquely oriented with the fragment displaced posteroinferior
 AP view
 double spinous process shadow is suggestive of displaced fracture
 CT
o indications
 method of choice
 routine CT imaging in high-energy trauma patients
 clinical criteria
 altered consciousness
 midline spinal pain or tenderness
 impaired CCJ motion
 lower cranial nerve paresis
 motor paresis
o views : fracture is best seen on lateral view
 MRI
o indications : not required in isolation

- 93 -
OrthoBullets2017 Spine Trauma | Cervical Trauma Procedures

Lateral view AP view- double spinous process CT


shadow
Treatment
 Nonoperative
o NSAIDS, rest, immobilization in hard collar for comfort
 indications : most common treatment for pain control
 modalities
 short term treatment with hard collar
 outcomes : usually high union rates and excellent clincal outcomes
 Operative
o surgical excision
 indications
 persistent pain or non-union
 failed conservative treatment

Complications
 Chronic pain
 Neck stiffness

D. Cervical Trauma Procedures

1. Closed Cervical Traction


Indications
 Indications
o subaxial cervical fractures with malalignment
o unilateral and bilateral facet dislocations
o displaced odontoid fractures
o select hangman's fractures
o C1-2 rotatory subluxation
 Contraindications
o patient who is not awake, alert, and cooperative
o presence of a skull fracture may be a contraindication

- 94 -
By Dr, AbdulRahman AbdulNasser Spine Trauma | Cervical Trauma Procedures
Patient position
 Preferred setting
o emergency room, operating room, ICU for close observation and frequent
fluoroscopy/radiographs
 Patient position
o supine with reverse trendelenburg or use of arm and leg weights can help prevent patient
migration to the top of the bed with addition of weights.
 Sedation
o small doses of diazepam can be administered to aid in muscle relaxation
o however patient must remain awake and able to converse

Pin Placement
 Pin placement (Gardner-Wells pins)
o pin placement is 1 cm above pinna, in line with external auditory meatus and below the equator
of the skull.
 if the pin is placed too anterior, the temporalis muscles and superficial temporal artery and
vein are at risk
 an anterior pin will apply an extension moment to the cervical spine
 if the pin is placed too posterior, it can apply a flexion moment to the cervical spine.
 a posterior pin with a flexion moment may facilitate reduction of a facet dislocation.
 Pin tightness
o On Gardner-Wells tongs, pins are tightened until spring loaded indicator protrudes 1 mm above
surface
 this is the equivalent of 139 newtons (31 lbs) of force
 overtightening by 0.3 mm leads to 448 newtons (100 lbs)
 failure of temporal bone occurs at 965 +/- 200 newtons (216 lbs)
 note Mayfield pins are tightened to 60 lbs
o overtightening of the pins can result in penetration of the
inner table of the calvarium
 this may cause cerebral hemorrhage or abscess
 Pin strength
o stainless steel pins have higher failure loads than titanium
and MRI-compatible graphite and should be used with
traction of > 50lbs.
Reduction with Serial Traction
 Serial traction
o an initial 10lbs is added.
o weights are increased by 10lb increments every 20 minutes
o serial exams and radiographs are taken after each weight is
placed
o maximal weight is controversial
 some authors recommend weight limits of 70 lbs
 recent studies report that up to 140 lbs is safe
 Reduction maneuvers
o reduction of a unilateral facet dislocation
 reduction maneuver performed after facet is distracted to a perched position
- 95 -
OrthoBullets2017 Spine Trauma | Cervical Trauma Procedures
 maintain axial load and rotate head 30-40 degrees past midline, in the direction of the
dislocation
 stop once resistance is felt, and confirm with radiographs
o reduction of bilateral facet dislocation
 reduction maneuver performed after facet is distracted to a perched position
 palpate the stepoff in the spinal process posteriorly and apply an anterior directed force
caudal to the level of the dislocation
 rotate the head 40 degrees beyond midline in one direction, and then rotate 40 degrees in the
other direction while axial traction is maintained.
Complications
 Failure to reduce
o a bilateral, irreducible facet dislocation is unstable and should be treatment with urgent open
reduction after an MRI is performed..
 Change in neurologic exam
o with any change in the neurologic exam the weights should be removed and an MRI should be
obtained.

2. Halo Orthosis Immobilization


Introduction
 Fixes skull relative to torso
o provides most rigid form of cervical spine external immobilization
o ideal for upper C-spine injury
 Allows intercalated paradoxical motion in the subaxial cervical spine
o therefore not ideal for lower cervical spine injuries (lateral bending least controlled)
 "snaking phenomenon"
 recumbent lateral radiograph shows focal kyphosis in midcervical spine
 yet, upright lateral radiograph shows maintained lordosis in midcervical spine

Indications
 Adult
o definitive treatment of cervical spine trauma including
 occipital condyle fx
 occipitocervical dislocation
 stable Type II atlas fx (stable Jefferson fx)
 type II odontoid fractures in young patients
 type II and IIA hangman’s fractures
o adjunctive postoperative stabilization following cervical spine surgery
 Pediatric
o definitive treatment for
 atlanto-occipital dissociation
 Jefferson fractures (burst fracture of C1)
 atlas fractures
 unstable odontoid fractures
 persistent atlanto-axial rotatory subluxation
 C1-C2 dissociations

- 96 -
By Dr, AbdulRahman AbdulNasser Spine Trauma | Cervical Trauma Procedures
 subaxial cervical spine trauma
o preoperative reduction in the patients with spinal deformity
Contraindications
 Absolute
o cranial fractures
o infection
o severe soft-tissue injury
 especially near proposed pin sites
 Relative
o polytrauma
o severe chest trauma
o barrel-shaped chest
o obesity
o advanced age
 recent evidence demonstrates an unacceptably high mortality rate in patients aged 79
years and older (21%)
Imaging
 CT scan prior to halo application
o indications
 clinical suspicion for cranial fracture
 children younger than 10 to determine thickness of bone

Adult Technique
 Adults
o torque
 tighten to 8 inch-pounds of torque
o location
 total of 4 pins
 2 anterior pins
 safe zone is a 1 cm region just above the lateral one third of the orbit (eyebrow) at or
below the equator of the skull
 this is anterior and medial to temporalis fossa/temporalis muscle
 this is lateral to supraorbital nerve
 2 posterior pins
 placed on opposite side of ring from anterior pins
o followup care
 can have patient return on day 2 to tighten again
 proper pin and halo care can be done to minimize chance of infection

- 97 -
OrthoBullets2017 Spine Trauma | Cervical Trauma Procedures

Pediatric Technique
 Pediatrics
o torque
 best construct involves more pins with less torque
 total of 6-8 pins

 lower torque (2-4 in-lbs or "finger-tight")


o pin locations
 place anterior pins lateral enough to avoid injury to the frontal sinus, supratrochlear and
supraorbital nerves
 place pins anterior enough to avoid the temporalis muscle
 place pins posteriorly opposite from anterior pins
o brace/vest
 custom fitted vest for children > 2 years
 children <2 yrs should use Minerva cast
o CT scans may help in pin placement
 can help facilitate avoiding cranial sutures
 can help facilitate avoiding thin regions of skull
 help limit risk of complications

Complications
 Higher complications in children (70%) than adults (35%)
 Loosening (36%)
o can be treated with retightening
o if continues to loosen, should be treated with pin exchange

- 98 -
By Dr, AbdulRahman AbdulNasser Spine Trauma | Cervical Trauma Procedures
 Infection (20%)
o can especially occur with posterior pin in temporalis fossa because
 pins hidden in hairline
 bone is thin
 temporalis muscle moves with chewing

o can be treated with oral antibiotics if pin not loose


 if pin infection and loose then pin should be removed
 Discomfort (18%)
o treated by loosening skin around pin
 Dural puncture (1%)
 Abducens nerve palsy
o epidemiology
 is most commonly injured cranial nerve with halo
o pathophysiology
 thought to be a traction injury to cranial nerve 6, which affects abducens nerve (innervate
lateral rectus muscles)
o symptoms
 diplopia
o physical exam
 loss of lateral gaze on affected side
o treatment
 observation as most resolve spontaneously
 Supraorbital nerve palsy
o injured by medially placed anterior pins
 Supratrochlear nerve palsy
o injured by medially placed anterior pins
 Medical complications
o pneumonia
o ARDS
o Arrhythmia

Collected By : Dr AbdulRahman
AbdulNasser
drxabdulrahman@gmail.com
In June 2017

- 99 -
OrthoBullets2017 Spine Trauma | Thoracolumbar Trauma

E. Thoracolumbar Trauma

1. Thoracic & Lumbar Trauma Introduction


Introduction
 Potentially devastating due to
o narrow canal
o precarious blood supply
 Anatomic regions of injury
o thoracic spine (T2-T10)
 fractures from T2-T10 are rare due to increased stability of thoracic spine (see anatomy
below)
 fractures include
 burst fracture
 osteoporotic compression fracture
 fracture dislocation (rare but leads to paralysis in 80%)
o thoracolumbar region (T11 to L2)
 more commonly affected by spine trauma due to fulcrum of motion (intersection between
stiff thoracic spine and increased motion of lumbar spine)
 more than 50% of all thoracic and lumbar fractures occur in this region
 common fracture patterns include
 thoracolumbar burst fracture
 Chance fracture

Anatomy
 Biomechanics
o thoracic spine from T2 to T10 has increased stiffness due to
 increased rigidity by articulation with ribs
 ribs articulate with sternum, adding secondary stability
 facet joints oriented in coronal plane
 disks are thin increasing stiffness and rotational stability
 kyphosis concentrates axial load on anterior column
o definitions of spinal stability
 Blood supply
o "watershed area" in middle thoracic spine
 is a vascular watershed area
 vascular injury can lead to cord ischemia
 Spinal cord
o spinal cord ends and cauda equina begins at level of L1/L2
 variable so valuable to identify beginning of cauda equina on MRI in relation to pathology
 injuries below L1 have a better prognosis because the nerve roots (cauda equina and nerve
roots within thecal sac) are affected as opposed to the spinal cord
Classification
 Magerl classification (of thoracic spine injuries)
o Type A : compression caused by axial loading

- 100 -
By Dr, AbdulRahman AbdulNasser Spine Trauma | Thoracolumbar Trauma
o Type B
 B1: ligamentous distraction injury posterior
 B2: osseoligamentous distraction injury posterior
o Type C
 multidirectional injuries, often fracture dislocations
 very unstable with high likelihood of neurologic injury
 AO classification (of thoracolumbar spinal fracture)
o Type A: Compression injuries
o Type B: Distraction injuries
o Type C: Torsional injury
 each type then broken down further into
 fracture morphology
 bony versus ligamentous failure
 direction of displacement

Imaging
 Radiographs
o obtain radiographs of entire spine (concomitant spine fractures in 20%)
 CT scan indications
o fracture on plain film
o neurologic deficit in lower extremity
o inadequate plain films
 MRI useful to evaluate for
o injury to anterior and posterior ligament complex
o spinal cord compression by disk or osseous material
o cord edema or hemorrhage

Treatment
 Treatment varies by condition, but the following should be considered
o degree of neurologic deficits seen on physical exam
o degree of spinal cord compression and imaging evidence of myelomalacia
o spinal stability
 Nonoperative
o indications
 most thoracic and thoracolumbar fractures (burst and compression) can be treated
nonoperatively when the patient is neurologically intact
 treat in orthosis for 6 to 12 weeks depending on degree of instability
 Operative
o indications for surgery
 progressive neurologic deficits
 myelomalacia seen on MRI
 gross spinal instability
 posterior osseoligamentous stability compromised

Surgical Techniques
 Approaches
o surgical approach is dictated by

- 101 -
OrthoBullets2017 Spine Trauma | Thoracolumbar Trauma
 site of compression (anterior or posterior)
 unlike thecal sack, the spinal cord can not be manipulated or medialized
 surgical window needed to restore spinal stability
 often times anterior column needs to be reconstructed
o thoracic approaches used include
 midline posterior approach
 indicated only when spinal cord compression is posterior
 costotransverse
 can be open or thoracosopic
 transthoracic

2. Thoracolumbar Burst Fractures


Introduction
 Defined as vertebral fracture with compromise of the anterior and middle column
o can be unstable because both anterior and middle columns are involved
 Mechanism
o axial loading with flexion
o at thoracolumbar junction there is fulcrum of increased motion that makes spine more vulnerable
to traumatic injury
 Neurologic deficits
o canal compromise often caused by retropulsion of bone
o maximum canal occlusion and neural compression at moment of impact
o retropulsed fragments resorb over time and usually do not cause progressive neurologic
deterioration
 Associated injuries
o concomitant spine fractures in 20%
o lamina fracture is associated with dural tear and entrapped nerve roots

Anatomy
 Denis three column system
o clinical relevance
 only moderately reliable in determining clinical degree of stability
o definitions
 anterior column
 anterior longitudinal ligament (ALL)
 anterior 2/3 of vertebral body and annulus
 middle column
 posterior longitudinal ligament (PLL)
 posterior 1/3 of vertebral body and annulus
 posterior column
 pedicles
 lamina
 facets
 ligamentum flavum
 spinous process
 posterior ligament complex (PLC)

- 102 -
By Dr, AbdulRahman AbdulNasser Spine Trauma | Thoracolumbar Trauma
 instability defined by
 injury to middle column
 as evidenced by widening of interpedicular distance on AP radiograph
 loss of height of posterior cortex of vertebral body
 disruption of posterior ligament complex combined with anterior and middle column
involvement
 Posterior Ligamentous Complex
o considered to be a critical predictor of spinal fracture stability
o consists of
 supraspinous ligament
 interspinous ligament
 ligamentum flavum
 facet capsule
o evaluation
 determining the integrity of the PLC can be challenging
 conditions where PLC is clearly ruptured
 bony chance fracture
 widening of interspinous distance
 progressive kyphosis with nonoperative treatment
 facet diastasis
 conditions where integrity of PLC is indeterminant
 MRI shows signal intensity between spinous process

Classification ‎II:19 Thoracolumbar Injury Classification and


Severity Score
 Dennis classification
o Type A: Fracture of both end-plates. The bone is retropulsed into the canal.
o Type B: Fracture of the superior end-plate. It is common and occurs due to a combination of
axial load with flexion.
o Type C: Fracture of the inferior end-plate.
o Type D: Burst rotation. This fracture could be misdiagnosed as a fracture-dislocation. The he
mechanism of this injury is a combination of axial load and rotation.
o Type E: Burst lateral flexion. This type of fracture differs from the lateral compression fracture
in that it presents an increase of the interpediculate distance on anteroposterior roentgenogram
 Thoracolumbar Injury Classification and Severity Score
o injury characteristic qualifier points
 injury morphology
 compression (1 point)
 burst (+1 point)
 rotation/translation (+3 point)
 distraction (+4 point)
 neurologic status
 intact (+0 point)
 nerve root (+2 point)
 incomplete Spinal cord or conus medullaris injury (+3 point)
 complete Spinal cord or conus medullaris injury (+2 point)
 cauda equina syndrome (+3 point)
 posterior ligamentous complex integrity
- 103 -
OrthoBullets2017 Spine Trauma | Thoracolumbar Trauma
 intact(+0 point)
 no interspinous ligament widening seen with flexion views. MRI shows no edema in
interspinous ligament region
 suspected/indeterminate (+2 point)
 MRI shows some signal in region of interspinous ligaments
 disrupted (+3 point)
 widening of interspinous distance seen
o TLICS treatment implications
 score < 4 points
 Nonsurgical management
 score = 4 points
 Nonsurgical or surgical managment
 score > 4 points
 surgical management indicated

Imaging
 Radiographs
o recommended views
 obtain radiographs of entire spine (concomitant spine fractures in 20%)

o AP shows
 widening of pedicles
 coronal deformity
o lateral shows
 retropulsion of bone into canal
 kyphotic deformity
 CT scan
o indications
 fracture on plain film
 neurologic deficit in lower extremity
 inadequate plain films
 MRI
o useful to evaluate for
 spinal cord or thecal sac compression by disk or osseous material
 cord edema or hemorrhage
 injury posterior ligament complex
 signal intensity in PLC is concerning for instability and may warrant surgical intervention

Treatment
 Nonoperative
o ambulation as tolerated with or without a thoracolumbosacral orthosis
 indications
 patients that are neurologically intact and mechanically stable
 posterior ligament complex preserved
 kyphosis < 30° (controversial)

- 104 -
By Dr, AbdulRahman AbdulNasser Spine Trauma | Thoracolumbar Trauma
 vertebral body has lost < 50% of body height (controversial)
 TLICS score = 3 or lower
 thoracolumbar orthosis
 recent evidence shows no clear advantage of TLSO on outcomes
 if it provides symptomatic relief, may be beneficial for patient
 outcomes
 retropulsed fragments resorb over time and usually do not cause neurologic deterioration
 Operative
o surgical decompression & spinal stabilization
 indications
 neurologic deficits with radiographic evidence of cord/thecal sac compression
 both complete and incomplete spinal cord injuries require decompression and
stabilization to facilitate rehabilitation
 TLICS score = 5 or higher
 unstable fracture pattern as defined by
 injury to the Posterior Ligament Complex (PLC)
 progressive kyphosis
 > 30°kyphosis (controversial)
 > 50% loss of vertebral body height (controversial)
 > 50% canal compromise (controversial)

Techniques
 Anterior decompression and stabilization (with or without posterior stabilization)
o indications
 indicated when neurologic deficits caused by anterior compression (bony retropulsion)
 scientific data has not shown a benefit to early decompression and stabilization
o technique
 usually includes corpectomy and strut grafting followed by anterior +/- posterior
instrumentation
 advantage is that you do not need to do a laminectomy which will further destabilize the
spine by compromising the posterior supporting structures
 Posterior Decompression and Fusion
o indications
 unstable fracture pattern with no need for neurologic decompression
 complete neurologic injury (allows earlier rehab)
o neural decompression
 direct decompression
 retropulsed bone can be removed via transpedicular approach
 indirect decompression
 via ligamentotaxis may occur by restoring height and sagittal alignment with posterior
instrumentation
o arthrodesis
 fusion should be performed with instrumentation
 instrumentation should be under distraction to restore vertebral body height and achieve
indirect decompression
 historically it was recommended to instrument three levels above and two levels below
 modern pedicle screws have changes this to one level above and one level below
- 105 -
OrthoBullets2017 Spine Trauma | Thoracolumbar Trauma
 avoid laminectomy if possible as it will further destabilize the spine by compromising the
posterior supporting structures
 Posterior Fusion Alone (no decompression)
o indications
 progression kyphosis or clear injury to posterior ligament complex, but with no significant
neurologic compression
Complications
 Entrapped nerve roots and dural tear
o from associated lamina fractures
 Pain
o most common
 Progressive kyphosis
o common with unrecognized injury to PLL
 Flat back
o leads to pain, a forward flexed posture, and easy fatigue
o post-traumatic syringomyelia

3. Chance Fracture (flexion-distraction injury)


Introduction
 Mechanism
o a flexion-distraction injury (seatbelt injury)
 may be a bony injury
 may be ligamentous injury (flexion-distraction injury)
 more difficult to heal

 middle and posterior columns fail under tension


 anterior column fails under compression

 Associated injuries
o high rate of gastrointestinal injuries (50%)
Imaging
 Radiographs

o obtain AP and lateral


o flexion-extension radiographs

- 106 -
By Dr, AbdulRahman AbdulNasser Spine Trauma | Thoracolumbar Trauma
 MRI
o important to evaluate for injury to the posterior elements
 CT
o important to evaluate degree of bone injury and retropulsion of posterior wall into canal
Treatment
 Nonoperative
o immobilization in cast or TLSO
 indications
 neurologically intact patients with
 stable injury patterns with intact posterior elements
 bony Chance fracture
 technique
 may cast or brace (TLSO) in extension
 must be followed for non-union and kyphotic deformity
 Operative
o surgical decompression and stabilization
 indications
 patients with neurologic deficits
 unstable spine with injury to the posterior ligaments (soft-tissue Chance fx)
 techniques
 anterior decompression and stabilization
 usually with vertebrectomy and strut grafting followed by instrumentation
 posterior indirect decompression and stabilization and compression fusion construct
 historically three levels above and two levels below
 modern pedicle screws have changed this to one level above and one level below
 distraction construct in burst fractures
 compression construct in Chance fractures

Complications
 Pain
o most common
 Deformity
o scoliosis
o progressive kyphosis
 common with unrecognized injury to PLL
o flat back
 leads to pain, a forward flexed posture, and easy fatigue
o post-traumatic syringomyelia
 Nonunion

‎II:20 Xray-Lumbar-Lat: Chance fracture in 28-year-old


woman. Cross-table lateral radiograph of lumbar spine
shows fanning of spinous processes (double-headed arrow)
and fracture extending through pedicle (between
arrowheads) and into L2 vertebral body (single

- 107 -
OrthoBullets2017 Spine Trauma | Thoracolumbar Trauma

4. Thoracolumbar Fracture-Dislocation
Introduction
 Fractures associated with posterior facet dislocation occuring at the thoracolumbar junction (AO type
C)
 Epidemiology
o incidence
 approx. 4% of spinal cord injuries admitted to Level 1 trauma centres
 50-60% of fracture-dislocations are associated with spinal cord injuries
o demographics
 4:1 male-to-female ratio
o location
 most commonly occur at the thoracolumbar junction
o risk factors
 high energy injuries
 motor vehicle accident (most common)
 falls
 sports
 violence
 Pathophysiology
o mechanism of injury
 acceleration/deceleration injuries
 resultng in hyperflexion, rotation and shearing of the spinal column
o associated injury
 neurologic deficits
 head injury
 concomitant injuries in thorax and abdomen
 Classification Systems
o Thoracolumbar Injury Classification System (TLICS)
 categorizes injuries based on
 morphology of injury
 neurologic injury
 posterior ligamentous complex integrity
 treatment recommendation based on total score
 nonsurgical = 3 or lower
 indeterminate = 4
 surgical = 5 or higher

Anatomy
 Lumbothoracic junction
o Definition
 T10 - L2
 transition zone between thoracic spine (kyphosis) and lumbar spine (lordosis)
o Pathoanatomy
 greater mobility in the lumbar spine compared to thoracic spine
 results in an area of the spine that is vulnerable to shearing forces

- 108 -
By Dr, AbdulRahman AbdulNasser Spine Trauma | Thoracolumbar Trauma
 high risk of injury to the spinal cord, conus or cauda equina depending on the patients
anatomy and degree of dislocation
Presentation
 Pre-hospital
o patients almost exclusively present as a major trauma with or without neurological deficit
o transportation to a trauma center using spine immobilization precautions with a spinal board and
cervical collar.
 Clinical Approach
o ATLS
 Airway, Breathing, Circulation
 Neurological assessment
 Inspection
 open injury
 deformity (e.g. kyphosis)
 Palpation
 point tenderness
 step-off deformity
 crepitus
 Neurological Impairment
 GCS
 ASIA Impairment score
 sensory, motor, or reflexes impairment
 rectal examination
 History
 Physical examination

Imaging
 Radiographs
o recommended views
 AP and lateral view of thoraco-lumbar spine
o indications
 suspected spinal column injury with bone tenderness
 recognize stable versus unstable spine injuries
o findings
 fracture type, pattern and dislocation
 CT scan
o indications
 better visualization of fracture pattern and type compared to plain radiographs (e.g. unilateral
facet dislocations, etc)
 blunt trauma patients requiring a CT scan to screen for other injuries
o findings
 cannot adequately visualize and describe the spinal canal and other associated ligaments
 MRI
o indications
 better visualisation of the spinal cord and supporting ligamentous structures
 level of neurological deficit does not align with apparent level of spinal injury

- 109 -
OrthoBullets2017 Spine Trauma | Thoracolumbar Trauma
o findings
 important to evaluate for injury to the posterior longitudinal ligament
Treatment
 Operative
o posterior open reduction with instrumented fusion
 indications
 most patients with thoracolumbar fracture dislocation
 unstable fracture patterns
 disrupted supporting ligamentous structures
 technique
 midline incision
 identify fracture-dislocation site
 use pedicle screws for distraction to obtain anatomical reduction
 insert posterior instrumentation two levels above and two levels below the site of injury
 outcomes
 early decompression and instrumentation has been shown to have better outcomes than
delayed surgery or non-operative treatment
 obtain postoperative CT/MRI to see if their is any residual anterior compression

Complications
 Neurological injury
 Cauda equina syndrome
 DVT
 Non-union after spinal fusion
 Post-traumatic pain
o most commoncomplication
o greater with increased kyphotic deformity
 Deformity
o scoliosis
o progressive kyphosis
 common with unrecognized injury to PLL
o flat back
 leads to pain, a forward flexed posture, and easy fatigue
o post-traumatic syringomyelia

5. Osteoporotic Vertebral Compression Fracture


Introduction
 A fragility fracture of the spine
 Epidemiology
o incidence
 vertebral compression fractures (VCF) are the most common fragility fracture
 700,000 VCF per year in US
 70,000 hospitalizations annually
 15 billion in annual costs
o demographics

- 110 -
By Dr, AbdulRahman AbdulNasser Spine Trauma | Thoracolumbar Trauma
 affects up to
 25% people over 70 years
 50% people over 80 years
o risk factors
 history of 2 VCFs
 is the strongest predictor of future vertebral fractures in postmenopausal women
 Pathoanatomy
o osteoporosis
 characteristics
 bone is normal quality but decreased in quantity
 cortices are thinned
 cancellous bone has decreased trabecular continuity
 bone mineral density in the lumbar spine (BMD)
 peaks at
 between 33 to 40 yrs in women
 between 19 to 33 years in men
 peak BMD is widely variable based on demographic factors and location in
body
 decreases with age following peak mass
 correlate well with bone strength and is a good predictor of fragility fracture
 definition
 WHO defines osteoporosis as T score below -2.5
 Associated conditions
o compromised pulmonary function
 increased kyphosis can affect pulmonary function
 each VCF leads up to 9% reduction in FV
 Prognosis
o mortality
 1-year mortality ~ 15% (less than hip fx)
 2-year mortality ~20% (equivalent to hip fx)

Presentation
 Symptoms
o pain
 25% of VCF are painful enough that patients seek medical attention
 pain usually localized to area of fracture
 but may wrap around rib cage if dermatomal distribution
 Physical exam
o focal tenderness
 pain with deep palpation of spinous process
o local kyphosis
 multiple compression fractures can lead to local kyphosis
o spinal cord injury
 signs of spinal cord compression are very rare
o nerve root deficits
 may see nerve root deficits with compression fractures of lumbar spine that lead to severe
foraminal stenosis
- 111 -
OrthoBullets2017 Spine Trauma | Thoracolumbar Trauma
Imaging
 Radiographs
o obtain radiographs of the entire spine (concomitant spine fractures in 20%)
o will see loss of anterior, middle, or posterior vertebral height by 20% or at least 4mm
 CT scan
o usually not necessary for diagnosis
o indications
 fracture on plain film
 neurologic deficit in lower extremity
 inadequate plain films
 MRI
o usually not necessary for diagnosis
o useful to evaluate for
 acute vs chronic nature of compression fracture
 injury to anterior and posterior ligament complex
 spinal cord compression by disk or osseous material
 cord edema or hemorrhage

Studies
 Laboratory
o a full medical workup should be performed with CBC, BMP
o ESR may help to rule out infection
o Urine and serum protein electrophoresis may help rule out multiple myeloma

Differential Diagnosis
 Metastatic cancer to the spine
o must be considered and ruled out
o the following variables should raise suspicion
 fractures above T5
 atypical radiographic findings
 failure to thrive and constitutional symptoms
 younger patient with no history of fall

Treatment
 Nonoperative
o observation, bracing, and medical management
 indications
 majority of patients can be treated with observation and gradual return to activity
 PLL intact (even if > 30 degrees kyphosis or > 50% loss of vertebral body height)
 technique
 if the fracture is less than five days old
 calcitonin can be used for four weeks to decrease pain
 medical management can consist of bisphosphonates
 to prevent future risk of fragility fractures
 some patients may benefit from an extension orthosis
 although compliance can be an issue

- 112 -
By Dr, AbdulRahman AbdulNasser Spine Trauma | Thoracolumbar Trauma
 Operative
o vertebroplasty
 indications
 not indicated
 AAOS recommends strongly against the use of vertebroplasty
 outcomes
 randomized, double-blind, placebo-controlled trials have shown no beneficial effect of
vertebroplasty
 vertebroplasty has higher rates of cement extravasation and associated complications than
kyphoplasty
o kyphoplasty
 indications
 patient continues to have severe pain symptoms after 6 weeks of nonoperative treatment
 AAOS recommend may be used, but recommendation strength is limited
 technique
 kyphoplasty is different than vertebroplasty in that a cavity is created by balloon
expansion and therefore the cement can be injected with less pressure
 pain relief thought to be from elimination of micromotion
o surgical decompression and stabilization
 indications
 very rare in standard VCF
 progressive neurologic deficit
 PLL injury and unstable spines
 technique
 to prevent possible failure due to osteoporotic bone I‎I:21 kyphoplasty
 consider long constructs with multiple fixation points
 consider combined anterior fixation

Techniques
 Kyphoplasty vs. vertebroplasty
o performed under fluoroscopic guidance
o percutaneous transpedicular approach used for cannula
o vertebroplasty
 PMMA injected directly into cancellous bone without cavity creation
 performed when cement is more liquid
 requires greater pressure because no cavity is created
 increased risk of extravasation into spinal canal is greater
o kyphoplasty
 cavity created with expansion device (e.g., balloon) prior to PMMA injection
 performed when cement is more viscous
 may be possible to obtain partial reduction of fracture with balloon expansion

Complications
 Neurological injury
o can be caused by extravasation of PMMA into spinal canal
 higher risk with vertebroplasty than kyphoplasty
 important to consider defects in the posterior cortex of the vertebral body

- 113 -
OrthoBullets2017 Spine Trauma | Thoracolumbar Trauma

6. Cauda Equina Syndrome


Introduction
 Cauda equina syndrome is defined by a constellation of symptoms that result from terminal spinal
nerve root compression in the lumbosacral region
o considered one of the few true medical emergencies in orthopaedics
o key features
 bilateral leg pain
 bowel and bladder dysfunction
 saddle anesthesia
 lower extremity sensorimotor changes
 Epidemiology
o 1-6% of lumbar disc herniations
 Pathophysiology
o space-occupying lesion within lumbosacral canal, including
 disc herniation (most common)
 spinal stenosis
 tumors
 trauma (retropulsion of fracture fragment, dislocation or collapse)
 spinal epidural hematoma
 epidural abscess
 Associated conditions
o deep vein thrombosis (DVT)
 is a well known complication after spinal trauma or spine surgery

Relevant Anatomy
 Spinal cord
o conus medullaris
 tapered, terminal end of the spinal cord
 terminates at T12 or L1 vertebral body
o filum terminale
 non-neural, fibrous extension of the conus medullaris that attaches to the coccyx
o cauda equina (horse's tail)
 collection of L1-S5 peripheral nerves within the lumbar canal
 compression considered to cause lower motor neuron lesions
 Bladder
o receives innervation from
 parasympathetic nervous system (pelvic splanchnic nerves and the inferior hypogastric
plexus) and
 sympathetic plexus (hypogastric plexus)
o external sphincter of the bladder is controlled by the pudendal nerve
o lower motor neuron lesions of cauda equina will interrupt the nerves forming the bladder reflex
arcs

- 114 -
By Dr, AbdulRahman AbdulNasser Spine Trauma | Thoracolumbar Trauma

Presentation
 History
o two distinct clinical presentations: acute (e.g. disc herniation, trauma) and insidious (e.g. spinal
stenosis, tumor)
 Symptoms
o bilateral leg pain
o saddle anesthesia
o impotence
o sensorimotor loss in lower extremity
o neurogenic bladder dysfunction
 disruption of bladder contraction and sensation leads to urinary retention and eventually to
overflow incontinence
o bowel dysfunction is rare
 Physical exam
o inspection
 lower extremity muscle atrophy with insidious presentations (e.g. spinal stenosis)
 fasciculations are rare
o palpation
 lower back pain/tenderness is not a distinguishing feature
 palpation of the bladder for urinary retention
o neurovascular examination
 bilateral lower extremity weakness and sensory disturbances
 decreased or absent lower extremity reflexes
o rectal/genital examination
 reduced or absent sensation to pinprick in the perianal region (S2-S4 dermatomes), perineum,
and posterior thigh
 decreased rectal tone or voluntary contracture
 diminished or absent anal wink test and a
bulbocavernosus reflex
Imaging
 MRI
o study of choice to evaluate neurologic compression
 CT myelography
o study of choice if patient unable to undergo MRI

- 115 -
OrthoBullets2017 Spine Trauma | Thoracolumbar Trauma
Treatment
 Operative
o urgent surgical decompression within 48 hours
 indications
 significant suspicion for CES
 severity of symptoms will increase the urgency of surgical decompression
 techniques
 diskectomy
 laminectomy
 outcomes
 studies have shown improved outcomes in bowel and bladder function and resolution of
motor and sensory deficits when decompression performed within 48 hours of the onset
of symptoms
Surgical Techniques
 Surgical decompression of neural elements
o approach
 posterior midline approach to lumbar spine
o diskectomy vs. wide laminectomy and diskectomy
 no comparison studies between microdiskectomy alone and wide decompression combined
with microdiskectomy.
Complications
 Delayed presentation or decompression
o sexual dysfunction
o urinary dysfunction requiring catheterization
o chronic pain
o persistent leg weakness

Collected By : Dr AbdulRahman
AbdulNasser
drxabdulrahman@gmail.com
In June 2017

- 116 -
By Dr, AbdulRahman AbdulNasser Upper Extremity | Thoracolumbar Trauma

ORTHO BULLETS

III.Upper Extremity

- 117 -
OrthoBullets2017 Upper Extremity | Shoulder

A. Shoulder

1. Brachial Plexus Injuries


Introduction
 Traumatic brachial plexus injuries (BPIs) can involve any degree of injury at any level of the plexus
o more severe injury such as rupture of plexal segments or root avulsions are associated with
higher energy trauma
 Brachial Plexus injuries include
o traumatic injury (this topic)
o obstetric brachial plexus injury
 Erb's palsy
 Klumpke palsy
o burners and stingers
o Parsonage-Turner Syndrome
 Epidemiology
o supraclavicular injuries
 complete involvement of all roots is
most common
 75%-80% of traumatic BPIs
 C5 and C6 upper trunk (Erb palsy)
 20%-25% of traumatic BPIs
 C8, T1 or lower (Klumpke palsy)
 0.6%-3.0% of traumatic BPIs
 Mechanism
o high speed vehicular accidents (mostly motorcycle)
 83% of traumatic BPIs
o caudally forced shoulder
 predominantly affect upper brachial plexus
 with high enough energy all roots can be affected
o forced arm abduction (as in grabbing onto something while falling)
 predominantly affects lower roots
 Prognosis
o recovery of reconstructed plexus can take up to 3 years
o nerve regeneration occurs at speed of 1mm/day
o infraclavicular plexus injuries have better prognosis than supraclavicular injuries
o upper plexus injuries have improved prognosis
 preservation of hand function
o root avulsion (preganglionic injuries) have worst prognosis
 not repairable
o other surgeries such as arthrodesis and tendon transfers may be needed

Anatomy
 Anatomy
o brachial plexus motor and sensory innervation

- 118 -
By Dr, AbdulRahman AbdulNasser Upper Extremity | Shoulder

Classification
 Preganglionic vs. postganglionic
o preganglionic
 avulsion proximal to dorsal root ganglion
 involves CNS which does not regenerate – little potential recovery of motor function
(poor prognosis)
 lesions suggesting preganglionic injury:
 Horner’s syndrome
 disruption of sympathetic chain
 winged scapula medially
 loss of serratus anterior (long thoracic nerve) rhomboids (dorsal scapular nerve) leads
to medial winging (inferior border goes medial)
 presents with motor deficits (flail arm)
 sensory deficits
 absence of a Tinel sign or tenderness to percussion in the neck
 normal histamine test (C8-T1 sympathetic ganglion)
 intact triple response (redness, wheal, flare)
 elevated hemidiaphragm (phrenic nerve
 rhomboid paralysis (dorsal scapular nerve)
 supraspinatus/infraspinatus (suprascapular nerve)
 latissimus dorsi (thoracodorsal)
 evaluation
 EMG may show loss of innervation to cervical paraspinals
o postganglionic
 involve PNS, capable of regeneration (better prognosis)
 presentation
 presents with motor deficit (flail arm)
 sensory deficits
 evaluation

- 119 -
OrthoBullets2017 Upper Extremity | Shoulder
 EMG shows maintained innervation to cervical paraspinals
 abnormal histamine test
 only redness and wheal, but NO flare
 Classification based on location

Upper Lesion: Erb's Palsy (C5,6)


Introduction Physical Exam
 Most common obstetric brachial  Clinically, arm will be adducted, internally rotated, at
plexopathy shoulder; pronated, extended at elbow (“waiter’s tip”)
 Results from excessive  C5 deficiency
displacement of head to opposite o axillary nerve deficiency (weakness in deltoid, teres
side and depression of shoulder minor)
on same side producing traction o suprascapular nerve deficiency (weakness in
on plexus supraspinatus, infraspinatus)
 Occurs during difficult delivery o musculocutaneous nerve deficiency (weakness to
in infants or fall onto shoulder in biceps)
adults  C6 deficiency
 Best prognosis o radial nerve deficiency (weakness in
brachioradialis, supinator)

Lower Lesion: Klumpke Palsy (C8,T1)


 Rare in obstetric palsy  Deficit of all of the small muscles of the hand (ulnar and
 Usually avulsion injuries caused median nerves)
by excessive abduction (person  Clinically, presents as “claw hand”
falling from height clutching on o wrist held in extreme extension because of the
object to save himself) unopposed wrist extensors
 Other causes may include o hyperextension of MCP due to loss of hand
cervical rib, or lung mets in intrinsics
lower deep cervical lymph nodes o flexion of IP joints due to loss of hand intrinsics
 Frequently associated with a
preganglion injury and Horner's
Syndrome
 Poor prognosis

Total Palsy (C5-T1)


 A form of brachial plexopathy  Leads to a flaccid arm
 Worst prognosis  Involves both motor and sensory

Presentation
 History
o high energy injury
 Physical exam
o Horner's syndrome
 features include
 drooping of the left eyelid
 pupillary constriction
 anhidrosis
 usually show up three days after injury
 represents disruption of sympathetic chain via C8 and/or T1 root avulsions
- 120 -
By Dr, AbdulRahman AbdulNasser Upper Extremity | Shoulder
o severe pain in anesthetized limb
 correlates with root avulsion
o important muscles to test
 serratus anterior (long thoracic nerve) and rhomboids (dorsal scapular nerve)
 if they are functioning then it is more likely the C5 injury is postganglionic
o pulses
 check radial, ulnar and brachial pulses
 arterial injuries common with complete BPIs

Imaging
 Radiographs
o chest radiograph
 recommended views
 PA and lateral
 fractures to the first or second ribs suggest damage to the overlying brachial plexus
 evidence of old rib fractures can be important in case intercostal nerve is needed for nerve
transfer
 inspiration and expiration can demonstrate a paralyzed diaphragm (indicates upper nerve root
injury)
o cervical spine series
 recommended views
 AP and lateral
 transverse process fracture likely indicates a root avulsion
o scapular and shoulder series
 recommended views
 at least AP and axillary (or equivalent)
 scapulothoracic dissociation is associated with root avulsion and major vascular injury
o clavicle
 recommended views
 orthogonal views
 fracture may indicate brachial plexus injury
 CT myelography
o indications
 gold standard for defining level of nerve root injury
o avulsion of cervical root causes dural sheath to heal with meningocele
o scan should be done 3-4 weeks after injury
 allows blood clot in the injured area to dissipate and meningocele to form
 MRI
o indications
 suspect injury is distal to nerve roots
 can visualize much of the brachial plexus
 CT/myelogram demonstrates only nerve root injury
o findings
 traumatic neuromas and edema
 mass lesions in nontraumatic neuropathy of brachial plexus and its branches
 consistent with injury include
 pseudomeningocele (T2 highlights water content present in a pseudomeningocele )
- 121 -
OrthoBullets2017 Upper Extremity | Shoulder
 empty nerve root sleeves (T1 images highlight fat content nerve roots and empty sleeves)
 cord shift away from midline (T1 highlights fat of cord)
Studies
 Electromyography (EMG)
o tests muscles at rest and during activity
o fibrillation potentials (denervation changes)
 as early as 10-14 days following injury in proximal muscles
 as late as 3-6 weeks in distal muscles
o can help distinguish preganglionic from postganglionic
 examine proximally innervated muscles that are innervated by root level motor branches
 rhomboids
 serratus anterior
 cervical paraspinals
 Nerve conduction velocity (NCV)
o performed along with EMG
o measures sensory nerve action potentials (SNAPs)
 distinguishes preganglionic from postganglionic
 SNAPs preserved in lesions proximal to dorsal root ganglia
 cell body found in dorsal root ganglia
 if SNAP normal and patient insensate in ulnar nerve distribution
 preganglionic injury to C8 and T1
 if SNAP normal and patient insensate in median nerve distribution
 preganglionic injury to C5 and C6
 Nerve action potential (NAPs)
o often intraoperative
o tests a nerve across a lesion
o if NAP positive across a lesion
 preserved axons
 or significant regeneration
o can detect reinnervation months before EMG
 NAP negative-neuropraxic lesion
 NAP positive- axonotmetic lesion
 Sensory and Motor Evoked Potential
o more sensitive than EMG and NCV at identifying continuity of roots with spinal cord (positive
finding)
 a negative finding can not differentiate location of discontinuity (root avulsion vs.
axonotmesis)
o perform 4-6 weeks after injury to allow for Wallerian degeneration to occur
o stimulation done at Erb's point and recording done over cortex with scalp electrodes
(transcranial)

Treatment
 Nonoperative
o observation alone waiting for recovery

- 122 -
By Dr, AbdulRahman AbdulNasser Upper Extremity | Shoulder
 indications
 most managed with closed observation
 guns shot wounds (in absence of major vascular damage can observe for three months)
 signs of neurologic recovery
 advancing Tinel sign is best clinical sign of effective nerve regeneration
 Operative
o immediate surgical exploration (< 1 week)
 indications
 sharp penetrating trauma (excluding GSWs)
 iatrogenic injuries
 open injuries
 progressive neurologic deficits
 expanding hematoma or vascular injury
 techniques
 nerve repair
 nerve grafting
 neurotization
o early surgical intervention (3-6 weeks)
 indicated for near total plexus involvement and with high mechanism of energy
o delayed surgical intervention (3-6 months)
 indications
 partial upper plexus involvement and low energy mechanism
 plateau in neurologic recovery
 best not to delay surgery beyond 6 months
 techniques
 usually involves tendon/muscle transfers to restore function

Surgical Techniques
 Direct nerve repair
o rarely possible due to traction and usually only possible for acute and sharp penetration injuries
 Nerve graft
o commonly used due to traction injuries (postganglionic)
o preferable to graft lesions of upper and middle trunk
 allows better chance of reinnervation of proximal muscles before irreversible changes at
motor end plate
o donor sites include sural nerve, medial brachial nerve, medial antebrachial cutaneous nerve
o vascularized nerve graft includes ulnar nerve when there is a proven C8 and T1 avulsion
(mobilized on superior ulnar collateral artery)
 Neurotization (nerve transfer)
o transfer working but less important motor nerve to a nonfunctioning more important denervated
muscle
o use extraplexal source of axons
 spinal accessory nerve (CN XI)
 intercostal nerves
 contralateral C7
 hypoglossal nerve (CN XII)
o intraplexal nerves
- 123 -
OrthoBullets2017 Upper Extremity | Shoulder
 phrenic nerve
 portion of median or ulnar nerves
 pectoral nerve
 Oberlin transfer
 ulnar nerve used for upper trunk injury for biceps function
 Muscle or tendon transfer
o indications
 isolated C8-T1 injury in adult (reinervation unlikely due to distance between injury site and
hand intrinsic muscles)
o priorities of repair/reconstruction
 elbow flexion (musculocutaneous nerve)
 shoulder stability (suprascapular nerve)
 brachial-thoracic pinch (pectoral nerve)
 C6-C7 sensory (lateral cord)
 wrist extension / finger flexion (lateral and posterior cords)
 wrist flexion / finger extension
 intrinsic function
o technique
 gracilis most common free muscle transfer

2. Sternoclavicular Dislocation
Introduction
 Traumatic or Atraumatic
o traumatic dislocation
 direction
 anterior (more common)
 posterior (mediastinal structures at risk)
 important to distinguish from medial clavicle physeal fracture (physis doesn't fuse until
age 20-25)
 mechanism : usually high energy injury (MVA, contact sports)
o atraumatic subluxation
 occurs with overhead elevation of the arm
 affected patients are younger
 many demonstrate signs of generalized ligamentous laxity
 subluxation usually reduces with lowering the arm
 treatment is reassurance and local symptomatic treatment

Anatomy
 Medial clavicle
o first bone to ossify and last physis to close (age 20-25)
 Sternoclavicular joint
o osteology
 diarthrodial saddle joint
 incongruous (~50% contact)
 fibrocartilage

- 124 -
By Dr, AbdulRahman AbdulNasser Upper Extremity | Shoulder
o stability
 stability depends on ligamentous structures
 posterior capsular ligament
 most important structure for anterior-posterior stability
 anterior sternoclavicular ligament
 primary restraint to superior displacement of medial clavicle
 costoclavicular (rhomboid) ligament
 anterior fasciculus resists superior rotation and lateral displacement
 posterior fasciculus resists inferior rotation and medial displacement
 intra-articular disk ligament
 prevents medial displacement of clavicle
 secondary restraint to superior clavicle displacement

Presentation
 Symptoms
o anterior dislocation
 deformity with palpable bump
o posterior dislocations
 dyspnea or dysphagia
 tachypnea and stridor worse when supine
 Physical exam
o palpation
 prominence that increases with
arm abduction and elevation
o ROM and instability : decreased arm ROM
o neurovascular
 parasthesias in affected upper extremity
 venous congestion or diminished pulse when
compared with contralateral side
o provocative maneuvers
 turning head to affected side may relieve pain

Imaging
 Radiographs
o recommended views
 AP and serendipity views
o findings
 difficult to visualize on AP
 serendipity views ( beam at 40 cephalic tilt)
 anterior dislocation : affected clavicle above contralateral clavicle
 posterior dislocation : affected clavicle below contralateral clavicle
 CT scan
o study of choice
 axial views can visualize mediastinal structures and injuries
 can differentiate from physeal fractures

- 125 -
OrthoBullets2017 Upper Extremity | Shoulder

CT Axial views showing posterior dislocation of sternoclavicular joint


Treatment
 Nonoperative
o reassurance and local symptomatic treatment
 indications
 atraumatic subluxation
 chronic anterior dislocation (> 3 weeks old)
 technique
 sling for comfort
 return to unrestricted activity by 3 months
o closed reduction under general anesthesia +/- thoracic surgery
 indications
 acute anterior dislocations (< 3weeks old)
 acute posterior dislocations (< 3weeks old)
 if reduction stable
 velpeau bandage for 6 weeks
 may need plaster jacket or figure of eight bandage to distract shoulder
 elbow exercises at 3 weeks
 return to sports at 3 months
 if reduction unstable
 accept deformity or medial clavicle excision (below)
 Operative
o open reduction and soft-tissue reconstruction with thoracic surgery back-up
 indications
 posterior dislocation with
 dysphagia
 shortness of breath
 decreased peripheral pulses
o medial clavicle excision
 indications
 chronic/recurrent SC dislocation (anterior or posterior)
 persistent sternoclavicular pain

- 126 -
By Dr, AbdulRahman AbdulNasser Upper Extremity | Shoulder
Techniques
 Closed reduction under general anesthesia
o reduction technique
 place patient supine with arm at edge of table and prep entire chest
 abduct and extend arm while applying axial traction and direct pressure
 simultaneously apply direct posterior pressure over medial clavicle
 manipulate medial clavicle with towel clip or fingers
 Medial clavicle excision
o approach

 incision made over medial clavicle


o resection
 costoclavicular ligaments must be preserved
 preserve by resecting < 15mm of medial clavicle
 repair if injured
Complications: Cosmetic deformity

3. Clavicle Fractures
Introduction
 Epidemiology
o incidence : clavicle fractures make up ~4% of all fractures
o demographics : often seen in young active patients
 Pathophysiology
o mechanism
 direct blow to lateral aspect of shoulder
 fall on an outstretched arm or direct trauma
o pathoanatomy
 in displaced fractures, the
sternocleidomastoid muscle pulls the
medial fragment posterosuperiorly,
while pectoralis and weight of arm pull
the lateral fragment inferomedially
 open fractures buttonhole through platysma
 Associated injuries
o are rare but include
 ipsilateral scapular fracture
 scapulothoracic dissociation
 should be considered with significantly
displaced fractures
 rib fracture
 pneumothorax
 neurovascular injury
 Pediatric Clavicle fractures
o fracture patterns include
 medial clavicle physeal injury
 distal clavicle physeal injury
- 127 -
OrthoBullets2017 Upper Extremity | Shoulder
Relevant Anatomy
 Acromioclavicular Joint Anatomy
 AC joint stability
o acromioclavicular ligament
 provides anterior/posterior stability
 has superior, inferior, anterior, and posterior components
 superior ligament is strongest, followed by posterior
o coracoclavicular ligaments (trapezoid and conoid)
 provides superior/inferior stability
 trapezoid ligament inserts 3 cm from end of clavicle
 conoid ligament inserts 4.5 cm from end of clavicle in the posterior border
 conoid ligament is strongest
o capsule, deltoid and trapezius act as additional stabilizers

Classification
 Allman Classification with Neer's Modification

Neer classification of Group II fracture clavicle

Group 1 nondisplaced Group 1 displaced

- 128 -
By Dr, AbdulRahman AbdulNasser Upper Extremity | Shoulder
Group I - Middle third (80-85%)
Nondisplaced  Less than 100% displacement Nonoperative
Displaced  Greater than 100% displacement
Operative
 Nonunion rate of 4.5%
Group II - Neer Classification of Lateral third (10-15%)
Type I  Fracture occurs lateral to coracoclavicular ligaments (trapezoid,
conoid) or interligamentous Nonoperative
 Usually minimally displaced
 Stable because conoid and trapezoid ligaments remain intact
Type IIA  Fracture occurs medial to intact conoid and trapezoid ligament
 Medial clavicle unstable Operative
 Up to 56% nonunion rate with nonoperative management
Type IIB  Fracture occurs either between ruptured conoid and intact
trapezoid ligament or lateral to both ligaments torn Operative
 Medial clavicle unstable
 Up to 30-45% nonunion rate with nonoperative management
Type III  Intraarticular fracture extending into AC joint
 Conoid and trapezoid intact therefore stable injury Nonoperative
 Patients may develop posttraumatic AC arthritis
Type IV  A physeal fracture that occurs in the skeletally immature
 Displacement of lateral clavicle occurs superiorly through a tear in
the thick periosteum Nonoperative
 Clavicle pulls out of periosteal sleeve
 Conoid and trapezoid ligaments remain attached to periosteum
and overall the fracture pattern is stable
Type V  Comminuted fracture
 Conoid and trapezoid ligaments remain attached to comminuted Operative
fragment
 Medial clavicle unstable
Group III - Medial third (5-8%)
Anterior  Most often non-operative
Nonoperative
displacement  Rarely symptomatic
Posterior  Rare injury (2-3%)
displacement  Often physeal fracture-dislocation (age < 25)
 Stability dependent on costoclavicular ligaments Operative
 Must assess airway and great vessel compromise
 Serendipity radiographs and CT scan to evaluate
 Surgical management with thoracic surgeon on standby

Neer I Neer IIa Neer IIb

- 129 -
OrthoBullets2017 Upper Extremity | Shoulder

Neer III Neer IV Neer V


Presentation
 Symptoms
o shoulder pain
 Physical exam
o deformity
o perform careful neurovascular exam
o tenting of skin (impending open fracture)

Imaging
 Radiographs
o standard AP view of bilateral shoulders
 to measure clavicular shortening
o 45° cephalic tilt determine superior/inferior displacement
o 45° caudal tilt determines AP displacement
 CT
o may help evaluate displacement, shortening, comminution, articular extension, and nonunion
o useful for medial physeal fractures and sternoclavicular injuries

Treatment
 Nonoperative
o sling immobilization with gentle ROM exercises at 2-4 weeks and strengthening at 6-10
weeks
 indications
 nondisplaced Group I (middle third)
 stable Group II fractures (Type I, III, IV)
 nondisplaced Group III (medial third)
 pediatric distal clavicle fractures (skeletally immature)
 outcomes
 nonunion (1-5%)
 risk factors for nonunion
 Group II (up to 56%)
 comminution
 100% displacement & shortening (>2 cm)
 advanced age and female gender
 poorer cosmesis
 decreased shoulder strength and endurance
 seen with displaced midshaft clavicle fracture healed with > 2 cm of shortening

- 130 -
By Dr, AbdulRahman AbdulNasser Upper Extremity | Shoulder
 Operative
o open reduction internal fixation
 indications
 absolute
 unstable Group II fractures (Type IIA, Type IIB, Type V)
 open fxs
 displaced fracture with skin tenting
 subclavian artery or vein injury
 floating shoulder (clavicle and scapula neck fx)
 symptomatic nonunion
 posteriorly displaced Group III fxs

 displaced Group I (middle third) with >2cm shortening


 relative and controversial indications
 brachial plexus injury (questionable b/c 66% have spontaneous return)
 closed head injury
 seizure disorder
 polytrauma patient
 outcomes
 advantages of ORIF
 improved results with ORIF for clavicle fractures with >2cm shortening and 100%
displacement
 improved functional outcome / less pain with overhead activity
 faster time to union
 decreased symptomatic malunion rate
 improved cosmetic satisfaction
 improved overall shoulder satisfaction
 increased shoulder strength and endurance
 disadvantages of ORIF
 increased risk of need for future procedures
 implant removal
 debridement for infection
o coracoclavicular ligament repair vs reconstruction
 indications
 Type IIb fractures

Techniques
 Sling Immobilization
o technique
 sling or figure-of-eight (prospective studies have not shown difference between sling and
figure-of-eight braces)
 after 2-4 weeks begin gentle range of motion exercises
 strengthening exercises begin at 6-10 weeks
 no attempt at reduction should be made

- 131 -
OrthoBullets2017 Upper Extremity | Shoulder

precontoured anatomic
plates
Superior plate Anterior plate

intramedullary screw or
3.5mm reconstruction plate hook plate nail fixation
Open Reduction Internal Fixation
o technique
 plate and screw fixation
 superior vs anterior (anteroinferior) plating
 superior plating biomechanically higher load to failure and bending
 superior plating better for inferior bony comminution
 superior plating has higher risk of neurovascular injury during drilling
 anteroinferior plating has longer screws
 anteroinferior plating has to remove portion of deltoid attachment
 limited contact dynamic compression plate
 3.5mm reconstruction plate
 locking plates
 precontoured anatomic plates
 lower profile needing less chance for removal surgery
 intramedullary screw or nail fixation
 higher complication rate including hardware migration
 hook plate
 AC joint spanning fixation
 postoperative rehabilitation
 sling for 7-10 days followed by active motion
 strengthening at ~ 6 weeks when pain free motion and radiographic evidence of union
 full activity including sports at ~ 3 month
 Coracoclavicular ligament repair
o technique
 coracoclavicular ligament primary repair (most common)
 most add supplementary suture (mersilene tape, fiberwire, ethibond) tied around coracoid
and either into or around clavicle
- 132 -
By Dr, AbdulRahman AbdulNasser Upper Extremity | Shoulder
 Coracoclavicular ligament reconstruction
o see AC separation Techniques section
 techniques include
 modified Weaver-Dunn
 free tendon graft

Complications
 Nonoperative treatment
o nonunion (1-5%)
 risks
 comminution
 Z deformity
 female
 older
 smoker
 distal clavicle higher risk than middle third
 treatment of nonunion
 if asymptomatic, no treatment necessary
 if symptomatic, ORIF with plate and bone graft (particularly atrophic nonunion)
 Operative treatment
o hardware prominence
 ~30% of patient request plate removal
 superior plates associated with increased irritation
o neurovascular injury (3%)
 superior plates associated with increased risk of subclavian artery or vein penetration
 subclavian thrombosis
o nonunion (1-5%)
o infection (~4.8%)
o mechanical failure (~1.4%)
o pneumothorax
o adhesive capsulitis
 4% in surgical group develop adhesive capsulitis requiring surgical intervention

Collected By : Dr AbdulRahman
AbdulNasser
drxabdulrahman@gmail.com
In June 2017

- 133 -
OrthoBullets2017 Upper Extremity | Shoulder

4. Scapula Fractures
Introduction
 Uncommon fracture pattern associated with high energy trauma
o 2-5% associated mortality rate
 usually pulmonary or head injury
 associated with Increased Injury Severity Scores
 Epidemiology
 incidence : less than 1% of all fractures
o location : 50% involve body and spine
 Associated injuries (in 80-90%)
o orthopaedic
 rib fractures (52%)
 ipsilateral clavicle fracture (25%)
 spine fracture (29%)
 brachial plexus injury (5%) : 75% of brachial plexus
injuries resolve
o medical
 pulmonary injury
 pneumothorax (32%)
 pulmonary contusion (41%)
 head injury (34%)
 vascular injury (11%) ‎III:1 Floating Shoulder
Classification
 Classification is based on the location of the fracture and includes
o coracoid fractures
o acromial fractures
o glenoid fractures
o scapular neck fractures
 look for associated AC joint separation or clavicle fracture
 known as "floating shoulder"
o scapular body fractures
 described based on anatomic location
o scapulothoracic dissociation

Coracoid Fracture Classification


Type I Fracture occurs proximal to the coracoclavicular ligament
Type II Fracture occurs towards the tip of the coracoids

Acromial Fracture Classification


Type I Nondisplaced or minimally displaced
Type II Displaced but does not compromise the subacromial space
Type III Displaced and compromises the subacromial space

- 134 -
By Dr, AbdulRahman AbdulNasser Upper Extremity | Shoulder
Ideberg Classification of Glenoid Fracture
Type Ia Anterior rim fracture
Type Ib Posterior rim fracture
Type II Fracture line through glenoid fossa exiting scapula laterally
Type III Fracture line through glenoid fossa exiting scapula superiorly
Type IV Fracture line through glenoid fossa exiting scapula medially
Type Va Combination of types II and IV
Type Vb Combination of types III and IV
Type Vc Combination of types II, III, and IV
Type VI Severe comminution

Coracoid fractures Acromion Fractures

Ideberg Classification of Glenoid Fracture


Anterior Rim Glenoid Fractures Posterior Rim Glenoid Fractures

Type II Type III Type IV Type V

- 135 -
OrthoBullets2017 Upper Extremity | Shoulder
Imaging
 Radiographs
o recommended views
 true AP, scapular Y and axillary lateral view
 CT
o intra-articular fracture
o significant displacement
o three-dimensional reconstruction useful

Treatment
 Nonoperative

o sling for 2 weeks, followed by early motion


 indications
 indicated for vast majority of scapula fractures
 90% are minimally displaced and acceptably
aligned
 outcomes
 union at 6 weeks
 can expect no functional deficits
 Operative
o open reduction internal fixation
 indications
 glenohumeral instability
 > 25% glenoid involvement with
subluxation of humerus
 > 5mm of glenoid articular surface step off
or major gap
 excessive medialization of glenoid
 displaced scapula neck fx
 with > 40 degrees angulation or 1 cm
translation
 open fracture
 loss of rotator cuff function
 coracoid fx with > 1cm of displacement
 outcomes
 70% good to excellent results with operative
treatment
Surgical Technqiues
 Open Reduction Internal Fixation of Scapula
o approach
 based on fracture location ‎III:2 Judet approach
 Judet approach is most common
 utilizes internervous plane between infraspinatus (suprascapular nerve) and teres minor
(axillary nerve)

- 136 -
By Dr, AbdulRahman AbdulNasser Upper Extremity | Shoulder

5. Scapulothoracic Dissociation
Introduction
 A traumatic disruption of the scapulothoracic articulation often associated with
o severe neurologic injuries
o vascular injuries
o orthopaedic injuries
 Mechanism
o usually caused by a lateral traction injury to the shoulder girdle
o involves significant trauma to heart, chest wall and lungs
 Associated conditions
o orthopaedic
 scapula fractures
 clavicle fractures
 AC dislocation/separation
 sternoclavicular dislocation
 flail extremity (52%)
 complete loss of motor and sensory function rendering the extremity non-functional
o vascular injury
 subclavian artery most commonly injured
 axillary artery
o neurologic injury (up to 90%)
 ipsilateral brachial plexus injury (often complete)
 neurologic injuries more common than vascular injuries
 Prognosis
o mortality rate of 10%
o functional outcome is dependent on neurologic injury
 if return of neurological function is unlikely, early amputation is recommended

Anatomy
 Scapulothoracic joint
o a sliding joint
o articulates with ribs 2-7
o moves into abduction at 2:1 ratio
 GH joint 120°
 ST joint 60°
 Neurovascular anatomy
o brachial plexus
o subclavian artery
o axillary artery

Presentation
 History
o history of high energy trauma
 Symptoms
o pain in involved upper extremity (UE)
o numbness/tingling in involved UE
- 137 -
OrthoBullets2017 Upper Extremity | Shoulder
 Physical exam
o inspection
 significantswelling in shoulder region
 bruising around shoulder
o vascular exam
 decreased or absent pulses in involved UE
o neurological exam
 neurologic deficits in UE
 neurological status critical part of exam

Imaging
 Radiographs
o required views
 AP chest ‎III:3 Laterally displaced scapula
o recommended view
 AP and lateral of shoulder as tolerated
 appropriate images of suspected fracture sites
o findings
 laterally displaced scapula
 edge of scapula displaced > 1 cm from spinous process as compared to contralateral side
 widely displaced clavicle fx
 AC separation
 sternoclavicular dislocation
 Angiogram
o indicated to detect injury to subclavian and axillary artery

Treatment
 Nonoperative
o immobilization/supportive care
 indications
 patients without significant vascular injury who are hemodynamically stable
 patients may have adequate collateral flow to UE even with injury
 Operative
o high lateral thoracotomy with vascular repair
 indications : axillary artery injury in hemodynamically unstable patient
o median sternotomy with vascular repair
 indications : more proximal arterial injury (i.e., subclavian artery) in a hemodynamically
unstable patient
o ORIF of the clavicle or AC joint
 indications : associated clavicle and AC injuries
o forequarter amputation
 indications : complete brachial plexus injury

- 138 -
By Dr, AbdulRahman AbdulNasser Upper Extremity | Shoulder

6. Flail Chest
Introduction
 Defined as 3 or more ribs with segmental fractures
 Epidemiology
o bimodal distribution
 younger patients involved in trauma
 older patients with osteopenia
 Mechanism
o blunt forces
o deceleration injuries
 Associated Injuries
o scapula fractures
o clavicle fractures
o hemo/pneumothorax ‎III:4 paradoxical respiration
 Prognosis
o varies depending on underlying pulmonary injury or other concomitant injuries

Anatomy
 Osteology
o 12 ribs per side
 the first seven pairs are connected with the sternum
 the next three are each articulated with the lower border of the cartilage of the preceding rib
 the last two have pointed extremities
o can have an accessory clavicular rib
o anterior ribs articulate with the sternum via the costal cartilage
 Blood Supply
o derived from intercostal vessels

Presentation
 Symptoms
o pain
o respiratory difficulty
o hemopneumothorax
 Exam
o paradoxical respiration
 area of injury "sinks in" with inspiration, and
expands with expiration (opposite of normal
chest wall mechanics) ‎III:5 may see associated hemothorax
o chest wall deformity can be seen
o bony or soft-tissue crepitus is often noted

Imaging
 Radiographs
o may be hard to distinguish non- or minimally-displaced rib fractures
o may see associated hemothorax
 CT : improved accuracy of diagnosis with CT (vs. radiographs)

- 139 -
OrthoBullets2017 Upper Extremity | Shoulder
Treatment
 Nonoperative
o observation
 indications
 no respiratory compromise
 no flail chest segment (>3 consecutive segmentally fractured ribs)
 techniques
 pain control
 systemic narcotics or local anesthetics
 positive pressure ventilation
 Operative
o open reduction internal fixation
 indications
 displaced rib fractures associated with intractable pain
 flail chest segment (3 or more consecutive ribs with segmental injuries)
 rib fractures associated with failure to wean from a ventilator
 open rib fractures
 technique
 approach
 full thoracotomy approach
 limited exposure approach
 open reduction and internal fixation
 plate and screw constructs
 intramedullary splinting
 postop
 early shoulder and periscapular range of motion

Complications
 Intercostal neuralgia
 Periscapular muscle weakness
 Pneumonia
 Restrictive type pulmonary function

- 140 -
By Dr, AbdulRahman AbdulNasser Upper Extremity | Humerus

B. Humerus

1. Proximal Humerus Fractures


Introduction
 Epidemiology
o incidence
 4-6% of all fractures
 third most common fracture pattern seen in elderly
o demographics
 2:1 female to male ratio
 increasing age correlates with increasing fracture risk in women
 Pathophysiology
o mechanism
 low-energy falls
 elderly with osteoporotic bone
 high-energy trauma
 young individuals
 concomitant soft tissue and neurovascular injuries
o pathoanatomy : vascularity of articular segment is more likely to be preserved if ≥ 8mm of calcar
is attached to articular segment
 Associated conditions
o nerve injury : axillary nerve palsy most common
o fracture-dislocations : more commonly associated with nerve
injuries
Anatomy
 Osteology
o anatomic neck : represents the old epiphyseal plate
o surgical neck : represents the weakened area below
 Vascular anatomy
o anterior humeral circumflex artery
 one of primary blood supplies to the humeral head
 branches
 anterolateral ascending branch : is a branch of the
anterior humeral circumflex artery
 arcuate artery : is the terminal branch
 course
 runs parallel to lateral aspect of tendon of
long head of biceps in the bicipital
groove
 has an interosseous anastomosis
o posterior humeral circumflex artery
 recent studies suggest it is the main blood supply
to humeral head

- 141 -
OrthoBullets2017 Upper Extremity | Humerus
Classification
 Valgus impacted
o not true 4-part fractures
o have preserved posterior medial capsular vascularity to the articular segment
 AO/OTA
o organizes fractures into 3 main groups and additional subgroups based on
 fracture location
 status of the surgical neck
 presence/absence of dislocation
 Neer classification
o based on anatomic relationship of 4 segments
 greater tuberosity
 lesser tuberosity
 articular surface
 shaft
o considered a separate part if
 displacement of > 1 cm
 45° angulation

Evaluation
 Symptoms
o pain and swelling
o decreased motion
 Physical exam ‎III:6 AO/OTA of proximal humeral frx
o inspection
 extensive ecchymosis of chest, arm, and forearm
o neurovascular exam
 45% incidence of nerve injury (axillary most common)
 distinguish from early deltoid atony and inferior subluxation of humeral head
 arterial injury may be masked by extensive collateral
circulation preserving distal pulses
Imaging
 Radiographs : recommended views
o complete trauma series
 true AP
 scapular Y
 axillary
o additional views
 apical oblique
 Velpeau
 West Point axillary
o findings
 combined cortical thickness (>4 mm)
 studies suggest correlation with increased lateral plate pullout strength

- 142 -
By Dr, AbdulRahman AbdulNasser Upper Extremity | Humerus

apical oblique Velpeau West Point axillary

 CT scan : indications
o preoperative planning
o humeral head or greater tuberosity position uncertain
o intra-articular comminution
 MRI : indications
o rarely indicated
o useful to identify associated rotator cuff injury

Treatment
 Nonoperative
o sling immobilization followed by progressive rehab
 indications
 85% of proximal humerus fractures are minimally displaced and can be treated
nonoperatively including
 minimally displaced surgical neck fracture (1-, 2-, and 3-part)
 greater tuberosity fracture displaced < 5mm
 fractures in patients who are not surgical candidates
 additional variables to consider
 age
 fracture type
 fracture displacement
 bone quality
 dominance
 general medical condition
 concurrent injuries
 technique
 start early range of motion within 14 days

- 143 -
OrthoBullets2017 Upper Extremity | Humerus
 Operative
o CRPP (closed reduction percutaneous pinning)
 indications
 2-part surgical neck fractures
 3-part and valgus-impacted 4-part fractures in patients with good bone quality, minimal
metaphyseal comminution, and intact medial calcar
o ORIF
 indications
 greater tuberosity displaced > 5mm
 2-,3-, and 4-part fractures in younger patients
 head-splitting fractures in younger patients
o intramedullary rodding
 indications
 surgical neck fractures or 3-part greater tuberosity fractures in
 younger patients
 combined proximal humerus and humeral shaft fractures
 outcomes
 85% success rate in younger patients
o hemiarthroplasty
 indications
 anatomic neck fractures in elderly (initial varus malalignment >20 degrees) or those that
are severely comminuted
 4-part fractures and fracture-dislocations (3-part if stable internal fixation unachievable)
 rotator cuff compromise
 glenoid surface is intact and healthy
 chronic nonunions or malunions in the elderly
 head-splitting fractures with incongruity of humeral head
 humeral head impression defect of > 40% of articular surface
 detachment of articular blood supply (most 3- and 4-part fractures)
 outcomes
 improved results if
 performed within 14 days
 accurate tuberosity reduction
 cerclage wire passed through hole in prosthesis and tuberosities
 poor results with
 tuberosity malunion
 proud prosthesis
 retroversion of humeral component > 40°
o total shoulder arthroplasty
 indications
 rotator cuff intact
 glenoid surface is compromised (arthritis, trauma)
o reverse shoulder arthroplasty
 indications
 elderly individuals with nonreconstructible tuberosities

- 144 -
By Dr, AbdulRahman AbdulNasser Upper Extremity | Humerus
Treatment by Fracture Type
One-Part Fracture (most common)
Surgical Neck fx • Most common type • if stable then early ROM
• if unstable then period of immobilization followed by ROM
once moves as a unit
Anatomic Neck fx • ORIF in young patient
• ORIF vs. hemiarthroplasty in elderly patient
• hemiarthroplasty if severely comminuted
Two-Part Fracture
Surgical Neck • Most common fx pattern (85%) Nonoperative
• Deforming forces: • Closed reduction often possible
1) pectoralis pulls shaft anterior and • Sling
medial 2) head and attached tuberosities Operative
stay neutral • indicated for >45° angulation
• Posterior angulation tolerated better • technique
than anterior and varus angulation - CRPP
- Plate fixation
- Enders rods with tension band
- IM device
Greater tuberosity • Often missed Nonoperative
• Deforming forces: GT pulled superior • indicated for GT displaced < 5 mm
and posterior by SS, IS, and TM Operative
• Can only accept minimal displacement • indicated for GT displacement > 5 mm
or else it will block ER and ABD •AP radiograph of a left shoulder demonstrates a 2-part
proximal humerus fracture at the surgical neck.
- isolated screw fixation only in young with good bone stock
- nonabsorbable suture technique for osteoporotic bone (avoid
hardware due to impingement)
- tension band wiring
Lesser tuberosity • Assume posterior dislocation until Operative
proven otherwise • ORIF if large fragment
• excision with RCR if small
Anatomic neck • Rare Operative
• ORIF in young
• ORIF vs. hemiarthroplasty in elderly patient
Three-Part Fracture
Surgical neck and GT • Subscap will internally rotate articular
segment
• Often associated with longitudinal RCT
Surgical neck and LT • Unopposed pull of external rotators lead • Trend towards nonoperative management with high
to articular surface to point anterior complications with ORIF
• Often associated with longitudinal RCT • Young patient
- percutaneous pinning (good results, protect axillary nerve)
- blade plate / fixed angle device
- IM fixation (violates cuff)
- T plate (poor results with high rate of AVN, impingement,
infection, and malunion)
• Elderly patient
- hemiarthroplasty with RCR or tuberosity repair
Four-Part Fracture
Valgus impacted 3- • Radiographically will see alignment • 74% good results with ORIF
and 4-part fracture between medial shaft and head • Low rate of AVN if posteromedial component intact thus
segments preserving intraosseous blood supply
• Surgical technique
1. raise articular surface and fill defects
2. repair tuberosities
4-part with articular • Characterized by removal of soft tissue • Young patient
surface and head- from fracture fragment leading to high - ORIF vs. hemiarthroplasty (nonreconstructible articular
splitting fracture risk of AVN (21-75%) surface, severe head split, extruded anatomic neck fracture)
• Deforming forces: 1) shaft pulled
medially by pectoralis • Elderly patient
- hemiarthroplasty

- 145 -
OrthoBullets2017 Upper Extremity | Humerus
Techniques
 CRPP (closed reduction percutaneous pinning)
o approach
 percutaneous
o technique
 use threaded pins but do not cross cartilage
 externally rotate shoulder during pin placement
 engage cortex 2 cm inferior to inferior border of humeral head
o complications
 with lateral pins
 risk of injury to axillary nerve
 with anterior pins
 risk of injury to biceps tendon, musculocutaneous n., cephalic vein
 ORIF
o approach
 shoulder anterior approach (deltopectoral)
 shoulder lateral (deltoid-splitting) approach
 indicated for GT and valgus-impacted 4-part fractures
 increased risk of axillary nerve injury
o technique
 heavy nonabsorbable sutures
 (figure-of-8 technique) should be used for greater tuberosity fx reduction and fixation
(avoid hardware due to impingement)
 isolated screw
 may be used for greater tuberosity fx reduction and fixation in young patients with good
bone stock
 locking plate
 has improved our ability to fix these fractures
 screw cut-out (up to 14%) is the most common complication following fixation of 3- and
4- part proximal humeral fractures and fractures treated with locking plates
 more elastic than blade plate making it a better option in osteoporotic bone
 place plate lateral to the bicipital groove and pectoralis major tendon to avoid injury to the
ascending branch of anterior humeral circumflex artery
 placement of an inferomedial calcar screw can prevent post-operative varus collapse,
especially in osteoporotic bone
 Intramedullary rodding
o approach
 superior deltoid-splitting approach
o technique
 lock nail with trauma or pathologic fractures
o complications
 rod migration in older patients with osteoporotic bone is a concern
 shoulder pain from violating rotator cuff
 nerve injury with interlocking screw placement

- 146 -
By Dr, AbdulRahman AbdulNasser Upper Extremity | Humerus
 Hemiarthroplasty
o approach
 shoulder anterior approach (deltopectoral)
o technique for fractures
 cerclage wire or suture passed through hole in prosthesis and tuberosities improves fracture
stability
 place greater tuberosity 10 mm below articular surface of humeral head (HTD = head to
tuberosity distance)
 impairment in ER kinematics and 8-fold increase in torque with nonanatomic placement
of tuberosities
 height of the prosthesis best determined off the superior edge of the pectoralis major tendon
 post-operative passive external rotation places the most stress on the lesser tuberosity
fragment
 Total shoulder arthroplasty
 Reverse shoulder arthroplasty

Rehabilitation
 Important part of management
 Best results with guided protocols (3-phase programs)
o early passive ROM for first 6 weeks
o active ROM and progressive resistance
o advanced stretching and strengthening program
 Prolonged immobilization leads to stiffness

Complications
 Screw penetration
o most common complication after locked plating fixation (up to 14%)
 Avascular necrosis
o risk factors
 4 part fractures
 head split
 short calcar segments
 disrupted medial hinge
o no relationship to type of fixation (plate or cerclage wires)
 Nerve injury
o axillary nerve injury (up to 58%)
 increased risk with anterolateral acromial approach
 axillary nerve is found 7cm distal to the tip of the acromion
o suprascapular nerve (up to 48%)
 Malunion
o usually varus apex-anterior or malunion of GT
o results inferior if converting from varus malunited fracture (with GT in varus necessitating
osteotomy) to TSA
 use reverse TSA instead
 Nonunion
o usually with surgical neck and tuberosity fx
o treatment of chronic nonunion/malunion in the elderly should include arthroplasty

- 147 -
OrthoBullets2017 Upper Extremity | Humerus
o lessertuberosity nonunion leads to weakness with lift-off testing
o greater tuberosity nonunion leads to lack of active shoulder elevation
o greatest risk factors for non-union are age and smoking
 Rotator cuff injuries and dysfunction
 Missed posterior dislocation
 Adhesive capsulitis
 Posttraumatic arthritis
 Infection

2. Proximal Humerus Fracture Malunion


Introduction
 Malposition of the humeral tuberosities; rotation, angulation, and/or offset of the head-shaft junction;
or articular incongruities
 Epidemiology
o incidence : proximal humerus fractures account for 4% to 5% of all fractures
o risk factors
 fracture characteristics
 3 or 4 part fracture patterns
 humeral head split
 displaced tuberosity fractures
 patient factors
 osteoporosis
 chronic renal disease
 chronic alcohol or steriod use
 Pathophysiology
o origin of malunion I‎II:7 displaced tuberosity fractures
 inital fracture displacement
 secondary displacement after loss of reduction
 failure of internal fixation
o humeral head malunion
 varus/valgus
 impacted (>1cm displacement)
 articular surface incongruity (e.g. head split)
o greater tuberosity malunion
 usually displaced posterior, superior and externally rotated
o lesser tuberosity malunion
 usually displaced medial
 Associated conditions
o rotator cuff tearing
o osteonecrosis of humeral head
o glenohumeral joint issues
 stiffness
 post-traumatic arthritis
 subluxation or dislocation I‎II:8 Normal anatomy Greater Tuberosity
 subacromial impingement position

- 148 -
By Dr, AbdulRahman AbdulNasser Upper Extremity | Humerus
Anatomy
 Humeral head
o shape : spheroidal in 90% of individuals
o size : average diameter is 43 mm
o orientation
 retroverted 30° from transepicondylar axis of the distal humerus
 neck-shaft agle usually 130° to 140°
 Greater tuberosity
o position important for rotator cuff muscle fuction
 horizontal position : medial edge of tuberosity is 10mm lateral to humeral canal axis
 vertical position : superior edge of tuberosity is 6mm inferior to upper edge of humeral head

Classification
 Beredjiklian et al.

Beredjiklian
TypeI • Malposition of the greater or lesser tuberosity ( e.g. >1 cm from native anatomical
position)
Type II • Articular incongruity ( e.g. intra-articular fracture extension, osteoarthritis)
Type III • Articular surface malalignment ( e.g. >45° of deformity with respect to the humeral shaft
in the coronal, sagittal, or axial planes

 Boileau et al.
Boileau
Type I • Humeral head necrosis or impaction
Type II • Chronic dislocations or fracture-dislocations
Type III • Nonunion of the surgical neck
Type IV • Severe malunion of the tuberosity

Presentation
 History
o initial evaluation
 date and mechanism of injury
 current and prior function
 handedness
 treatment to date
 specific goals of treatment
 Symptoms
o pain and weakness
o limitations
 Physical exam
‎III:9 Humeral head orientation
o inspection
 features of systemic disease
 muscle atrophy
 diffuse tenderness
- 149 -
OrthoBullets2017 Upper Extremity | Humerus
o motion
 active and passive shoulder range-of-motion
 blocks or crepitus should be noted
 rotator cuff
 greater tuberosity malunion = weakness with abduction, external rotation
 lesser tuberosity malunion = weakness with internal rotation
 instability
 humeral head malunion = apprehension test
o neurovascular

Imaging
 Radiographs
o recommended views
 true AP, scapular Y, axillary
o optional views
 apical oblique
 Velpeau
 West Point axillary
o findings ‎III:10 fracture fragment displacement
 neck-shaft angle = varus or valgus
 greater tuberosity = superiorly and posteriorly displaced, externally rotated
 lesser tuberosity = medialized
o measurements
 humeral head
 > 45° of deformity in any plane
 symptomatic articular incongruity
 neck-shaft angle <120° or >150°
 greater or lesser tuberosity
 >1 cm from native anatomical position
 CT scan
o indications
 preoperative planning
 assess bone stock, orientation and articular surface
o findings
 humeral head and greater tuberosity displacement
 glenoid version and glenoid bone stock
 articular injury
 MRI
o indications
 preoperative planning
 soft-tissue structures
o findings
 rotator cuff or labral injury
 deltoid atrophy secondary to axillary nerve injury
 long-head biceps injury
 osteonecrosis

- 150 -
By Dr, AbdulRahman AbdulNasser Upper Extremity | Humerus
Studies
 Labs : CBC, ESR, CRP, blood cultures to rule out infection
 Electrodiagnositcs : concern for nerve dysfunction

Treatment
 Nonoperative
o NSAIDS, physical therapy, occasional corticosteriod injection
 indications
 low-demand patient
 painless shoulder limitations
 unable to comply with rehabilitation protocol
 modalities
 physical therapy
 maximize ROM and strengthening program
 outcomes
 impacted varus and valgus fractures show good-to-excellent results
 return to 90% of normal fuction
 Operative
o humeral head preserving techniques
 indications
 symptomatic malunion following
 nonoperative treatment
 failed internal fixation
 anatomical requirements
 adequate bone stock for fixation
 preserved articular surface
 intact blood supply to humeral head
 techniques
 humeral head deformities
 minor deformity techniques
 open/arthroscopic tuberoplasty +/- acromioplasty +/- capsular release +/-
bursectomy
 severe deformity techniques
 varus/valgus osteotomy +/- rotational osteotomy and lateral plate fixation
 treated with corrective osteotomy/fixation if patient is young or active
 augmentation with strut allograft for poor bone stock
 greater tuberosity deformities
 <1.5 cm displacement
 arthroscopic subacromial decompression +/- rotator cuff repair
 >1.5 cm displacement
 open/arthroscopic tuberosity osteotomy +/- subacromial decompression
 outcomes
 complication rates associated with surgical management of malunions are higher than
those associated with acute fractures

- 151 -
OrthoBullets2017 Upper Extremity | Humerus
o humeral head replacing techniques
 indications
 symptomatic malunion following
 nonoperative treatment
 failed internal fixation
 anatomical requirements
 inadequate bone stock for fixation techniques
 articular incongruity, destruction or collapse (e.g. osteonecrosis or head-split)
 compromised blood supply
 chronic dislocation
 techniques
 hemiarthroplasty
 total shoulder arthroplasty
 reverse total shoulder arthroplasty

Complications
 Persistent pain and weakness
 Stiffness
 Loss of fixation
 Infection
 Bleeding

3. Humeral Shaft Fractures


Introduction
 Incidence
o 3-5% of all fractures
o bimodal age distribution
 young patients with high-energy trauma
 elderly, osteopenic patients with low-energy injuries
Relevant Anatomy
 Osteology : humeral shaft is cylindrical
 Muscles
o insertion for
 pectoralis major
 deltoid
 coracobrachialis
o origin for
 brachialis
 triceps
 brachioradialis
 Nerve
o radial nerve
 courses along spiral groove
 14cm proximal to the lateral epicondyle
 20cm proximal to the medial epicondyle

- 152 -
By Dr, AbdulRahman AbdulNasser Upper Extremity | Humerus
Classification
 OTA
o bone number: 1
o fracture location: 2
o fracture pattern: simple:A, wedge:B, complex:C
 Descriptive
o fracture location: proximal, middle or distal third
o fracture pattern: spiral, transverse, comminuted
 Holstein-Lewis fracture
o a spiral fracture of the distal one-third of the humeral shaft commonly
associated with neuropraxia of the radial nerve (22% incidence)
Presentation
 Symptoms
o pain ‎III:11 Holstein-Lewis fracture
o extremity weakness
 Physical exam
o examine overall limb alignment
o preoperative or pre-reduction neurovascular exam is critical
 examine and document status of radial nerve pre and post-reduction

Imaging
 Radiographs : views
o AP and lateral
 be sure to include joint above and below the site of injury
o transthoracic lateral
 may give better appreciation of sagittal plane deformity
o traction views
 may be necessary for fractures with significant shortening, proximal
or distal extension but not routinely indicated
‎III:12 AP radiograph fracture
Treatment humerus
 Nonoperative
o coaptation splint followed by functional brace
 indications
 indicated in vast majority of humeral shaft fractures
 criteria for acceptable alignment include:
 < 20° anterior angulation
 < 30° varus/valgus angulation
 < 3 cm shortening
 absolute contraindications
 severe soft tissue injury or bone loss
 vascular injury requiring repair
 brachial plexus injury
 relative contraindications ‎III:13 transthoracic lateral
 see relative operative indications section
 radial nerve palsy is NOT a contraindication to functional bracing

- 153 -
OrthoBullets2017 Upper Extremity | Humerus
 outcomes
 90% union rate
 increased risk with proximal third oblique or spiral fracture
 varus angulation is common but rarely has functional or cosmetic sequelae
o damage control orthopaedics (DCO)
 closed humerus fractures, including low velocity GSW, should be initially managed with a
splint or sling
 type of fixation after trauma should be directed by acceptable fracture alignment parameters,
fracture pattern and associated injuries
 Operative
o open reduction and internal fixation
 absolute indications
 open fracture
 vascular injury requiring repair
 brachial plexus injury
 ipsilateral forearm fracture (floating elbow)
 compartment syndrome
 relative indications
 bilateral humerus fracture
 polytrauma or associated lower extremity fracture I‎II:14 standard
 allows early weight bearing through humerus functional brace

 pathologic fractures
 burns or soft tissue injury that precludes bracing
 fracture characteristics
 distraction at fracture site
 short oblique or transverse fracture pattern
 intraarticular extension
o intramedullary nailing (IMN)
 relative indications
 pathologic fractures
 segmental fractures
 severe osteoporotic bone
 overlying skin compromise limits open approach
 polytrauma I‎II:15 open reduction and
internal fixation
Techniques
 Coaptation Splint & Functional Bracing
o coaptation splint
 applied until swelling resolves
 adequately applied splint will extend up to axilla and over shoulder
 common deformities include varus and extension
 valgus mold to counter varus displacement
o functional bracing
 extends from 2.5 cm distal to axilla to 2.5 cm proximal to humeral condyles
 sling should not be used to allow for gravity-assisted fracture reduction

- 154 -
By Dr, AbdulRahman AbdulNasser Upper Extremity | Humerus
 shoulder extension used for more proximal fractures
 Open Reduction Internal Fixation with Plating
o approaches
 anterolateral approach to humerus
 used for proximal third to middle third shaft fractures
 distal extension of the deltopectoral approach
 radial nerve identified between the brachialis and
brachioradialis distally
 posterior approach to humerus
 used for distal to middle third shaft fractures although
can be extensile
 triceps may either be split or elevated with a lateral
paratricipital exposure ‎III:16apply plate in bridging mode n severe
 radial nerve is found medial to the long and lateral comminution
heads and 2cm proximal to the deep head of the
triceps
 radial nerve exits the posterior compartment through lateral intramuscular septum 10 cm
proximal to radiocapitellar joint
 lateral brachial cutaneous/posterior antebrachial cutaneous nerve serves as an anatomic
landmark leading to the radial nerve during a paratricipital approach
o techniques
 plate osteosynthesis commonly with 4.5mm plate (narrow or broad)
 3.5mm plates may function adequately
 absolute stability with lag screw or compression plating in simple patterns
 apply plate in bridging mode in the presence of significant comminution
o postoperative
 full crutch weight bearing shown to have no effect on union
 Closed Intramedullary Nailing (IMN)
o techniques : can be done antegrade or retrograde
o complication
 nonunion
 nonunion rates not shown to be different between IMN and
plating in recent meta-analyses
 IM nailing associated with higher total complication rates
 shoulder pain
 increased rate when compared to plating (16-37%)
 nerve injury
 radial nerve is at risk with a lateral to medial distal locking
screw
 musculocutaneous nerve is at risk with an anterior-posterior
locking screw
I‎II:17 Closed Intramedullary
o postoperative Nailing
 full weight bearing allowed and had no effect on union

- 155 -
OrthoBullets2017 Upper Extremity | Humerus
Complications
 Malunion
o varus angulation is common but rarely has functional or cosmetic sequelae
o risk factors : transverse fracture patterns
 Nonunion
o incidence
 2 to 10% in nonoperative management
 5 to 10% with surgical management
o risk factors
 distraction at the fracture site on injury films
 open fracture
 metabolic/endocrine abnormalities (Vitamin D deficiency most common)
 segmental fracture
 infection
 shoulder or elbow stiffness (motion directed to fracture site)
 patient factors (smoking, obesity, malnutrition, noncompliance)
o treatment
 compression plating with bone grafting
 shown to be superior to both IM nailing with bone grafting and compression plating
alone
 lateral, posterior or paratricipital (Gerwin) approach to allow exploration of the radial
nerve
 Radial nerve palsy
o incidence
 seen in 8-15% of closed fractures
 increased incidence distal one-third fractures
 neuropraxia most common injury in closed fractures and neurotomesis in open fractures
 85-90% of improve with observation over 3 months
 spontaneous recovery found at an average of 7 weeks, with full recovery at an average of 6
months
o treatment
 observation
 indicated as initial treatment in closed humerus fractures
 obtain EMG at 3-4 months

 wrist extension in radial deviation is expected to be regained first


 brachioradialis first to recover, extensor indicis is the last
 surgical exploration
 indications
 open fracture with radial nerve palsy (likely neurotomesis injury to the radial nerve)
 closed fracture that fails to improve over ~ 3-6 months
 fibrillations (denervation) seen at 3-4 months on EMG

- 156 -
By Dr, AbdulRahman AbdulNasser Upper Extremity | Humerus

4. Distal Humerus Fractures


Introduction
 Consists of
o supracondylar fractures
o single column (condyle) fractures
o bicolumn fractures
o coronal shear fractures
 Epidemiology
o incidence : distal intercondylar fractures are the most common fracture pattern
o demographics : most common in young males and older females
 Pathophysiology
o mechanism
 low energy falls in elderly
 high energy impact in younger population
o pathoanatomy
 elbow position affects fracture type
 elbow flexed < 90 degrees
 axial load leads to transcolumnar fracture
 direct posterior blow leads to olecranon fracture with or without distal humerus
involvement
 elbow flexed > 90 degrees
 may lead to intercondylar fracture
 Associated injuries
o elbow dislocation
o terrible triad injury
o floating elbow
o Volkmann contracture : result of a missed forearm compartment syndrome
 Prognosis
o majority of patients regain 75% of elbow motion and strength
o goal is to restore elbow ROM 30-130 degrees of flexion
o unsatisfactory outcomes in up to 25%
 treatment of these fractures is complex due to:
 low fx line of one or both columns
 metaphyseal fragmentation of one or both columns
 articular comminution
 poor bone quality

Anatomy
 Osteology
o elbow is a hinged joint
o trochlea
 articulates with sigmoid notch
 allows for flexion and extension
o capitellum
 articulates with proximal radius : allows for forearm rotation

- 157 -
OrthoBullets2017 Upper Extremity | Humerus
 Muscles
o common flexors (originate from medial epicondyle)
 pronator teres
 flexor carpi radialis
 palmaris longus
 FDS
 FCU
o common extensors (originate from lateral epicondyle)
 anconeus
 ECRL
 ECRB
 extensor digitorum comminus
 EDM
 ECU
 Ligaments
o medial collateral ligament
 anterior bundle originates from distal medial epicondyle
 inserts on sublime tubercle
 primary restraint to valgus stress at the elbow from 30 to 120 deg
 tight in pronation
o lateral collateral ligament
 originates from distal lateral epicondyle
 inserts on crista supinatorus
 stabilizer against posterolateral rotational instability
 taut in supination
 Nerves
o ulnar nerve : resides in cubital tunnel in a subcutaneous position below the medial condyle
o radial nerve
 resides in spiral groove 15cm proximal to distal humeral articular surface
 between brachioradialis and brachialis proximal to elbow
 divides into PIN and superficial radial nerve at level of radial head
Classification
 Can be classified as
o supracondylar fractures
o distal single column fractures
 subclassified using Milch classification system (see table)
 lateral condyle more common than medial
o distal bicolumnar fractures
 classified using Jupiter classification system (see table)
 5 major articular fragments have been identified
 capitellum/lateral trochlea
 lateral epicondyle
 posterolateral epicondyle
 posterior trochlea
 medial trochlea/epicondyle

- 158 -
By Dr, AbdulRahman AbdulNasser Upper Extremity | Humerus

AO/OTA Classification of Distal Humerus Fractures


Type A Extraarticular (supracondylar fracture)
80% are extension type
Type B Intraarticular-Single column (partial articular-isolated condylar, coronal
shear, epicondyle)
Type C Intraarticular-Both columns fractured and no portion of the joint contiguous
with the shaft (complete articular)

Each type further divided by degree and location of fracture comminution

Milch Classification of Single Column Condyle Fractures


Milch Type I Lateral trochlear ridge intact
Milch Type Fracture through lateral trochlear ridge
II

Jupiter Classification of Two-Column Distal Humerus Fractures


High-T Transverse fx proximal to or at upper olecranon fossa
Low-T Transverse fx just proximal to trochlea (common)
Y Oblique fx line through both columns with distal vertical fx line
H Trochlea is a free fragment (risk of AVN)
Medial lambda Proximal fx line exists medially
Lateral lambda Proximal fx line exists laterally

Multiplane T T type with additional fracture in coronal plane

Illustrated AO/OTA classification of distal humerus fractures: type


A, extra-articular; type B, partial articular; and type C, complete Milch Classification of Single Column Condyle Fractures
articular. Each type has further subdivisions based on the increasing
complexity of the fracture pattern.

- 159 -
OrthoBullets2017 Upper Extremity | Humerus
Presentation
 Symptoms : elbow pain and swelling
 Physical exam
o gross instability often present
 avoid ROM due to risk of neurovascular damage
o neurovascular exam
 check function of radial, ulnar, and median nerve
 check distal pulses
 brachial artery may be injured
 if pulse decreased, obtain noninvasive vascular studies; consult vascular surgery if
abnormal
o monitor carefully for forearm compartment syndrome
Imaging
 Radiographs
o recommended views
 obtain AP and lateral of humerus and elbow
 include entire length of humerus and forearm
o additional views
 obtain wrist radiographs if elbow injury present or distal tenderness on exam
 oblique radiographs may assist in surgical planning
 traction radiographs may assist in surgical planning
 specifically evaluate if there is continuity of the trochlear fragment to medial epicondylar
fragment, this can influence hardware choice
 CT
o often obtained for surgical planning
o especially helpful when shear fractures of the capitellum and trochlea are suspected
o 3D CT scan improves the intraobserver and interobserver reliability of several classification
systems
 MRI
o usually not indicated in acute injury

Treatment
 Nonoperative
o cast immobilization
 indications
 nondisplaced Milch Type I fractures
 technique
 immobilize in supination for lateral condyle fractures
 immobilize in pronation for medial condyle fractures
 Operative
o closed reduction and percutaneous pinning
 indications
 displaced Mich Type I fractures
o open reduction internal fixation
 indications
 supracondylar fractures

- 160 -
By Dr, AbdulRahman AbdulNasser Upper Extremity | Humerus
 intercondylar / bicolumnar fractures
 Milch Type II fractures
o total elbow arthroplasty
 indications
 distal bicolumnar fractures in elderly patients

Techniques
 Open Reduction Internal Fixation
o positioning
 lateral decubitus position
 on foam mattress with radiolucent arm board
 prone position
 useful in patients with spine injuries or contralateral extremity fractures
 supine positioning
 can be used in a polytrauma situation or with contraindications to other positioning
 obtain test imaging before prepping and draping
 prep entire arm from shoulder to hand
o approach
 articular surface exposure
 olecranon osteotomy 57%
 triceps-reflecting 46%
 triceps-splitting 35%
 posterior superficial approach
 raise full thickness medial and lateral soft tissue flaps
 elevate deep fascia to identify ulnar and radial nerves
 triceps splitting (Campbell)
 split triceps tendon in midline down to olecranon
 tricep sparing (known as paratricipital, Alonso-Llames, medial and lateral windows)
 indications
 extra articular fractures or fractures with simple articular split)

- 161 -
OrthoBullets2017 Upper Extremity | Humerus
 can be converted to olecranon osteotomy if needed
 medial side
 identify ulnar nerve and dissect it 15cm proximal to elbow joint proximally, and
distally to first motor branch to FCU
 elevate triceps from posterior aspect of humerus on medial side and free it from
medial intermuscular septum
 posterior band of MCL is elevated and posterior joint capsule entered to visualize
trochlea
 lateral side
 identify radial nerve proper proximally if fracture is distal
 if fracture is distal and does not require long plates, proper radial nerve does not
need to be exposed
 elevate remainder of tricep from posterior aspect of humerus
 anconeus may be divided or dissected on lateral side to improve exposure
 olecranon osteotomy
 indications : complex intra articular fragments and/or presence of coronal splint)
 contraindications : total elbow arthroplasty is planned/may be required
 technique
 identify bare area of sigmoid notch medially and laterally
 pre-drill (for 6.5mm screw) or plate prior to making bone cut
 pass sponge through ulnohumeral joint to protect articular surface while making cut
 fluoroscopy is used to confirm location of osteotomy
 shallow chevron (apex distal) is cut down to subchondral bone (95% cut)
 finish cut (remaining 5%) with osteotome
 peel olecranon and triceps proximally and wrap with saline soaked sponge
 fixation
 screw, K wires and tension band or plate
 clamp osteotomy from medial and lateral side with large pointed reduction clamps
 insert 6.5, 7.0 or 7.3mm screw (or plate) in previously drilled hole
 apply tension band
 still preferable for posterior trochlea fx and medial epicondyle fx
 complications
 AIN nerve injury
 check ability to flex thumb interphalangeal joint in recovery
 triceps reflecting (Bryan-Morrey)
 reflect triceps tendon, forearm fascia and periosteum from medial to lateral off olecranon
 repair through transosseous drill holes
 immobilize to protect triceps repair for 4-6wk postop
 triceps-reflecting anconeous pedicle (O'Driscoll)
 elevate anconeous subperiosteally from proximal ulna
 medial exposure is Bryan-Morrey triceps reflecting approach
 lateral muscles interval
 is an alternative to visualize the articular
 elevate ECRB and part of ECRL of supracondylar ridge
 usually able to work anterior to and sacrifice LCL
 if fx of lateral column, utilize and mobilize

- 162 -
By Dr, AbdulRahman AbdulNasser Upper Extremity | Humerus
 sublux joint to assist in articular visualization
o fixation principles (O'Driscoll)
 fixation in the distal fragment must be maximized
 all fixation in distal fragments should contribute to stability between the distal fragments and
the shaft.
o fixation objectives (O'Driscoll)
 every screw in the distal fragments should pass through a plate
 engage a fragment on the opposite side that is also fixed to a plate
 as many screws as possible should be placed in the distal fragments
 each screw should be as long as possible
 each screw should engage as many articular fragments as possible
 the screws in the distal fragments should lock together by interdigitation, creating a fixed-
angle structure
 this creates the architecural equivalent of an arch, which gives the most biomechanical
stability
 plates should be applied such that compression is achieved at the supracondylar level for both
columns
 the plates must be strong enough and stiff enough to resist breaking or bending before union
occurs at the supracondylar level.
o fixation
 countersunk / headless screw to fix articular fragments 1st after provisional reduction with k-
wires
 if metaphyseal injury is not comminuted, reducing one column to the metaphysis first
may be beneficial
 consider using positional screws when reducing trochlea to avoid narrowing it with
compression
 then address condyles and epitrochlear ridge
 lateral epicondyle may be fix with tension band wire or plate
 two plates in orthogonal planes used to fix articular segment to shaft
 place 3.5-mm LCDC plate or one of equivalent strength on lateral side
 place 2.7-mm or 3.5-mm LCDC plate on medial side
 interdigitate screws if possible to increase strength
 new literature supports parallel plates
 if ulnar nerve contacts medial hardware during flexion/extension, can transpose however
literature does not support decreased ulnar n. symptoms with transposition
 postoperative
 place in splint with elbow in approx 70 degrees of flexion
 remove splint at 48 hours post-operatively, initiate ROM exercises
 if osteotomy performed patient may do active and active assisted flexion and
extension for 6 weeks; no active extension against gravity or resistance
 if not osteotomy, permitted to do active motion against gravity without restrictions
 no restrictions to rotation
 start gentle strengthening program at 6 weeks, and full strengthening program at 3 months

- 163 -
OrthoBullets2017 Upper Extremity | Elbow
Complications
 Elbow stiffness : most common
 Heterotopic ossification
o reported rate of 8%
o routine prophylaxis is not warranted
 increased rate of nonunion in patients treated with indomethacin
 Nonunion
o low incidence
o avoid excessive soft-tissue stripping
 Malunion
o avoided by proper surgical technique
 cubitus valgus (lateral column fxs)
 cubitus varus (medial column fxs)
 DJD
 Ulnar nerve injury
 AIN Injury : can be seen with olecranon osteotomy

C. Elbow

1. Elbow Dislocation
Introduction
 Epidemiology
o incidence
 elbow dislocations are the most common major joint dislocation second to the shoulder
 account for 10-25% of injuries to the elbow
 posterolateral is the most common type of dislocation (80%)
o demographics : predominantly affects patients between age 10-20 years old
 Pathophysiology
o mechanism
 usually a combination of
 axial loading
 supination/external rotation of the forearm
 posterolateral based valgus force
 a varus posteromedial mechanism has also been reported
 posterior dislocations may involve more than one injury mechanism
o pathoanatomy
 associated with complete or near complete circular disruption of capsuloligamentous
stabilizers
 pathoanatomic cascade
 progression of injury is from lateral to medial
 LCL fails first (primary lesion)
 by avulsion of the lateral epicondylar origin
 midsubstance LCL tears are less common but do occur
 MCL fails last depending on degree of energy

- 164 -
By Dr, AbdulRahman AbdulNasser Upper Extremity | Elbow
Anatomy
 Static and dynamic stabilizers confer stability to the elbow
o static stabilizers (primary)
 ulnohumeral joint
 anterior bundle of the MCL
 LCL complex (includes the LUCL)
o static stabilizers (secondary)
 radiocapitellar joint
 capsule
 origins of the flexor and extensor tendons
o dynamic stabilizers : includes muscles crossing elbow joint
 anconeus
 brachialis
 triceps
 See complete Anatomy and Biomechanics of Elbow

Classification
 Anatomic description
o based on anatomic location of olecranon relative to humerus
 posterolateral : most common
 Simple vs. complex
o simple
 no associated fracture
‎III:19 lateral radiograph of terrible triad
 account for 50-60% of elbow dislocations injury
o complex
 associated fracture present
 may take form of
 terrible triad injury
 involves a disruption of the LUCL, a radial head fracture, a coronoid tip fracture and a
dislocation of the elbow
 varus posteromedial rotatory instability
 the coronoid fracture may be comminuted
 medial facet of the coronoid is usually involved

Presentation
 Symptoms : pain may be the primary symptom
 Physical exam
o important to assess
 the status of the skin
 presence of compartment syndrome
 neurovascular status

‎III:20 AP radiograph of terrible triad


injury

- 165 -
OrthoBullets2017 Upper Extremity | Elbow
Imaging
 Radiographs
o recommended views
 AP and lateral films
 need to check the status of the congruency of the joint
o optional views
 oblique views
 may give clearer sense of periarticular bony involvement
 CT scan
o indications
 suspicion of complex injury pattern
 useful to identify osseous involvement

Treatment
 Nonoperative
o reduction and splinting at 90° for 7-10 days, early therapy
 indications
 acute simple stable dislocations
o reduction splinting in hinged brace at 90° for 2-3 weeks
 indications
 acute simple unstable elbow dislocations (unstable with extension following reduction)
 Operative
o ORIF (coronoid, radial head, olecranon) , LCL repair, +/- MCL repair
 indications
 acute complex elbow dislocations
 persistent instability after reduction
 reduction blocked by entrapped soft tissue or osteochondral fragments
 outcomes
 improved with use of this systematic algorithm
o open reduction, capsular release, and dynamic hinged elbow fixator
 indications
 chronic dislocations
 postoperative
 hinged external fixator indicated in chronic dislocation to protect the reconstruction and
allow early range of motion
Nonoperative Technique
 Closed reduction with splinting
o reduction maneuver
 inline traction to correct coronal displacement
 supination to clear the coronoid beneath trochlea
 flexion of elbow while placing pressure on tip of olecranon
o assess post reduction stability
 elbow is often unstable in extension
 if LCL is disrupted then usually more stable in pronation
 if MCL is disrupted then usually more stable in supination

- 166 -
By Dr, AbdulRahman AbdulNasser Upper Extremity | Elbow
o immobilize and obtain post-reduction radiographs
 check for concentric reduction of joint
 if concentric then immobilize (5-7 days) and start early therapy
 Rehabilitation
o initial
 immobilize for 5-7 days
o early
 supervised (therapist) active and active assist range-of-motion exercises within stable arc
 extension block brace is used for 3-4 weeks
 proceed with light duty use 2 weeks from injury
o late rehabilitation
 extension block is decreased such that by 6-8 weeks after the injury full stable extension is
achieved
Operative Technique
 ORIF of coronoid, radial head, repair of LCL +/- MCL
o approach
 posterior utility approach used
 Kocher interval laterally (ECU/anconeus)
o reconstruction
 coronoid
 fixation can usually be completed laterally via radial head fracture
 severe comminution may necessitate medial approach
 radial head
 ORIF
 when placing fixation on the proximal radius, one must be aware of the "safe zone" (a
90° arc in the radial head that does not articulate with the proximal ulna)
 the "safe zone" can be identified by its relationship to Lister's tubercle and the
radial styloid
 radial head arthroplasty
 indicated if radial head can not be reconstructed
 if radial head is replaced the replacement should be anatomic and restore normal
length/size
 this improves the varus and external rotatory stability of the elbow, but stability
isn't restored until LCL is addressed
 excision of the radial head leads to varus/external rotatory instability when the
LCL function is absent
 LCL
 reconstructed or repaired relative to the anatomic axis of rotation
 extensor origin avulsion is common and may be repaired
 MCL
 if instability persists following LCL repair, the MCL is repaired or reconstructed
o postoperative care
 depending on stability of the elbow, active ROM exercises may commence while using a
brace
 an extension block may or may not be used

- 167 -
OrthoBullets2017 Upper Extremity | Elbow
Complications
 Varus Posteromedial instability

o injury to the LCL and fracture of the anteromedial facet of the coronoid
o solid fixation of the anteromedial facet is critical for functional outcome and prevention of
arthrosis
 Loss of motion
o loss of terminal extension is the most common sequelae after closed treatment of a simple elbow

dislocation
o early active ROM can help prevent this from occurring
o static, progressive splinting can be utilized after inflammation has diminished
 Neurovascular injuries (ulnar/median nerves)
 Compartment syndrome
 Damage to articular surface
 Chronic instability
 Heterotopic ossification
o may require excision to improve elbow range of motion
 Contracture/stiffness
o correlated with immobilization beyond 3 weeks

2. Radial Head Fractures


Introduction
 Epidemiology
o incidence
 1.5-4% of all fractures
 radial head fractures are among the most common elbow fractures (33%)
 Pathophysiology
o mechanism of injury
 fall on outstretched hand
 elbow in extension + forearm in pronation
 most force transmitted from wrist to radial head
 Associated injuries
o 35% have associated soft tissue or skeletal injuries including
 ligamentous injury
 lateral collateral ligament (LCL) injury
 most common (up to 80% on MRI)
 medial collateral ligament (MCL) injury
 combined LCL/MCL
 Essex-Lopresti injury
 distal radioulnar joint (DRUJ) injury
 interosseous membrane disruption
 other elbow fractures
 coronoid fracture
 olecranon fracture

- 168 -
By Dr, AbdulRahman AbdulNasser Upper Extremity | Elbow
 elbow dislocation
 terrible triad (elbow dislocation, radial head fracture, coronoid fracture)
 carpal fractures
 scaphoid fracture

Anatomy
 Osteology
o elbow joint contains two articulations
 ulnohumeral (hinge)
 radiocapitellar (pivot)
 60% load transfer across elbow joint
o proximal radius
 nonarticular portion of the radial head is a ~90 degree arc from radial styloid to Lister's
tubercle (safe zone for hardware placement)
 Ligaments
o lateral collateral ligament complex
 lateral ulnar collateral ligament (LUCL)
 primary stabilizer to varus and external rotation stress
 deficiency results in posterolateral rotatory instability
 radial collateral ligament (RCL)
 accessory lateral collateral ligament
 annular ligament
 stabilizes proximal radioulnar joint
o medial (ulnar) collateral ligament (MCL)
 three bundles
 anterior bundle
 primary stabilizer to valgus stress
(radial head is second)
 posterior bundle
 transverse bundle
 Biomechanics
o radial head confers two types of stability to the elbow
 valgus stability
 secondary restraint to valgus load at the elbow, important if MCL deficient
 longitudinal stability
 restraint to proximal migration of the radius
 contributions from interosseous membrane and DRUJ
 load-sharing from wrist to radiocapitellar joint, dependant on radiocapitellar surface area
 loss of longitudinal stability occurs when
 radial head fracture + DRUJ injury + interosseous membrane disruption (Essex-
Lopresti)
 radial head must be fixed or replaced to restore stability, preventing proximal
migration of the radius and ulnocarpal impaction

- 169 -
OrthoBullets2017 Upper Extremity | Elbow
Classification
Mason Classification (Modified by Hotchkiss and Broberg-Morrey)
Nondisplaced or minimally displaced (<2mm), no mechanical block to
Type I
rotation
Displaced >2mm or angulated, possible mechanical block to forearm
Type II
rotation
Type III Comminuted and displaced, mechanical block to motion
Type IV Radial head fracture with associated elbow dislocation

Presentation
 Symptoms
o pain and tenderness along lateral aspect of elbow
o limited elbow or forearm motion, particularly supination/pronation
 Physical exam
o range of motion
 evaluate for mechanical blocks to elbow motion
 flexion/extension and pronation/supination
 aspiration of joint hematoma and injection of local anesthesia aids in evaluation of
mechanical block
o stability
 elbow
 lateral pivot shift test (tests LUCL)
 valgus stress test (tests MCL)
 DRUJ
 palpate wrist for tenderness
 translation in sagittal plane > 50% compare to contralateral side is abnormal

- 170 -
By Dr, AbdulRahman AbdulNasser Upper Extremity | Elbow
may be difficult to determine on exam, can get dynamic CT scan in neutral, pronation
and supination for subtle injury
 interosseous membrane
 palpate along interosseous membrane for tenderness
 radius pull test
 >3mm translation concerning for longitudinal forearm instability (Essex-Lopresti)

Imaging
 Radiographs
o recommended views
 AP and lateral elbow
 check for fat pad sign indicating occult minimally displaced fracture
o additional views
 radiocapitellar view (Greenspan view)
 oblique lateral view of elbow
 beam angled 45 degrees cephalad
 allows visualization of the radial head without coronoid overlap
 helps detect subtle fractures of the radial head
 CT
o further delineate fragments in comminuted fractures
o identify associated injuries in complex fracture dislocations

‎III:21 The radiocapitellar (Greenspan) view is obtained by aiming the beam 45 degree cephalad,
lessening the overlap between the proximal radius and olecranon, making subtle radial head fractures easier to identify

Treatment
 Nonoperative
o short period of immobilization followed by early ROM
 indications
 isolated minimally displaced fractures with no mechanical blocks (Mason Type I)
 outcomes
 elbow stiffness with prolonged immobilization
 good results in 85% to 95% of patients
 Operative
o ORIF

- 171 -
OrthoBullets2017 Upper Extremity | Elbow
 indications
 Mason Type II with mechanical block
 Mason Type III where ORIF feasible
 presence of other complex ipsilateral elbow injuries
 outcomes
 # fragments
 ORIF shown to have worse outcome with 3 or more fragments compared to ORIF
with < 3 fragments
 isolated vs. complex
 ORIF isolated radial head fractures versus complex radial head fractures (other
associated fracture/dislocation) show no significant difference in outcomes at 4 years
 isolated fractures trended towards better Patient-Rated Elbow Evaluation score, lower
complication rate and lower rate of secondary capsular release
o fragment excision (partial excision)
 indications
 fragments less than 25% of the surface area of the radial head or 25%-33% of capitellar
surface area
 outcomes : even small fragment excision may lead to instability
o radial head arthroplasty
 indications
 comminuted fractures (Mason Type III) with 3 or more fragments where ORIF not
feasible and involves greater than 25% of the radial head
 elbow fracture-dislocations or Essex Lopresti lesions
 radial head excision will exacerbate elbow/wrist instability and may result in proximal
radial migration and ulnocarpal impingement
 outcomes
 radial head fractures requiring replacement have shown good clinical outcomes with
metallic implants
 compared to ORIF for fracture-dislocations and Mason Type III fractures, arthroplasty
results in greater stability, lower complication rate and higher patient satisfaction
o radial head resection
 indications
 low demand, sedentary patients
 in a delayed setting for continued pain of an isolated radial head fracture
 contraindications
 presence of destabilizing injuries
 forearm interosseous ligament injury (>3mm translation with radius pull test)
 coronoid fracture
 MCL deficiency

Techniques
 Approaches to Radial Head
o overview
 PIN crosses the proximal radius from anterior to posterior within the supinator muscle 4cm
distal to radial head
 in both Kocher and Kaplan approaches, the forearm should be pronated to protect PIN
 pronation pulls the nerve anterior and away from the surgical field
- 172 -
By Dr, AbdulRahman AbdulNasser Upper Extremity | Elbow
o Kocher approach
interval
 between ECU (PIN) and anconeus (radial n.)
 key steps
 incise posterior fibers of the supinator
 incise capsule in mid-radiocapitellar plane
 anterior to crista supinatoris to avoid damaging LUCL
 pros
 less risk of PIN injury than Kaplan approach (more posterior)
 cons
 risk of destabilizing elbow if capsule incision is too posterior and LUCL is
violated, which lies below the equator of the capitellum
o Kaplan approach
 interval
 between EDC (PIN) and ECRB (radial n.)
 key steps
 incise mid-fibers of supinator
 incise capsule anterior to mid-radiopatellar plane (have access)
 pros
 less risk of disrupting LUCL and destabilizing elbow than Kocher approach (more
anterior)
 better visualization of the coronoid
 cons
 greater risk of PIN and radial nerve injury

The Kaplan approach uses the Pronation of the forearm pulls the PIN
more anterior interval between anteromedially and away from the lateral surgical
ECRB and EDC. The Kocher field.
approach uses the more
posterior interval between ECU
and anconeus.

- 173 -
OrthoBullets2017 Upper Extremity | Elbow
 ORIF
o approach
 Kocher or Kaplan approach
o plates
 fracture involved head and neck
 posterolateral plate placement
 safe zone (nonarticular area) consists of 90-110 degree arc from radial styloid to Lister's
tubercle, with arm in neutral rotation to avoid impingement of ulna with forearm rotation
 bicipital tuberosity is the distal limit of plate placement
 anything distal to that will endanger PIN
 countersink implants on articular surface
o screws
 headless compression screws (Hebert) if placed in articular surface
 better elbow range of motion and functional outcome scores at 1 year compared to plate
fixation
 Radial head arthroplasty
o approach
 Kocher or Kaplan approach
o technique
 metal prostheses
 loose stemmed prosthesis
 that acts as a stiff spacer
 bipolar prosthesis
 that is cemented into the neck of the radius
 silicon replacements are no longer used
‎III:22 Safe zone
 indepedent risk factor for revision surgery
o complications
 overstuffing of joint that leads to capitellar wear problems and malalignment instability
 overstuffing of joint is best assessed under direct visualization
 Radial head resection
o approach
 Kocher or Kaplan approach
o complications after excision of the radial head include
 muscle weakness
 wrist pain
 valgus elbow instability
 heterotopic ossification
 arthritis
 proximal radial migration
 decreased strength
 cubitus valgus

Complications
 Displacement of fracture
o occurs in less than 5% of fractures; serial radiographs do not change management
 Posterior interosseous nerve injury (with operative management)
 Loss of fixation
- 174 -
By Dr, AbdulRahman AbdulNasser Upper Extremity | Elbow
 Loss of forearm rotation
 Elbow stiffness
o first-line management incluides supervised exercise therapy with static or dynamic progressive
elbow splinting over a 6 month period
 Radiocapitellar joint arthritis
 Infection
 Heterotopic ossification
 Hardware loosening
 Complex regional pain syndrome

3. Coronoid Fractures
Introduction
 Coronoid fractures are pathognomonic of an episode of elbow instability
o may be
 isolated coronoid fracture : less common than previously thought
 coronoid fracture + associated injuries
 commonly occur with elbow dislocation
 associated with recurrent instability after dislocation
 Mechanism
o traumatic shear injury
 typically occurs as distal humerus is driven against coronoid with an episode of severe varus
stress or posterior subluxation
 not an avulsion injury as nothing inserts on tip
 Pathoanatomy
o fractures at the coronoid base can amplify elbow instability given that
 anterior bundle of the medial ulnar collateral ligament attaches to the sublime tubercle 18 mm
distal to tip
 anterior capsule attaches 6 mm distal to the tip of the coronoid
 Epidemiology
o incidence : 10-15% of elbow injuries
 Associated conditions
o posteromedial rotatory instability
 coronoid anteromedial facet fracture and LCL disruption I‎II:23 anteromedial facet fracture

 results from a varus deforming force


o posterolateral rotatory instability
 coronoid tip fracture, radial head fracture, and LCL injury
o olecranon fracture-dislocation
 usually associated with a large coronoid fracture
o terrible triad of elbow
 coronoid fracture (transverse fracture pattern), radial head
I‎II:24 coronoid tip fracture
fracture, and elbow dislocation
 Prognosis : complications and reoperation rates are high

Anatomy
 Coronoid osteology
o coronoid tip
- 175 -
OrthoBullets2017 Upper Extremity | Elbow
 is an intraarticular structure
 can be visualized during elbow arthroscopy
o medial facet
 important for varus stability
 provides insertion for the medial ulnar collateral ligament
 Coronoid biomechanics
o coronoid functions as an anterior buttress of the olecranon greater sigmoid notch
 important in preventing recurrent posterior subluxation
o primary resistor of elbow subluxation or dislocation

Classification
Regan and Morrey Classification
Type I coronoid process tip fracture
Type II fracture of 50% or less of height
Type III fracture of more than 50% of height

O'Driscoll Classification
 Subdivides coronoid injuries based on location and number of coronoid
fragments
 Recognizes anteromedial facet fractures caused by varus posteromedial rotatory
force

Regan and Morrey Classification

- 176 -
By Dr, AbdulRahman AbdulNasser Upper Extremity | Elbow
Presentation
 Symptoms
o elbow deformity & swelling
o elbow pain
o forearm or wrist pain may be a sign of associated injuries
 Physical exam
o inspection & palpation
 varus or valgus deformity
 ecchymosis & swelling
 diffuse tenderness
o range of motion & instability
 document flexion-extension and pronation-supination
 crepitus should be noted
 varus/valgus instability stress test
 challenging but important for an accurate diagnosis
o neurovascular exam

Imaging
 Radiographs
o recommended views : AP and lateral elbow views I‎ II:25 anteromedial facet
coronoid fracture ap and
o findings : interpretation may be difficult due to overlapping structures lateral radiographs
 CT scan : useful for high grade injuries and comminuted fractures

Treatment
 Nonoperative
o brief period of immobilization, followed by early range of motion
 indications : Type I, II, and III that are minimally displaced with stable elbow
 Operative
o ORIF with medial approach
 indications
 Type I, II, and III with persistent elbow instability
 posteromedial rotatory instability
o ORIF with posterior approach
 indications
 olecranon fracture dislocation
 terrible triad of elbow
o hinged external fixation
 indications
 large fragments
 poor bone quality
 difficult revision cases to help maintain stability

Techniques
 ORIF with medial approach
o approach
‎III:26 ORIF with buttress
plate fixation and screws
 medial exposure through an interval between two heads of FCU
 exposure more anteriorly through a split in flexor pronator mass

- 177 -
OrthoBullets2017 Upper Extremity | Elbow
o technique
 cerclage wire or No. 5 suture through ulna drill holes for Type I injuries
 ORIF with retrograde cannulated screws or plate for Type II or III injuries
 ORIF with buttress plate fixation or pins and lateral ligament repair for posteromedial
rotatory instability
o postoperative rehabilitation
 depends on intraoperative exam following the procedure
 thermoplastic resting splint
 applied with elbow at 90° and forearm in neutral
 restrict terminal 30° extension for 2-4 weeks
 avoid shoulder abduction for 4-6 weeks
 to prevent varus moment on arm
 early active motion
 dynamic muscle contraction may improve gapping of the ulnohumeral joint after surgical
repair
 ORIF with posterior approach
o approach : posterior
o technique
 mobilize olecranon fracture to access coronoid fracture for associated olecranon fracture-
dislocations
 repair coronoid fragment first prior to reducing main ulnar fracture
 olecranon ORIF with dorsal plate and screws

Complications
 Recurrent elbow instability : especially medial-sided
 Elbow stiffness
 Posttraumatic arthritis
 Heterotopic ossification
 Early failure : associated with failure to recognize and repair underlying elbow instability

4. Terrible Triad Injury of Elbow


Introduction
 A traumatic injury pattern of the elbow characterized by
o elbow dislocation (often associated with posterolateral
dislocation or LCL injury)
o radial head fracture
o coronoid fracture
 Pathophysiology
o mechanism
 fall on extended arm that results in a combination of
 valgus, axial, and posterolateral rotatory forces
 produces posterolateral dislocation
 structures of elbow fail from lateral to medial
 anterior bundle of MCL last to fail ‎III:27 Medial collateral ligament
 LCL disrupted in most cases

- 178 -
By Dr, AbdulRahman AbdulNasser Upper Extremity | Elbow
Anatomy
 Radial head
o forearm in neutral rotation, lateral portion of articular
margin devoid of cartilage
 roughly between radial styloid and listers tubercle
o provides anterior and valgus buttress
 Coronoid process
o provides an anterior and varus buttress
 Medial collateral ligament
o anterior bundle, posterior bundle, and transverse
ligament components ‎III:28Lateral collateral ligament
o anterior bundle most important to stability, restraint to
valgus and posteromedial rotatory instability
 inserts on sublime tubercle (anteromedial facet of coronoid)
 specifically inserts 18.4mm dorsal to tip of coronoid process
 Lateral collateral ligament
o inserts on supinator crest distal to lesser sigmoid notch
o restraint to varus and posterolateral rotatory instability
o two components
 lateral ulnar collateral ligament (most important for stability)
 lateral radial collateral ligament : attaches to annular ligament

Presentation
 Symptoms : patients complain of pain, clicking and locking with elbow
in extension
 Physical exam
o varus instability
o may show valgus instability if injury to MCL

Imaging
 Radiographs
o evaluate for concentricity of ulnohumeral and radiocapitellar joints
o line drawn through center of radial neck should intersect the center of the capitellum regardless
of radiographic projection
o evaluate lateral radiograph for coronoid fracture
 CT
o better evaluation of coronoid fracture
o 3D imaging for determining fracture line propagation

Treatment
 Nonoperative
o immobilize in 90 deg of flexion for 7-10 days
 indications (rare)
 ulnohumeral and radiocapitellar joints must be concentrically reduced
 elbow should extend to at least 30 degrees before becoming unstable
 CT must show insignificant radial head/neck fx, no block to motion
 coronoid fx limited to tip

- 179 -
OrthoBullets2017 Upper Extremity | Elbow
 technique
 active motion initiated with resting splint at 90 degrees, avoiding terminal extension
 static progressive extension splinting at night after 4-6 weeks
 strengthening protocol after 6 weeks
 Operative
o acute radial head stabilization, coronoid ORIF, and LCL reconstruction, MCL
reconstruction if needed
 indications
 terrible triad elbow injury that includes a unstable radial head fracture, a type III coronoid
fracture, and an associated elbow dislocation
Techniques
 Acute radial head stabilization, coronoid ORIF, and LCL reconstruction, MCL reconstruction
if needed
o approach
 posterior skin incision advantageous
 allows access to both medial and lateral aspect of elbow
 lower risk of injury to cutaneous nerves
 more cosmetic
o technique
 radial head ORIF vs. arthroplasty
 radial head arthroplasty indicated for comminuted radial head fracture
 use of modular prosthesis preferable
 sizing based on fragments removed from elbow
 implant should articulate 2mm distal to the tip of the coronoid process
 radial head resection without replacement is NOT indicated in presence of Essex-
Lopresti lesion or in young active patient
 it <25% head damaged or fragments not reconstructable and nonarticulating, can excise
fractured portion if elbow stable (rarely indicated)
 radial head ORIF indicated if non comminuted with good bone stock and fracture
involves < 40% articular surface
 1.5, 2.0, or 2.4mm countersunk screws
 plating if necessary; 2.0 plates cause minimal loss of motion even when placed on
radial neck
 coronoid ORIF
 can be fixed through radial head defect laterally
 fix with suture passed through 2 drill holes, or posterior to anterior lag screws if fragment
large
 basal coronoid fxs (rare) fixed with anteromedial or medial plate on proximal ulna
 LCL repair
 usually avulsed from origin on lateral epicondyle
 reattach with suture anchors or transosseous sutures
 must be reattached at center of capitellar curvature on lateral epicondyle
 if MCL is intact, LCL is repaired with forearm in pronation
 if MCL injured, LCL is repaired with forearm in supination to avoid medial gapping due
to overtightening
 repairs are performed with elbow at 90 degrees of flexion
- 180 -
By Dr, AbdulRahman AbdulNasser Upper Extremity | Elbow
 MCL repair
 indicated if instability on exam after LCL and fracture fixation, especially with extension
beyond 30 degrees
 instability after complete bone and soft tissue repair indicates need for hinged or static elbow
fixator application
 postoperative
 immobilize elbow in flexion with forearm pronation to provide stability against posterior
subluxation
 if both MCL and LCL were repaired, splint in flexion and neutral rotation.

Complications
 Instability : more common following type I or II coronoid fractures
 Failure of internal fixation
o most common following repair of radial neck fractures
 poor vascularity leading to osteonecrosis and nonunion
 Posttraumatic stiffness
o very common
o initiate early ROM to prevent
 Heterotopic ossification
o consider prophylaxis in pts with head injury or in setting of revision surgery
 Posttraumatic arthritis : due to chondral damage at time of injury and/or residual instability

5. Olecranon Fractures
Introduction
 Epidemiology
o bimodal distribution
 high energy injuries in the young
 low energy falls in the elderly
 Pathophysiology
o mechanism
 direct blow
 usually results in comminuted fracture
 indirect blow
 fall onto outstretched upper extremity
 usually results in transverse or oblique fracture

Anatomy
 Osteology
o together with coronoid process, forms the greater sigmoid (semilunar) notch
o greater sigmoid notch articulates with trochlea
 provides flexion-extension movement
 adds to stability of elbow joint
 Muscles
o triceps
 inserts onto posterior, proximal ulna
 blends with periosteum

- 181 -
OrthoBullets2017 Upper Extremity | Elbow
 innervated by radial nerve (C7)
o anconeus
 insertson lateral aspect of olecranon
 innervate by radial nerve (C7)

Classification
Mayo Classification
Based on comminution, displacement, fracture-
dislocation

 Type I : undisplaced fractures


 Type II displaced stable fractures
 Type III fractures with unstable ulnohumeral joint
Each type subdivivded according comminution

Colton Classification
Nondisplaced - Displacement does not increase with elbow flexion
Avulsion (displaced)
Oblique and Transverse (displaced)
Comminuted (displaced)
Fracture dislocation

Schatzker Classification
Type A Simple transverse fracture
Type B Transverse impacted fracture
Type C Oblique fracture
Type D Comminuted fracture
Type E More distal fracture, extra-articular
Type F Fracture-dislocation

AO Classification
Type A Extra-articular
Type B Intra-articular
Type C Intra-articular fractures of both the radial head and olecranon

- 182 -
By Dr, AbdulRahman AbdulNasser Upper Extremity | Elbow

Colton Classification

Nondisplaced Avulsion Oblique and Comminuted Fracture


colton type I (displaced) Transverse (displaced) dislocation
(displaced)

Schatzker Classification

Type A Type B

Type C Type D Type E Type F


Presentation
 Symptoms : pain well localized to posterior elbow
 Physical exam
o palpable defect : indicates displaced fracture or severe comminution
o inability to extend elbow : indicates discontinuity of triceps (extensor) mechanism

Imaging
 Radiographs
o recommended views
 AP/lateral radiographs
 true lateral essential for determination of fracture pattern
o additional views
 radiocapitellar may be helpful for
 radial head fracture
 capitellar shear fracture
 CT : may be useful for preoperative planning in comminuted fractures

- 183 -
OrthoBullets2017 Upper Extremity | Elbow
Treatment
 Nonoperative
o immobilization
 indications
 nondisplaced fractures
 displaced fracture is low demand, elderly individuals
 technique
 immobilization in 45-90 degrees of flexion initially
 begin motion at 1 week
 Operative
o tension band technique
 indications
 transverse fracture with no comminution
 outcomes
 excellent results with appropriate indications
o intramedullary fixation
 indications
 transverse fracture with no comminution (same as tension band technique)
o plate and screw fixation
 indications
 comminuted fractures
 Monteggia fractures
 fracture-dislocations
 oblique fractures that extend distal to coronoid
o excision and triceps advancement
 indications
 elderly patients with osteoporotic bone
 fracture must involve <50% of joint surface
 nonunions
 outcomes
 salvage procedure that leads to decreased extension strength
 may result in instability if ligamentous injury is not diagnosed before operation

Tension band technique • Plate and screw • Intramedullary fixation Excision


fixation

- 184 -
By Dr, AbdulRahman AbdulNasser Upper Extremity | Elbow
Surgical Techniques
 Tension band technique: technique
 converts distraction force of triceps into a compressive force
 engaging anterior cortex of ulna with Kirschner wires may prevent wire migration
 avoid overpenetration of wires through anterior cortex
 may injury anterior interosseous nerve (AIN)
 may lead to decreased forearm rotation
 use 18-gauge wire in figure-of-eight fashion through drill holes in ulna
o cons
 high % of second surgeries for hardware removal (40-80%)
 does not provide axial stability in comminuted fractures
 Intramedullary fixation: technique
 can be combined with tension banding
 intramedullary screw must engage distal intramedullary canal
 Plate and screw fixation
o technique
 place plate on dorsal (tension) side
 oblique fractures benefit from lag screws in addition to plate fixation
 one-third tubular plates may not provide sufficient strength in comminuted fractures
 may advance distal triceps tendon over plate to avoid hardware prominence
o pros : more stable than tension band technique
o cons : 20% need second surgery for plate removal
 Excision and triceps advancement
o technique : triceps tendon reattached with nonabsorbable sutures passed through drill holes in
proximal ulna
Complications
 Symptomatic hardware : most frequent reported complication
 Stiffness : occurs in ~50% of patients , usually doesn't alter functional capabilities
 Heterotopic ossification : more common with associated head injury
 Posttraumatic arthritis
 Nonunion : rare
 Ulnar nerve symptoms
 Anterior interosseous nerve injury
 Loss of extension strength

6. Capitellum Fractures
Introduction
 Coronal fracture of the distal humerus at capitellum
 Epidemiology : 1% of elbow fractures
 Mechanism of injury : fall on outstretched hand
 Prognosis
o most patients will gain functional range of motion but have residual stiffness
o surgical treatment results are generally favorable
 reoperation rates as high as 48%

- 185 -
OrthoBullets2017 Upper Extremity | Elbow
Classification
Bryan and Morrey Classification (with McKee modification)
Type I Large osseous piece of the capitellum involved
Can involve trochlea
Type II Kocher-Lorenz fracture
Shear fracture of articular cartilage
Articular cartilage separation with very little subchondral bone attached
Type III Broberg-Morrey fracture
Severely comminuted
Multifragmentary
Type IV McKee modification
Coronal shear fracture that includes the capitellum and trochlea

Type I Type II

Type III

Presentation
 History : fall on outstretched arm
 Symptoms : elbow pain, swelling
 Physical exam : may have mechanical block to flexion and extension

Imaging
 Radiographs : recommended
o AP and lateral of the elbow
 best demonstrated on lateral radiograph
 CT : delineates fracture anatomy and classification

- 186 -
By Dr, AbdulRahman AbdulNasser Upper Extremity | Elbow
Treatment
 Nonoperative
o posterior splint immobilization for < 3 weeks
 indications : nondisplaced Type I and Type II fractures (<2 mm displacement)
 Operative
o open reduction and internal fixation
 indications
 displaced Type I fractures (>2mm)
 Type IV fractures
o fragment excision
 indications
 displaced (>2mm) Type II fractures
 displaced (>2mm) Type III fractures
o total elbow arthroplasty
 indications : unreconstructable capitellar fractures in elderly patients with associated medial
column instability
Technique
 ORIF
o approach
 lateral approach recommended for Type IV fx
 posterior approach can be used if associated with other elbow injuries
o screw fixation
 headless screw fixation
 minifragment screw using posterior to anterior fixation
 counter sink screw using anterior to posterior fixation
o avoid disruption of the blood supply that comes from the posterolateral aspect of the elbow

Complications
 Elbow contracture (most common)
 Nonunion (1-11% with ORIF)
 Ulnar nerve injury
 Heterotopic ossification (4% with ORIF)
 AVN of capitellum
 Nonunion of olecranon osteotomy

- 187 -
OrthoBullets2017 Upper Extremity | Forearm

D. Forearm

1. Monteggia Fractures
Introduction
 Injury defined as proximal 1/3 ulnar fracture with associated radial head dislocation/instability
 Epidemiology
o rare in adults
o more common in children with peak incidence between 4 and 10 years of age
 different treatment protocol for children
 Associated injuries
o may be part of complex injury pattern including
 olecranon fracture-dislocation
 radial head fx
 coronoid fx
 LCL injury
 terrible triad of elbow
 Prognosis : if diagnosis is delayed greater than 2-3 weeks complication rates increase significantly

Anatomy
 Ligament : annular ligament

Classification
Bado Classification
Type I Fracture of the proximal or middle third of the ulna with anterior
60% dislocation of the radial head (most common in children and young
adults)
Type II 15% Fracture of the proximal or middle third of the ulna with posterior
dislocation of the radial head (70 to 80% of adult Monteggia fractures)
Type III Fracture of the ulnar metaphysis (distal to coronoid process) with lateral
20%
dislocation of the radial head
Type IV Fracture of the proximal or middle third of the ulna and radius with
5%
dislocation of the radial head in any direction

Type I Type II Type III

- 188 -
By Dr, AbdulRahman AbdulNasser Upper Extremity | Forearm

Jupiter Classification of Type II Monteggia Fracture-


Dislocations
Type IIA Coronoid level
Type IIB Metaphyseal-diaphyseal junction
Type IIC Distal to coronoid
Type IID Fracture extending to distal half of ulna

Presentation
 Symptoms
o pain and swelling at elbow joint
‎III:29 Jupiter Classification of Type II Monteggia
 Physical exam Fracture-Dislocations
o inspection
 may or may not be obvious dislocation at radiocapitellar joint
 should evaluate skin integrity
o ROM & instability : may be loss of ROM at elbow due to dislocation
o neurovascular exam
 PIN neuropathy
 radial deviation of hand with wrist extension
 weakness of thumb extension
 weakness of MCP extension
 most likely nerve injury

Imaging
 Radiographs
o recommended view
 AP and Lateral of elbow, wrist, and forearm
 CT scan : helpful in fractures involving coronoid, olecranon, and radial head

Treatment
 Nonoperative
o closed reduction
 indications
 more common and successful in children
 must ensure stabilty and anatomic alignment of ulna fracture
 technique : cast in supination for Bado I and III
 Operative
o ORIF of ulna shaft fracture
 indications
 acute fractures which are open or unstable (long oblique)
 comminuted fractures
 most Monteggia fractures in adults are treated surgically
o ORIF of ulna shaft fracture, open reduction of radial head
 indications
 failure to reduce radial head with ORIF of ulnar shaft only
- 189 -
OrthoBullets2017 Upper Extremity | Forearm
 ensure ulnar reduction is correct
 complex injury pattern
o IM Nailing of ulna
 indications : transverse or short oblique fracture

Techniques
 ORIF of ulnar shaft fracture
o approach
 lateral decubitus position with arm over padded support
 midline posterior incision placed lateral to tip of olecranon
 develop interval between flexor carpi ulnaris and anconeus along ulnar border proximally,
and interval between FCU and ECU distally
o techniques
 with proper alignment of ulna radial head usually reduces and open reduction of radial head is
rarely needed
 failure to align ulna will lead to chronic dislocation of radial head
 ORIF of radial head
o approach : posterolateral (Kocher) approach
o technique
 annular ligament often found interposed in radiohumeral joint preventing anatomic reduction
after ulnar ORIF
 treatment based on involved components (radial head, coronoid, LCL)

Complications
 PIN neuropathy
o up to 10% in acute injuries
o treatment
 observation for 2-3 months
 spontaneously resolves in most cases
 if no improvement obtain nerve conduction studies
 Malunion with radial head dislocation
o usually caused by failure to obtain anatomic alignment of ulna
o treatment
 ulnar osteotomy and open reduction of the radial head

Collected By : Dr AbdulRahman
AbdulNasser
drxabdulrahman@gmail.com
In June 2017

- 190 -
By Dr, AbdulRahman AbdulNasser Upper Extremity | Forearm

2. Radius and Ulnar Shaft Fractures


Introduction
 "Both-bone" forearm fractures
 Epidemiology
o more common in men than women
o ratio of open to closed fractures is higher than for any other bone except tibia
 Mechanism
o direct trauma
 often while protecting one's head
o indirect trauma
 motor vehicle accidents
 falls from height
 athletic competition
 Associated conditions
o elbow injuries
 evaluate DRUJ and elbow for
 Galeazzi fractures
 Monteggia fractures
o compartment syndrome
 evaluate compartment pressures if concern for compartment syndrome
 Prognosis
o functional results depend on restoration of radial bow

Anatomy
 Osteology
o axis of rotation of forearm runs through radial head (proximal) and ulna fovea (distal)
 distal radius effectively rotates around the distal ulna in pronosupination
 Interosseous membrane (IOM)
o occupies the space between the radius and ulna
o comprised of 5 ligaments
 central band is key portion of IOM to be reconstructed
 accessory band
 distal oblique bundle
 proximal oblique cord
 dorsal oblique accessory cord

Classification
 Descriptive
o closed versus open
o location
o comminuted, segmental, multifragmented
o displacement
o angulation
o rotational alignment ‎III:30 Interosseous membrane

- 191 -
OrthoBullets2017 Upper Extremity | Forearm
 OTA classification
o radial and ulna diaphyseal fractures
 Type A : simple fracture of ulna (A1), radius (A2), or both bones (A3)
 Type B : wedge fracture of ulna (B1), radius (B2), or both bones (B3)
 Type C : complex fractures

Presentation
 Symptoms
o gross deformity, pain, swelling
o loss of forearm and hand function
 Physical exam
o inspection
 open injuries
 check for tense forearm compartments
o neurovascular exam
 assess radial and ulnar pulses
 document median, radial, and ulnar nerve function
o pain with passive stretch of digits
 alert to impending or present compartment syndrome

Imaging
 Radiographs
o recommended views

 AP and lateral views of the forearm


o additional views
 oblique forearm views for further fracture definition
 ipsilateral wrist and elbow
 to evaluate for associated fractures or dislocation
 radial head must be aligned with the capitellum on all views

Treatment
 Nonoperative
o functional fx brace with good interosseous mold
 indications
 isolated nondisplaced or distal 2/3 ulna shaft fx (nightstick fx) with
 < 50% displacement and
 < 10° of angulation
 outcomes
 union rates > 96%
 acceptable to fix surgically due to long time to union
 Operative
o ORIF without bone grafting
 indications
 displaced distal 2/3 isolated ulna fxs
 proximal 1/3 isolated ulna fxs
 all radial shaft fxs (even if nondisplaced)
 both bone fxs
- 192 -
By Dr, AbdulRahman AbdulNasser Upper Extremity | Forearm

 Gustillo I, II, and IIIa open fractures may be treated with primary ORIF
 outcomes
 most important variable in functional outcome is to restore the radial bow
o ORIF with bone grafting
 indications
 cancellous autograft is indicated in radial and ulnar fractures with bone loss
 bone loss that is segmental or associated with open injury
 nonunions of the forearm
o external fixation
 indications
 Gustillo IIIb and IIIc open fractures
o IM nailing
 indications
 poor soft-tissue integrity
 not preferred due to lack of rotational and axial stability and difficulty maintaining radial
bow (higher nonunion rate)
Techniques
 ORIF
o approach
 usually performed through separate approaches due to risk
of synostosis
 radius
 volar (Henry) approach to radius
 best for distal 1/3 and middle 1/3 radial fx
 dorsal (Thompson) approach to radius
 best for middle and proximal 1/3 radial fx
 ulna
 subcutaneous approach to ulna shaft
o technique
 3.5 mm DCP plate (AO technique) is standard
 longer plates are preferred due to high torsional stress in forearm
 locked plates are increasingly indicated over conventional plates in osteoporotic bone and
in bridging comminuted fractures
 bone grafting
 vascularized fibula grafts can be used for large defects and have a lower rate of infection
o postoperative care
 early ROM unless there is an injury to proximal or distal joint
 should be managed with a period of non-weight bearing due to risk of secondary
displacement of the fracture
Complications
 Synostosis
o uncommon with an incidence of 3 to 9%
o associated with ORIF using a single incision approach

- 193 -
OrthoBullets2017 Upper Extremity | Forearm
o heterotopic bone excision can be performed with low recurrence risk as early as 4-6 months post-
injury when prophylactic radiation therapy and/or indomethacin are used postoperatively
 Infection
o 3% incidence with ORIF
 Compartment syndrome
o increased risk with
 high energy crush injury
 open fxs
 low velocity GSWs
 vascular injuries
 coagulopathies (DIC)
 Nonunion
o commonly result from technical error or use of IM fixation
o atrophic nonunions can be treated with 3.5 mm plates and autogenous cancellous bone grafting
 Malunion
o direct correlation between restoration of radial bow and functional outcome
 Neurovascular injury
o uncommon except
 PIN injury with Monteggia fxs and Henry (volar) approach to middle and upper third radial
diaphysis
 Type III open fxs
o observe for three months to see if nerve function returns
 explore if no return of function after 3 months
 Refracture
o increased risk with
 removing plate too early
 large plates (4.5 mm)
 comminuted fx
 persistent radiographic lucency
o do not remove plates before 15 mos.
o wear functional forearm brace for 6 weeks and protect activity for 3 mos. after plate removal

3. Radioulnar Synostosis
Introduction
 Bony bridge which develops between radius and ulna secondary to a specific event
o must differentiate from congenital radioulnar synostosis
 Epidemiology
o incidence : 3% to 9%
o risk factors
 trauma related
 Monteggia fracture
 both bone forearm fractures at the same level
 open fracture,
 significant soft-tissue lesion
 comminuted fracture
 high energy fracture
- 194 -
By Dr, AbdulRahman AbdulNasser Upper Extremity | Forearm
 associated head trauma
 bone fragments on the interosseous membrane
 treatment related
 use of one incision for both radius and ulna
 delayed surgery > 2 weeks
 screws that penetrate interosseous membrane
 bone grafting into interosseous membrane
 prolonged immobilization

Anatomy
 Forearm anatomy

Presentation
 History : previous trauma or surgery in forearm
 Symptoms
o pain with incomplete synostosis
o no pain with complete synostosis
 Physical exam : pronation and supination blocked both actively and passively

Imaging
 Radiographs
o recommended views : AP and lateral of forearm, elbow, and wrist
o findings : bony bridge between radius and ulna

Treatment
 Operative
o surgical resection of synostosis, irradiation, and indomethacin
 indications
 mature post-traumatic synostosis that impairs function
 excision indicated at 4-6 months
 timing is controversial
 excision too early can lead to recurrence
 excision too late can lead to surrounding joint contractures
 results : results of resection are poor except for midshaft synostosis
o proximal radial excision
 indications
 reserved for patients who have a proximal radioulnar synostosis that is too extensive to
allow a safe resection, involves the articular surface, and is associated with an
anatomic deformity.
 results
 can provide forearm rotation
 associated with radioulnar and/or elbow instability

- 195 -
OrthoBullets2017 Upper Extremity | Forearm

4. Distal Radius Fractures


Introduction
 Most common orthopaedic injury with a bimodal distribution
o younger patients - high energy
o older patients - low energy / falls
 50% intra-articular
 Associated injuries
o DRUJ injuries must be evaluated
o radial styloid fx - indication of higher energy
o soft tissue injuries in 70%
 TFCC injury 40%
 scapholunate ligament injury 30%
 lunotriquetral ligament injury 15%
 Osteoporosis
o high incidence of distal radius fractures in women >50
o distal radius fractures are a predictor of subsequent fractures
 DEXA scan is recommended in woman with a distal radius fracture

Classification
 Fernandez: based on mechanism of injury

- 196 -
By Dr, AbdulRahman AbdulNasser Upper Extremity | Forearm

Fernandez classification:
This is a mechanism-based classification system.
Type I: Metaphyseal bending fracture with the inherent problems of loss of palmar tilt and
radial shortening relative to the ulna (DRUJ injury)
Type II: Shearing fracture requiring reduction and often buttressing of the articular segment
Type III: Compression of the articular surface without the characteristic fragmentation; also the
potential for significant interosseous ligament injury
Type IV: Avulsion fracture or radiocarpal fracture dislocation
Type V: Combined injury with significant soft tissue involvement owing to high-energy injury
From Koval, Kenneth J.; Zuckerman, Joseph D. Handbook of Fractures, 3rd Edition

 Frykman: based on joint involvement (radiocarpal and/or radioulnar) +/- ulnar styloid fx

Frykman Classification
Distal Ulna Fracture
Distal Radius Fracture
Absent present
Extraarticular I II
Intraarticular involving radiocarpal joint III IV
Intraarticular involving distal radioulnar joint
V VI
(DRUJ)
Intraarticular involving radiocarpal and DRUJ VII VIII
From Koval, Kenneth J.; Zuckerman, Joseph D. Handbook of Fractures, 3rd Edition

- 197 -
OrthoBullets2017 Upper Extremity | Forearm

 Melone: divides intra-articular fxs into 4 types based on displacement

 AO: comprehensive but cumbersome

- 198 -
By Dr, AbdulRahman AbdulNasser Upper Extremity | Forearm
 Eponyms: see table for list of commonly used eponyms
Eponyms
Die-punch fxs A depressed fracture of the lunate fossa of the articular surface of the distal radius
Barton's fx Fx dislocation of radiocarpal joint with intra-articular fx involving the volar or dorsal lip
(volar Barton or dorsal Barton fx)
Chauffer's fx Radial styloid fx
Colles' fx Low energy, dorsally displaced, extra-articular fx
Smith's fx Low energy, volar displaced, extra-articular fx

Die-punch fxs Barton fx

Colles’ Fx Smith Fx Chauffeur Fx


Imaging
 Radiographs

View Measurement Normal Acceptable criteria

AP Radial height 13 mm
<5 mm shortening

Radial inclination 23 degrees


change <5°
Articular stepoff congruous <2 mm stepoff
dorsal angulation <5° or within 20° of contralateral distal
LAT Volar tilt 11 degrees
radius

- 199 -
OrthoBullets2017 Upper Extremity | Forearm

 CT scans : important to evaluate intra-articular involvement and for surgical planning


 MRI useful to evaluate for soft tissue injury
o TFCC injuries
o scapholunate ligament injuries (DISI)
o lunotriquetral injuries (VISI)

Treatment
 Successful outcomes correlate with
o accuracy of articular reduction
o restoration of anatomic relationships
o early efforts to regain motion of wrist and fingers
 Nonoperative
o closed reduction and cast immobilization
 indications
 extra-articular
 <5mm radial shortening
 dorsal angulation <5° or within 20° of contralateral distal radius
 technique (see below)
 Operative
o surgical fixation (CRPP, External Fixation, ORIF)
 indications: radiographic findings indicating instability (pre-reduction radiographs best
predictor of stability)
 displaced intra-articular fx
 volar or dorsal comminution
 articular margins fxs
 severe osteoporosis
 dorsal angulation >5° or >20° of contralateral distal radius
 >5mm radial shortening
 comminuted and displaced extra-articular fxs (Smith's fx)
 progressive loss of volar tilt and loss of radial length following closed reduction and
casting
 associated ulnar styloid fractures do not require fixation
Closed reduction and cast immobilization
 Indications : most extra-articular fxs

- 200 -
By Dr, AbdulRahman AbdulNasser Upper Extremity | Forearm
 Technique
o rehabilitation
 no significant benefit of physical therapy over home exercises for simple distal radius
fractures treated with cast immobilization
 Outcomes : repeat closed reductions have 50% less than satisfactory results
 Complications
o acute carpal tunnel syndrome : (see complications below)
o EPL rupture : (see complications below)
Percutaneous Pinning
 Indications
o can maintain sagittal length/alignment in extra-articular fxs with stable volar cortex
o cannot maintain length/alignment when unstable or comminuted volar cortex
 Techniques
o Kapandji intrafocal technique
o Rayhack technique with arthroscopically assisted reduction
 Outcomes : 82-90% good results if used appropriately
External Fixation
 Indications
o alone cannot reliably restore 10 degree palmar tilt
 therefore usually combined with percutaneous pinning technique or plate fixation
 Technical considerations
o relies on ligamentotaxis to maintain reduction
o place radial shaft pins under direct visualization to avoid injury to superficial radial nerve
o nonspanning ex-fix can be useful if large articular fragment
o avoid overdistraction (carpal distraction < 5mm in neutral position) and excessive volar flexion
and ulnar deviation
o limit duration to 8 weeks and perform aggressive OT to maintain digital ROM
 Outcomes : important adjunct with 80-90% good/excellent results
 Complications
o malunion/nonunion
o stiffness and decreased grip strength
o pin complications (infections, fx through pin site, skin difficulties)
 pin site care comprising daily showers and dry dressings recommended
o neurologic (iatrogenic injury to radial sensory nerve, median neuropathy, RSD)
ORIF
 Indications
o significant articular displacement (>2mm)
o dorsal and volar Barton fxs
o volar comminution
o metaphyseal-diaphyseal extension
o associated distal ulnar shaft fxs
o die-punch fxs
 Technique
o volar plating
 volar plating preferred over dorsal plating
 volar plating associated with irritation of both flexor and extensor tendons

- 201 -
OrthoBullets2017 Upper Extremity | Forearm
 rupture of FPL is most common with volar plates
 associated with plate placement distal to watershed area, the most volar margin of the
radius closest to the flexor tendons
 new volar locking plates offer improved support to subchondral bone
o dorsal plating
 dorsal plating historically associated with extensor tendon irritation and rupture
 dorsal approach indicated for displaced intra-articular distal radius fracture with dorsal
comminution
o other technical considerations
 can combine with external fixation and PCP
 bone grafting if complex and comminuted
 study showed improved results with arthroscopically assisted reduction
 volar lunate facet fragments may require fragment specific fixation to prevent early post-
operative failure
Complications
 Median nerve neuropathy (CTS)
o most frequent neurologic complication
o 1-12% in low energy fxs and 30% in high energy fxs
o prevent by avoiding immobilization in excessive wrist flexion and ulnar deviation (Cotton-Loder
Position)
o treat with acute carpal tunnel release for:
 progressive paresthesias, weakness in thumb opposition
 paresthesias do not respond to reduction and last > 24-48 hours
 Ulnar nerve neuropathy : seen with DRUJ injuries
 EPL rupture
o nondisplaced distal radial fractures have a higher rate of spontaneous rupture of the extensor
pollicis longus tendon
 extensor mechanism is felt to impinge on the tendon following a nondisplaced fracture and
causes either a mechanical attrition of the tendon or a local area of ischemia in the tendon.
o treat with transfer of extensor indicis proprius to EPL
 Radiocarpal arthrosis (2-30%)
o 90% young adults will develop symptomatic arthrosis if articular stepoff > 1-2 mm
o may be nonsymptomatic
 Malunion and Nonunion
o Intra-articular malunion : treat with revision at > 6 weeks
o Extra-articular angulation malunion
 treat with opening wedge osteotomy with ORIF and bone grafting
o Radial shortening malunion
 radial shortening associated with greatest loss of wrist function and degenerative changes in
extra-articular fxs
 treat with ulnar shortening
 ECU or EDM entrapment : entrapment in DRUJ injury
 Compartment syndrome
 RSD/CRPS
o AAOS 2010 clinical practice guidelines recommend vitamin C supplementation to prevent
incidence of RSD postoperatively
- 202 -
By Dr, AbdulRahman AbdulNasser Upper Extremity | Forearm

5. Distal Radial Ulnar Joint (DRUJ) Injuries


Introduction
 Frequently occur with distal radius fractures but must be considered independently
o common cause of pain and limited ROM after distal radius fractures
o often underappreciated and ignored
 Associated conditions
o ulnar styloid and distal ulna fractures
o TFCC tears
o ulnar impaction syndrome
o Essex-Lopresti injuries
o Galeazzi fractures
 Prognosis
o primary method to prevent disability related to DRUJ injuries is anatomic reduction of the distal
radius which often results in an anatomically-reduced DRUJ

Anatomy
 DRUJ
o arthrology
 articulation occurs between the ulnar head and sigmoid notch (a shallow concavity found
along ulnar border of distal radius)
 most stable in supination
o primary stabilizers
 volar and dorsal radioulnar ligaments
 TFCC
 TFCC attaches to the fovea at the base of the ulnar styloid
 components include
 central articular disc

- 203 -
OrthoBullets2017 Upper Extremity | Forearm
 meniscal homologue
 volar
and dorsal radioulnar ligaments
 ulnolunate and ulnotriquetral ligament origins
 floor of the ECU tendon sheath
o biomechanics : joint motion includes both rotation and translation

Presentation
 Symptoms
o pain and instability with acute DRUJ dislocation
o dorsal wrist pain and limited pronosupination with post-traumatic arthritis
 Physical exam
o post-traumatic arthritis
 snapping and crepitus
 proximal rotation of the forearm with compression of the ulna against the radius elicits pain
 decreased grip strength

Imaging
 Radiographs
o AP shows widening of the DRUJ
o lateral shows dorsal displacement : instability of the DRUJ is present when the ulnar head is
subluxed from the sigmoid notch by its full width with the arm in neutral rotation
 Dynamic CT
o useful in the diagnosis of subtle chronic DRUJ instability
o sequential CT scans are performed with the forearm in neutral and full supination and pronation
o >50% translation compared to the contralateral side is abnormal
 MRI : useful in the identification of TFCC injuries

Treatment
 Nonoperative
o closed reduction, immobilization
 indications : DRUJ instability resulting from purely ligamentous
injury
 techniques
 closed reduction and immobilization in a position of stability ‎III:31 MRI showig TFCC tear
for 4 weeks
 dorsal instability is stable with the forearm in supination
 volar instability is stable in pronation

- 204 -
By Dr, AbdulRahman AbdulNasser Upper Extremity | Forearm
 outcomes : interposition of ECU may impede closed reduction
 Operative
o DRUJ pinning, radioulnar ligament repair
 indications : highly unstable DRUJ
 techniques : pinning across joint with 0.062-inch K-wires

Ulnar Styloid Fractures


 Reflects high degree of initial fracture displacement
 Fractures through base often associated with TFCC rupture and instability
 Painful hypertrophic nonunions can occur in the absence of instability
 Treatment
o nonoperative
 cast immobilization
 indications : nondisplaced fractures proximal to the ulnar styloid
o operative
 ORIF, symptomatic fragment excision
 indications
 displaced fractures through the base with associated instability
 sigmoid notch fractures
 Galeazzi fracture patterns
 TFCC avulsions in the face of an unstable DRUJ
 techniques : preserve ulnar attachments of TFCC with fragment excision

TFCC Tears
 Mechanism of injury
o wrist extension, forearm pronation
 in pronation, volar ligaments prevent dorsal subluxation
 in supination, dorsal ligaments prevent volar subluxation
 Classification
o type I - traumatic
o type II - degenerative (ulnocarpal impaction)
 IIA - TFCC thinning ‎III:32 Darrach procedure
 IIB - IIA + lunate and/or ulnar chondromalacia
 IIC - IIB + TFCC perforation
 IID - IIC + LT ligament disruption
 IIE - IID + ulnocarpal and DRUJ arthritis
 Treatment
o nonoperative
 immobilization, NSAIDS
 indications : all acute traumatic TFCC tears
o operative
 arthroscopic vs. open debridement and/or repair
 indications
 failure of nonoperative management
‎III:33Sauve-Kapandji procedure
 persistent symptoms
 techniques
 type I injuries

- 205 -
OrthoBullets2017 Upper Extremity | Forearm
 arthroscopic vs. open debridement and/or repair
 type II injuries
 TFCC pathology treated with arthroscopic or open debridement
 ulnocarpal impaction treated with ulnar shortening osteotomy (in the absence of
DRUJ arthrosis) or wafer resection of the ulnar head
Ulnar Impaction Syndrome
 Radial shortening leads to positive ulnar variance and altered mechanics
 Sequelae includes
o lunate chondromalacia
o degenerative TFCC tears
 Operative treatment
o TFCC debridement
o radial osteotomy
o ulnar shortening
o distal ulnar resection (Wafer procedure)
 preserve ulnar attachment of TFCC

Essex-Lopresti Injuries
 Radial head fracture with an interosseous membrane injury extending
to DRUJ
o unstable relationship between ulna and radius
o leads to proximal migration of the radius
o results in secondary DRUJ pathology and ulnocarpal abutment
 Treatment
o treat bony pathology (radial head or shaft)
o pin DRUJ for 6 weeks in neutral to facilitate ligamentous healing
o if pinning fails (or the initial injury is missed) radial head
replacement may be required
Galeazzi Fractures
 Distal one-third fracture of the radius and a DRUJ injury
 ECU entrapment may cause DRUJ to be irreducible
 Treatment
o nonoperative
 splint in supination
 indications : rarely indicated for stable injuries
o operative
 radial ORIF and DRUJ pinning
 indications : often required to achieve a stable reduction

Complications ‎III:34 Essex-Lopresti Injuries


 DRUJ arthrosis
o treatment
 resection arthroplasty (resect distal ulna)
 matched resection vs. Darrach
 Darrach procedure
 reserved for low-demand, elderly patients
- 206 -
By Dr, AbdulRahman AbdulNasser Upper Extremity | Forearm
 an unstable, painful proximal ulna stump may result
 hemiresection or interposition arthroplasty
 ulnar insertion of TFCC is preserved
 radioulnar impingement is prevented by soft tissue interposition
 Sauve-Kapandji procedure
 DRUJ fusion with creation of a proximal ulnar neck pseudoarthrosis
 ulnar head prosthetic replacement
 creation of a one-bone forearm
 ultimate salvage procedure that eliminates forearm rotation

6. Galeazzi Fractures
Introduction
 Definition
o distal 1/3 radius shaft fx AND
o associated distal radioulnar joint (DRUJ) injury
 Incidence of DRUJ instability
o if radial fracture is <7.5 cm from articular surface : unstable in 55%
o if radial fracture is >7.5 cm from articular surface : unstable in 6%
 Mechanism
o direct wrist trauma : typically dorsolateral aspect
o fall onto outstretched hand with forearm in pronation

Anatomy
 DRUJ
o sigmoid notch
 found along ulnar border of distal radius
 is a shallow concavity for the articulating ulnar head
o volar and dorsal radioulnar ligaments
 function as the primary stabilizers of the DRUJ
o most stable in supination

Classification
 OTA classification of radius/ulna
o included under subgroups and qualifications

OTA classification of radius/ulna


22-A2.3 Radius/ulna, diaphyseal, simple fracture of radius with dislocation of DRUJ
Radius/ulna, diaphyseal, simple fracture of both bones (distal zone radius) with
22-A3.3
dislocation of DRUJ
22-B2.3 radius/ulna, diaphyseal, wedge fracture of radius with dislocation of DRUJ

22-B3.3 radius/ulna, diaphyseal, wedge of both bones with dislocation of DRUJ

- 207 -
OrthoBullets2017 Upper Extremity | Forearm

Presentation
 Symptoms : pain, swelling, deformity
 Physical exam
o point tenderness over fracture site
o ROM : test forearm supination and pronation for instability
o DRUJ stress : causes wrist or midline forearm pain

Imaging
 Radiographs
o recommended views
 AP and lateral views of forearm, elbow, and wrist
o findings : signs of DRUJ injury
 ulnar styloid fx
 widening of joint on AP view
 dorsal or volar displacement on lateral view
 radial shortening (≥5mm)

Treatment
 Operative
o ORIF of radius with reduction and stabilization of DRUJ
 indications
 all cases, as anatomic reduction of DRUJ is required
 acute operative treatment far superior to late reconstruction

Surgical Techniques
 ORIF of radius
o approach : volar (Henry) approach to radius
o plate fixation
 perform anatomic plate fixation of radial shaft
 radial bow must be restored/maintained
 Reduction & stabilization of DRUJ
o approach : dorsal capsulotomy
o reduction technique
 immobilization in supination (6 weeks)
 indicated if DRUJ stable following ORIF of radius
 percutaneous pin fixation

- 208 -
By Dr, AbdulRahman AbdulNasser Upper Extremity | Forearm
 indicated if DRUJ reducible but unstable following ORIF of radius
 cross-pin ulna to radius : leave pins in place for 4-6 weeks
 open surgical reduction
 indicated if reduction is blocked
 suspect interposition of ECU tendon
 open reduction internal fixation
 indicated if a large ulnar styloid fragment exists
 fix styloid and immobilize in supination

Complications
 Compartment syndrome
o increased risk with
 high energy crush injury
 open fractures
 vascular injuries or coagulopathies
o diagnosis
 pain with passive stretch is most sensitive
 Neurovascular injury : uncommon except type III open fractures
 Refracture
o usually occurs following plate removal
o increased risk with
 removing plate too early
 large plates (4.5mm)
 comminuted fractures
 persistent radiographic lucency
o prevention
 do not remove plates before 18 months after insertion
 amount of time needed for complete primary bone healing
 Nonunion
 Malunion
 DRUJ subluxation : displaced by gravity, pronator quadratus, or brachioradialis

Collected By : Dr AbdulRahman
AbdulNasser
drxabdulrahman@gmail.com
In June 2017

- 209 -
OrthoBullets2017 Hand Trauma | Forearm

ORTHO BULLETS

IV. Hand Trauma

- 210 -
By Dr, AbdulRahman AbdulNasser Hand Trauma | Tendon Injuries

A. Tendon Injuries

1. Flexor Tendon Injuries


Introduction
 Commonly result from volar lacerations and may have concomitant neurovascular injury
 Classified by the zone of injury (see table below)
o basic concepts in repair are similar for different zones
o location of laceration directly affects healing potential

Phases of Tendon Healing


Phase Days Histology Strength
Inflammatory 0-5 cellular proliferation none
fibroblastic proliferation with disorganized
Fibroblastic 5-28 increasing
collagen
will tolerate controlled active
Remodeling >28 linear collagen organization
motion

Anatomy
 Muscles
o flexor digitorum profundus (FDP)
 functions as a flexor of the DIP joint
 assists with PIP and MCP flexion
 shares a common muscle belly in the forearm
o flexor digitorum superficialis (FDS)
 functions as a flexor of the PIP joint
 assists with MCP flexion
 individual muscle bellies exist in the forearm
 FDS to the small finger is absent in 25% of people
o flexor pollicis longus (FPL)
 located within the carpal tunnel as the most radial structure
o flexor carpi radialis (FCR) ‎IV:1 Campers chiasm
 primary wrist flexor
 inserts on the base of the second metacarpal

 closest flexor tendon to the median nerve


o flexor carpi ulnaris (FCU)
 primary wrist flexor
 inserts on the pisiform, hook of hamate, and the base of the 5th metacarpal
 Blood supply
o 2 sources exist
 diffusion through synovial sheaths
 occurs when flexor tendons are located within a sheath
 it is the more important source distal to the MCP joint
 direct vascular supply
 nourishes flexor tendons located outside of synovial sheaths
- 211 -
OrthoBullets2017 Hand Trauma | Tendon Injuries
 Campers chiasm
o located at the level of the proximal phalanx where FDP splits FDS
 Pulley system
o digits 1-4 contain
 5 annular pulleys (A1 to A5)
 3 cruciate pulleys (C1 to C3)
 A2 and A4 are the most important pulleys to prevent flexor tendon bowstringing
o thumb contains
 2 annular pulley
 interposed oblique pulley (most important)

Tendon sheath Blood supply of flexor tendon

Pulley system of thumb Pulley system of flexor lateral four digits

- 212 -
By Dr, AbdulRahman AbdulNasser Hand Trauma | Tendon Injuries
Classification

Zone Definition Introduction Treatment


I Distal to FDS
insertion Jersey finger
II FDS insertion Zone is unique in that FDP and Direct repair of both tendons followed by early ROM
to distal FDS in same tendon sheath (both (Duran, Kleinert). Be sure to preserve A2 and A4
palmar crease injured within the flexor pulley. This zone historically had very poor results
retinaculum) but results have improved due to advances in
postoperative motion protocols
III Palm Often associated with Direct tendon repair. Good results from direct repair
neurovascular injury which carries can be expected due to absence of retinacular
a worse prognosis structures (if no neurovascular injury)
IV Carpal tunnel Often complicated by postoperative Direct tendon repair. Transverse carpal ligament
adhesions due to close quarters and should be repaired in a lengthened fashion
synovial sheath of the carpal tunnel
V Wrist to Often associated with Direct tendon repair
forearm neurovascular injury which carries
a worse prognosis
Thumb TI, TII, TIII Outcomes different than fingers. Direct end-to-end repair of FPL is advocated. Try to
Early motion protocols do not avoid Zone III to avoid injury to the recurrent motor
improve long-term results and there branch of the median nerve. Oblique pulley is more
is a higher re-rupture rate than important than the A1 pulley; however both may be
flexor tendon repair in fingers incised if necessary. Attempt to leave one pulley
intact to prevent bowstringing

Presentation
 Symptoms
o loss of active flexion strength or motion of the involved digit(s)
 Physical exam
o inspection
 observe resting posture of the hand and assess the digital cascade
 evidence of malalignment or malrotation may indicate an underlying fracture
 assess skin integrity to help localize potential sites of tendon injury
 look for evidence of traumatic arthrotomy
o range of motion
 passive wrist flexion and extension allows for assessment of the tenodesis effect
 normally wrist extension causes passive flexion of the digits at the MCP, PIP, and DIP
joints
 maintenance of extension at the PIP or DIP joints with wrist extension indicates flexor
tendon discontinuity
 active PIP and DIP flexion is tested in isolation for each digit
o neurovascular exam
 important given the close proximity of flexor tendons to the digital neurovascular bundles

- 213 -
OrthoBullets2017 Hand Trauma | Tendon Injuries
Treatment
 Nonoperative
o wound care and early range of motion
 indications
 partial lacerations < 60% of tendon width
 outcomes
 may be associated with gap formation or triggering
 Operative
o flexor tendon repair and controlled mobilization
 indications
 lacerations > 60% of tendon width
 outcomes
 depends on zone of injury
o flexor tendon reconstruction and intensive postoperative rehabilitation
 indications
 failed primary repair
 chronic untreated injuries
 outcomes
 subsequent tenolysis is required more than 50% of the time
o FDS4 transfer to thumb
 single stage procedure
 indication
 chronic FPL rupture

Surgical Technique
 Flexor Tendon Repair of Complete Lacerations
o approach
 incisions should always cross flexion creases transversely or obliquely to avoid contractures
(never longitudinal)
o timing of repair
 perform repair within three weeks of injury (2 weeks ideal)
 waiting longer leads to difficulty due to tendon retraction
o technique
 # of suture strands that cross the repair site is more important than the number of grasping

loops
 linear relationship between strength of repair and # of sutures crossing repair
 4-6 strands provide adequate strength for early active motion
 high-caliber suture material increases strength and stiffness and decreases gap formation
 locking-loops decrease gap formation
 ideal suture purchase is 10mm from cut edge
 core sutures placed dorsally are stronger
 meticulous atraumatic tendon handling minimizes adhesions
 circumferential epitendinous suture
 improves tendon gliding
 improves strength of repair (adds 20% to tensile strength)
 allows for less gap formation (first step in repair failure)

- 214 -
By Dr, AbdulRahman AbdulNasser Hand Trauma | Tendon Injuries
 simple running suture is recommended
 sheath repair is controversial
 theoretically improves tendon nutrition through synovial pathway
 clinical studies show no difference with or without sheath repair
 most surgeons will repair if it is easy to do
 pulley management
 critical to preserve A2 and A4 pulleys in digits and oblique pulley in thumb
 FDS repair
 in zone 2 injuries, repair of one slip alone improves gliding when compared to repair of
both slips
o outcomes
 repair failure
 tendon repairs are weakest between postoperative day 6 and 12
 repair usually fails at suture knots
 Flexor Tendon Repair of Partial Lacerations
o indications
 >75% laceration
 ≥50-60% laceration with triggering
 epitendinous suture at the laceration site is sufficient
 no benefit of adding core suture
 Wide-Awake Flexor Tendon Repair
o performed under tumescent local anesthesia using lidocaine with epinephrine
 dosing
 usually epinephrine 1:100,000 and 7mg/kg lidocaine
 from 1:400,000 to 1:1000 is safe
 if <50cc is needed
 1% lidocaine with 1:100,000 epi for a 70kg person
 if 50-100cc is needed
 dilute with saline (50:50) to get 0.5% lidocaine, 1:200,000 epi
 if 100-200cc is needed for large fields (tendon transfer, spaghetti wrist)
 dilute with 150cc saline to get 0.25% lidocaine and 1:400,000 epi
 for longer surgery >2h
 add 10cc of 0.5% bupivacaine with 1:200,000 epi
 location
 proximal and middle phalanges, use 2ml
 distal phalanx, use 1ml
 palm, use 10-15ml
o no tourniquet, no sedation
o 4 advantages
 allows intraoperative assessment for repair gaps by getting awake patient to actively flex digit
 reduces need for postop tenolysis by allowing intraoperative assessment of whether repair
will fit through pulleys
 allows on-the-spot debulking of bunched repairs
 allows division of A4 pulley and venting (partial division) of A2 pulleys
 allows repair of tendons inside tendon sheaths as patients can demonstrate that the inside of
the sheath has not been inadvertently caught

- 215 -
OrthoBullets2017 Hand Trauma | Tendon Injuries
 facilitates postop early active motion
 immobilize for 3 days
 begin active midrange motion after day 3 (form a partial fist with 45 degree flexion at
MP, PIP and DIP joints, or "half a fist 45/45/45 regime")
 Reconstruction Technique
o requirements
 supple skin
 sensate digit
 adequate vascularity
 full passive range of motion of adjacent joints
o techniques of reconstruction involving silicone rods
 Hunter-Salisbury two-stage procedure
 Stage I - silicone rod is placed to create a favorable tendon bed
 Stage II (3-4 months) - retrieve SR and pass a tendon graft through the mesothelium lined
pseudosheath
 only perform a single-stage reconstruction if the flexor sheath is pristine and the digit has
full ROM
 pulvertaft weave proximally and end-to-end tenorrhaphy distally
 Paneva-Holevich two-stage technique
 Stage I - SR is placed in the flexor sheath, pulleys are reconstructed (as needed), and a
loop between the proximal stumps of FDS and FDP is created in the palm
 Stage II - SR is retrieved, FDS is cut proximally and reflected distally through
pseudosheath and attached directly to FDP stump/or secured with button
 advantages
 graft (FDS) size is known at the time of silicone rod selection
 less graft diameter-rod diameter mismatch
 FDS graft is intrasynovial (fewer adhesions than extrasynovial grafts)
 only relying on 1 tenorrhaphy site (distal or proximal) to heal at any one time (vs
Hunter technique where 2 tennoprhaphy sites are healing simultaneously)
 disadvantage
 graft tensioning is at the distal end during stage II
 the proximal end has already healed after stage I
o graft choices
 palmaris longus (absent in 15% of population)
 most common
 plantaris (absent in 19%)
 indicated if longer graft is needed
 long toe extensor
o pulley reconstruction
 one pulley should be reconstructed proximal and distal to each joint
 methods include belt loop method and FDS tail method
 Tenolysis
o indications
 localized tendon adhesions with minimal to no joint contracture and full passive digital
motion
 may be required if a discrepancy between active and passive motion exists after therapy

- 216 -
By Dr, AbdulRahman AbdulNasser Hand Trauma | Tendon Injuries
o timing of procedure
 wait for soft tissue stabilization (> 3 months) and full passive motion of all joints
o technique
 careful technique to preserve A2 and A4 pulleys
o postoperative care
 follow with extensive therapy

Postoperative Rehabilitation
 Postoperative controlled mobilization has been the major reason for improved results with tendon
repair
o especially in zone II
o leads to improved tendon healing biology
o limits restrictive adhesions and leads to increased tendon excursion
 Early active motion protocols
o moderate force and potentially high excursion
o dorsal blocking splint limiting wrist extension
o perform “place and hold” exercises with digits
 Early passive motion protocols
o Duran protocol
 low force and low excursion
 active finger extension with patient-assisted passive finger flexion
o Kleinert protocol
 low force and low excursion
 active finger extension, dynamic splint-assisted passive finger flexion
o Mayo synergistic splint
 low force and high tendon excursion
 adds active wrist motion which increases flexor tendon excursion the most
 Immobilize children and noncompliant patients
o Children should be immobilized following repair
o Casts or splints are applied with the wrist and MCP joints positioned in flexion and the IP joints
in extension
Complications
 Tendon adhesions : most common complication following flexor tendon repair
 Rerupture
o 15-25% rerupture rate
o treatment
 if <1cm of scar is present, resect the scar and perform primary repair
 if >1cm of scar is present, perform tendon graft
 if the sheath is intact and allows passage of a pediatric urethral catheter or vascular
dilator, perform primary tendon grafting
 if the sheath is collapsed, place Hunter rod and perform staged grafting
 Joint contracture : rates as high as 17%
 Swan-neck deformity
 Trigger finger
 Lumbrical plus finger
 Quadrigia

- 217 -
OrthoBullets2017 Hand Trauma | Tendon Injuries

2. Jersey Finger
Introduction
 Refers to an avulsion injury of FDP from insertion at base
of distal phalanx
o a Zone I flexor tendon injury
 Epidemiology
o ring finger involved in 75% of cases
 during grip ring fingertip is 5 mm more prominent
than other digits in ~90% of patients
 therefore ring finger exposed to greater average force than other fingers during pull-away
 Pathophysiology
o FDP muscle belly in maximal contraction during forceful DIP extension

Anatomy
 Muscles
o Flexor Digitorum Profundus (ulnar n. and AIN n.)
 Flexor zones : zone I extends from insertion of FDS distally

Classification
Leddy and Packer classification
(based on level of tendon retraction and presence of fracture)
Type Description Treatment
Type I FDP tendon retracted to palm. Leads to disruption of Prompt surgical treatment within 7 to 10
the vascular supply days
Type II FDP retracts to level of PIP joint Attempt to repair within several weeks for
opitmal outcome
Type III Large avulsion fracture limits retraction to the level of Attempt to repair within several weeks for
the DIP joint opitmal outcome
Type IV Osseous fragment and simultaneous avulsion of the If tendon separated from fracture
tendon from the fracture fragment ("Double avulsion” fragment, first fix fracture via ORIF then
with subsequent retraction of the tendon usually into reattach tendon as for Type I/II injuries
palm)
Ruptured tendon with bone avulsion with bony
Type V comminution of the remaining distal phalanx (Va,
extraarticular; Vb, intra-articular)

Presentation
 Physical exam
o pain and tenderness over volar distal finger
o finger lies in slight extension relative to other fingers in resting position
o no active flexion of DIP
o may be able to palpate flexor tendon retracted proximally along flexor sheath

- 218 -
By Dr, AbdulRahman AbdulNasser Hand Trauma | Tendon Injuries
Imaging
 Radiograhs
o may see avulsion fragement

Treatment
 Operative
o direct tendon repair or tendon reinsertion with dorsal button
 indications
 acute injury (< 3 weeks)
 technique
 advancement of > 1 cm carries risk of a DIP flexion contracture or quadrigia
 postoperative rehab should include either
 early patient assisted passive ROM (Duran) or
 dynamic splint-assisted passive ROM (Kleinert)
o ORIF fracture fragment
 indications
 types III and IV (for type IV then repair as for Type
I/II injuries)
 techniques
 with K-wire, mini frag screw or pull out wire
 examine for symmetric cascade once fixation
completed
o two stage flexor tendon grafting
 indications
 chronic injury (> 3 months) in patient with full
PROM of the DIP joint
o DIP arthrodesis
 indicated as salvage procedure in chronic injury (> 3
months) with chronic stiffness
Complications
 Quadrigia
o advancement of > 1 cm carries risk of a DIP flexion contracture or quadrigia

- 219 -
OrthoBullets2017 Hand Trauma | Tendon Injuries

3. Extensor Tendon Injuries


Introduction
 Injury can be caused by laceration, trauma, or overuse
 Epidemiology
o most commonly injured digit is the long finger
o zone VI is the most frequently injured zone
 Mechanism
o Zone I
 forced flexion of extended DIP joint
o Zone II
 dorsal laceration or crush injury
o Zone V
 commonly from "fight bite"
 sagittal band rupture ("flea flicker injury")
 forced extension of flexed digit
 most common in long finger

Classification

Zones of Extensor Tendon Injuries


Zone I • Disruption of terminal extensor tendon distal to or at the DIP joint of the fingers
and IP joint of the thumb (EPL)
• Mallet Finger
Zone II • Disruption of tendon over middle phalanx or proximal phalanx of thumb (EPL)
Zone III • Disruption over the PIP joint of digit (central slip) or MCP joint of thumb (EPL and
EPB
• Boutonniere deformity
Zone IV • Disruption over the proximal phalanx of digit or metacarpal of thumb (EPL and
EPB)
Zone V • Disruption over MCP joint of digit or CMC joint of thumb (EPL and EPB)
•"Fight bite" common
• Sagittal band rupture
Zone VI • Disruption over the metacarpal
• Nerve and vessel injury likely
Zone VII • Disruption at the wrist joint
• Must repair retinaculum to prevent bowstringing
• Tendon repair followed by immobilization with wrist in 40° extension and MCP
joint in 20° flexion for 3-4 weeks
Zone VIII • Disruption at the distal forearm
Zone VIII • Extensor muscle belly
• Usually from penetrating trauma
• Often have associated neurologic injury
• Tendon repair followed by immobilization with elbow in flexion and wrist
in extension

- 220 -
By Dr, AbdulRahman AbdulNasser Hand Trauma | Tendon Injuries
Presentation
 Zone I
o Inability to extend at the DIP joint
 Zone III
o Elson test
 flex the patient's PIP joint over a table 90 degrees and ask them to extend against resistance
 if central slip is intact, DIP will remain supple
 if central slip disrupted, DIP will be rigid
 Zone V
o extensor lag and flexion loss common
o junctura tendinae may allow partial/temporary extension by connecting with intact adjacent
extensor tendons
o sagittal band rupture
 rupture of stronger radial fibers of sagittal band may lead to extensor tendon subluxation
 finger held in flexed position at MCP joint with no active extension

Imaging
 Radiographs
o AP and lateral of digit to verify no bony avulsion (boney mallet)

Treatment
 Nonoperative
o immobilization with early protected motion
 indications
 lacerations < 50% of tendon in all zones if patient can extend digit against resistance
o DIP extension splinting
 indications
 acute (<12 weeks) Zone 1 injury (mallet finger)
 nondisplaced bony mallet
 chronic mallet finger (>12 weeks) if joint supple, congruent
 techniques
 full-time splinting for six weeks
 part-time splinting for four to six weeks
 avoid hyperextension, which may cause skin necrosis
 maintain PIP motion
 outcomes
 noncompliance is a common problem
‎IV:2 Mallet Finger
o PIP extension splinting
 indications
 closed central slip injury (zone III)
 techniques
 full-time splinting for six weeks
 part-time splinting for four to six weeks
 maintain DIP flexion
I‎V:3 Boutonniere deformity
o MCP extension splinting
 indications
 closed zone V sagittal band rupture
- 221 -
OrthoBullets2017 Hand Trauma | Tendon Injuries
 techniques
 full-time splinting for four to six weeks
 Operative
o immediate I&D
 indications
 fight bite to MCP joint
 techniques
 close loosely or in delayed fashion
‎IV:4 Sagittal band rupture
 treat with culture-specific antibiotics, although Eikenella corrodens is a common mouth
organism
o tendon repair
 indications
 laceration > 50% of tendon width in all zones
o fixation of bony avulsion
 indications
 boney mallet finger with P3 volar subluxation
 techniques
 closed reduction and percutaneous pinning through DIP joint
 extension block pinning
 ORIF if it involves >50% of the articular surface
o tendon reconstruction
 indications
 chronic tendon injury or when repair not possible
o central slip reconstruction
 techniques
 tendon graft
 extensor turndown
 lateral band mobilization
 transverse retinacular ligament
 FDS slip
o EIP to EPL tendon transfer
 indications
 chronic EPL rupture

Surgical Techniques
 Tendon Repair
o incision technique
 utilize laceration, when present, and extend incision as needed to gain appropriate exposure
 longitudinal incision may be utilized across joints on the dorsum of digits, unlike the palmar
side
o suture technique
 # of suture strands that cross the repair site is more important than the number of grasping
loops
 in general strength increases with increasing number of sutures crossing the repair site,
thickness of the suture, and locking of the stitch
 4-6 strands provide adequate strength for early active motion

- 222 -
By Dr, AbdulRahman AbdulNasser Hand Trauma | Tendon Injuries
o circumferential epitendinous suture
 Optional for reinforcement
o repair failure
 tendon repairs are weakest between postoperative day 6 and 12
 repair usually fails at knots
 Tendon Reconstruction
o usually done as two stage procedure
 first a silicon tendon implant is placed to create a favorable tendon bed
 wait 3-4 months and then place biologic tendon graft
 only perform single stage reconstruction if flexor sheath is pristine and digit has full ROM
o available grafts include
 palmaris longus (absent in 15% of population)
 most common
 plantaris (absent in 19%)
 indicated if longer graft is needed
 long toe extensor
o pulley reconstruction
 one pulley should be reconstructed proximal and distal to each joint
 methods include belt loop method and FDS tail method
 Tenolysis
o indications
 adhesion formation with loss of finger flexion
 wait for soft tissue stabilization (> 3 months) and full passive motion of all joints
o postoperative
o follow with extensive therapy

Rehabilitation
 Early active short-arc motion (SAM)
o indications
 after zone III central slip repair
o advantages over static immobilization
 increases total arc of motion
 decrease duration of therapy
 increase DIP motion
 creates 4mm of tendon excursion and prevents adhesions.

Complications
 Adhesion formation
o leads to loss of finger flexion
o common in zone IV and VII and older patients
o prevented with early protected ROM and dynamic splinting (zone IV)
o treatment
 extensor tenolysis with early motion indicated after failure of nonoperative management,
usually 3-6 months
 tenolysis contraindicated if done in conjunction with other procedures that require joint
immobilization

- 223 -
OrthoBullets2017 Hand Trauma | Tendon Injuries
 Tendon rupture
o causes include poor suture material or surgical technique, aggressive therapy, and
noncompliance
o incidence
 5%
 most frequently during first 7 to 10 days post-op
o treatment
 early recognition may allow revision repair
 tendon reconstruction for late rupture or rupture with excessive scarring
 Swan neck deformity
o caused by prolonged DIP flexion with dorsal subluxation of lateral bands and PIP joint
hyperextension
o treatment
 Fowler central slip tenotomy
 spiral oblique ligament reconstruction
 Boutonniere deformity (DIP hyperextension)
o caused by central slip disruption and lateral band volar subluxation
o treatment
 dynamic splinting or serial casting for maximal passive motion
 terminal extensor tenotomy, PIP volar plate release

4. Mallet Finger
Introduction
 A finger deformity caused by disruption of the terminal extensor
tendon distal to DIP joint
o the disruption may be bony or tendinous
 Epidemiology
o risk factors
 usually occur in the work environment or during participation
in sports
o demographics
 common in young to middle-aged males and older females
o body location
 most frequently involves long, ring and small fingers of dominant hand
 Pathophysiology
o mechanism of injury
 traumatic impaction blow
 usually caused by a traumatic impaction blow (i.e. sudden forced flexion) to the tip of the
finger in the extended position.
 forces the DIP joint into forced flexion
 dorsal laceration
 a less common mechanism of injury is a sharp or crushing-type laceration to the dorsal
DIP joint
Classification
 Doyle's Classification

- 224 -
By Dr, AbdulRahman AbdulNasser Hand Trauma | Tendon Injuries
Doyle's Classification of Mallet Finger Injuries
Type I • Closed injury with or without small dorsal avulusion fracture
Type II • Open injury (laceration)
Type III • Open injury (deep soft tissue abrasion involving loss skin and tendon substance)
Type IV • Mallet fracture
A = distal phalanx physeal injury (pediatrics)
B = fracture fragment involving 20% to 50% of articular surface (adult)
C = fracture fragment >50% of articular surface (adult)

Presentation
 Symptoms
o primary symptoms
 painful and swollen DIP joint following impaction injury to finger
 often in ball sports
 Physical exam
o inspection
 fingertip rest at ~45° of flexion
o motion
 lack of active DIP extension
 Imaging
 Radiographs
o findings
 usually see bony avulsion of distal phalanx
 may be a ligamentous injury with normal bony anatomy

Treatment
 Nonoperative
o extension splinting of DIP joint for 6-8 weeks
 indications
 acute soft tissue injury (< than 12 weeks)
 nondisplaced bony mallet injury
 technique Bony avulsion Ligamintous injury
 maintain free movement of the PIP joint
 worn for 6-8 weeks
 volar splinting has less complications than dorsal splinting
 avoid hyperextension
 begin progressive flexion exercises at 6 weeks
 Operative
o CRPP vs ORIF
 indications
 absolute indications
 volar subluxation of distal phalanx
 relative indications
 >50% of articular surface involved
 >2mm articular gap
o surgical reconstruction of terminal tendon
 indications

- 225 -
OrthoBullets2017 Hand Trauma | Tendon Injuries
 chronic injury (> 12 weeks) with healthy joint
 outcomes
 tendon reconstruction has a high complication rate (~ 50%)
o DIP arthrodesis
 indications
 painful, stiff, arthritic DIP joint
o Swan neck deformity correction
 indications : Swan neck deformity present

Techniques
 CRPP vs ORIF
o approach
 dorsal midline incision
o fixation
 simple pin fixation
 dorsal blocking pin
 Surgical reconstruction of terminal tendon
o repair
 this may be done with direct repair/tendon advancement, tenodermodesis, or spiral oblique
retinacular ligament reconstruction
 Swan neck deformity correction
o techniques to correct Swan neck deformity include
 lateral band tenodesis
 FDS tenodesis
 Fowler central slip tenotomy
 for deformities of <35° extensor lag
 minimal Swan Neck deformities may correct with treatment of the DIP pathology alone

Complications
 Extensor lag
o a slight residual extensor lag of < 10° may be present at completion of closed treatment
 Swan neck deformities
o occurs due to
 attenuation of volar plate and transverse retinacular ligament at PIP joint
 dorsal subluxation of lateral bands
 resulting PIP hyperextension
 contracture of triangular ligament maintains deformity

- 226 -
By Dr, AbdulRahman AbdulNasser Hand Trauma | Tendon Injuries

5. Sagittal Band Rupture (traumatic extensor tendon dislocation)


Introduction
 Sagittal band (SB) rupture leads to dislocation of the extensor tendon
o also known as "boxer's knuckle"
 Epidemiology
o demographics
 more common in pugilists
 index and middle finger in professionals
 ring and little finger in amateurs
o location
 the middle finger is most commonly involved
 index 14%
 middle 48%
 ring 7%
 little 31%
 the radial SB is more commonly involved
 radial:ulnar = 9:1
 Mechanisms
o traumatic
 forceful resisted flexion or extension
 laceration of extensor hood
 direct blow to MCP joint
o atraumatic
 inflammatory (e.g. rheumatoid arthritis)
 spontaneously during routine activities
 Associated conditions
o MCP joint collateral ligament injuries

Anatomy
 Extensor mechanism comprises
o tendons
 EDC/EIP/EDM
 lumbricals
 interossei
o retinacular system
 sagittal bands
 the sagittal bands are part of a closed cylindrical tube (or girdle) that surrounds the
metacarpal head and MCP along with the palmar plate
 origin
 volar plate and intermetacarpal ligament at the metacarpal neck
 insertion
 extensor mechanism (curving around radial and ulnar side of MCP joint)
 retinacular ligaments
 triangular ligament
 Sagittal band

- 227 -
OrthoBullets2017 Hand Trauma | Tendon Injuries
o function
 the SB is the primary stabilizer of the extensor tendon at the MCP joint
 juncturae tendinum are the secondary stabilizers
 resists ulnar deviation of the tendon, especially during MCP flexion
 prevents tendon bowstringing during MCP joint hyperextension
o biomechanics
 ulnar sagittal band
 partial or complete sectioning does not lead to extensor tendon dislocation
 radial sagittal band
 distal sectioning does not produce extensor tendon instability
 complete sectioning leads to extensor dislocation
 sectioning of 50% of the proximal SB leads to extensor tendon subluxation
 extensor tendon
 instability after sectioning is greater with wrist flexion
 instability after sectioning is greater in the central digits (than border digits)
 the least stable tendon is the middle finger
 the most stable tendon is the little finger
 junctura tendinum stabilize the small finger

Classification
Rayan and Murray Classification of Closed SB Injury
Type Description
Type I SB injury without extensor tendon instability
Type II SB injury with tendon subluxation
Type III SB injury with tendon dislocation

Presentation
 Symptoms
o MCP soreness
 Physical exam
o tendon snapping
o ulnar deviation of the digits at the MCP joint (rheumatoid arthritis)
o inability to initiate extension
o pseudo-triggering
o extensor tendon dislocation into intermetacarpal gully
 most unstable during MCP flexion with wrist flexed
 least unstable during MCP flexion with wrist extended
o provocative test
 pain when extending MCP joint against resistance (with both IP joints extended)

Imaging
 Radiographs
o required views
 hand PA, lateral, oblique

- 228 -
By Dr, AbdulRahman AbdulNasser Hand Trauma | Tendon Injuries
o optional view
 Brewerton view
 AP with dorsal surface of fingers touching the cassette and MCP joints flexed 45deg
 stress view
 to rule out collateral ligament avulsion/injury
o findings
 exclude mechanical/bony pathology limiting extension, or predisposing to sagittal band
rupture
 may show dropped fingers and ulnar deviation in rheumatoid arthritis
 Ultrasound (dynamic)
o indications : when swelling obscures the physical exam
o findings : subluxation of EDC tendon relative to metacarpal head on MCP flexion
 MRI
o indications
 to establish diagnosis of SB disruption (radial or ulnar SB)
 may show underlying etiology e.g. synovitis in rheumatoid arthritis
o views
 axial images at the level of the long MCP
 with MCP joint flexed for maximum EDC tendon displacement
o findings
 poor definition, focal discontinuity and focal thickening in acute injury
 subluxation of extensor tendon in radial direction due ulnar SB defect
 dislocation of extensor tendon into ulnar intermetacarpal gully radial SB defect

Differentials
 MCP joint collateral ligament injury
 EDC tendon rupture
 Trigger finger
 Junctura tendinum disruption
 Congenital sagittal band deficiency
 MCP joint arthritis

Treatment
 Nonoperative
o extension splint for 4-6 weeks
 indications ‎IV:5 extension splint ‎IV:6 direct repair
(Kettlekamp)
 acute injuries (within one week)
 Operative
o direct repair (Kettlekamp)
 indications
 chronic injuries (more than one week) where primary repair is possible
 professional athlete
o extensor centralization procedure
 indications
 chronic injuries (more than one week) where primary repair is NOT possible
 professional athlete

- 229 -
OrthoBullets2017 Hand Trauma | Tendon Injuries
Techniques
 Extensor Centralization Procedures
o various techniques described including
 trapdoor flap
 ulnar based partial thickness capsular flap created
 tendon placed deep to flap
 flap resutured to capsule
 Kilgore tendon slip
‎IV:7 trapdoor flap
 distally based slip of EDC tendon on radial side
 looped around radial collateral ligament
 sutured to itself after tensioning to centralize tendon
 Carroll tendon slip
 distally based slip of EDC tendon on ulnar side
 routed deep to affected tendon and around radial collateral ligamnt
 sutured to itself after tensioning to centralize tendone
 McCoy tendon slip
 proximally based slip of EDC tendon
 looped around lumbrical insertion
 sutured to itself after tensioning to centralize tendon I‎V:8 McCoy tendon slip
 Watson EDC tendon transfer
 distally based slip of EDC tendon slip
 looped under deep transverse metacarpal ligament
 weaved to remaining EDC tendon after tensioning to centralize tendon
 Wheeldon junctural reinforcement
 for a middle finger radial SB rupture, the juncturae tendinum (JT) of the ring finger
is divided close to the ring finger,
 bring JT over the extensor tendon
 attach JT to the torn SB
 fascial strips or free tendon graft

‎IV:9 Carroll tendon slip

- 230 -
By Dr, AbdulRahman AbdulNasser Hand Trauma | Wrist Trauma

B. Wrist Trauma

1. Scaphoid Fracture
Introduction
 Scaphoid is most frequently fractured carpal bone
 Epidemiology
o incidence : accounts for up to 15% of acute wrist injuries
o location
 incidence of fracture by location
 waist -65%
 proximal third - 25%
 distal third - 10%
 distal pole is most common location in kids due to ossification sequence
 Pathoanatomy
o most common mechanism of injury is axial load across hyper-extended and radially deviated
wrist
 common in contact sports
o transverse fracture patterns are considered more stable than vertical or oblique oriented fractures
 Associated conditions
o SNAC (Scaphoid Nonunion Advanced Collapse)
 Prognosis
o incidence of AVN with fracture location
 proximal 5th AVN rate of 100%
 proximal 3rd AVN rate of 33%

Anatomy
 Articular surface
‎IV:10 Blood supply of scaphoid
o > 75% of scaphoid bone is covered by articular cartilage
 Blood supply
o major blood supply is dorsal carpal branch (branch of the radial artery)
 enters scaphoid in a nonarticular ridge on the dorsal surface and supplies proximal 80% of
scaphoid via retrograde blood flow
o minor blood supply from superficial palmar arch (branch of volar radial artery)
 enters distal tubercle and supplies distal 20% of scaphoid
 Motion
o both intrinsic and extrinsic ligaments attach and surround the scaphoid
o the scaphoid flexes with wrist flexion and radial deviation and it extends during wrist extension
and ulnar deviation (same as proximal row)
 Also see Wrist Ligaments and Biomechanics for more detail

Presentation
 Physical exam
o anatomic snuffbox tenderness dorsally
o scaphoid tubercle tenderness volarly
o pain with resisted pronation
‎IV:11 scaphoid tubercle tenderness
- 231 -
OrthoBullets2017 Hand Trauma | Wrist Trauma
Imaging
 Radiographs
o recommended views
 AP and lateral
 scaphoid view
 30 degree wrist extension, 20 degree ulnar deviation
 45° pronation view
o findings
 if radiographs are negative and there is a high clinical suspicion
 should repeat radiographs in 14-21 days
 Bone scan
o effective to diagnose occult fractures at 72 hours
 specificity of 98%, and sensitivity of 100%, PPV 85% to 93% when done at 72 hours
 MRI
o indications
 most sensitive for diagnosis occult fractures < 24 hours
 immediate identification of fractures / ligamentous injuries
 assessment of vascular status of bone (vascularity of proximal pole)
 proximal pole AVN best determined on T1 sequences
 CT scan with 1mm cuts
o less effective than bone scan and MRI to diagnose occult fracture
o can be used to evaluate location of fracture, size of fragments, extent of collapse, and progression
of nonunion or union after surgery

Xray Bone scan scaphoid view MRI CT

Treatment
 Nonoperative
o thumb spica cast immobilization
 indications
 stable nondisplaced fracture (majority of fractures)
 if patient has normal xrays but there is a high level of suspicion can immobilize in thumb
spica and reevaluate in 12 to 21 days
 technique
 start immobilization early (nonunion rates increase with delayed immobilization of > 4
weeks after injury)
 long arm spica vs short arm casting is controversial
 with no consensus
 duration of casting depends on location of fracture
 distal-waist for 3 months

- 232 -
By Dr, AbdulRahman AbdulNasser Hand Trauma | Wrist Trauma
 mid-waist for 4 months
 proximal third for 5 months
 athletes should not return to play until imaging shows a healed fracture
 may opt to augment with pulsed electromagnetic field (studies show beneficial in delayed
union)
 outcomes
 scaphoid fractures with <1mm displacement have union rate of 90%
 Operative
o ORIF vs percutaneous screw fixation
 indications
 in unstable fractures as shown by
 proximal pole fractures
 displacement > 1 mm
 15° scaphoid humpback deformity
 radiolunate angle > 15° (DISI)
 intrascaphoid angle of > 35°
 scaphoid fractures associated with perilunate dislocation I‎V:12 screw fixation of scaphoid
 comminuted fractures
 unstable vertical or oblique fractures
 in non-displaced waist fractures
 to allow decreased time to union, faster return to work/sport, similar total costs
compared to casting
 outcomes
 union rates of 90-95% with operative treatment of scaphoid fractures
 CT scan is helpful for evaluation of union

Technique
 ORIF vs percutaneous screw fixation
o approach
 dorsal approach
 indicated in proximal pole fractures
 care must be taken to preserve the blood supply when entering the dorsal ridge by limiting
exposure to the proximal half of the scaphoid
 percutaneous has higher risk of unrecognized screw penetration of subchondral bone
 volar approach
 indicated in waist and distal pole fractures and fractures with humpback flexion
deformities
 allows exposure of the entire scaphoid
 uses the interval between the FCR and the radial artery
 arthroscopic assisted approach
 has also been described
o fixation
 rigidity is optimized by long screw placed down the central axis of the scaphoid
o radial styloidectomy
 should be performed if there is evidence of impaction osteoarthritis between radial styloid
and scaphoid

- 233 -
OrthoBullets2017 Hand Trauma | Wrist Trauma
Complications
 Scaphoid Nonunion
o treatment
 inlay (Russe) bone graft
 indications
 if minimal deformity and there is no adjacent carpal collapse or excessive flexion
deformity (humpback scaphoid)
 outcomes
 92% union rate
 interposition (Fisk) bone graft
 indications
 if there is adjacent carpal collapse and excessive flexion deformity (humpback
scaphoid)
 technique
 an opening wedge graft that is designed to restore scaphoid length and angulation
 outcomes
 results show 72-95% union rates
 vascular bone graft from radius
 indications
 gaining popularity and a good option for
proximal pole fractures with osteonecrosis
confirmed by MRI
 technique
 1-2 intercompartmental supraretinacular artery (branch of radial artery) is
harvested to provide vascularized graft from dorsal aspect of distal radius
 vascular bone graft from medial femoral condyle
 corticoperiosteal flap that provides highly osteogenic periosteum
 indications
 proximal pole fractures with osteonecrosis
 lack of pancarpal arthritis and collapse
 technique
 utilize the descending genicular artery pedicle (from the superficial femoral artery)
 if DGA is too small, use superomedial genicular artery (from popliteal artery)
 identify and protect MCL (distal to flap)
o SNAC wrist (scaphoid nonunion advanced collapse)

‎IV:13 Humpback deformity

- 234 -
By Dr, AbdulRahman AbdulNasser Hand Trauma | Wrist Trauma

2. Lunate Dislocation (Perilunate dissociation)


Introduction
 High energy injury with poor functional outcomes
 Commonly missed (~25%) on initial presentation
 Categories
o perilunate dislocation
 lunate stays in position while carpus dislocates
 4 types
 transcaphoid-perilunate
 perilunate
 transradial-styloid
 transcaphoid-trans-capitate-perilunar ‎IV:15 sequence of events ‎IV:14 Volar displacement
o lunate dislocation of lunate
 lunate forced volar or dorsal while carpus remains aligned
 Mechanism
o traumatic, high energy
o occurs when wrist extended and ulnarly deviated
 leads to intercarpal supination
 Pathoanatomy
o sequence of events
 scapholunate ligament disrupted -->
 disruption of capitolunate articulation -->
 disruption of lunotriquetral articulation -->
 failure of dorsal radiocarpal ligament --> I‎V:16 Blue line represent
 lunate rotates and dislocates, usually into carpal tunnel greater arc Red line
represent lesser arc
o dislocation can course through
 greater arc
 ligamentous disruptions with associated fractures of the radius, ulnar, or carpal bones
 lesser arc
 purely ligamentous

Anatomy
 Normal wrist anatomy
 Osseous
o proximal row
 scaphoid
 lunate
 triquetrum
 pisiform
o distal row
 trapezium
 trapezoid
 capitate ‎IV:17 Normal wrist anatomy
 hamate
 Ligaments

- 235 -
OrthoBullets2017 Hand Trauma | Wrist Trauma
o interosseous ligaments
 run between the carpal bones
 scapholunate interosseous ligament
 lunotriquetral interosseous ligament
 major stabilizers of the proximal carpal row
o intrinsic ligaments
 ligaments the both originate and insert among the carpal bones
 dorsal intrinsic ligaments
 volar intrinsic ligaments
o extrinsic ligaments
 connect the forearm bones to the carpus
 volar extrinsic carpal ligaments
 dorsal extrinsic carpal ligaments

Classification
Mayfield Classification
Stage I • scapholunate dissociation
Stage II • + lunocapitate disruption
Stage III • + lunotriquetral disruption, "perilunate"
Stage IV • lunate dislocated from lunate fossa (usually volar)
• associated with median nerve compression

Stage I Stage II Stage III Stage IV

Presentation
 Symptoms
o acute wrist swelling and pain
o median nerve symptoms may occur in ~25% of patients
 most common in Mayfield stage IV where the lunate dislocates into the carpal tunnel

Imaging
 Radiographs
o required views
 PA/lateral wrist radiographs
o findings
 AP
 break in Gilula's arc
 lunate and capitate overlap
 lunate appears triangular "piece-of-pie sign"
‎IV:18 lateral xray

- 236 -
By Dr, AbdulRahman AbdulNasser Hand Trauma | Wrist Trauma

 lateral
 lossof colinearity of radius, lunate, and
capitate
 SL angle >70 degrees
 MRI
o usually not required for diagnosis
Treatment
‎IV:20 piece-of-pie sign ‎IV:19 Abnormal alignment
 Nonoperative
of scaphoid , lunate and
o closed reduction and casting triquetrum
 indications
 no indications when used as definitive management
 outcomes
 universally poor functional outcomes with non-operative management
 recurrent dislocation is common
 Operative
o emergent closed reduction/splinting followed by open reduction, ligament repair, fixation,
possible carpal tunnel release
 indications
 all acute injuries <8 weeks old
 outcomes
 emergent closed reduction leads to
 decreased risk of median nerve damage
 decreased risk of cartilage damage
 return to full function unlikely
 decreased grip strength and stiffness are common
o proximal row carpectomy
 indications
 chronic injury (defined as >8 weeks after initial injury)

 not
uncommon, as initial diagnosis frequently missed
o total wrist arthrodesis
 indications
 chronic injuries with degenerative changes

Techniques
 Closed Reduction
o technique
 finger traps, elbow at 90 degrees of flexion
 hand 5-10 lbs traction for 15 minutes
 dorsal dislocations are reduced through wrist extension, traction, and flexion of wrist
 apply sugar tong splint
 follow with surgery
 Open reduction, ligament repair and fixation +/- carpal tunnel release
o approach (controversial)
 dorsal approach

- 237 -
OrthoBullets2017 Hand Trauma | Wrist Trauma
 longitudinal incision centered at Lister's tubercle
 excellent exposure of proximal carpal row and midcarpal joints
 does not allow for carpal tunnel release
 volar approach
 extended carpal tunnel incision just proximal to volar wrist crease
 combined dorsal/volar
 pros
 added exposure
 easier reduction
 access to distal scaphoid fractures
 ability to repair volar ligaments
 carpal tunnel decompression
 cons
 some believe volar ligament repair not necessary
 increased swelling
 potential carpal devascularization
 difficulty regaining digital flexion and grip
o technique
 fix associated fractures
 repair scapholunate ligament
 suture anchor fixation
 protect scapholunate ligament repair
 controversy of k-wire versus intraosseous cerclage wiring
 repair of lunotriquetral interosseous ligament
 decision to repair based on surgeon preference as no studies have shown improved results
o post-op
 short arm thumb spica splint converted to short arm cast at first post-op visit
 duration of casting varies, but at least 6 weeks
 Proximal row carpectomy
o technique
 perform via dorsal and volar incisions if median nerve compression is present
 volar approach allows median nerve decompression with excision of lunate
 dorsal approach facilitates excision of the scaphoid and triquetrum

3. Hook of Hamate Fracture


Introduction
 Epidemiology
o incidence
 2% of carpal fractures
o risk factors
 often seen in
 golf
 baseball
 hockey
- 238 -
By Dr, AbdulRahman AbdulNasser Hand Trauma | Wrist Trauma
 Pathophysiology
o typically caused by a direct blow
 grounding a golf club
 checking a baseball bat
 Associated conditions
o bipartite hamate will have smooth cortical surfaces

Anatomy
 Hamate
o one of carpal bones, distal and radial to the pisiform
o articulates with
 fourth and fifth metacarpals
 capitate
 triquetrum
o hook of hamate
 forms part of Guyon's canal, which is formed by
 roof - superficial palmar carpal ligament
 floor - deep flexor retinaculum, hypothenar muscles
 ulnar border - pisiform and pisohamate ligament
 radial border - hook of hamate
 one of the palpable attachments of the flexor retinaculum
 deep branch of ulnar nerve lies under the hook

Presentation
 Symptoms
o hypothenar pain
o pain with activities requiring tight grip
 Physical examination
o provocative maneuvers
 tender to palpation over the hook of hamate
 hook of hamate pull test:
 hand held in ulnar deviation as patient flexes DIP joints of the ulnar 2 digits, the flexor
tendons act as a deforming force on the fracture site, positive test elicits pain
o motion and strength : decreased grip strength
o neurovascular exam
 chronic cases
 parasthesia in ring and small finger
 motor weakness in intrinsics

- 239 -
OrthoBullets2017 Hand Trauma | Wrist Trauma
Carpal Tunnel View

Imaging
 Radiographs
o recommended views
 AP and carpal tunnel view
o findings
 fracture best seen on carpal tunnel view
 CT
o indications
 establish diagnosis if radiographs are negative

Treatment
 Nonoperative
o immobilization 6 weeks
 indications
 acute hook of hamate fractures
 body of hamate fx (rare)
 Operative
o excision of hamate fracture fragment
 indications : chronic hook of hamate fxs with non-union
o ORIF
 indications : ORIF is possible but has little benefit

Complications
 Non-union
 Scar sensitivity
 Iatrogenic injury to ulnar nerve
 Closed rupture of the flexor tendons to the small finger

- 240 -
By Dr, AbdulRahman AbdulNasser Hand Trauma | Wrist Trauma

4. Hamate Body Fracture


Introduction
 A rare carpal fracture
 Epidemiology
o incidence
 <2% of all carpal fractures
 Pathophysiology
o mechanism of injury
 main cause for these lesions is a direct impact against a hard
surface with a clenched fist
 Associated conditions
o may be associated with 4th or 5th metacarpal base fractures or
dislocations
 present in ~ 15%

Anatomy
 Hamate Bone
o osteology
 triangular shaped carpal bone
 composed of hook and body
o location
 most ulnar bone in the distal carpal row
o articulation
 4th and 5th metacarpals
 capitate
 triquetrum

Classification
 Milch Classification of Hamate Fractures

Milch Classification
Type I Hook of Hamate Fx (most common)
Type II Body of Hamate Fx

Presentation
 Symptoms
o ulnar-sided wrist pain and swelling
 Physical exam
o inspection
 focal tenderness over hamate

Imaging
 Radiographs
o recommended views
 oblique radiographs (30°) are usually required to visualize fracture
 AP and lateral radiographs are less reliable
- 241 -
OrthoBullets2017 Hand Trauma | Wrist Trauma
o additional views
 carpal tunnel view radiographs
 CT
o usually required to delineate fracture pattern and determine operative
plan
Treatment
 Nonoperative
o immobilization

indications
 ‎IV:21 30°oblique view
 rarely may be used for extra-articular nondisplaced fracture
 Operative
o ORIF
 indications
 most fracture are intra-articular and require open reduction
 technique
 interfragmentary screws +/- k-wires for temporary stabilization

Surgical Techniques
 Open Reduction Internal Fixation
o approach : dorsal most common approach
o fixation technique

fixation may be obtained with K wires or screws ‎IV:22 sagittal CT


o postoperative care
 immobilize for 6-8 weeks

Complications
 Stiffness
 Malunion
 Infection

5. Pisiform Fracture
Introduction
 A rare carpal fracture
 Epidemiology
o incidence
 <1% of carpal fractures
 rare injury and often missed
 Pathophysiology
o mechanism of injury
 usually occurs by direct impact against a hard surface
 fall on outstretched hand
 Associated conditions
o 50% occur as isolated injuries
o 50% occur in association with other carpal fractures or distal radius fractures

- 242 -
By Dr, AbdulRahman AbdulNasser Hand Trauma | Wrist Trauma
Anatomy
 Pisiform Bone
o osteology
 pea shaped, seasmoid bone
o location
 most ulnar and palmar carpal bone in proximal row
 located within the FCU tendon
o function
 contributes to the stability of the ulnar column by preventing triquetral subluxation

Presentation
 Symptoms
o ulnar sided wrist pain after a fall
o grip weakness
 Physical exam
o inspection
 hypothenar tenderness and swelling
 rule out associated injury to other carpal bones and distal radius

Imaging
 Radiographs
o recommended views
 AP and lateral views of wrist
o additional views
 pronated oblique and supinated oblque views
 carpal tunnel view
o findings
 best seen with 30 deg of wrist supination or utilizing the carpal tunnel view
 CT
o indications
 may be required to delineate fracture pattern and determine treatment plan
 MRI
o indications
 suspected carpal fracture with negative radiographs
o findings
 may show bone marrow edema within the pisiform indicating fracture

Treatment
 Nonoperative
o early immobilization
 indications
 first line of treatment
 technique
 short arm cast with 30 degrees of wrist flexion and ulnar deviation for 6-8 weeks
 outcomes
 most often go on to heal without posttraumatic osteoarthritis
 Operative

- 243 -
OrthoBullets2017 Hand Trauma | Wrist Trauma
o pisiformectomy
 indications
 severely displaced and symptomatic fractures
 painful nonunion
 outcomes
 studies show a pisiformectomy is a reliable way to relieve this pain and does not impair
wrist function
Complications
 Malunion
 Non-union
 Chronic ulnar sided pain
 Decreased grip strength

6. Seymour Fracture
Introduction
 Displaced distal phalangeal physeal fracture with an associated nailbed injury
 Epidemiology
o incidence : 20% to 30% of phalangeal fractures involve the physis in children
o body location
 middle finger injury is most common
 type of the distal phalangeal physeal fracture:
 metaphyseal fractures 1 to 2 mm distal to the epiphyseal plate
 Salter-Harris I fractures
 Salter-Harris II fractures
 type of nailbed injury:
 nailbed laceration
 nail plate subluxation
 interposition of soft tissue at fracture site (usually germinal matrix)
 Pathophysiology
o mechanism of injury : direct trauma or crush injury (e.g. caught in door, heavy object or sport)
o pathoanatomy
 similar mechanism to mallet finger in adults
 injury causes flexed posturing of the distal phalanx
 deformity results from an imbalance between the flexor and the extensor tendons at the level
of the fracture
 imbalance occurs due to different insertion sites of flexor and extensor tendons
 extensor tendon inserts into the epiphysis of the distal phalanx
 flexor tendon inserts into metaphysis of the distal phalanx
 widened physis likely to have interposed tissue in the fracture site
 Prognosis
o operative intervention is warranted to ensure that there is no interposed tissue in the fracture site
o failure to recognize injury may result in:
 nailplate deformity
 physeal arrest
 chronic osteomyelitis

- 244 -
By Dr, AbdulRahman AbdulNasser Hand Trauma | Wrist Trauma
Presentation
 Physical exam
o apparent mallet deformity
o echymosis and swelling
o nail plate lying superficial to the eponychial fold

Imaging
 Radiographs
o AP : may appear normal on posteroanterior view
o lateral view
 widened physis or displacement between epiphysis/metaphysis
 flexion deformity at fracture site

Differential Diagnosis
 Mallet finger
o pediatric mallet finger is usually osseous avulsion (SH III and SH IV)
o mallet finger fracture line enters DIPJ, while Seymour fracture line traverses physis (does not
enter DIPJ)
Treatment
 Nonoperative
o closed reduction and splinting
 indications
 minimally displaced, closed fracture
 no interposition of soft tissue at fracture site
 Operative
o closed reduction and pinning across DIPJ
 indications
 displaced, closed fracture
 no interposition of soft tissue at fracture site
o antibiotics, open reduction and pinning across DIPJ, nailbed repair
 open management has fewer complications than closed management
 indications : open fracture
 technique
 hyperflexion of the digit will permit removal of the interposed soft tissue from the
fracture site
 thorough irrigation and debridement
 anatomical reduction and retrograde k-wire fixation crossing the fracture site and DIP
joint
 nailbed injury repair

Complications
 Nail dystrophy
 Growth disturbance of the distal phalanx and nail
 Secondary fracture displacement
 Chronic osteomyelitis (failure to treat as open fracture)
 Flexion deformity

- 245 -
OrthoBullets2017 Hand Trauma | Wrist Trauma

7. TFCC Injury
Introduction
 Mechanism of TFCC injury
o Type 1 traumatic injury
 mechanism
 most common is fall on extended wrist with forearm pronation
 traction injury to ulnar side of wrist
 traction injury to ulnar wrist
o Type 2 degenerative injury
 associated with positive ulnar variance
 associated with ulnocarpal impaction

Anatomy
 TFCC made up of
o dorsal and volar radioulnar ligaments
 deep ligaments known as ligamentum subcruentum
o central articular disc
o meniscus homolog
o ulnar collateral ligament
o ECU subsheath
o origin of ulnolunate and ulnotriquetral ligaments
 Blood supply
o periphery is well vascularized (10-40% of the periphery)
o central portion is avascular
 Origin
o dorsal and volar radioulnar ligaments originate at the sigmoid notch of the radius
 Insertion
o dorsal and volar radioulnar ligaments converge at the base of the ulnar styloid

Classification
Class 1 - Traumatic TFCC Injuries
1A Central perforation or tear
1B Ulnar avulsion (without ulnar styloid fx)
1C Distal avulsion (origin of UL and UT ligaments)
1D Radial avulsion

- 246 -
By Dr, AbdulRahman AbdulNasser Hand Trauma | Wrist Trauma
Class 2 - Degenerative TFCC Injuries
2A TFCC wear and thinning
2B Lunate and/or ulnar chondromalacia + 2A
2C TFCC perforation + 2B
2D Ligament disruption + 2C
2E Ulnocarpal and DRUJ arthritis + 2D

Presentation
 Symptoms
o wrist pain
o turning a door key often painful
 Physical exam
o positive "fovea" sign
 tenderness in the soft spot between the ulnar styloid and flexor carpi ulnaris tendon, between
the volar surface of the ulnar head and the pisiform
 95% sensitivity and 87% specificity for foveal disruptions of TFCC or ulnotriquetral
ligament injuries
o pain elicited with ulnar deviation (TFCC compression) or radial deviation (TFCC tension)

Imaging
 Radiographs
o usually negative
o zero rotation PA view evaluates ulnar variance
o dynamic pronated PA grip view may show pathology
 Arthography
o joint injection shows extravasation
 MRI
o has largely replaced arthrography
o tear at ulnar part of lunate indicates ulnocarpal impaction
o sensitivity = 74-100%
 Arthroscopy
o most accurate method of diagnosis
o indicated in symptomatic patients after failing several months of splinting and activity
modification
Differential
 Differential for ulnar sided wrist pain
See table next page

- 247 -
OrthoBullets2017 Hand Trauma | Wrist Trauma

Treatment
 Nonoperative
o immobilization, NSAIDS, steroid injections
 indications
 all acute Type I injuries
 first line of treatment for Type 2 injuries
 Operative
o arthroscopic debridement
 indications
 type 1A
 diagnostic gold standard
o arthroscopic repair
 indications
 type 1B, 1C, 1D
 best for ulnar and dorsal/ulnar tears
 generally acute, athletic injuries more amenable to repair than chronic injuries
 outcomes
 patient should expect to regain 80% of motion and grip strength when injuries are
classified as acute (<3 months)
o ulnar diaphyseal shortening
 indications
 Type II with ulnar positive variance is > 2mm
 advantage of effectively tightening the ulnocarpal ligaments and is favored when LT
instability is present
o Wafer procedure
 indications
 Type II with ulnar positive variance is < 2mm
 type 2A-C

- 248 -
By Dr, AbdulRahman AbdulNasser Hand Trauma | Wrist Trauma
o limited ulnar head resection
indications : type 2D
o Darrach procedure
 indications
 contraindicated due to problems with ulnar stump instability

Techniques
 Arthroscopic debridement
o approach
 arthroscopic approach to the wrist
 performed through combination of 3-4 and 6R portal
o technique
 maintain 2 mm rim peripherally otherwise joint can become unstable
o pros & cons
 not effective if patient has ulnar positive variance
 80% of patients obtain good relief of pain
 Arthroscopic repair
o approach
 arthroscopic approach to the wrist
o technique
 many techniques exist such as outside-in and inside-out
 generally suture based repair
o pros & cons
 only works for peripheral tears where blood supply is present
 patient immobilized for 6 weeks
o complications
 ECU tendonitis from suture knot
 dorsal sensory nerve injury
 Ulnar diaphyseal shortening
o approach
 dorsal approach to the forearm
o technique
 osteotomy of the diaphysis or metaphysis followed by plate fixation
o pros & cons
 can address > 2 mm ulnar variance
 requires immobilization and time for fracture healing
 can help tension the ulnocarpal ligaments
o complications
 nonunion
 hardware irritation necessitating removal
 Wafer procedure
o approach : dorsal approach to the forearm
o technique
 ulnar cortex is not disrupted
 do not extend bone removal into the DRUJ
o pros & cons
 intrinsic stability of ECU, TFCC, and ulnar periosteum obviate need for plate fixation
- 249 -
OrthoBullets2017 Hand Trauma | Finger Trauma
 Limited ulnar head resection
o approach
 arthroscopic approach to the wrist
o technique
 removal of approximately 2-4 mm of bone under the TFCC
 distal ulnar burred through central TFCC defect
o pros & cons
 can be technically difficult to obtain level shortening through TFCC window
 only applicable when patient has < 2mm of ulnar variance
 Darrach procedure
o approach
 dorsal approach to the forearm
o technique
 resection of the distal 1-2cm of the distal ulna
 TFCC should be approximated to the wrist capsule
o pros & cons
 salvage procedure for pain relief only
 distal joint is unstable
o complications : ECU tendon can sublux over remaining ulna causing pain

C. Finger Trauma

1. Metacarpal Fractures
Introduction
 Metacarpal fractures
o divided into fractures of metacarpal head, neck, shaft
o treatment based on which metacarpal is involved and location of fracture
o acceptable angulation varies by location
o no degree of malrotation is acceptable
 Epidemiology
o incidence
 metacarpal fractures account for 40% of all hand injuries
o demographics
 men aged 10-29 have highest incidence of metacarpal injuries
o location
 metacarpal neck is most common site of fracture
 fifth metacarpal is most commonly injured
 Mechanism of injury
o direct blow to hand or rotational injury with axial load
o high energy injuries (ie. automobile) may result in multiple fractures
 Associated conditions
o wounds may indicate open fractures or concomitant soft tissue injury
 tendon laceration
 neurovascular injury
o compartment syndrome

- 250 -
By Dr, AbdulRahman AbdulNasser Hand Trauma | Finger Trauma
 closed injuries with multiple fractures or dislocations
 crush injuries
Anatomy
 Metacarpal anatomy
o concave on palmar surface
o 1st, 4th, and 5th digits form mobile borders
o 2nd and 3rd digits form stiffer central pillar
 index metacarpal is the most firmly fixed, while the thumb metacarpal articulates with the
trapezium and acts independently from the others
o three palmar and four dorsal interossei muscles arise from metacarpal shafts
 Insertional anatomy
o extensor carpi radialis longus/brevis
 insert on the base of metacarpal II, III (respectively); assist with wrist extension and radial
flexion of the wrist
o extensor carpi ulnaris
 inserts on the base of metacarpal V; extends and fixes wrist when digits are being flexed;
assists with ulnar flexion of wrist
o abductor pollicis longus
 inserts on the trapezium and base of metacarpal I; abducts thumb in frontal plane; extends
thumb at carpometacarpal joint
o opponens pollicis
 inserts on metacarpal I; flexes metacarpal I to oppose the thumb to the fingertips
o opponens digiti minimi
 inserts on the medial surface of metacarpal V; Flexes metacarpal V at carpometacarpal joint
when little finger is moved into opposition with tip of thumb; deepens palm of hand.
Presentation
 Physical exam
o inspect for open wounds and associated injuries
 fight wounds over MCP joint are open until proven otherwise
 extensor tendon can be lacerated and retracted
 dorsal wounds over metacarpal fractures are almost always open
fractures
o deformity indicates location
 deformity at metacarpal base may indicate CMC dislocation
 shortening can be assessed by comparing contralateral hand
 malrotation assessed by lining up fingernail in partial flexion and full flexion if possible,
compare to contralateral side
o motor examination
 typically no motor deficits unless open wounds present
 check integrity of flexor/extensor tendons in presence of open wounds
o neurovascular examination
 dorsal wounds may affect dorsal sensory branch of radial/ulnar nerve
 volar wounds can involve digital nerves
 test for radial and ulnar border two-point discrimination on the injured digit before any
regional/hematoma block or attempted reduction
- 251 -
OrthoBullets2017 Hand Trauma | Finger Trauma

Imaging
 Radiographs
o standard AP, oblique, and lateral films
o oblique radiographs
 for evaluation of CMC joint and improved visualization
of affected digit
 30°pronated lateral
 to see 4th and 5th CMC fx/dislocation
 30°supinated view
 to see 2nd and 3rd CMC fx/dislocation
o Brewerton view for metacarpal head fractures
o Roberts view for thumb CMC joint
 CT
o indications
 inconclusive radiographs of CMC fractures/dislocations
 multiple CMC dislocations
 complex metacarpal head fractures

General Treatment
 Nonoperative
o immobilization
 indications
 must be stable pattern
 no rotational deformity
 acceptable angulation & shortening (see table)

Acceptable Shaft Acceptable Shaft Acceptable neck


Angulation (degrees) Shortening (mm) Angulation
Index & Long Finger 10-20 2-5 10-15
Ring Finger 30 2-5 30-40
Little Finger 40 2-5 50-60

 Operative
o operative treatment
 general indications
 intra-articular fxs
 rotational malalignment of digit
 significantly displaced fractures (see above criteria)

 multiple metacarpal shaft fractures


 loss inherent stability from border digit during healing process
 postoperative
 early motion is critical
 remove pins/ cast at ~ 4 weeks

- 252 -
By Dr, AbdulRahman AbdulNasser Hand Trauma | Finger Trauma
Treatment - Metacarpal Head Fractures
 Operative
o ORIF
 indications
 no degree of articular displacement acceptable
 majority requires surgical fixation
o external fixation
 indications
 severely comminuted fractures
o MCP arthroplasty
 indications
 severely comminuted fractures
o MCP fusion
 indications
 arthritis late disease
 Techniques
o ORIF
 approach
 dorsal incision
 either centrally split extensor apparatus or release and repair sagittal band
 fixation
 hardware cannot protrude from joint surface
 fix with multiple small screws in collateral recess, headless screws, or k-wires
 ideal fixation should allow for early motion
 Complications
o stiffness
 most common
 prevented with early motion

Treatment - Metacarpal Shaft Fractures


 Nonoperative
o immobilization
 indications
 nondisplaced metacarpal neck fractures
 acceptable angulation (see above table)
 no malrotation
 shortening (aesthetic problem only)
 immobilize MCP joints in 70-90 degrees of flexion
 cast for 4 weeks
 Operative
o ORIF vs. CRPP
 indications
 open fractures
 unacceptable angulation (see above table)
 any malrotation
 multiple fractures

- 253 -
OrthoBullets2017 Hand Trauma | Finger Trauma
 Techniques
o closed reduction percutaneous pinning
 place antegrade through metacarpal base or retrograde through collateral recess
 remove pins at 4 weeks
o open reductions with lag screw
 can use multiple lag screws for long spiral fractures
 try to get at least two lag screws
o open reduction with dorsal plating
 works best for transverse fractures
 try to cover plate with periosteum to prevent tendon irritation
 begin early motion to prevent tendon irritations

Treatment - Metacarpal Neck Fractures


 Nonoperative
o reduction and casting
 acceptable degrees of apex dorsal angulation (varies by study, see above table)
 immobilize MCP joints in 70-90 degrees of flexion, leave PIP joints free
 cast for 4 weeks
 reduce using Jahss technique
 90 degrees MCP flexion, dorsal pressure through proximal phalanx while stabilizing
metacarpal shaft
 Operative
o reduction and fixation
 indications
 unacceptable angulation (see above table)
 open fractures
 any malrotation
 intraarticular fractures
 Technique
o CRPP with MCP's flexed
 antegrade through metacarpal base
 retrograde through collateral recess
o ORIF
 perform if cannot get reduction for CRPP
 difficult to plate because limited bone for distal fixation

2. MCP Dislocations
Introduction
 Epidemiology
o dorsal dislocations most common
o index finger most commonly involved
 Mechanism
o a hyperextension injury

Classification
 Simple vs. Complex

- 254 -
By Dr, AbdulRahman AbdulNasser Hand Trauma | Finger Trauma
o simple
 volar plate not interposed in joint
 treated with closed reduction
o complex
 complex dislocations have interposition of volar plate and/or sesamoids
 in index finger flexor tendon displaces ulnarly and lumbrical displaces radially which
tighten around metacarpal neck preventing reduction
 in small finger flexor tendons and lumbrical displace radially and the abductor digiti
minimi and flexor digiti minimi ulnarly preventing closed reduction
 may require open reduction
 Kaplan's lesion (rare)
o most common in index finger
o complex dorsal dislocation of finger, irreducible
o metacarpal head buttonholes into palm (volarly)
o volar plate is interposed between base of proximal phalanx and metacarpal head

Presentation
 Physical exam
o skin dimpling often seen in complex dislocations but absent in simple dislocations

Imaging
 Radiographs
o lateral view best shows dislocation
o joint space widening may indicate interposition of volar plat
o useful to detect associated chip fractures

Treatment
 Nonoperative
o closed reduction
 indications
 simple dislocations
 technique
 reduction technique involve applying direct pressure over proximal phalanx while the
wrist is held in flexion to take tension off the intrinsic and extrinsic flexors
 avoid longitudinal traction and hyperextension during closed reduction, may pull volar
plate into joint
 Operative
o open reduction
 indications
 complex dislocations

Surgical Techniques
 Open reduction
o approach
 dorsal approach
 split extensor tendon to expose joint
 may be able to push volar plate out with freer elevator
 usually need to split volar plate to remove from joint
- 255 -
OrthoBullets2017 Hand Trauma | Finger Trauma
 use this approach for volar dislocations
 volar approach
 places neurovascular structures at risk
 release A1 pulley to expose volar plate

3. Phalanx Fractures
Introduction
 Common hand injuries that can be broken into the following injuries
o proximal phalanx
o middle phalanx
o distal phalanx
 Epidemiology
o incidence
 most common injuries to the skeletal system
 account for 10% of all fractures
 distal phalanx is most common fractured bone in the hand
 Pathophysiology
o mechanism
 depends on age
 10-29 years of age: sports is most common
 40-69 year of age: machinery is most common
 >70 year of age: falls are most common
o pathoanatomy
 proximal phalanx fx
 deformity is usually apex volar angulation due to
‎IV:23 Proximal Phalynx fractures
 proximal fragment in flexion (from interossei)
 distal fragment in extension (from central slip)
 middle phalanx
 deformity is usually apex dorsal OR volar angulation
 apex dorsal if fracture proximal to FDS insertion (from extension of proximal
fragment through pull of the central slip)
 apex volar if fracture distal to FDS insertion (prolonged insertion from just distal to
the flare at the base to within a few mm of the neck)
 a fracture through the middle third may angulate in either direction or not at all
secondary to the inherent stability provided by an intact and prolonged FDS insertion
 Associated conditions
o nail bed injuries
 associated with distal phalanx fractures

Presentation
 Symptoms
o pain
 Physical exam
o local tenderness
o deformity
o look carefully for open wounds

- 256 -
By Dr, AbdulRahman AbdulNasser Hand Trauma | Finger Trauma
Imaging
 Radiographs
o finger xrays
 must get true lateral of joint
o hand xrays to rule out associated fractures
 30°pronated lateral to see 4th and 5th CMC x/dislocation
 30°supinated view to see 2nd and 3rd CMC fx/dislocation

Treatment - Proximal Phalanx Fracture


 Nonoperative
o buddy taping
 indications
 extraarticular with < 10° angulation or < 2mm shortening and no rotational deformity
 3 weeks of immobilization followed by aggressive motion
o reduction and splinting
 indications : most distal phalanx fx
 Operative
o CRPP vs. ORIF
 indications
 irreducible or unstable fracture pattern
 transverse fractures (all angulate volarly) with > 10° angulation or 2mm shortening or
rotationally deformed
 long oblique proximal phalanx fractures
 techniques
 crossed k-wires
 Eaton-Belsky pinning through metacarpal head
 minifragment fixation with plate and lag screws, or lag screws alone
 lag screws alone indicated in presence of long oblique fracture

Treatment - Middle Phalanx Fracture


 Nonoperative
o buddy taping
 indications
 extraarticular with < 10° angulation or < 2mm
shortening and no rotational deformity
 technique
 3 weeks of immobilization followed by aggressive
motion
 Operative
o CRPP vs. ORIF
 indications
 irreducible or unstable fracture pattern
 transverse fractures with > 10° angulation or 2mm shortening or rotationally deformed
 techniques
 crossed k-wires
 collateral recess pinning
 minifragment fixation with plate and lag screws
- 257 -
OrthoBullets2017 Hand Trauma | Finger Trauma
Treatment - Distal Phalanx Fracture
 Nonoperative
o reduction and splinting
 indications
 most cases
 nail matrix may be incarcerated in fx and block reduction
 Operative
o remove nail, repair nailbed, and replace nail to maintain epi fold
 indications : when distal phalanx associated with a nailbed injury
 see nail bed injuries
o ORIF +/- bone grafting
 indications : non-unions

Complications
 Loss of motion
o most common complication
o predisposing factors include prolonged immobilization, associated joint injury, and extensive
surgical dissection
o treat with rehab, and surgical release as a last resort
 Malunion
o malrotation, angulation, shortening
o surgery indicated when associated with functional impairment
 corrective osteotomy at malunion site (preferred)
 metacarpal osteotomy (limited degree of correction)
 Nonunion
o uncommon
o most are atrophic and associated with bone loss or neurovascular compromise
o surgical options
 resection, bone grafting, plating
 ray amputation or fusion

4. Phalanx Dislocations
Introduction
 Common hand injuries can be broken into the following
o PIP joint
 dorsal dislocations
 dorsal fracture-dislocations
 volar dislocation
 volar fracture-dislocation
 rotatory dislocations
o DIP joint
 dorsal dislocations & fracture-dislocations
 Associated conditions
o swan neck deformity
o nail bed injuries
 associated with distal phalanx fractures
- 258 -
By Dr, AbdulRahman AbdulNasser Hand Trauma | Finger Trauma
Imaging
 Radiographs
o finger xrays
 must get true lateral of joint
o hand xrays to rule out associated fractures
 30°pronated lateral to see 4th and 5th CMC x/dislocation
 30°supinated view to see 2nd and 3rd CMC fx/dislocation

Dorsal PIP Dislocations


 Introduction
o more common than volar dislocation
o leads to injury to the volar plate and at least one collateral ligament, and if untreated a swan neck
deformity will result
 Classification
o simple
 middle phalanx in contact with condyles of proximal phalanx
o complex
 base of middle phalanx not in contact with condyle of proximal phalanx, bayonet appearance
 volar plate acts as block to reduction with longitudinal traction
 Treatment
o nonoperative
 reduce and buddy tape to adjacent finger (3-6 weeks)
 indications
 dislocation is reducible
 usually performed by patient
 technique
 if complex, reduce with hyperextension of middle phalanx followed by palmar force
 complications
 a PIP flexion contracture (pseudoboutonniere)
 may develop but usually resolves with therapy
 swan neck deformity
 occurs secondary to a volar plate injury
o operative
 open reduction and extraction of the volar plate
 indication
 failed reduction
 technique
 in closed injuries incomplete reduction usually due to volar plate interposition
 in open injuries incomplete reduction usually caused by dislocated FDP tendon
 perform dorsal approach with incision between central slip and lateral band

Dorsal PIP Fracture-Dislocations


 Classification
o Hastings classification (based on amount of P2 articular surface involvement)
o volar lip fractures are the most common fracture pattern
 Type I-Stable
 <30%-treat with dorsally based extension block splint
- 259 -
OrthoBullets2017 Hand Trauma | Finger Trauma
 Type II-Tenuous
 30-50%-if reducible in flexion, dorsally based extension block splint
 Type III-Unstable
 >50%-ORIF, hamate autograft, or volar plate arthroplasty
 Treatment
o nonoperative
 dorsal extension block splinting
 indications
 if < 40% joint involved and stable
 outcome
 regardless of treatment, must achieve adequate joint
reduction for favorable long-term outcome
o operative
 ORIF or CRPP
 indications
 if > 40% joint involved and unstable
 technique
 reduction of the middle phalanx on the condyles of the proximal phalanx is the
primary goal
 adequate volar exposure of the volar plate requires resection of
 proximal portion of C2 pulley
 entire A3 pulley
 distal C1 pulley
 outcomes
 articular surface reconstruction is desirable, but not necessary for a good clinical
outcome
 PIP subluxation inhibits the gliding arc of the joint and portends a poor clinical
outcome
 dynamic distraction external fixation
 indications
 highly comminuted "pilon" fracture-dislocations
 technique
 follow with early mobilization
 volar plate arthroplasty
 indications
 chronic injuries
 arthrodesis
 indications
 chronic injuries

Volar PIP Dislocation & Fracture-dislocations


 Introduction
o less common than dorsal dislocation
o leads to an injury to the central slip and at least one collateral ligament, and a failure to treat will
lead to boutonneire deformity
 Treatment
o dislocation only
- 260 -
By Dr, AbdulRahman AbdulNasser Hand Trauma | Finger Trauma
 nonoperative
 splinting in extension for 6-8 weeks
 indications
 most PIP dislocations
o fracture-dislocation
 nonoperative
 splinting in extension for 6-8 weeks
 indications
 if < 40% joint involved and stable
 operative
 ORIF or CRPP
 reduction of the middle phalanx on the condyles of the proximal phalanx is the
primary goal
 if > 40% joint involvement

Rotatory PIP dislocation


 Introduction
o one of phalangeal condyles is buttonholed between central slip and lateral band
 Treatment
o nonoperative
 only if reduction is successful
 reduce by applying traction to finger with MP and PIP joints in 90 degrees of flexion
 flexion relaxes volarly displaced lateral band, allowing it to slip back dorsally
 reduction is confirmed with post-reduction true lateral radiograph
o operative
 open reduction
 indications
 required in most cases

Dorsal DIP Dislocations & Fracture-Dislocations


 Treatment
o nonoperative
 closed reduction, immobilization in slight flexion with a dorsal splint for 2 weeks
 indications
 first line of treatment
 tuft fractures require no specific treatment
 can consider temporary splinting
o operative
 open reduction
 indications : if two reduction attempts fail
 technique
 volar plate interposition is most common block to reduction in irreducible closed DIP
joint dislocation
 FDP may be blocking reduction if injury is open
 may require percutaneous pinning to support nail bed repair
 amputation
 consider in highly comminuted injuries with significant soft tissue loss

- 261 -
OrthoBullets2017 Hand Trauma | Finger Trauma

5. Digital Collateral Ligament Injury


Introduction
 Trauma to the digit injuring the radial or ulnar collateral ligaments
 Mechanism
o depends on the joint involved
o usually the result of a "jammed finger"
o doral or volar dislocation events can tear one or both of the collateral ligaments

Anatomy
 Collateral ligaments of the digits
o located on the lateral aspect of the DIP, PIP and MCP joints
o crucial for opposing pinch stability

Presentation
 Symptoms
o Pain at involved joint
o Instabilty with pinch once pain resolved
 Physical exam
o inspection
 swelling at involved joint
 deformity of joint
o provocative tests
 varus and valgus stress tests

Imaging
 Radiographs
o recommended views
 AP, lateral, and oblique views of digit
 varus/valgus stress views may aid in diagnosis
 MRI
o indicated if equivocal physical exam findings

Treatment
 Nonoperative
o buddy taping for 3 weeks
 indications
 simple tears
o buddy taping for 6 weeks
 indications
 complete tears
 Operative
o collateral ligament repair
 indications
 radial ligament injuries of index finger (ligament needed for pinch stability)

- 262 -
By Dr, AbdulRahman AbdulNasser Hand Trauma | Thumb Trauma

D. Thumb Trauma

1. Base of Thumb Fractures


Introduction
 Base of the thumb metacarpal fractures
include
o Bennett fracture (intra-articular)
o Rolando fracture (intra-articular)
o extra-articular fractures
 Epidemiology
o incidence
 80% of thumb fractures involve the metacarpal base
 most common variant is the Bennet fracture
 Pathophysiology
o mechanism of injury
 most fractures caused by axial force applied to the thumb
o pathoanatomy
 three muscles provide deforming forces at base of thumb
 abductor pollicis longus (PIN)
 extensor pollicis longus (PIN)
 adductor pollicis (Ulnar n.)
 the thumb has extensive CMC motion in sagittal plane
 allows for angulation up to 30 degrees in this plane

Bennett Fracture
 Intra-articular fracture/dislocation of base of 1st metacarpal characterized by
o volar lip of metacarpal based attached to volar oblique ligament
 ligament holds this fragment in place
 small fragment of 1st metacarpal continues to articulate with trapezium
 Pathoanatomy
o lateral retraction of distal 1st metacarpal shaft by APL and
adductor pollicis
 shaft pulled into adduction
 metacarpal base supinated
 Prognosis
o better than Rolando fx
 Imaging
o radiographs
 recommended views
 fracture best seen with hyper-pronated thumb view
 findings
 minimal joint step-off considered best
 Treatment
o nonoperative
 closed reduction & cast immobilization

- 263 -
OrthoBullets2017 Hand Trauma | Thumb Trauma
 indications
 nondisplaced fractures
 technique
 reduction maneuver with traction, extension, pronation, and abduction
o operative
 closed reduction and percutaneous pinning
 indications
 volar fragment is too small to hold a screw
 anatomic reduction unstable
 technique
 can attempt reduction of shaft to trapezium to hold reduction
 ORIF
 indications
 large fragment
 2mm+ joint displacement
 Complications
o post-traumatic arthritis
 there is no agreement regarding the relationship of post-fixation joint incongruity and post-
traumatic arthritis
Rolando Fracture
 Intra-articular fracture of base of 1st metacarpal characterized by
o intra-articular comminution
 Epidemiology
o less common than Bennett's fracture
 Pathoanatomy
o deforming forces are the same as Bennett's fracture
 volar fragment should have volar oblique ligament
attached
 shaft pulled dorsally
o typically the base is split into a volar and dorsal fragment
 commonly called a 'Y' fracture
o often have more than two proximal fragments
 Prognosis
o worse than Bennett fx
 Treatment
o nonoperative
 immobilization
 indications
 for severe comminution, stable
 start early range of motion
o operative
 external fixation, CRPP
 indications
 for severe comminution, unstable
 technique
 can approximate large fragments with k-wires
- 264 -
By Dr, AbdulRahman AbdulNasser Hand Trauma | Thumb Trauma
ORIF
 indications
 most common fixation method
 technique
 use t-plate or blade plate
 can use k-wires of fragments are too small for screw purchase
 Complications
o commonly results in post-traumatic osteoarthritis

Extra-articular fracture
 Extra-articular fracture of base of 1st metacarpal
o can be transverse or oblique in nature
 Treatment
o nonoperative
 spica casting
 indications
 if joint is reduced and there is less than 30 degrees of angulation
o operative
 CRPP
 indications
 if reduction cannot be held to result in less than 30 degrees of angulation
 outcome
 these fractures typically have the best outcome

2. Thumb CMC dislocation


Introduction
 Thumb CMC dislocation is mostly dorsal
o volar dislocation is rare
 Epidemiology
o incidence
 makes up <1% of hand injuries
 Pathophysiology
o mechanism
 axial force on a flexed thumb (more
common)
st
 dorsal force applied in 1 web space (e.g.
handlebar driven into a motorcyclist’s
thumb on impact) (less common) ‎IV:24 Dorsal dislocation
o pathoanatomy
 dorsoradial ligament is torn
st
 anterior oblique ligament is stripped/peeled off the 1 metacarpal base but remains
continuous
Anatomy
 dorsal side ligaments are the primary stabilizers to dorsal/dorsoradial forces
 16 total ligaments that stabilize the TMC joint
- 265 -
OrthoBullets2017 Hand Trauma | Thumb Trauma
o superficial anterior oblique
 does NOT stabilize joint in flexion
 does NOT prevent dorsal subluxation
 provides for laxity of TMCJ to allow pronation during opposition
 forms “voluminous pouch” to accommodate metacarpal translation
o deep anterior oblique (beak) ligament
 pivot for TMCJ for pronation of thumb
o dorsoradial ligament
 stabilizer (“check rein”) to radial subluxation
 becomes taut with radial/dorsoradial subluxation before other ligaments
 if all other ligaments but this one are cut, CMC still remains reduced
 lax in stage IV arthritis
o posterior oblique ligament
o ulnar collateral ligament
o intermetacarpal ligament
o dorsal intermetacarpal
o dorsal trapeziotrapeziod
o volar trapeziotrapeziod
o dorso trapezio-II metacarpal
o volar trapezio-II metacarpal
o trapezio-III metacarpal
o transverse carpal ligament
o trapeziocapitate I‎V:25 volar dislocation is rare
o volar scaphotrapezial
o radial scaphotrapezial
 7 main stabilizers of TMCJ – SAOL, dAOL, DRL, POL, UCL, IML and DIML
 9 stabilizers of trapezium – DTT, VTT, DT-II MC, VT-II MC, T-III MC, VST, RST,
trapeziocapitate and transverse carpal
 4 key ligamentous restraints of the thumb
o anterior oblique ligament
 remains attached to volar fragment in Bennett/Rolando fracture
o posterior oblique ligament
o intermetacarpal ligament
o dorsoradial ligament

Presentation
 History
o collide onto fixed object/axial force on a flexed thumb
st
o dorsal force applied to 1 web space
 e.g. handlebar driven into a motorcyclist’s thumb on impact)
 Symptoms
o pain over thenar eminence
 Physical exam
o swelling, bruising over thenar eminence
o unable to form a fist

- 266 -
By Dr, AbdulRahman AbdulNasser Hand Trauma | Thumb Trauma
Imaging
 Radiographs
o radiographs
 hand AP, lateral, oblique
 MRI
o indications
 persistent/recurrent instability after reduction
 guide to ligamentous reconstruction

Treatment
 Nonoperative
o closed reduction and immobilization in extension and pronation
 indications
 stable on reduction (implying the AOL is intact)
 Operative
o closed reduction and temporary pinning
o reconstruction of the dorsal capsuloligamentous complex with tendon autograft +
temporary pinning
 recommended treatment
 indications
 grossly unstable joint (AOL possibly torn as well)
 results
 better abduction and pinch strength than closed reduction and pinning

Complications
 Anterior osteophyte often visible
 Low incidence of recurrent dislocation

3. Thumb Collateral Ligament Injury


Introduction
 Thumb collateral ligament injuries include
o radial collateral ligament
 rare
o ulnar collateral ligament
 most common
 eponyms for ulnar collateral ligament (UCL) injury are
 Gamekeeper's thumb for chronic injury
 skiers thumb for acute injury
 Stener lesion
 avulsed ligament with or without bony attachment is displaced above the adductor
aponeurosis
 will not heal without surgical repair
 Epidemiology
o UCL more common than radial collateral ligament
 Mechanism
o hyper abduction or extension at the MCP joint

- 267 -
OrthoBullets2017 Hand Trauma | Thumb Trauma
Anatomy
 UCL is composed of
o proper collateral ligament
 resists valgus load with thumb in flexion
o accessory collateral ligament and volar plate
 resists valgus load with thumb in extension
 valgus laxity in both flexion and extension is indicative of a complete UCL rupture

Presentation
 History
o hyperabduction injury
 Symptoms
o pain at ulnar aspect of thumb MCP joint
 Physical exam
o inspection and palpation
 mass from torn ligament and possible bony avulsion may be present
o stress joint with radial deviation both at neutral and 30° of flexion
 instability in 30° of flexion indicates injury to proper UCL
 instability in neutral indicates injury to accessory and proper UCL and/or volar plate
 compare to uninjured thumb MCP joint

Imaging
 Radiographs
o recommended views
 AP, lateral and oblique of thumb
 valgus stress view may aid in diagnosis if a bony avulsion has already
been ruled out
 MRI
o can aid in diagnosis if exam equivocal

Treatment
 Nonoperative
o immobilization for 4 to 6 weeks
 indications
 partial tears with < 20° side to side variation of varus/valgus instability
 Operative
o ligament repair
 indications
 acute injuries with
 > 20° side to side variation of varus/valgus instability
 >35° of opening
 Stener lesion
 avulsed ligament with or without bony attachment is displaced above the adductor
aponeurosis
 will not heal without surgical repair

- 268 -
By Dr, AbdulRahman AbdulNasser Hand Trauma | Other Traumatic Injuries
 technique
 can use suture, suture anchors, or small screw to repair ligament
o reconstruction of ligament with tendon graft, MCP fusion, or adductor advancement
 indications
 chronic injury

Radial Collateral Ligament Injury


 Rare
 Treatment
o nonoperative
 immobilization
‎IV:26 Stener lesion
 indicated in most cases
 Stener's lesion does not occur

E. Other Traumatic Injuries

1. Human Bite
Introduction
 Epidemiology
o incidence
 third most common bite behind dog and cat
o demographics
 more common in males
o location
 typically dorsal aspect of 3rd or 4th MCP joint
 "fight bite"
 Pathophyiology
o mechanism
 most often result of direct clenched-fist trauma (from tooth) after punching another individual
in the mouth
 can also result from direct bite (i.e. child biting another child)
o pathoanatomy
 tooth penetrates capsule of MCP joint
 flora (bacteria) from mouth enter joint
 bacteria become trapped within joint as fist is released from clenched position
 bacteria now caught under extensor tendon and/or capsule
o microbiology
 typically polymicrobial
 most common organisms
 alpha-hemolytic streptococcus (S. viridans) and staphylococcus aureus
 eikonella corrodens in 7-29%
 other gram negative organisms
 Associated conditions
o extensor tendon lacerations
 can be missed due to proximal tendon retraction

- 269 -
OrthoBullets2017 Hand Trauma | Other Traumatic Injuries
Presentation
 History
o direct clenched-fist trauma to another individual's mouth
 often overlooked
 must have high index of suspicion as patients often unwilling to reveal history
 consider the injury a "fight-bite" until proven otherwise
o possible delay in presentation until symptoms become intolerable
 Symptoms
o progressive development of pain, swelling, erythema, and drainage over wound
 Physical exam
o fight bite
 small wound over dorsal aspect of MCP joint
 wound often transverse, irregular
 typically 3rd and/or 4th MCPs, but can involve any digit
 erythema, warmth, and/or edema overlying wound and joint
 ± purulent drainage
 must assess for integrity of extensor tendon function
 possible pain with passive ROM of MCP joint
 typically no involvement of volar/flexor surface of digit
 neurovascular status typically preserved

Imaging
 Radiographs
o indicated to assess for foreign body (i.e. tooth fragment) and for fracture

Studies
 Culture
o not routinely obtained in ED due to contamination
o deep culture obtained in OR
 aerobic and anaerobic

Treatment
 Operative
o I&D, IV antibiotics
 indications
 fight bite
 joints or tendon shealths are involved
 antibiotics
 IV antibiotics directed at Staph, Strep, and gram-negative organisms
 ampicillin/sulbactam (unasyn)
 PO antibiotics upon discharge for 5 to 7 days
 amoxicillin/clavulanic acid (augmentin)
 debridement
 debridement of wound and joint capsule
 wound left open for drainage
 obtain gram stain and culture

- 270 -
By Dr, AbdulRahman AbdulNasser Hand Trauma | Other Traumatic Injuries

2. Dog and Cat Bites


Introduction
 Bites by domestic animals are common
o delayed presentation (>1week) is common
o required care is often underestimated
 Epidemiology
o incidence (dog > cat > human)
 dog bites are most common animal bites in the US (90%)
 dog is known to victim in 90% of cases
 cat bites are 2nd most common (10%)
 human bites are 3rd most common
o demographics
 more common in males
 more frequent at ages 2-19 years
o location
 upper extremity > lower extremity
 Pathophysiology
o mechanism
 dog bites
 cause crush, puncture, avulsion, tears and abrasions
 large dogs' jaws exert >450lbs/ square inch
 more likely to cause structural damage to nerves, vessels, joints
 cat bites
 penetrate bones and joints, and cause septic arthritis and osteomyelitis
 small, sharp teeth cause puncture wounds that seal immediately
 penetrate joints and flexor tendons
 higher risk for infection than dog bites
 Associated conditions
o secondary bacterial infection
 most bites do not become infected
 risk factors for infection
 bite to hand, foot, or major joint
 puncture wounds or crush injuries
 treatment delay >12h
 age >50y
 preexisting host disease
 immune suppression (steroids, asplenism)
 chronic alcoholism
 diabetes mellitus
 vascular disease
 existing edema of extremity
 microbiology
 most infections are polymicrobial, with > 1 anerobe
 most common isolate from both cats and dogs is Pasteurella sp.
 gram-negative, facultative, anaerobic, pleomorphic coccobacillus

- 271 -
OrthoBullets2017 Hand Trauma | Other Traumatic Injuries
 inform lab about potential for Pasteurella
 cultures require appropriate growth media and take 1wk to grow
 dog bites
 Pasteurella (50% of dog bite infections)
 Pasteurella canis
 Staphylococcus aureus
 Streptococcus alpha-hemolytic
 Corynebacterium
 anerobes (e.g. Bacteroides)
 Capnocytophaga canimorsus
 rare, potentially fatal (in splenectomy patients)
 causes cellulitis, sepsis, endocarditis, meningitis, DIC, ARDS and death
 highest mortality in immunocompromised (30-60%)
 cat bites
 Pasteurella (most common, 70-80% of cat bite infections)
 Pasteurella multocida and Pasteurella septica
 causes intense pain, swelling in 48h
 other organisms similar to dog bites
o rabies
 caused by a rhabdovirus
 common animal carriers include dogs, raccoons, bats, foxes
 increased risk with open wounds, scratches/abrasions, mucous membranes
 Prognosis
o serious and fatal bites include
 large, aggressive dogs
 small children
 head and neck bites

Presentation
 History
o important to determine
 type of animal
 time since injury
 presence of comorbidities
 Symptoms
o pain and swelling
o bleeding
o signs of local or systemic sepsis
 Physical Exam
o evaluate depth of puncture wound and presence of crush injury
o check for neurovascular status
o look for joint penetration
o important to photograph wounds

Imaging
 Radiographs
o indications to obtain

- 272 -
By Dr, AbdulRahman AbdulNasser Hand Trauma | Other Traumatic Injuries
 crush injuries
 suspected fracture
 suspected foreign body

Studies
 Culture
o indications
 if signs of infection are present
 routine culture not indicated
o technique
 deep aerobic and anaerobic culture

Treatment
 Noperative
o copious irrigation, prophylactic antibiotics, tetanus toxoid, +/- rabies prophylaxis
 copious irrigation in emergency room
 saline (>150ml) irrigation with 18-19G needle or plastic catheter
 use povidone-iodine solution if high risk of rabies
 indications for antibiotics

 cat bites
 presentation >8h
 immune compromised or diabetic
 hand bite
 deep bites
 choice of antibiotics
 amoxicillin/clavulanic acid effective against Pasteurella multocida
 cefuroxime
 ceftriaxone
 rabies prophylaxis
 indicated when any suspicion for rapid animal
 suspect if unprovoked attack by animal with bizarre behavior
 human diploid cell vaccine and human rabies immunoglobulin
 immobilization
 immobilize and elevate extremity
 Operative
o formal surgical debridement
 indications
 crush or devitalized tissue
 foreign body
 bites to digital pulp space, nail bed, flexor tendon sheath, deep spaces of the palm, joint
spaces
 tenosynovitis
 septic arthritis
 abscess formation

- 273 -
OrthoBullets2017 Hand Trauma | Other Traumatic Injuries

3. Nail Bed Injury


Introduction
 Nail bed injuries are the result of direct trauma to the fingertip. Injury types include
o subungual hematoma (details below)
o nail bed laceration
o nail bed avulsion
 Epidemiology
o nail bed injuries are included under the umbrella of fingertip injuries
 finger tip injuries are the most common hand injuries seen in the hospital emergency
department
 Pathophysiology
o mechanisms of injury include
 crushing fingertip between two objects
 catching finger in a closing door
 saw injury
 snowblower injury
 direct blow from a hammer
 Associated conditions : DIP fractures or dislocations
 Prognosis
o early treatment of acute injuries results in the best outcomes with minimal morbidity

Anatomy
 Nailbed and surrounding tissue
o perionychium
 nail
 nailbed
 surrounding skin
o paronychium
 lateral nail folds
o hyponychium
 skin distal distal and palmar to the nail
o eponychium
 dorsal nail fold
 proximal to nail fold
o lunula : white part of the proximal nail
o matrix
 sterile
 soft tissue deep to nail
 distal to lunula
 adheres to nail
 germinal
 soft tissue deep to nail
 proximal to sterile matrix
 responsible for most of nail development
 insertion of extensor tendon is approximately 1.2 to 1.4 mm proximal to germinal matrix

- 274 -
By Dr, AbdulRahman AbdulNasser Hand Trauma | Other Traumatic Injuries
Presentation
 Symptoms
o pain
 Physical exam
o examine for subungual hematoma
o inspect nail integrity
 Imaging
 Radiographs
o recommended
 AP, lateral and oblique of finger
 to rule out fracture of distal phalanx

Subungual Hematoma
 Most commonly caused by a crushing-type injury
o causes bleeding beneath nail
 Treatment
o drainage of hematoma by perforation
 indications
 less than 50% of nail involved
 techniques
 puncture nail using sterile needle
 electrocautery to perforate nail
o nail removal, D&I, nail bed repair
 indications
 > 50 % nail involved
 technique
 nail bed repair (see techniques)

Nail Bed Lacerations


 Laceration of the nail and underlying nail bed
o usually present with the nail intact and a subungual hematoma greater than 50% of nail surface
area
 Treatment
o nail removal with D&I, nail bed repair
 indications
 most cases
 modalities
 tetanus and antibiotic prophylaxis

Avulsion Injuries
 Avulsion of nail and portion of underlying nail bed
 Mechanism
o usually caused by higher energy injuries
 Associated conditions
o commonly associated with other injuries including
 distal phalanx fracture
 if present reduction is advocated

- 275 -
OrthoBullets2017 Hand Trauma | Other Traumatic Injuries
 Treatment
o nail removal, nail bed repair, +/- fx fixation
 indications
 avulsion injury with minimal or no loss of nail matrix, with or without fracture
 technique
 always give tetanus and antibiotics
 fracture fixation depends on fracture type
o nail removal, nail bed repair, split thickness graft vs. nail matrix transfer, +/- fx fixation
 indications
 avulsion or crush injury with significant loss of nail matrix
 technique
 always give tetanus and antibiotics
 nail matrix transfer from adjacent injured finger or nail matrix transfer from second toe
 fracture fixation depends on fracture type

Techniques
 Nail bed repair
o nail removal
 soak nail in Betadine while repairing nail bed
o nail bed repair
‎IV:27 Hook nail
 repair nail bed with 6-0 or smaller absorbable suture
 RCT has demonstrated quicker repair time using 2-octylcyanoacrylate (Dermabond) instead
of suture with comparable cosmetic and functional results
o splint eponychial fold
 splint eponychial fold with original nail, aluminum, or non-adherent gauze

Complications
 Hook nail
o caused by advancement of the matrix to obtain coverage without adequate bony support
 Treatment : remove nail and trim matrix to level of bone
 Split nail
o caused by scarring of the matrix following injury to nail bed
 Treatment
 excise scar tissue and replace nail matrix
 graft may be needed

4. High-Pressure Injection Injuries


Introduction
 Characterized by extensive soft tissue damage associated with a benign high-pressure entry wound
 Epidemiology
o demographics
 most common in laborers in industry using paint, automotive grease, solvents and diesel oil
o location
 the non-dominant index finger is the most commonly affected
 Pathophysiology
o force delivered from 3,000 to 10,000 PSI and up to 400mph

- 276 -
By Dr, AbdulRahman AbdulNasser Hand Trauma | Other Traumatic Injuries
o leads to dissection along planes of least resistance (along neurovascular bundles)
o vascular occlusion may lead to local soft tissue necrosis
 Prognosis
o Up to 50% amputation rate for organic solvents (paint, paint thinner, diesel fuel, jet fuel, oil)
o severity of the injury is dependent on
 time from injury to treatment
 force of injection
 volume injected
 composition of material
 grease, latex, chloroflourocarbon & water based paints are less destructive
 industrial solvents & oil based paints cause more soft tissue necrosis

Presentation
 History
o important to document duration since event
 Physical exam
o inspection
 entry wound often benign looking
 vascular occlusion may lead to local soft tissue necrosis

Imaging
 Radiographs
o may be useful to detect spread of radio-opaque dye

Treatment
 Nonoperative
o tetanus prophylaxis, parenteral antibiotics, limb elevation, early mobilization, monitoring
for compartment syndrome
 indications
 for injection of air and water
 Operative
o irrigation & debridement, foreign body removal and broad-spectrum antibiotics
 indications
 most cases require immediate surgical debridement
 technique
 it is important to remove as much of the foreign material as possible
 broad spectrum antibiotic coverage is important to reduce risk of post operative infection
 outcomes
 higher rates of amputation are seen when surgery is delayed greater than 10 hours after
injury
Complications
 Amputation
o amputation rates approach 50% with oil-based paint injection injuries
 Infection
o necrotic tissue is a good culture medium for bacterial growth

- 277 -
OrthoBullets2017 Hand Trauma | Other Traumatic Injuries

5. Frostbite
Introduction
 Definition
o extensive soft tissue damage associated with exposure to temperatures below freezing point
 Epidemiology
o demographics
 males (m:f = 10:1)
 age 30-50 years
o risk factors
 host factors
 alcohol abuse
 mental illness
 peripheral vascular disease
 peripheral neuropathy
 malnutrition
 chronic illness
 tobacco use
 race
 African descent more likely to sustain frostbite than Caucasians who have better cold
induced vasodilatation
 smoking
 reduces nitric oxide (vasodilator)
 potentiates thrombosis by increasing fibrinogen levels and platelet activity
 environmental factors
 degree of cold temperature
 risk of frostbite is low at > -10°C
 risk of frostbite is high at < -25°C
 duration of exposure
 windchill
 tissues at -18°C freeze in 1h at windspeed of 10mph
 tissues at -18°C freeze in 10min at windspeed of 40mph
 altitude >17,000 feet
 contact with conductive materials (water, ice, metal)
 Pathophysiology
o with hypothermia (CBT <35°C) circulation shunted from periphery to maintain core body
temperature (CBT)
o cardiac effects
 basal metabolic rate, HR and cardiac output drop
 myocardial irritability (abnormal EKG)
o neurological effects
 disorientation, coma
 shivering (anaerobic) until CBT drops below 30-32°C
 below 30-32°C, shivering stops and muscle rigidity ensures (like rigor mortis)
 resembles death (absent respirations, dilated pupils, muscle rigidity)
 must be rewarmed before pronounced dead (“no one is dead until warm and dead”)

- 278 -
By Dr, AbdulRahman AbdulNasser Hand Trauma | Other Traumatic Injuries
o limbs (4 phases)
 phase I (cooling and freezing)
 vasoconstriction/vasospasm followed by transient arteriovenous shunting (hunting
response) of cycles of vasodilatation/vasoconstriction every 10min
 those who do not have this response are more prone to cold injury
 with persistent cold, cycles cease and temperature in tissue drops to freezing point of
tissue (<-2°C)
 ice crystals
 extracellular ice crystals causes sludging/stasis and intracellular dehydration
(because of osmotic gradient)
 intracellular ice crystals destroy cell membranes
 interstitial crystallization is exothermic, maintains latent heat to keep limb above
freezing temperature
 when crystallization is complete, limb temperature falls to ambient temperature
 phase II (rewarming)
 reverses freezing process
 limb absorbs heat, intra/extracellular ice crystals melt
 intracellular swelling occurs
 endothelial cells of capillaries become permeable
 fluid extravasation leads to blisters/edema
 important to prevent re-freezing (freeze-thaw has severe effects on tissues)
 phase III (progressive tissue injury)
 inflammation, stasis/thrombosis, tissue necrosis
 diminished prostaglandin E2 (vasodilator, antiplatelet)
 elevated prostaglandin F2a and thromboxane B2 (vasoconstrictors, platelet-aggregating)
o phase IV (resolution)
 complete healing with no symptoms
 healing with sequelae
 early tissue necrosis/gangrene
o cell biology
 leads to movement of water from intracellular location to extracellular location
 cellular dehydration leads to cell death
o biochemistry
o
 ice crystal formation occurs within the extracellular fluid at -2 to -15°C
 sensory nerve dysfunction occurs at -10°C
 Associated conditions
o frostnip
 mildest cold exposure injury
 only affects superficial layers of skin (blanching, numbness) but no dermis damage
 reversible
o chilblain (pernio)
 occurs in cold, nonfreezing temperatures in dry conditions
 burning sensation, with pruritus, swelling, erythema
 may have blisters, ulceration
 resolves in 2 weeks

- 279 -
OrthoBullets2017 Hand Trauma | Other Traumatic Injuries
 may leave chronic vasculitis esp in young/middle-aged women
o trench foot (immersion foot)
 military personnel
 prolonged wet nonfreezing condition <10°C
o frostbite
 results in localized/extensive tissue necrosis
 may require amputation
o hypothermia
 when core body temperature is affected
 can be fatal
 Prognosis
o the severity is increased with
 alcohol consumption/intoxication
 contact of skin with metal or ice
 elevated wind chill factor

Presentation
 Physical exam
o hypothermia (mild, 32-35°C; moderate, 28-32°C; severe, <28°C)
 tachycardia followed by bradycardia, decreased cardiac output, arrythymia (atrial and
ventricular fibrillation)
 decreased respiratory rate
 CO2 retention leads to hypoxia/respiratory acidosis
 disorientation, comatose
o frostbite (similar to burns)
 traditional classification
st
 1 degree – central whitish area with surrounding erythema
nd
 2 degree – clear/cloudy blisters within 24h
rd
 3 degree – hemorrhagic blisters / hard black eschars
th
 4 degree – tissue necrosis
 newer classification
st nd
 superficial (1 and 2 degree) has good prognosis
rd th
 deep (3 and 4 degree) has poor prognosis
 blisters form 6-24 hours after rewarming
 superficial lesions present as clear blisters
 deeper lesions form hemorrhagic blisters which may be painless

Imaging
 MRI
o T2-weighted images shows enhanced signal in necrotic muscles because of disrupted cell
membranes and increased extracellular fluid
99m
 Serial bone scans ( Tc)
o can be used to evaluate the severity of the soft-tissue damage
o 1st scan at 2 days after initial injury
 absence of uptake has poor prognosis but may not indicate necrosis
o 2nd scan at 5 days after initial injury
 normal blood/bone pool = treat expectantly

- 280 -
By Dr, AbdulRahman AbdulNasser Hand Trauma | Other Traumatic Injuries
 diminished blood/bone pool = observation, with potential early debridement
 absent blood/bone pool = early debridement or amputation
Treatment for Hypothermia
 protect patient from further exposure to freezing temperature
 rewarming
o only after confirmation that the patient can be maintained in a constant warm environment (avoid
freeze-thaw cycles)
o external-surface rewarming (for mild hyperthermia)
 passive
 dry clothes and warm room
 active
 disadvantage is too-rapid vasodilatation leads to metabolic waste rushing to core, leading
to paradoxical drop in core temperature (“afterdrop”) that can worsen arrythmia
 heat lamps, radiant heaters, heating blanket, immersion in warm water with cardiac
monitoring
o internal-core rewarming (for moderate and severe hypothermia)
 warmed oxygen, warm IV fluid
 body cavity lavage (invasive)
 cardiac bypass
 requires systemic heparinization
 continuous arteriovenous rewarming
 blood from femoral arterial catheter into fluid heat exchanger
 returns to body through subclavian venous catheter
 achieves 1°C every 15min
o avoid alcohol/sedatives
 dulls shivering response and further lowers CBT

Treatment for Frostbite


 Nonoperative
o prevention
 footwear thermal insulation is the most important factor for protection against cold induced
injury
o protect limb from mechanical trauma e.g. walking, rubbing
 pad/splint, wrap with blanket for transportation
o initial resuscitation with warm IV fluids, tetanus prophylaxis, NSAIDS, silver sulfadiazine
ointment or topical antibiotics to open wounds, rapid rewarming
 indications
 superficial frostbite
 water bath 40-42°C with mild antibacterial agent x 30min
 successful when skin becomes pliable and red-purple
 avoid repetitive freeze-thaw cycles
 IV analgesia / conscious sedation
 wound care with topical aloe vera, extremity elevation and splinting
o IV antibiotics
 if secondarily infected
o rehabilitation

- 281 -
OrthoBullets2017 Hand Trauma | Other Traumatic Injuries
 whirlpool hydrotherapy
 PT and OT for preserve joint motion
 Adjunctive (low molecular weight dextran, anticoagulants, tissue plasminogen activator)
o intravenous tPA within 24h reduces rate of digital amputations
 indications
 no blood flow on bone scan
 2nd or 3rd degree (NOT superficial frostbite)
 contraindications
 general contraindications
 alcoholic patients (risk of bleeding from concomitant head injuries)
 active internal bleeding
 intracranial hemorrhage/surgery within past 3 months
 concurrent trauma
 major surgery within previous 14 days
 known aneurysm or vascular malformation
 known bleeding diathesis
 pregnancy
 labile hypertension
 cold-related contraindications
 > 24 hours of cold exposure
 warm ischemia times >6h
 multiple freeze-thaw cycles
o hyperbaric oxygen (anecdotal evidence)
 Operative
o immediate surgical escharotomy
 circumferentially constrictive lesion of digit
o fasciotomy
 for compartment syndrome
o debride clear blisters and apply aloe vera
 reduces high levels of prostaglandin F2 and thromboxane B2
o drain/aspirate hemorrhagic blisters (represents deep injury) but leave intact
 prevents dessication of underlying dermis
o late debridement/amputation for necrosis
 “frostbite in January, amputate in July”
 after demarcation occurs at 1-3months
o surgical sympathectomy
 reduces duration of pain and time to demarcation of tissue
 does not reduce extent of necrosis

Complications
 Adults
o persistent pain (50%)
 intolerable in 15%
o cold intolerance
o vasospastic disease (Raynauds phenomenon, cold sensitivity, persistent color changes,
hyperhidrosis)
 treatment
- 282 -
By Dr, AbdulRahman AbdulNasser Hand Trauma | Other Traumatic Injuries
 calcium channel blockers, vasodilators, beta blockers, surgical sympathetectomy
 indications
 late, persistent vasospastic disease
o neuropathy (cold/heat hypersensitivity, hypesthesia, paresthesia)
 decreased motor/sensory NCV
 treatment
 decompression e.g. carpal tunnel release
o musculoskeletal (osteopenia)
 subchondral bone loss (frostbite arthropathy), joint contractures esp in DIPJ > PIPJ of hands
and feet
 treatment
 joint arthroplasty, resection arthroplasty
 Children
o premature growth plate closure
 1-2 years after exposure
 secondary to chondrocytic injury
o joint laxity, angular deformities, short digits, excess skin, degenerative joint changes
 seen after age 10 in patients with prior frost bite injuries
 treatment
 physeal arrest, osteotomy, arthrodesis

Collected By : Dr AbdulRahman
AbdulNasser
drxabdulrahman@gmail.com
In June 2017

- 283 -
OrthoBullets2017 Pelvis Trauma | Other Traumatic Injuries

ORTHO BULLETS

V. Pelvis Trauma

- 284 -
By Dr, AbdulRahman AbdulNasser Pelvis Trauma | Pelvis

A. Pelvis

1. Pelvic Ring Fractures


Introduction
 Mechanism typically high energy blunt trauma
 Mortality rate 15-25% for closed fractures, as much as 50% for open fractures
o hemorrhage is leading cause of death overall
 closed head injury is the most common for lateral compression injuries
o increased mortality associated with
 systolic BP <90 on presentation
 age >60 years
 increased Injury Severity Score (ISS) or Revised Trauma Score (RTS)
 need for transfusion > 4 units
 Associated injuries
o chest injury in up to 63%
o long bone fractures in 50%
o sexual dysfunction up to 50%
o head and abdominal injury in 40%
o spine fractures in 25%
 Prognosis
o high prevalence of poor functional outcome and chronic pain
o poor outcome associated with
 SI joint incongruity of > 1 cm
 high degree initial displacement
 malunion or residual displacement
 leg length discrepancy > 2 cm
 nonunion
 neurologic injury
 urethral injury
 Pediatric pelvic ring fractures
o children with open triradiate cartilage have different fracture patterns than do children whose
triradiate cartilage has closed
 if triradiate cartilage is open the iliac wing is weaker than the elastic pelvic ligaments,
resulting in bone failure before pelvic ring disruption
 for this reason fractures usually involve the pubic rami and iliac wings and rarely require
surgical treatment
Anatomy
 Osteology
o ring structure made up of the sacrum and two innominate bones
o stability dependent on strong surrounding ligamentous structures
o displacement can only occur with disruption of the ring in two places
o neurovascular structures intimately associated with posterior pelvic ligaments
 high index of suspicion for injury of internal iliac vessels or lumbosacral plexus

- 285 -
OrthoBullets2017 Pelvis Trauma | Pelvis
 Ligaments
o anterior
 symphyseal ligaments
 resist external rotation
o pelvic floor
 sacrospinous ligaments
 resist external rotation
 sacrotuberous ligaments
 resist shear and flexion
o posterior sacroiliac complex (posterior tension band)
 strongest ligaments in the body
 more important than anterior structures for pelvic ring stability
 anterior sacroiliac ligaments
 resist external rotation after failure of pelvic floor and anterior structures
 interosseous sacroiliac
 resist anterior-posterior translation of pelvis
 posterior sacroiliac
 resist cephalad-caudad displacement of pelvis
 iliolumbar
 resist rotation and augment posterior SI ligaments

Physical Exam
 Symptoms
o pain & inability to bear weight
 Physical exam
o inspection
 test stability by placing gentle rotational force on each iliac crest
 low sensitivity for detecting instability
 perform only once
 look for abnormal lower extremity positioning
 external rotation of one or both extremities
 limb-length discrepancy
o skin
 scrotal, labial or perineal hematoma, swelling or ecchymosis ‎V:1 Morel-Lavallee lesion
 flank hematoma
 lacerations of perineum
 degloving injuries (Morel-Lavallee lesion)
o neurologic exam
 rule out lumbosacral plexus injuries (L5 and S1 are most common)
 rectal exam to evaluate sphincter tone and perirectal sensation
o urogenital exam
 most common finding is gross hematuria
 more common in males (21% in males, 8% in females)
o vaginal and rectal examinations
 mandatory to rule out occult open fracture

- 286 -
By Dr, AbdulRahman AbdulNasser Pelvis Trauma | Pelvis
Imaging
 Radiographs
o AP Pelvis
 part of initial ATLS evaluation
 look for asymmetry, rotation or
displacement of each hemipelvis
 evidence of anterior ring injury needs
further imaging
o inlet view
 X-ray beam angled ~45 degrees caudad
(may be as little as 25 degrees)
 adequate image when S1 overlaps S2
body
 ideal for visualizing: ‎V:2 Normal AP pelvis

 anterior or posterior translation of the hemipelvis


 internal or external rotation of the hemipelvis
 widening of the SI joint
 sacral ala impaction
o outlet view
 X-ray beam angled ~45 degrees cephalad (may be as much as 60 degrees)
 adequate image when pubic symphysis overlies S2 body
 ideal for visualizing:
 vertical translation of the hemipelvis
 flexion/extension of the hemipelvis
 disruption of sacral foramina and location of sacral fractures
o radiographic signs of instability
 > 5 mm displacement of posterior
sacroiliac complex
 presence of posterior sacral
fracture gap
 avulsion fractures (ischial spine,
ischial tuberosity, sacrum,
transverse process of 5th lumbar
vertebrae)
 CT
o routine part of pelvic ring injury
evaluation
o better characterization of posterior
ring injuries
o helps define comminution and
fragment rotation
o visualize position of fracture lines
relative to sacral foramina

- 287 -
OrthoBullets2017 Pelvis Trauma | Pelvis

Inlet view radiograph CT pelvis Outlet view radiograph

Classification & Treatment


 Tile classification
o A: stable
 A1: fracture not involving the ring (avulsion or iliac wing fracture)
 A2: stable or minimally displaced fracture of the ring
 A3: transverse sacral fracture (Denis zone III sacral fracture)
o B - rotationally unstable, vertically stable
 B1: open book injury (external rotation)
 B2: lateral compression injury (internal rotation)
 B2-1: with anterior ring rotation/displacement through ipsilateral rami
 B2-2-with anterior ring rotation/displacement through contralateral rami (bucket-handle
injury)
 B3: bilateral
o C - rotationally and vertically unstable
 C1: unilateral
 C1-1: iliac fracture
 C1-2: sacroiliac fracture-dislocation
 C1-3: sacral fracture
 C2: bilateral with one side type B and one side type C
 C3: bilateral with both sides type C

- 288 -
By Dr, AbdulRahman AbdulNasser Pelvis Trauma | Pelvis
Young-Burgess Classification

Descriptions Treatment
Anterior Posterior Compression (APC)
APC I Symphysis widening < 2.5 cm Non-operative. Protected weight
bearing
APC II Symphysis widening > 2.5 cm. Anterior SI joint Anterior symphyseal plate or external
diastasis. Posterior SI ligaments intact. Disruption fixator +/- posterior fixation
of sacrospinous and sacrotuberous ligaments.
APC III Disruption of anterior and posterior SI ligaments Anterior symphyseal multi-hole plate or
(SI dislocation). Disruption of sacrospinous and external fixator and posterior
sacrotuberous ligaments. stabilization with SI screws or
APCIII associated with vascular injury plate/screws
Lateral Compression (LC)
LC Type I Oblique or transverse ramus fracture and Non-operative. Protected weight
ipsilateral anterior sacral ala compression bearing (complete, comminuted sacral
fracture. component. Weight bearing as
tolerated (simple, incomplete sacral
fracture).
LC Type II Rami fracture and ipsilateral posterior ilium Open reduction and internal fixation of
fracture dislocation (crescent fracture). ilium
LC Type III Ipsilateral lateral compression and contralateral Posterior stabilization with plate or SI
APC (windswept pelvis). screws as needed. Percutaneous or
Common mechanism is rollover vehicle accident open based on injury pattern and
or pedestrian vs auto. surgeon preference.
Vertical Shear
Vertical shear Posterior and superior directed force. Posterior stabilization with plate or SI
Associated with the highest risk of hypovolemic screws as needed. Percutaneous or
shock (63%); mortality rate up to 25% open based on injury pattern and
surgeon preference.

- 289 -
OrthoBullets2017 Pelvis Trauma | Pelvis
Young-Burgess Classification

APC I APC II x-ray APC II CT

APC III x-ray APC III CT LC Type I x-ray

LC Type I CT LC Type II x-ray LC Type II CT

LC Type III xray LC Type III CT Vertical shear

Bleeding & Initial Treatment


 Bleeding Source
o intraabdominal
o intrathoracic
o retroperitoneal
o extremity (thigh compartments)
o pelvic
 common sources of hemorrhage
- 290 -
By Dr, AbdulRahman AbdulNasser Pelvis Trauma | Pelvis
 venous injury (80%)
 shearing injury of posterior thin walled venous plexus
 bleeding cancellous bone
 uncommon sources of hemorrhage
 arterial injury (10-20%)
 superior gluteal most common (posterior ring injury, APC pattern)
 internal pudendal (anterior ring injury, LC pattern)
 obturator (LC pattern)
 Treatment
o resuscitation
 PRBC:FFP:Platelets ideally should be transfused 1:1:1
 this ratio shown to improve mortality in patients requiring massive
transfusion
o pelvic binder/sheet
 indications
 initial management of an unstable ring injury
 contraindications
 hypothetical risk of over-rotation of hemipelvis and hollow viscus injury (bladder) in
pelvic fractures with internal rotation component (LC)
 no clinical evidence exists of this complication occurring
 technique
 centered over greater trochanters to effect indirect reduction
 do not place over iliac crest/abdomen
 ineffective and precludes assessment of abdomen
 may augment with internal rotation of lower extremities and taping at ankles
 transition to alternative fixation as soon as possible
 prolonged pressure from binder or sheet may cause skin necrosis
 working portals may be cut in sheet to place percutaneous fixation
o external fixation
 indications
 pelvic ring injuries with an external rotation component (APC, VS, CM)
 unstable ring injury with ongoing blood loss
 contraindications
 ilium fracture that precludes safe application
 acetabular fracture
 technique
 theoretically works by decreasing pelvic volume
 stability of bleeding bone surfaces and venous plexus in order to form clot
 pins inserted into ilium
 supra-acetabular pin insertion
 single pin in column of supracetabular bone from AIIS towards PSIS
 obturator outlet view
 helps to identify pin entry point
 iliac oblique view
 helps to direct pin above greater sciatic notch
 obturator oblique inlet view

- 291 -
OrthoBullets2017 Pelvis Trauma | Pelvis
 helps to ensure pin placement within inner and outer table
 AIIS pins can place the lateral femoral cutaneous nerve at risk
 pedicle screws with internal subcutaneous bar may be used
 superior iliac crest pin insertion
 multiple half pins in the superior iliac crest
 place in thickest portion of ilium (gluteal pillar)
 may be placed with minimal fluoroscopy
 should be placed before emergent laparotomy
o angiography / embolization
 indications
 controversial and based on multiple variables including:
 protocol of institution, stability of patient, proximity of angiography suite , availability
and experience of IR staff
 CT angiography useful for determining presence or absence of ongoing arterial
hemorrhage (98-100% negative predictive value)
 contraindications
 not clearly defined
 technique
 selective embolization of identifiable bleeding sources
 in patients with uncontrolled bleeding after selective embolization, bilateral temporary
internal iliac embolization may be effective
 complications include gluteal necrosis and impotence

Definitive Treatment
 Nonoperative
o weight bearing as tolerated
 indications
 mechanically stable pelvic ring injuries including
 LC1
 anterior impaction fracture of sacrum and oblique ramus fractures with < 1cm of
posterior ring displacement
 APC1
 widening of symphysis < 2.5 cm with intact posterior pelvic ring
 isolated pubic ramus fractures
 parturition-induced pelvic diastasis
 bedrest and pelvic binder in acute setting with diastasis less than 4cm
 Operative
o ORIF
 indications
 symphysis diastasis > 2.5 cm
 SI joint displacement > 1 cm
 sacral fracture with displacement > 1 cm
 displacement or rotation of hemipelvis
 open fracture
 chronic pain and diastasis in parturition-induced diastasis or acute setting >6cm
 technique
 for open fractures aggressive debridement according to open fracture principles
- 292 -
By Dr, AbdulRahman AbdulNasser Pelvis Trauma | Pelvis
o anterior subcutaneous pelvic fixator (INFIX)
 indications : same indications as anterior external fixation and symphyseal plating
o diverting colostomy
 indications
 consider in open pelvic fractures
 especially with extensive perineal injury or rectal involvement

Techniques
 Anterior ring stabilization
o single superior plate
 apply through rectus-splitting Pfannenstiel approach
 may perform in conjunction with laparotomy or GU procedure
 Posterior ring stabilization
o anterior SI plating
 risk of L4 and L5 injury with placement of anterior sacral retractors
o iliosacral screws (percutaneous)
 good for sacral fractures and SI dislocations
 safe zone is in S1 vertebral body
 outlet radiograph view best guides superior-inferior screw placement
 inlet radiograph view best guides anterior-posterior screw placement
 L5 nerve root injury complication with errors in screw placement
 entry point best viewed on lateral sacral view and pelvic outlet views
 risk of loss of reduction highest in vertical sacral fracture patterns
o posterior SI "tension" plating
 can have prominent HW complications
 Anterior and posterior ring stabilization
o necessary in vertically unstable injuries
 Ipsilateral acetabular and pelvic ring fractures
o reduction and fixation of the pelvic ring should be performed first

Complications
 Neurologic injury
o L5 nerve root runs over sacral ala joint
o may be injured if SI screw is placed to anterior
o anterior subcutaneous pelvic fixator may give rise to LFCN injury (most common) or femoral
nerve injury
 DVT and PE
o DVT in ~ 60%, PE in ~ 27%
o prophylaxis essential
 mechanical compression
 pharmacologic prevention (LMWH or Lovenox)
 vena caval filters (closed head injury)
 Chronic instability
o rare complication; can be seen in nonoperative cases
o presents with subjective instability and mechanical symptoms

o diagnosed with alternating single-leg-stance pelvic radiographs

- 293 -
OrthoBullets2017 Pelvis Trauma | Pelvis
Urogenital Injuries
 Present in 12-20% of patients with pelvic fractures
o higher incidence in males (21%)
 Includes
o posterior urethral tear
 most common urogenital injury with pelvic ring fracture
o bladder rupture

 may see extravasation around the pubic symphysis


 associated with mortality of 22-34%
 Diagnosis
o made with retrograde urethrocystogram
o indications for retrograde urethrocystogram include
 blood at meatus
 high riding or excessively mobile prostate
 hematuria
 Treatment
o suprapubic catheter placement
 suprapubic catheter is a relative contraindication to anterior ring plating
o surgical repair
 rupture should be repaired at the same time or prior to definitive fixation in order to minimize
infection risk
 Complications
o long-term complications common (up to 35%)
 urethral stricture - most common
 impotence
 anterior pelvic ring infection
 incontinence
 parturition sequelae (i.e. caesarean section)

2. SI Dislocation & Crescent Fractures


Introduction
 Spectrum of injuries that include
o incomplete (Sacroiliac) SI dislocation
 posterior SI ligaments remain intact
 rotationally unstable
o complete SI dislocations
 posterior SI ligaments ruptured
 vertically and rotationally unstable
o SI fracture-dislocation (crescent fracture)
 iliac wing fracture that enters the SI joint
 injury to posterior ligaments vary
 combination of vertical iliac fx and SI dislocation
 posterior ilium remains attached to sacrum by posterior SI ligaments
 anterior ilium dislocates from sacrum with internal rotation deformity
 when ilium fragment remains with sacrum it is termed a crescent fracture
- 294 -
By Dr, AbdulRahman AbdulNasser Pelvis Trauma | Pelvis
 Pathophysiology
o mechanism of injury
 lateral compression force
 usually high energy
o pathoanatomy
 degree of injury to posterior structures determines pelvic stability
 Iliac wing fractures may be associated with open wounds and may involve bowel entrapment
 Prognosis : primarily based on accurate and stable reduction of SI joint

Anatomy
 Ligaments
o the SI joint is stabilized by the posterior pelvic ligaments
 sacrospinous
 sacrotuberous
 anterior sacroiliac
 posterior sacroiliac
 Nerves
o the L5 nerve root crosses the sacral ala approximately 2 cm
medial to SI joint
 Blood supply
o the superior gluteal artery runs across SI joint
o exits pelvis via greater sciatic notch

Classification
 No classification system specifically for SI injury
o included in Young- Burgess and Tile classification of pelvic fractures
o crescent fractures described as LC-2 injury according to Young-Burgess

Presentation
 Symptoms : pelvic pain
 Physical Exam
o assess hemodynamic status
o perform detailed neurological exam
o abdominal assessment to look for distention
o rectal exam
o examine urethral meatus for blood

Imaging
 Radiographs : recommended views
 AP pelvis
 inlet and outlet views

 CT scan
o evaluation of sacral fractures
o posterior pelvis better delineated

Treatment
 Operative

- 295 -
OrthoBullets2017 Pelvis Trauma | Pelvis
o immediate skeletal traction
 indications : vertical translation of the hemipelvis
o anterior ring ORIF
 indications
 incomplete SI dislocations with pubic symphyseal diastasis
o anterior and posterior ring ORIF
 indications
 complete SI dislocations
 vertically unstable require anterior and posterior pelvic ring fixation
o ORIF of ilium
 indications
 crescent fracture : required to restore posterior SI ligaments and pelvic stability

Techniques
 Closed Reduction and Percutaneous Fixation
o positioning
 intraoperative traction may aid in reduction
 small midline bump under sacrum may assist with SI screw placement
o imaging
 inlet view : shows anterior-posterior position of SI joint(s) for screw placement
 outlet view : shows cephalad-caudad position of SI joint(s) for screw placement
 lateral sacral view
 ensures safe placement of SI or sacral screws relative to the anterior cortex of the sacral
ala and the nerve root tunnel
o complications
 L5 nerve root at risk with anterior perforation of iliosacral screw as nerve goes inferiorly over
sacral ala
 ORIF
o approach
 anterior approach : lateral window with elevation iliacus back to SI joint
 posterior approach : for fixation of crescent fragment to intact ilium
o fixation
 plates
 iliosacral lag screws (SI screws)

Complications
 DVT : 35%-50%
 Neurological injury
 Loss of reduction and failure of fixation

- 296 -
By Dr, AbdulRahman AbdulNasser Pelvis Trauma | Pelvis

3. Sacral Fractures
Introduction
 Under-diagnosed and often mistreated fractures that may result in neurologic compromise
o common in pelvic ring injuries (30-45%)
o 25% are associated with neurologic injury
o frequently missed
 75% in patients who are neurologically
intact
 50% in patients who have a neurologic
deficit
 Epidemiology
o young adults : as a result of high energy trauma
o elderly : as a result of low energy falls
 Prognosis
o presence of a neurologic deficit is the most
important factor in predicting outcome
o mistreated fractures may result in
 lower extremity deficits
 urinary dysfunction
 rectal dysfunction
 sexual dysfunction

Anatomy
 Osteology
o formed by fusion of 5 sacral vertebrae
o articulates with
 5th lumbar vertebra proximally
 coccyx distally
 ilium laterally at sacroiliac joints
o contains 4 foramina which transmit sacral nerves
 Nerves
o L5 nerve root runs on top of sacral ala
o S1-S4 nerve roots are transmitted through the sacral

foramina ‎V:3 Nerves of the sacrum , P = pudendal nerve , SN =


 S1 and S2 nerve roots carry higher rate of injury sciatic nerve, LST = lumbosacral trunk
o lower sacral nerve roots (S2-S5)
 function
 anal sphincter tone / voluntary contracture
 bulbocavernosus reflex
 perianal sensation
 unilateral preservation of nerves is adequate for bowel and bladder control
 Biomechanics
o transmission of load distributed by first sacral segment through iliac wings to the acetabulum

- 297 -
OrthoBullets2017 Pelvis Trauma | Pelvis
Classification
 Denis classification
o zone 1
 fracture lateral to foramina
 characteristics
 most common (50%)
 nerve injury rare (5%)
 usually occurs to L5 nerve root
o zone 2
 fracture through foramina
 characteristics
 may be
 stable
 unstable
 zone 2 fracture with shear component highly unstable
 increased risk of nonunion and poor functional outcome
o zone 3
 fracture medial to foramina into the spinal canal
 characteristics
 highest rate of neurologic deficit (60%)
 bowel, bladder, and sexual dysfunction
 Transverse sacral fractures
o higher incidence of nerve dysfunction
 U-type sacral fractures
o results from axial loading
o represent spino-pelvic dissociation
o high incidence of neurologic complications

Zone 1 Zone 2 Zone 3

Transverse sacral fractures U-shape sacral fractures

- 298 -
By Dr, AbdulRahman AbdulNasser Pelvis Trauma | Pelvis
Presentation
 History
o motor vehicle accident or fall from height most common
o repetitive stress
 insufficiency fracture in osteoporotic adults
 Symptoms
o peripelvic pain
 Physical exam
o inspection
 soft tissue trauma around pelvis should raise concerns for pelvic or sacral fracture
o palpation
 test pelvic ring stability by internally and externally rotating iliac wings
 palpate for subcutaneous fluid mass indicative of lumbosacral fascial degloving (Morel-
Lavallee lesion)
 perform vaginal exam in women to rule-out open injury
o neurologic exam
 rectal exam
 light touch and pinprick sensation along S2-S5 dermatomes
 perianal wink
 bulbocavernosus and cremasteric reflexes
o vascular exam
 distal pulses
 if different consider ankle-brachial index or angiogram

Imaging
 Radiographs
o only show 30% of sacral fractures
o recommended views
 AP pelvis
 inlet view
 best assessment of sacral spinal canal and superior view of S1
 outlet view
 provides true AP of sacrum
o additional views
 cross-table lateral
 effective screening tool for sacral fractures
 often of poor quality
o findings
 L4 or L5 transverse process fractures
 asymmetric foramina
 CT
o diagnostic study of choice
o recommend coronal and sagittal reconstruction views
 MRI
o recommended when neural compromise is suspected ‎V:4 Cross table lateral view

- 299 -
OrthoBullets2017 Pelvis Trauma | Pelvis
Treatment
 Nonoperative
o progressive weight bearing +/- orthosis
 indications
 <1 cm displacement and no neurologic deficit
 insufficiency fractures
 Operative
o surgical fixation
 indications
 displaced fractures >1 cm
 soft tissue compromise
 persistent pain after non-operative management
 displacement of fracture after non-operative management
o surgical fixation with decompression
 indications
 any evidence of neurologic injury

Surgical Techniques
 Percutaneous screw fixation
o screws may be placed as sacroiliac, trans-sacral or trans-iliac trans-sacral
o useful for sagittal plane fractures
o technique
 screws placed percutaneously under fluoroscopy
 beware of L5 nerve root
 avoid overcompression of fracture
 may cause iatrogenic nerve dysfunction
o cons
 may result in loss of fixation or malreduction
 does not allow for removal of loose bone fragments
 do not use in osteoporotic bone
 Posterior tension band plating
o approach : posterior two-incision approach
o technique
 may use in addition to iliosacral screws
o pros : allows for direct visualization of fracture
o cons : wound healing complications
 Iliosacral and lumbopelvic fixation
o approach
 posterior approach to lower lumbar spine and sacrum
o technique
 pedicle screw fixation in lumbar spine
 iliac screws parallel to the inclination angle of outer table of ilium
 longitudinal and transverse rods
o pros
 shown to have greatest stiffness when used for an unstable sacral fracture
o cons
 invasive
- 300 -
By Dr, AbdulRahman AbdulNasser Pelvis Trauma | Pelvis
 Decompression of neural elements
o technique
 indirect
 reduction through axial traction
 direct
 posterior approach followed by laminectomy or foraminotomy

Complications
 Venous thromboembolism
o often as a result of immobility
 Iatrogenic nerve injury
o may result from
 overcompression of fracture
 improper hardware placement
 Malreduction
o more common with vertically displaced fractures

4. Ilium Fractures
Introduction
 Most are unstable fractures
 Typically progress from iliac crest to greater sciatic notch
 Iliac wing fractures have high incidence of associated injuries
o open injuries
o bowel entrapment
o soft tissue degloving

Anatomy
 Osteology
o pelvic girdle is comprised of
 sacrum
 2 innominate (coxal) bones
 each formed from the union of 3 bones: ilium, ischium, and pubis
o ilium
 2 important anterior prominences
 anterior-superior iliac spine (ASIS)
 origin of sartorius and transverse and internal abdominal muscles
 anterior-inferior iliac spine (AIIS)
 origin of direct head of rectus femoris and iliofemoral ligament (Y ligament of
Bigelow)
 posterior prominences
 posterior-superior iliac spine (PSIS)
 located 4-5 cm lateral to the S2 spinous process
 posterior-inferior iliac spine (PIIS)

Imaging
 Plain radiographs

- 301 -
OrthoBullets2017 Pelvis Trauma | Pelvis
o standard set of AP pelvis, inlet/outlet, and judet views
 helpful for evaluating the iliac wing in addition to pelvic stability and possible acetabular
involvement
 CT scan
o carefully assess CT scan for signs of bowel entrapment
o evaluate for presence of gas or air in the soft tissues which can be associated with open injury or
bowel disruption
Classification
 No specific classification for iliac wing fractures
 Generally described as specific subtypes of more common classification systems
o Tile Classification
 stable (intact posterior arch)
 A1-1: iliac spine avulsion injury
 A1-2: iliac crest avulsion
 A2-1: iliac wing fractures often from a direct blow
 partially stable (incomplete disruption of posterior arch)
 B2-3: incomplete posterior iliac fracture
 unstable (complete disruption of posterior arch)
 C1-1: unilateral iliac fracture

Treatment
 Nonoperative
o mobilization with an assist device
 indications
 nondisplaced fractures
 isolated iliac wing fractures
 Operative
o open reduction and internal fixation
 indications
 displaced fractures of ilium

Operative Techniques
 Wound Management
o evaluate all wounds for
 soft tissue disruption or internal degloving injury
 possible soft tissue or bowel entrapment in the fracture site
o prophylactic antibiotics as appropriate
o serial debridements as necessary
 Open Reduction Internal Fixation
o approach
 posterior approach
 ilioinguinal approach
 Stoppa approach (lateral window)
o recommend early reconstruction
 single pelvic reconstruction plate or lag screw along the iliac crest

- 302 -
By Dr, AbdulRahman AbdulNasser Pelvis Trauma | Acetabulum
 supplemented with a second reconstruction plate or lag screw at the level of the pelvic brim
or sciatic buttress
o coordination with trauma team
 injury to bowel may require diversion procedures
 plan surgical intervention with trauma team to minimize recurrent trips to the operating room

Complications
 Malunion with deformity of the iliac wing
 Internal iliac artery injury
 Bowel perforation
 Lumbosacral plexus injury

B. Acetabulum

1. Acetabular Fractures
Introduction
 Epidemiology
o demographics
 bimodal distribution
 high energy blunt trauma for young patients
 low energy (fall from standing height) for elderly patients
o location
 posterior wall fractures are most common
 Pathoanatomy
o fracture pattern determined by
 force vector
 position of femoral head at time of injury
 Associated conditions
o orthopaedic manifestations
 extremity injury (36%)
 nerve palsy (13%)
 spine injury (4%)
o systemic injuries
 head injury (19%)
 chest injury (18%)
 abdominal injury (8%)
 genitourinary injury (6%)
 Classification Systems
o Judet and Letournel
 classifed as 5 elementary and 5 associated fracture patterns
o AO/OTA Classification

Anatomy
 Osteology
o acetabular inclination & anteversion
 mean lateral inclination of 40 to 48 degrees
- 303 -
OrthoBullets2017 Pelvis Trauma | Acetabulum
anteversion of 18 to 21 degrees
o column theory
 acetabulum is supported by two columns of bone
 form an "inverted Y"
 connected to sacrum through sciatic buttress
 posterior column
 comprised of
 quadrilateral surface
 posterior wall and dome
 ischial tuberosity
 greater/lesser sciatic notches
 anterior column
 comprised of
 anterior ilium (gluteus medius tubercle)
 anterior wall and dome
 iliopectineal eminence
 lateral superior pubic ramus
 Vascular ‎V:5 column theory form inverted Y
o corona mortis
 anastomosis of external iliac (epigastric) and internal iliac (obturator) vessels
 at risk with lateral dissection over superior pubic ramus

‎V:6 Corona mortis


‎ :7 Anterior column in blue , posterior column in
V
brown

- 304 -
By Dr, AbdulRahman AbdulNasser Pelvis Trauma | Acetabulum
Letournel Classification

Elementary
• Most common
Posterior wall • "gull sign" on obturator oblique view
Posterior column • check for injury to superior gluteal NV bundle
Anterior wall • Very rare
Anterior column • More common in elderly patients with fall from standing (most common in
elderly is "anterior column + medial wall")
Transverse • Axial CT shows anterior to posterior fx line
• Only elementary fx to involve both columns
Associated
Associated Both Column • Characterized by dissociation of the articular surface from the inonimate bone
• will see "spur sign" on obturator oblique
Transverse + Post. Wall • Most common associated fx
T Shaped • May need combined approach
Anterior column or wall + • Common in elderly patients
Post. hemitransverse
Post. column + Post. wall • Only associated fracture that does not involve both columns

- 305 -
OrthoBullets2017 Pelvis Trauma | Acetabulum

CT posterior wall fx Oblique view post wall fx CT anterior wall fx

CT Posterior column fx posterior column fx X-ray anterior wall

x-ray anterior column fx CT anterior column fx x-ray and CT of transverse fx

Associated Both Column fx . AP and both oblique views

Transverse + Post. Wall CT Transverse + Post. Wall x-ray Transverse + Post. Wall CT

- 306 -
By Dr, AbdulRahman AbdulNasser Pelvis Trauma | Acetabulum
Imaging
 Radiographs
o recommended views
 AP pelvis, Judet views, inlet and outlet if
concerned for pelvic ring involvement
o 6 radiographic landmarks of the acetabulum
 iliopectineal line (anterior column)
 ilioischial line (posterior column)
 anterior rim
 posterior rim
 teardrop
 weight bearing roof
o superior acetabular rim may show os acetabuli
marginalis superior which can be confused for fracture in adolescents
o Judet views (45 degree oblique views)
 obturator oblique
 shows profile of obturator foramen
 shows anterior column and posterior wall
 iliac oblique
 shows profile of involved iliac wing
 shows posterior column and anterior wall
o roof arc measurements
 show intact weight bearing dome if > 45 degrees on
AP, obturator, and iliac oblique
 not applicable for associated both column or posterior
wall pattern because no intact portion of the
acetabulum to measure
 CT scan
o important to
 define fragment size and orientation
 identify marginal impaction
 identify loose bodies
 look for articular gap or step-off

obturator oblique iliac oblique roof arc measurements Normal CT

- 307 -
OrthoBullets2017 Pelvis Trauma | Acetabulum
Treatment
 Nonoperative
o protected weight bearing for 6-8 weeks
 indications
 minimally displaced fracture (< 2mm)
 < 20% posterior wall fractures
 treatment based on size of posterior wall is controversial
 exam under anesthesia using fluoroscopy best method to test stability
 femoral head remains congruent with weight bearing roof (out of traction)
 both column fracture with secondary congruence (out of traction)
 displaced fracture with roof arcs > 45 degrees in AP and Judet views
 relative contraindications to surgery
 morbid obesity
 open contaminated wound
 presence of DVT
 technique
 lowest joint reactive forces seen with toe-touch weight
bearing and passive hip abduction
 greatest joint contact force seen when rising from a
chair on the affecdted extremity
 close radiographic follow-up
 skeletal traction rarely indicated as definitive treatment
 Operative treatment
o open reduction and internal fixation
 indications
 displacement of roof (>2mm)
 posterior wall fracture involving > 40-50%
 marginal impaction
 intra-articular loose bodies
 irreducible fracture-dislocation
 pregnancy is not contraindication to surgical fixation
 outcomes
 associated hip dislocations should be reduced within 12 hours for improved outcomes
 clinical outcome correlates with quality of articular reduction
 earlier operative treatment associated with increased chance of anatomic reduction
 postoperative CT scan is most accurate way to determine posterior wall accuracy of
reduction which has greatest correlation with clinical outcome
 greatest stress on acetabular repair occurs when rising from a seated position using the
affected leg, and occurs in the posterior superior portion of the acetabulum
 functional outcomes most strongly correlate with hip muscle strength and restoration of
gait postoperatively
o open reduction and internal fixation with acute total hip arthroplasty
 indications
 significant osteopenia and/or significant comminution
 outcomes
 up to 78% 10-year implant survival noted

- 308 -
By Dr, AbdulRahman AbdulNasser Pelvis Trauma | Acetabulum
 worse outcomes in males, patients <50 years old or >80kg, or if a significant acetabular
defect remains
o percutaneous fixation with column screws
 indications
 anterior column screws

Techniques
 Percutaneous fixation with column screws
o approach
 anterograde (from iliac wing to ramus)
 retrograde (from ramus to iliac wing)
 posterior column screws
o imaging
 obturator oblique best view to rule out joint penetration
 inlet iliac oblique view best to determine anteroposterior position of screw within the pubic
ramus
 obturator oblique inlet view best to determine position of a supraacetabular screw within
tables of the ilium
 ORIF
o approaches
 approach depends on fracture pattern
 two approaches can be combined

- 309 -
OrthoBullets2017 Pelvis Trauma | Acetabulum
Approaches Indications Risks
Anterior Approach • anterior wall and anterior column • femoral nerve injury
(Ilioinguinal) • both column fracture • LFCN injury
• posterior hemitransverse • thrombosis of femoral vessels
• laceration of corona mortis in 10-15%.
Posterior Approach • posterior wall and posterior column fx • increased HO risk compared with
(Kocher-Langenbach) • most transverse and T-shaped anterior approach
• combination of above •sciatic nerve injury (2-10%)
• damage to blood supply of femoral head
(medial femoral circumflex)
Extensile Approach • only single approach that allows direct • massive heterotopic ossification
(extended iliofemoral) visualization of both columns • posterior gluteal muscle necrosis
• associated fracture pattern 21 days after
injury
• some transverse fxs and T types
• some both column fxs (if posterior
comminution is present)
Modified Stoppa • access to quadrilateral plate to buttress • Corona mortis must be exposed and
Approach comminuted medial wall fractures ligated in this approach

Complications
 Post-traumatic DJD
o most common complication
o 80% survival noted at 20 years for patients s/p ORIF
o risk factors for DJD include
 age >40
 associated fracture patterns
 concomitant femoral head injury
o treat with hip fusion or THA
 Heterotopic Ossification
o highest incidence with extensile approach
 treat with
 indomethacin x 5 weeks post-op
 low dose external radiation (no difference shown in direct comparison)
o lowest incidence with anterior ilioinguinal approach
 Osteonecrosis
o 6-7% of all acetabular fractures
o 18% of posterior fracture patterns
 DVT and PE
 Infection
 Bleeding
 Neurovascular injury
 Intraarticular hardware placement
 Abductor muscle weakness

- 310 -
By Dr, AbdulRahman AbdulNasser Pelvis Trauma | Acetabulum

2. Hip Dislocation
Introduction
 Epidemiology
o rare, but high incidence of associated injuries
o mechanism is usually young patients with high energy trauma
 Hip joint inherently stable due to
o bony anatomy
o soft tissue constraints including
 labrum
 capsule
 ligamentum teres ‎V:8 Dashboard injury

Classification
 Simple vs. Complex
o simple
 pure dislocation without associated fracture
o complex
 dislocation associated with fracture of acetabulum or
proximal femur
 Anatomic classification
o posterior dislocation (90%)
 occur with axial load on femur, typically with hip flexed ‎V:9 Clinical picture of
and adducted posterior dislocation
 axial load through flexed knee (dashboard injury)
 position of hip determines associated acetabular injury
 increasing flexion and adduction favors simple dislocation
 associated with
 osteonecrosis
 posterior wall acetabular fracture
 femoral head fractures
 sciatic nerve injuries
 ipsilateral knee injuries (up to 25%)
o anterior dislocation
 associated with femoral head impaction or chondral injury
 occurs with the hip in abduction and external rotation
 inferior ("obturator") vs. superior ("pubic")
 hip extension results in a superior (pubic) dislocation
 Clinically hip appears in extension and external rotation
 flexion results in inferior (obturator) dislocation
 Clinically hip appears in flexion, abduction, and external rotation

Presentation
 Symptoms
o acute pain, inability to bear weight, deformity
 Physical exam
o ATLS
- 311 -
OrthoBullets2017 Pelvis Trauma | Acetabulum
 95% of dislocations with associated injuries
o posterior dislocation (90%)
 hip and leg in slight flexion, adduction, and internal rotation
 detailed neurovascular exam (10-20% sciatic nerve injury)
 examine knee for associated injury or instability
 chest X-ray ATLS workup for aortic injury
o anterior dislocation
 hip and leg in flexion, abduction, and external rotation

Imaging
 Radiographs
o can typically see posterior dislocation on AP pelvis ‎V:10 Anteior dislocation
 femoral head smaller then contralateral side
 Shenton's line broken
 lesser trochanter shadow reveals internally rotated limb as compared to contralateral side
 scrutinize femoral neck to rule out fracture prior to attempting closed reduction
 CT
o helps to determine direction of dislocation, loose bodies, and associated fractures
 anterior dislocation
 posterior dislocation
o post reduction CT must be performed for all traumatic hip
dislocations to look for
 femoral head fractures
 loose bodies
 acetabular fractures
 MRI
o controversial and routine use is not currently supported
o useful to evaluate labrum, cartilage and femoral head vascularity

Loose fragment in
Anterior dislocation Posterior dislocation Associated neck fx the joint AP view posterior dislocation

Treatment
 Nonoperative
o emergent closed reduction within 6 hours
 indications
 acute anterior and posterior dislocations
 contraindications
 ipsilateral displaced or non-displaced femoral neck fracture
 Operative

o open reduction and/or removal of incarcerated fragments ‎V:11 Shenton’s line

 indications
 irreducible dislocation
 radiographic evidence of incarcerated fragment

- 312 -
By Dr, AbdulRahman AbdulNasser Pelvis Trauma | Acetabulum
 delayed presentation
 non-concentric reduction
 should be performed on urgent basis
o ORIF
 indications
 associated fractures of
 acetabulum
 femoral head
 femoral neck : should be stabilized prior to reduction
o arthroscopy
 indications
 no current established indications
 potential for removal of intra-articular fragments
 evaluate intra-articular injuries to cartilage, capsule, and labrum

Techniques
 Closed reduction
o perform with patient supine and apply traction in line with deformity regardless of direction of
dislocation
o must have adequate sedation and muscular relaxation to perform reduction
o assess hip stability after reduction
o post reduction CT scan required to rule out
 femoral head fractures
 intra-articular loose bodies/incarcerated fragments
 may be present even with concentric reduction on plain films
 acetabular fractures
o post-reduction : for simple dislocation, follow with protected weight bearing for 4-6 weeks
 Open reduction
o approach
 posterior dislocation : posterior (Kocher-Langenbeck) approach
 anterior dislocation : anterior (Smith-Petersen) approach
o technique
 may place patient in traction to reduce forces on cartilage due to incarcerated fragment or in
setting of unstable dislocation
 repair of labral or other injuries should be done at the same time

Complications
 Post-traumatic arthritis
o up to 20% for simple dislocation, markedly increased for complex dislocation
 Femoral head osteonecrosis : 5-40% incidence
o Increased risk with increased time to reduction
 Sciatic nerve injury : 8-20% incidence
o associated with longer time to reduction
 Recurrent dislocations : less than 2%

- 313 -
OrthoBullets2017 Lower Extremity | Acetabulum

ORTHO BULLETS

VI. Lower Extremity

- 314 -
By Dr, AbdulRahman AbdulNasser Lower Extremity | Femur

A. Femur

1. Femoral Head Fractures


Introduction
 A rare fracture pattern that is usually associated with hip dislocations
o the location and size of the fracture fragment and degree of comminution depend on the position
of the hip at the time of dislocation
 Epidemiology
o incidence
 rare
 increasing because of more MVA and better
resuscitation
 Mechanism
o impaction, avulsion or shear forces involved
 unrestrained passenger MVA (knee against dashboard)
 falls from height
 sports injury
 industrial accidents
o 5-15% of posterior hip dislocations are associated with a femoral head fracture
 because of contact between femoral head and posterior rim of acetabulum
o anterior hip dislocations usually associated with impaction/indentation fractures of the femoral
head
 Associated conditions
o femoral neck fracture (see Pipkin Classification below)
o acetabular fracture (see Pipkin Classification below)
o sciatic nerve neuropraxia
o femoral head AVN
o ipsilateral knee ligamentous instability (knee vs dashboard)
 Anatomy
 Blood supply
o the femoral head has 3 sources of arterial supply
 extracapsular arterial ring
 medial circumflex femoral artery (main supply to the head)
 from profunda femoris
 lateral circumflex femoral artery
 ascending cervical branches
 artery to the ligamentum teres
 from the obturator artery or MCFA
 supplies perifoveal area

- 315 -
OrthoBullets2017 Lower Extremity | Femur
Classification
Pipkin Classification
Type I Fx below fovea/ligamentum (small)
Does not involve the weightbearing portion of the femoral
head

Type II Fx above fovea/ ligamentum (larger)


Involves the weightbearing portion of the femoral head

Type III Type I or II with associated femoral neck fx


High incidence of AVN

Type IV Type I or II with associated acetabular fx (usually posterior


wall fracture)

Pipkin I Pipkin II Pipkin III Pipkin IV

Presentation
 History
o frontal impact MVA with knee striking dashboard
o fall from height
 Symptoms
o localized hip pain
o unable to bear weight
o other symptoms associated with impact
 Physical exam
o inspection
 shortened lower limb
 with large acetabular wall fractures, little to no rotational asymmetry is seen
 posterior dislocation
 limb is flexed, adducted, internally rotated
 anterior dislocation
 limb is flexed, abducted, externally rotated
o neurovascular
 may have signs of sciatic nerve injury

Imaging
 Radiographs
o recommended views
- 316 -
By Dr, AbdulRahman AbdulNasser Lower Extremity | Femur
AP pelvis, lateral hip and Judet views
 both pre-reduction and post-reduction
 inlet and outlet views
 if acetabular or pelvic ring injury suspected
 CT scan
o indications
 after reduction
 to evaluate:
 concentric reduction
 loose bodies in the joint
 acetabular fracture
 femoral head or neck fracture
o findings
 femoral head fracture
 intra-articular fragments
 posterior pelvic ring injury
 impaction
 acetabular fracture

Treatment
 Nonoperative
o hip reduction
 indications ‎VI:1 Fixation of head femur by scews
 acute dislocations
 reduce hip dislocation within 6 hours
 technique
 obtain post reduction CT
o TDWB x 4-6 weeks, restrict adduction and internal rotation
 indications
 Pipkin I
 undisplaced Pipkin II with < 1mm step off
 no interposed fragments
 stable hip joint
 technique
 perform serial radiographs to document maintained reduction
 Operative
o ORIF
 indications
 Pipkin II with > 1mm step off
 if performing removal of loose bodies in the joint
 associated neck or acetabular fx (Pipkin type III and IV)
 polytrauma
 irreducible fracture-dislocation
 Pipkin IV
 treatment dictated by characteristics of acetabular fracture
 small posterior wall fragments can be treated nonsurgically and suprafoveal fractures
can then be treated through an anterior approach
- 317 -
OrthoBullets2017 Lower Extremity | Femur
 outcomes
 outcomes mimic those of their associated injuries (hip dislocations and femoral neck
fractures)
 poorer outcomes associated with
 use of posterior (Kocher-Langenbeck) approach
 use of 3.0mm cannulated screws with washers
o arthroplasty
 indications
 Pipkin I, II (displaced), III, and IV in older patients
 fractures that are significantly displaced, osteoporotic or comminuted

Surgical Techniques
 ORIF of femoral head (Pipkin I, II, III)
o approach
 anterior (Smith-Peterson) approach
 the anterior (Smith-Peterson) and anterolateral (Watson-Jones) approaches provide the
best visualization of the head compared with the posterior approach
 utilizes internervous plane between the superior gluteal and femoral nerves
 no increased risk of AVN
 shorter surgical time
 less blood loss
 ease of reduction and fixation
 because femoral head fragment is commonly anteromedial
 can use surgical hip dislocation if needed

 anterolateral (Watson-Jones)
 utilizes intermuscular plane between the tensor
fascia lata and gluteus medius (both superior
gluteal nerve)
o exposure
 periacetabular capsulotomy to preserve blood supply
to femoral head
o fixation
 two or more 2.7mm or 3.5mm lag screws
 countersink the heads of the screws to avoid screw head
prominence
 headless compression screws
 bioabsorbable screws
o postop
 rehabilitation
 mobilization
 immediate early range of motion
 weightbearing
 delay weight bearing for 6-8 weeks
 stress strengthening of the quadriceps and abductors
 radiographs
 radiographs after 6 months to evaluate for AVN and osteoarthritis

- 318 -
By Dr, AbdulRahman AbdulNasser Lower Extremity | Femur
 ORIF of femoral head and acetabulum (Pipkin IV)
o approach
 posterior (Kocher-Langenbeck) approach with digastric osteotomy
 provides the best visualization of femoral head fracture and acetabular posterior wall
fracture
 preserves the medial circumflex artery supply to the femoral head
 utilizes plane created by splitting of gluteus maximus (no true internervous plane
 gluteus maximus is not denervated because it receives nerve supply well medial to the
split
 anterior (Smith-Peterson) approach
 for fixation of suprafoveal fractures (if posterior wall fragments are small, they can be
treated nonsurgically)
 Arthroplasty
o approach
 can use any hip approach for arthroplasty
 posterior (Kocher-Langenbeck) approach provides the best visualization of
acetabular posterior wall fracture
o pros & cons
 allows immediate postoperative mobilization and weightbearing
 hemiarthroplasty can be utilized if no acetabular fracture present
 total hip arthroplasty favored if patient physiologically younger or if acetabular fracture
present
Complications
 Heterotopic ossification
o overall incidence is 6-64%
 anterior approach has increased heterotopic ossification
compared with posterior approach
o treatment
 administer radiation therapy if there is concern for HO
 especially if there is associated head injury
 AVN
o incidence is 0-23%
 risk is greater with delayed reduction of dislocated hip
 the impact of anterior incision on AVN is unknown
‎VI:2 Heterotopic ossification
 Sciatic nerve neuropraxia
o incidence is 10-23%
 usually peroneal division of sciatic nerve
 spontaneous recovery of function in 60-70%
 DJD
o incidence 8-75%
o due to joint incongruity or initial cartilage damage
o Decreased internal rotation : may not be clinically problematic or cause disability

- 319 -
OrthoBullets2017 Lower Extremity | Femur

2. Femoral Neck Fractures


Introduction
 Epidemiology
o increasingly common due to aging population
o women > men
o whites > blacks
o United states has highest incidence of hip fx rates worldwide
o most expensive fracture to treat on per-person basis
 Mechanism
o high energy in young patients
o low energy falls in older patients
 Pathophysiology
o healing potential
 femoral neck is intracapsular, bathed in synovial
fluid
 lacks periosteal layer
 callus formation limited, which affects healing
 Associated injuries
o femoral shaft fractures
 6-9% associated with femoral neck fractures
 treat femoral neck first followed by shaft
 Prognosis
o mortality
 ~25-30% at one year (higher than vertebral compression fractures)
o predictors of mortality
 pre-injury mobility is the most significant determinant for post-operative survival
 in patients with chronic renal failure, rates of mortality at 2 years postoperatively, are close to
45%
Anatomy
 Osteology
o normal neck shaft-angle 130 +/- 7 degrees
o normal anteversion 10 +/- 7 degrees
 Blood supply to femoral head
o major contributor is medial femoral circumflex (lateral epiphyseal artery)
o some contribution to anterior and inferior head from lateral femoral circumflex
o some contribution from inferior gluteal artery
o small and insignificant supply from artery of ligamentum teres
o displacement of femoral neck fracture will disrupt the blood supply and cause an intracapsular
hematoma (effect is controversial)

- 320 -
By Dr, AbdulRahman AbdulNasser Lower Extremity | Femur

Classification

Garden Classification
(based on AP radiographs and does not consider lateral or sagittal plane alignment)
Type I Incomplete, ie. valgus impacted
Type II Complete fx. nondisplaced
Type III Complete, partially displaced
Type IV Complete, fully displaced
Posterior roll-off and/or angulation of femoral head leads to increased reoperation rates

Simplified Garden Classification


Nondisplaced Includes Garden I and II
Displaced Includes Garden IIII and IV

Pauwels Classification
(based on vertical orientation of fracture line)
Type I < 30 deg from horizontal
Type II 30 to 50 deg from horizontal
Type III > 50 deg from horizontal (most unstable with highest risk of nonunion and AVN)

Garden Type I Garden Type II

Garden Type III Garden Type IV

- 321 -
OrthoBullets2017 Lower Extremity | Femur
Presentation
 Symptoms
o impacted and stress fractures
 slight pain in the groin or pain referred along the medial side of the thigh and knee
o displaced fractures
 pain in the entire hip region
 Physical exam
o impacted and stress fractures
 no obvious clinical deformity
 minor discomfort with active or passive hip range of motion, muscle spasms at extremes of
motion
 pain with percussion over greater trochanter
o displaced fractures
 leg in external rotation and abduction, with shortening

Imaging
 Radiographs
o recommended views
 obtain AP pelvis and cross-table lateral, and full length femur film of ipsilateral side
 consider obtaining dedicated imaging of uninjured hip to use as template intraop
 traction-internal rotation AP hip is best for defining fracture type
 Garden classification is based on AP pelvis
 CT
o helpful in determining displacement and degree of comminution in some patients
 MRI
o helpful to rule out occult fracture
o not helpful in reliably assessing viability of femoral head after fracture
 Bone scan
o helpful to rule out occult fracture
o not helpful in reliably assessing viability of femoral head after fracture
 Duplex Scanning
o indication
 rule out DVT if delayed presentation to hospital after hip fracture

Treatment
 Nonoperative
o observation alone
 indications
 may be considered in some patients who are non-ambulators, have minimal pain, and who
are at high risk for surgical intervention
 Operative
o ORIF
 indications
 displaced fractures in young or physiologically young patients
 ORIF indicated for most pts <65 years of age

o cannulated screw fixation


- 322 -
By Dr, AbdulRahman AbdulNasser Lower Extremity | Femur
 indications
 nondisplaced transcervical fx
 Garden I and II fracture patterns in the physiologically elderly
 displaced transcervical fx in young patient
 considered a surgical emergency
 achieve reduction to limit vascular insult
 reduction must be anatomic, so open if necessary
o sliding hip screw
 indications
 basicervical fracture
 vertical fracture pattern in a young patient
 biomechanically superior to cannulated screws
 consider placement of additional cannulated screw above sliding hip screw to prevent
rotation
o hemiarthroplasty
 indications
 controversial
 debilitated elderly patients
 metabolic bone disease
o total hip arthoplasty
 indications
 controversial
 older active patients
 patients with preexisting hip osteoarthritis
 more predictable pain relief and better functional outcome than hemiarthroplasty
 arthroplasty for Garden III and IV in patient < 85 years

Techniques
 General Surgical Consideration
o time to surgery
 controversial
 reduction method and quality has more pronounced effect on healing than surgical timing
 elderly patients with hip fractures should be brought to surgery as soon as medically optimal
 the benefits of early mobilization cannot be overemphasized
 improved outcomes in medically fit patients if surgically treated less than 4 days from
injury
o treatment approach based on
 degree of displacement
 physiologic age of the patient (young is < than 50
 ipsilateral femoral neck and shaft fractures
 priority goes to fixing femoral neck because anatomic reduction is necessary to avoid
complications of AVN and nonunion
o fixation with implants that allow sliding
 permit dynamic compression at fx site during axial loading
 can cause shortening of femoral neck
 prominent implants
 affects biomechanics of hip joint
- 323 -
OrthoBullets2017 Lower Extremity | Femur
 lower physical function on SF-36
 decreased quality of life
 anatomic reduction with intraop compression and placement of length stable devices decrease
shortening
o open versus closed reduction
 worse outcomes with displacement > 5 mm (higher rate of osteonecrosis and nonunions)
 no consensus on which reduction approach is superior
 multiple closed reduction attempts are associated with higher risk of osteonecrosis of the
femoral head
 ORIF
o approach
 limited anterior Smith-Peterson
 10cm skin incision made beginning just distal to AIIS
 incise deep fascia
 develop interval between sartorious and TFL
 external rotation of thigh accentuates dissection plane
 LFCN is identified and retracted medially with sartorius
 identify tendinous portion of rectus femoris, elevate off hip capsule
 open capsule to identify femoral neck
 Watson-Jones
 used to gain improved exposure of lower femoral neck fractures
 skin incision approx 2cm posterior and distal to ASIS, down toward tip of greater
trochanter
 incision curved distally and extended 10cm along anterior portion of femur
 incise deep fascia
 develop interval between TFL and gluteus medius
 anterior aspect of gluteus medius and minimus is retracted posteriorly to visualize
anterior hip capsule
 capsule sharply incised with Z-shape incision
 capsulotomy must remain anterior to lesser trochanter at all times to avoid injury to
medial femoral circumflex artery
 reduction (method may vary)
 evacuate hematoma
 place A to P k-wires into femoral neck/head proximal to fracture to use as joysticks for
reduction
 insert starting k-wire (for either cannulated screw or sliding hip screw) into appropriate
position laterally, up to but not across the fracture
 once reduction obtained, drive starting k-wire across fracture
 insert second threaded tipped k-wire if adding additional fixation
 Cannulated Screw Fixation
o technique
 three screws if noncomminuted (3 screw inverted triangle shown to be superior to two
screws)
 order of screw placement (this varies)
 1-inferior screw along calcar
 2-posterior/superior screw

- 324 -
By Dr, AbdulRahman AbdulNasser Lower Extremity | Femur
 3-anterior/superior screw
 obtain as much screw spread as possible in femoral neck
 inverted triangle along the calcar (not central in the neck) has stronger fixation and higher
load to failure
 four screws considered for posterior comminution
 clear advantage of additional screws not proven in literature
 starting point at or above level of lesser trochanter to avoid fracture
 avoid multiple cortical perforations during guide pin or screw placement to avoid
development of lateral stress riser
 Hemiarthroplasty
o approach
 posterior approach has increased risk of dislocations
 anterolateral approach has increased abductor weakness
o technique
 cemented superior to uncemented
 unipolar vs. bipolar
 Total Hip Replacement
o technique
 should consider using the anterolateral approach and selective use of larger heads in the
setting of a femoral neck fracture
o advantages
 improved functional hip scores and lower re-operation rates compared to hemiarthroplasty
o complications
 higher rate of dislocation with THA (~ 10%)
 about five times higher than hemiarthroplasty

Complications
 Osteonecrosis
o incidence of 10-45%
o recent studies fail to demonstrate association between time to fracture reduction and subsequent
AVN
o increased risk with
 increase initial displacement
 AVN can still develop in nondisplaced injuries
 nonanatomical reduction
o treatment
 major symptoms not always present when AVN develops
 young patient
 > 50% involvement then treat with FVFG vs THA
 older patient
 prosthetic replacement (hemiarthroplasty vs THA)
 Nonunion
o incidence of 5 to 30%
 increased incidence in displaced fractures
 no correlation between age, gender, and rate of nonunion

- 325 -
OrthoBullets2017 Lower Extremity | Femur
o varus malreduction most closely correlates with failure of fixation after reduction and cannulated
screw fixation.
o treatment
 valgus intertrochanteric osteotomy
 indicated in patients after femoral neck nonunion
 can be done even in presence of AVN, as long as not severely collapsed
 turns vertical fx line into horizontal fx line and decreases shear forces across fx line
 free vascularized fibula graft (FVFG)
 indicated in young patients with a nonviable femoral head
 arthroplasty
 indicated in older patients or when the femoral head is not viable
 also an option in younger patient with a nonviable femoral head as opposed to FVFG
 revision ORIF
 Dislocation
o higher rate of dislocation with THA (~ 10%)
 about seven times higher than hemiarthroplasty

3. Intertrochanteric Fractures
Introduction
 Extracapsular fractures of the proximal femur between the greater and lesser trochanters
 Epidemiology
o incidence
 roughly the same as femoral neck fractures
o demographics
 female:male ratio between 2:1 and 8:1
 typically older age than patients with femoral neck fractures
o risk factors
 proximal humerus fractures increase risk of hip fracture for 1 year
 Pathophysiology
o mechanism
 elderly
 low energy falls in osteoporotic patients
 young
 high energy trauma
 Prognosis
o nonunion and malunion rates are low
o 20-30% mortality risk in the first year following fracture
o factors that increase mortality
 male gender (25-30% mortality) vs female (20% mortality)
 higher in intertrochanteric fracture (vs femoral neck fracture)
 operative delay of >2 days
 age >85 years
 2 or more pre-existing medical conditions
 ASA classification (ASA III and IV increases mortality)
o surgery within 48 hours decreases 1 year mortality

- 326 -
By Dr, AbdulRahman AbdulNasser Lower Extremity | Femur
o earlymedical optimization and co-management with medical hospitalists or geriatricians can
improve outcomes
Anatomy
 Osteology
o intertrochanteric area exists between greater and lesser trochanters
o made of dense trabecular bone
o calcar femorale
 vertical wall of dense bone that extends from posteromedial aspect of femoral shaft to
posterior portion of femoral neck
 helps determine stable versus unstable fracture patterns

Classification
 Stability of fracture pattern is arguably the most reliable method of classification
o stable
 definition
 intact posteromedial cortex
 clinical significance
 will resist medial compressive loads once reduced
o unstable
 definition
 comminution of the posteromedial cortex
 clinical significance
 fracture will collapse into varus and retroversion when loaded
 examples
 fractures with a large posteromedial fragment
 i.e., lesser trochanter is displaced
 subtrochanteric extension
 reverse obliquity
 oblique fracture line extending from medial cortex both laterally and distally

Presentation
 Physical Exam
o painful, shortened, externally rotated lower extremity

Imaging
 Radiographs
o recommended views
 AP pelvis
 AP of hip, cross table lateral
 full length femur radiographs
 CT or MRI
o useful if radiographs are negative but physical exam consistent with fracture

Treatment
 Nonoperative
o nonweightbearing with early out of bed to chair
 indications

- 327 -
OrthoBullets2017 Lower Extremity | Femur
nonambulatory patients
patients at high risk for perioperative mortality
 outcomes
 high rates of pneumonia, urinary tract infections, decubiti, and DVT
 Operative
o sliding hip compression screw
 indications
 stable intertrochanteric fractures
 outcomes
 equal outcomes when compared to intramedullary hip screws for stable fracture patterns
o intramedullary hip screw (cephalomedullary nail)
 indications
 stable fracture patterns
 unstable fracture patterns
 reverse obliquity fractures
 56% failure when treated with sliding hip screw
 subtrochanteric extension
 lack of integrity of femoral wall
 associated with increased displacement and collapse when treated with sliding hip
screw
 outcomes
 equivalent outcomes to sliding hip screw for stable fracture patterns
 use has significantly increased in last decade
o arthroplasty
 indications
 severely comminuted fractures
 preexisting symptomatic degenerative arthritis
 osteoporotic bone that is unlikely to hold internal fixation
 salvage for failed internal fixation

Techniques
 Sliding hip compression screw
o technique
 must obtain correct neck-shaft relationship
 lag screw with tip-apex distance >25 mm is associated with increased failure rates
 4 hole plates show no benefit clinically or biomechanically over 2 hole plates
o pros
 allows dynamic interfragmentary compression
 low cost
o cons
 open technique
 increased blood loss

 not advisable in unstable fracture patterns


 may result in
 collapse
 limb shortening

- 328 -
By Dr, AbdulRahman AbdulNasser Lower Extremity | Femur
 medialization of shaft
 can cause anterior spike malreduction in left-sided, unstable fractures due to screw torque
 Intramedullary hip screw
o technique
 short implants with optional distal locking
 standard obliquity fractures
 long implants
 standard obliquity fractures
 reverse obliquity fractures
 subtrochanteric extension
o pros
 percutaneous approach
 minimal blood loss
 may be used in unstable fracture patterns
o cons
 increased incidence of screw cutout
 periprosthetic fracture
 higher cost than sliding hip screw
 Arthroplasty
o technique
 calcar-replacing prosthesis often needed
 must attempt fixation of greater trochanter to shaft
o pros
 possible earlier return for full weight bearing
o cons
 increased blood loss
 may require prosthesis that some surgeons are unfamiliar with

Complications
 Implant failure and cutout
o incidence
 most common complication
 usually occurs within first 3 months
o cause
 tip-apex distance >45 mm associated with 60% failure rate
o treatment
 young
 corrective osteotomy and/or revision open reduction and internal fixation
 elderly
 total hip arthroplasty
 Anterior perforation of the distal femur
o incidence
 can occur following intramedullary screw fixation
o cause
 mismatch of the radius of curvature of the femur (shorter) and implant (longer)
 Nonunion
o incidence : <2%
- 329 -
OrthoBullets2017 Lower Extremity | Femur
o treatment
 revision ORIF with bone grafting
 proximal femoral replacement
 Malunion
o incidence
 varus and rotational deformities are common
o treatment : corrective osteotomies

4. Subtrochanteric Fractures
Introduction
 Subtrochanteric typically defined as area from lesser trochanter to 5cm distal
o fractures with an associated intertrochanteric component may be called
 intertrochanteric fracture with subtrochanteric extension
 peritrochanteric fracture
 Epidemiology
o usually in younger patients with a high-energy mechanism
o may occur in elderly patients from a low-energy mechanism
 rule out pathologic or atypical femur fracture
 denosumab or bisphosphonate use, particularly
alendronate, can be risk factor
 Pathoanatomy
o deforming forces on the proximal fragment are
 abduction ‎ I:3 atypical subtrochanteric fracture
V
with thickening of lateral cortix
 gluteus medius and gluteus minimus
(bisphosphonate use )
 flexion
 iliopsoas
 external rotation
 short external rotators
o deforming forces on distal fragment
 adduction & shortening
 adductors

Anatomy
 Biomechanics
o weight bearing leads to net compressive forces on medial cortex
and tensile forces on lateral cortex
Classification
Russel-Taylor Classification
Type I No extension into piriformis fossa
Type II Extension into greater trochanter with involvement of piriformis fossa
• look on lateral xray to identify piriformis fossa extension
• Historically used to differentiate between fractures that would amenable to an intramedullary nail (type I) and those
that required some form of a lateral fixed angle device (type II)
• Current interlocking options with both trochanteric and piriformis entry nails allow for treatment of type II fractures
with intramedullary implants

- 330 -
By Dr, AbdulRahman AbdulNasser Lower Extremity | Femur

AO/OTA Classification Examples


32-A3.1 Simple (A), Transverse (3), Subtrochanteric fracture (0.1)
32-B3.1 Wedge (B), Fragmented (3), Subtrochanteric fracture (0.1)
32-C1.1 Complex (C), Spiral (1), Subtrochanteric fracture (0.1)
Facture Location
• Femur (3) , Diaphysis (2), Subtrochanteric region (0.1)
Fracture Pattern
• Simple (A), Wedge (B), Complex (C)

ASBMR Task Force Case Definition of Atypical Femur Fractures (AFFs)

All major features should be present to designate a fracture as atypical; minor features may or may not be
present in individual cases
Major • Located anywhere along the femur from just distal to the lesser trochanter to just proximal to
Criteria the supracondylar flare
• Associated with no trauma or minimal trauma, as in a fall from a standing height or less
• Transverse or short oblique configuration
• Noncomminuted
• Complete fractures extend through both cortices and may be associated with a medial spike;
incomplete fractures involve only the lateral cortex
Minor • Localized periosteal reaction of the lateral cortex
Criteria • Generalized increase in cortical thickness of the diaphysis
• Prodromal symptoms such as dull or aching pain in the groin or thigh
• Bilateral fractures and symptomscomplete fractures involve only the lateral cortex
• Delayed healing
• Comorbid conditions (eg, vitamin D deficiency, rheumatoid arthritis, hypophosphatasia)
• Use of pharmaceutical agents (eg, BPs, glucocorticoids, proton pump inhibitors)
• Specifically excluded are fractures of the femoral neck, intertrochanteric fractures with spiral
subtrochanteric extension, pathological fractures associated with primary or metastatic bone
tumors, and periprosthetic fractures

Russel-Taylor Classification

- 331 -
OrthoBullets2017 Lower Extremity | Femur

32-A3.1 transverse fx 32-B3.1 32-C1.1

Presentation
 History
o long history of bisphosphonate or denosumab
o history of thigh pain before trauma occurred
 Symptoms
o hip and thigh pain
o inability to bear weight
 Physical exam
o pain with motion
o typically associated with obvious deformity (shortening and varus alignment)
o flexion of proximal fragment may threaten overlying skin

Imaging
 Radiographs
o required views
 AP and lateral of the hip
 AP pelvis
 full length femur films including the knee
o additional views
 traction views may assist with defining fragments in comminuted patterns but is not required
o findings
 bisphosphonate-related fractures have
 lateral cortical thickening
 transverse fracture orientation
 medial spike
 lack of comminution

Treatment
 Nonoperative
o observation with pain management
 indications
 non-ambulatory patients with medical co-morbidities that would not allow them to
tolerate surgery
- 332 -
By Dr, AbdulRahman AbdulNasser Lower Extremity | Femur
 limited role due to strong muscular forces displacing fracture and inability to mobilize
patients without surgical intervention
 Operative
o intramedullary nailing (usually cephalomedullary)
 indications
 historically Russel-Taylor type I fractures
 newer design of intramedullary nails has expanded indications
 most subtrochanteric fractures treated with IM nail
o fixed angle plate
 indications
 surgeon preference
 associated femoral neck fracture
 narrow medullary canal
 pre-existing femoral shaft deformity

Techniques
 Intramedullary Nailing
o position
 lateral positioning
 advantages
 allows for easier reduction of the distal fragment to the flexed proximal fragment
 allows for easier access to entry portal, especially for piriformis nail
 supine positioning
 advantages
 protective to the injured spine
 address other injuries in polytrauma patients
 easier to assess rotation
o techniques
 1st generation nail (rarely used)
 2nd generation reconstruction nail
 cephalomedullary nail
 trochanteric or piriformis entry portal
 piriformis nail may mitigate risk of iatrogenic malreduction from proximal valgus bend of
trochanteric entry nail
o pros
 preserves vascularity
 load-sharing implant
 stronger construct in unstable fracture patterns
o cons
 reduction technically difficult
 nail can not be used to aid reduction
 fracture must be reduced prior to and during passage of nail
 may require percutaneous reduction aids or open clamp placement to achieve and
maintain reduction
 mismatch of the radius of curvature
 nails with a larger radius of curvature (straighter) can lead to perforation of the anterior
cortex of the distal femur
- 333 -
OrthoBullets2017 Lower Extremity | Femur
o complications
 varus malreduction (see complications
below)
 Fixed angle plate
o approach
 lateral approach to proximal femur
 may split or elevate vastus lateralis
off later intermuscular septum
 dangers include perforating branches of profunda femoris
o technique
 95 degree blade plate or condylar screw
 sliding hip screw is contraindicated due to high rate of malunion and failure
 blade plate may function as a tension band construct
 femur eccentrically loaded with tensile force on the lateral cortex converted to
compressive force on medial cortex
o cons
 compromise vascularity of fragments
 inferior strength in unstable fracture patterns

Complications
 Varus/ procurvatum malunion
o the most frequent intraoperative complication with antegrade nailing of a subtrochanteric femur
fracture is varus and procurvatum (or flexion) malreduction
 Nonunion : can be treated with plating : allows correction of varus malalignment
 Bisphosphonate fractures
o nail fixation
 increased risk of iatrogenic fracture : because of brittle bone and cortical thickening
 increased risk of nonunion with nail fixation resulting in increased need for revision surgery
o plate fixation : increased risk of plate hardware failure
 because of varus collapse and dependence on intramembranous healing inhibited by
bisphosphonates

5. Femoral Shaft Fractures


Introduction
 High energy injuries frequently associated with life-threatening conditions
 Epidemiology : incidence : 37.1 per 100,000 person-years
 Mechanism
o traumatic
 high-energy
 most common in younger population
 often a result of high-speed motor vehicle accidents
 low-energy
 more common in elderly
 often a result of a fall from standing
 gunshot
‎VI:4 anterior bow
- 334 -
By Dr, AbdulRahman AbdulNasser Lower Extremity | Femur
 Associated conditions
o orthopaedic
 ipsilateral femoral neck fracture
 2-6% incidence
 often basicervical, vertical, and nondisplaced
 missed 19-31% of time
 bilateral femur fractures Linea Aspera
 significant risk of pulmonary complications
 increased rate of mortality as compared to unilateral fractures

Anatomy
 Osteology
o largest and strongest bone in the body
o femur has an anterior bow
o linea aspera
 rough crest of bone running down middle third of posterior femur
 attachment site for various muscles and fascia
 acts as a compressive strut to accommodate anterior bow to femur
 Muscles
o 3 compartments of the thigh
 anterior
 sartorius
 quadriceps
 posterior
 biceps femoris
 semitendinosus
 semimembranosus
 adductor
 gracilis
 adductor longus
 adductor brevis
 adductor magnus
 Biomechanics
o musculature acts as a deforming force after fracture
 proximal fragment
 abducted
 gluteus medius and minimus abduct as they insert on
greater trochanter
 flexed
 iliopsoas flexes fragment as it inserts on lesser trochanter
 distal segment
 varus
 adductors inserting on medial aspect of distal femur
 extension
 gastrocnemius attaches on distal aspect of posterior femur

- 335 -
OrthoBullets2017 Lower Extremity | Femur
Classification
Winquist and Hansen Classification
Type 0 • No comminution
Type I • Insignificant amount of comminution
Type II • Greater than 50% cortical contact
Type III • Less than 50% cortical contact
Type IV • Segmental fracture with no contact between proximal and distal fragment

OTA Classification
32A - Simple • A1 - Spiral
• A2 - Oblique, angle > 30 degrees
• A3 - Transverse, angle < 30 degrees
32B - Wedge • B1 - Spiral wedge
• B2 - Bending wedge
• B3 - Fragmented wedge
32C - Complex • C1 - Spiral
• C2 - Segmental
• C3 - Irregular

Winquist type 0 Winquist type 1 Winquist type 2

Winquist type 3 Winquist type 4

- 336 -
By Dr, AbdulRahman AbdulNasser Lower Extremity | Femur
Presentation
 Initial evaluation
o Advanced Trauma Life Support (ATLS) should be initiated ‎VI:5 OTA classification

 Symptoms
o pain in thigh
 Physical exam
o inspection
 tense, swollen thigh
 blood loss in closed femoral shaft fractures is 1000-1500ml
 for closed tibial shaft fractures, 500-1000ml
 blood loss in open fractures may be double that of closed fractures
 affected leg often shortened
 tenderness about thigh
o motion
 examination for ipsilateral femoral neck fracture often difficult secondary to pain from
fracture
o neurovascular : must record and document distal neurovascular status

Imaging
 Radiographs
o recommended views
 AP and lateral views of entire femur
 AP and lateral views of ipsilateral hip
 important to rule-out coexisting femoral neck fracture
 AP and lateral views of ipsilateral knee
 CT
o indications
 may be considered in midshaft femur fractures to rule-out associated femoral neck fracture

Treatment
 Nonoperative
o long leg cast
 indications
 nondisplaced femoral shaft fractures in patients with multiple medical comorbidities
 Operative
o antegrade intramedullary nail with reamed technique
 indications
 gold standard for treatment of diaphyseal femur fractures
 outcomes
 stabilization within 24 hours is associated with
 decreased pulmonary complications (ARDS)
 decreased thromboembolic events
 improved rehabilitation
 decreased length of stay and cost of hospitalization
 exception is a patient with a closed head injury ‎ I:6 A piriformis entry B trochanteric entry
V
 critical to avoid hypotension and hypoxemia
 consider provisional fixation (damage control)
- 337 -
OrthoBullets2017 Lower Extremity | Femur
o retrograde intramedullary nail with reamed technique
 indications
 ipsilateral femoral neck fracture
 floating knee (ipsilateral tibial shaft fracture)
 use same incision for tibial nail
 ipsilateral acetabular fracture
 does not compromise surgical approach to acetabulum
 multiple system trauma
 bilateral femur fractures
 avoids repositioning
 morbid obesity
 outcomes
 results are comparable to antegrade femoral nails
 immediate retrograde or antegrade nailing is safe for early treatment of gunshot femur
fractures
o external fixation with conversion to intramedullary nail within 2-3 weeks
 indications
 unstable polytrauma victim
 vascular injury
 severe open fracture
o ORIF with plate
 indications
 ipsilateral neck fracture requiring screw fixation
 fracture at distal metaphyseal-diaphyseal junction
 inability to access medullary canal
 outcomes
 inferior when compared to IM nailing due to increased rates of:
 infection
 nonunion
‎ I:7 piriformis entry
V
 hardware failure

Surgical Techniques
 Antegrade intramedullary nailing
o approach
 3 cm incision proximal to the greater trochanter in line with the femoral canal
o technique
 starting points
 piriformis entry
 pros
 colinear trajectory with long axis of femoral shaft
 cons
 starting point more difficult to access, especially in obese patients
 causes the most significant damage to
 abductor muscles and tendons
 may result in abductor limp
 blood supply to the femoral head
 may result in AVN in pediatric patients
- 338 -
By Dr, AbdulRahman AbdulNasser Lower Extremity | Femur
 trochanteric entry
 pros
 minimizes soft tissue injury to abductors
 easier starting point than piriformis entry nail
 cons
 not colinear with the long axis of femoral shaft
 must use nail specifically designed for trochanteric entry
 use of a straight nail may lead to varus malalignment
 reaming
 reamed nailing superior to unreamed nailing, with:
 increased union rates
 decreased time to union
 no increase in pulmonary complications
 indications for unreamed nail
 consider for patient with bilateral pulmonary injuries ‎VI:8 trochanteric entry
 interlocking screws
 technique
 computer-assisted navigation for screw placement decreases
radiation exposure
 widening/overlap of the interlocking hole in the proximal-
distal direction
 correct with adjustment in the abduction/adduction plane
 widening/overlap of the interlocking hole in the anterior-
posterior plane
 correct with adjustment in the internal/external rotation
plane
o postoperative care
 weight-bearing as tolerated
 range of motion of knee and hip is encouraged
o pros
 98-99% union rate
 low complication rate
 infection risk 2%
o cons
 not indicated for use with ipsilateral femoral neck fracture
 increased rate of HO in hip abductors with antegrade nailing
 increased rate of hip pain compared with retrograde nailing
 mismatch of the radius of curvature of the femoral shaft and intramedullary nails can lead to
anterior perforation of the distal femur
 Retrograde intramedullary nailing
o approach
 2 cm incision starting at distal pole of patella
 medial parapatellar versus transtendinous approaches
 nail inserted with knee flexed to 30-50 degrees
o technique
 entry point

- 339 -
OrthoBullets2017 Lower Extremity | Femur
 center of intercondylar notch on AP view
 extension of Blumensaat's line on lateral
 posterior to Blumensaat's line risks damage to
cruciate ligaments
o postoperative care
 weight-bearing as tolerated
 range of motion of knee and hip is encouraged
o pros
 technically easier
 union rates comparable to those of antegrade nailing
‎VI:9 entry point of retrogade nail
 no increased rate of septic knee with retrograde nailing
of open femur fractures
o cons
 knee pain
 increased rate of interlocking screw irritation
 cartilage injury
 cruciate ligament injury with improper starting point
 External fixation with conversion to intramedullary nail within 2-3 weeks
o technique
 safest pin location sites are anterolateral and direct lateral regions of the femur
 2 pins should be used on each side of the fracture line
o pros
 prevents further pulmonary insult without exposing patient to risk of major surgery
 may be converted to IM fixation within 2-3 weeks as a single stage procedure
o cons
 pin tract infection
 knee stiffness
 due to binding/scarring of quadriceps mechanism
 Special considerations
o ipsilateral femoral neck fracture
 priority goes to fixing femoral neck because anatomic reduction is necessary to avoid
complications of AVN and nonunion
 technique
 preferred methods
 screws for neck with retrograde nail for shaft
 screws for neck and plate for shaft
 compression hip screw for neck with retrograde nail for shaft
 less preferred methods
 antegrade nail with screws anterior to nail
 technically challenging

Complications
 Heterotopic ossification
o incidence
 25%
o treatment
 rarely clinically significant ‎VI:10 ipsilateral femoral neck fracture
- 340 -
By Dr, AbdulRahman AbdulNasser Lower Extremity | Femur
 Pudendal nerve injury
o incidence
 10% when using fracture table with traction
 Femoral artery or nerve injury
o incidence
 rare
o cause
 can occur when inserting proximal interlocking screws during a retrograde nail
 Malunion and rotational malalignment
o most accurately determined by the Jeanmart method
 angle between a line drawn tangential to the femoral condyles and a line drawn through the
axis of the femoral neck
o incidence
 proximal fractures 30%
 distal fractures 10%
o risk factors
 use of a fracture table increases risk of internal rotation deformities when compared to
manual traction
 fracture comminution
 night-time surgery
o treatment
 if noticed intraoperatively, remove distal interlocking screws and manually correct rotation
 if noticed after union, osteotomy is required
 Delayed union
o treatment
 dynamization of nail with or without bone grafting
 Nonunion
o incidence
 <10%
o risk factors
 postoperative use of nonsteroidal anti-inflammatory drugs
 smoking is known to decrease bone healing in reamed antegrade exchange nailing for
atrophic non-unions
o treatment
 reamed exchange nailing
 Infection
o incidence
 < 1%
o treatment
 removal of nail and reaming of canal
 external fixation used if fracture not healed
 Weakness
o quadriceps and hip abductors are expected to be weaker than contralateral side
 Iatrogenic fracture etiologies
o risk factors
 antegrade starting point 6mm or more anterior to the intramedullary axis

- 341 -
OrthoBullets2017 Lower Extremity | Femur
 however, anterior starting point improves position of screws into femoral head
 failure to overream canal by at least .5mm
 Mechanical axis deviation (MAD)
o lengthening along the anatomical axis of the femur leads to lateral MAD
o shortening along the anatomical axis of the femur leads to medial MAD
 Anterior cortical penetration.

6. Distal Femur Fractures


Introduction
 Epidemiology
o traditionally young patients but increasing in geriatric population
o bimodal distribution: young, healthy males, elderly osteopenic females
o periprosthetic fractures becoming more common
 Mechanism
o young patients
 high energy with significant displacement
o older patients
 low energy, often fall from standing, in osteoporotic bone,
usually with less displacement
Anatomy
 Osteology
o distal femur becomes trapezoidal in cross section towards knee
o medial condyle extends more distal than lateral
o posterior halves of both condyles are posterior to posterior cortex of femoral shaft
o anatomical axis of distal femur is 6-7 degrees of valgus
o lateral cortex of femur slopes ~10 degrees, medial cortex slopes ~25 degrees in axial plane
 Pathomechanics
o gastrocnemius: extends distal fragment (apex posterior)
o hamstring and extensor mechanism: cause shortening
o adductor magnus: leads to distal femoral varus

Classification
 Descriptive
o supracondylar
o intercondylar
 OTA: 33
o A: extraarticular
o B: partial articular
 portion of articular surface remains in continuity with shaft
 33B3 is in coronal plane (Hoffa fragment)
o C: complete articular
 articular fragment separated from shaft

- 342 -
By Dr, AbdulRahman AbdulNasser Lower Extremity | Femur
Presentation
 Physical exam
o pain, deformity, swelling localizing to distal thigh/knee
o evaluate skin integrity
o vascular evaluation
 potential for injury to popliteal artery if significant displacement
 if no pulse after gross alignment restored then angiography is indicated
 Full trauma evaluation if high energy mechanism

Imaging
 Radiographs
o obtain standard AP and Lateral
o traction views
 AP, Lateral, and oblique traction views can help characterize
injury but are painful for patient
 in elderly patients, evaluate for any pre-existing knee DJD
 consider views of the remainder of the extremity to rule out
‎VI:11 vascular evaluation
associated injuries
 consider views of contralateral femur for pre-operative planning and templating
 CT
o obtain with frontal and sagittal reconstructions
o useful for
 establishing intra-articular involvement
 identifying separate osteochondral fragments in the area of the intercondylar notch
 identifying coronal plane fx (Hoffa fx): 38% incidence of Hoffa fractures in Type C fractures
 preoperative planning
o if temporizing external fixation required, CT obtained after external
fixation
 Angiography
o indicated when diminished distal pulses after gross alignment restored
o consider if associated with knee dislocation

Treatment
 Nonoperative
o hinged knee brace with immediate ROM, NWB for 6 weeks
 indications (rare)
 nondisplaced fractures
 nonambulatory patient
 patient with significant comorbidities presenting unacceptably high degree of
surgical/anesthetic risk
 Operative
o external fixation
 temporizing measure until soft tissues permit internal fixation, or until patient is stable
 avoid pin placement in area of planned plate placement if possible
o open reduction internal fixation
 indications
 displaced fracture
- 343 -
OrthoBullets2017 Lower Extremity | Femur
 intra-articular fracture
 nonunion
 goals
 need anatomic reduction of joint
 stable fixation of articular component to shaft to permit early motion
 preserve vascularity
 technique (see below)
 postoperative
 early ROM of knee important
 non-weight bearing or toe touch
weight-bearing for 6-8 weeks, up
to 10-12 weeks if comminuted
 quadriceps and hamstring
strength exercises
o retrograde IM nail ‎VI:12 retrograde nail
 indications
 good for supracondylar fx without significant comminution
 preferred implant in osteoporotic bone
 traditionally, 4 cm of intact distal femur needed but newer implants with very distal
interlocking options may decrease this number, can perform independent screw
stabilization of intercondylar component of fracture around nail
o distal femoral replacement
 indications
 unreconstructable fracture
 fracture around prior total knee arthroplasty with loose component

Surgical Techniques
 ORIF Approaches
o anterolateral
 fractures without articular involvement or with simple articular extension
 incision from tibial tubercle to anterior 1/3 of distal femoral condyle
 extend up midlateral femoral shaft as needed
 minimally invasive plate osteosynthesis: small lateral incision, slide plate proximally, use
stab incisions for proximal screw placement
o lateral parapatellar
 fractures with complex articular extension
 extend incision into quad tendon to evert patella
 can be used for Hoffa fracture
o medial parapatellar
 typical TKA approach
 used for complex medial femoral condyle fractures
o medial/lateral posterior
 used for very posterior Hoffa fragment fixation
 patient placed in prone position
 midline incision over popliteal fossa
 develop plane between medial and lateral gastrocnemius m.
 capsulotomy to visualize fracture
- 344 -
By Dr, AbdulRahman AbdulNasser Lower Extremity | Femur
 Blade Plate Fixation
o indications
 not commonly used, technically difficult
 contraindicated in type C3 fractures
o technique
 placed 1.5 cm from articular surface
 Dynamic Condylar Screw Placement ‎VI:13 Blade plate
o indications : identical to 95 degree angled blade plate
o technique
 precise sagittal plane alignment is not necessary
 placed 2.0 cm from articular surface
o cons ‎VI:14 DCS
 large amount of bone removed with DCS
 difficult to place

Locked plate DCS Blade plate Retrograde nail


 Locked Plate Fixation
o indications
 fixed-angle locked screws provide improved fixation in short distal femoral block
 supracondylar periprosthetic femur fractures in cruciate retaining TKA
 TKA component must be well-fixed to proceed with fracture fixation
o technique
 lag screws with locked screws (hybrid construct)
 useful for intercondylar fractures (usually in conjunction with locked plate)
 useful for coronal plane fractures
 helps obtain anatomic reduction of joint
 required in displaced articular fractures
o pros
 percutaneous lateral application can minimize soft tissue stripping and obviate need for
medial plate
o cons
 potential to create too stiff a construct leading to nonunion or plate failure
 Non-fixed angle plates
o indications : now largely obsolete due to tendency for varus malalignment
 Retrograde interlocked IM nail
o indications
 good for supracondylar fractures without significant comminution

- 345 -
OrthoBullets2017 Lower Extremity | Femur
 preferred implant in osteoporotic bone
 short nail rarely indicated, implant should at least reach lesser trochanter
o approach
 medial parapatellar
 no articular extension present
 2.5 cm incision parallel to medial aspect of patellar tendon
 stay inferior to patella
 no attempt to visualize articular surface
 articular extension present
 continue approach 2-8 cm cephalad
 incise extensor mechanism 10 mm medial to
patella
 eversion of patella not typically necessary
 need to stabilize articular segments prior to
nail placement
o pros : requires minimal dissection of soft tissue
o cons
 less axial and rotational stability
 postoperative knee pain

Complications
 Symptomatic hardware
‎VI:15 Distal femur malunion
o lateral plate
 pain with knee flexion/extension due to IT band contact with plate
o medial screw irritation
 excessively long screws can irritate medial soft tissues
 determine appropriate intercondylar screw length by obtaining an
AP radiograph of the knee with the leg internally rotated 30 degrees
 Malunions
o most commonly associated with plating, usually valgus
o functional results satisfactory if malalignment is within 5 degrees in
any plane
 Nonunions
o up to 19%, most commonly in metaphyseal area, with articular portion
healed (comminution, bone loss and open fractures more likely in
metaphysis)
o decreasing with less invasive techniques
o treatment with revision ORIF and autograft indicated
o consider changing fixation technique to improve biomechanics
 Infection
o treat with debridement, culture-specific antibiotics, hardware removal
if fracture stability permits ‎ I:16 Non union
V

 Implant failure
o up to 9%
o titanium plates may be superior to stainless steel

- 346 -
By Dr, AbdulRahman AbdulNasser Lower Extremity | Knee

B. Knee
Meniscal tears and ligamintous injuries of knee discussed in Volume 3 (Sport)

1. Patella Fracture
Introduction
 Patella fractures account for 1% of all skeletal injuries
o occur either by direct impact injury or indirect eccentric contraction
o male to female 2:1
o most fractures occur in 20-50 year olds
 Patella sleeve fracture
o seen in pediatric population (8-10 year olds)
o high index of suspicion required
 Bipartite patella
o may be mistaken for patella fracture
o affects 8% of population
o characteristic superolateral position
‎ I:17 Bipartite patella
V

o bilateral in 50% of cases

Anatomy
 Patella is largest sesamoid bone in body
 Articular cartilage thickest in body (up to 1cm)
 Most important blood supply to the patella is located at the inferior pole

Classification
 Can be described based on fracture pattern
o nondisplaced
o transverse
o pole or sleeve (upper or lower)
o vertical
o marginal
o osteochondral
o comminuted (stellate)

Presentation
 Physical exam
o palpable patellar defect
o significant hemarthrosis
o unable to perform straight leg raise indicates failure of extensor mechanism
 retinaculum disrupted

Imaging
 Radiographs
o patella alta
o fracture displacement
 best evaluated on lateral x-ray
‎VI:18 palpable patellar defect
 degree of fracture displacement correlates with degree of retinacular disruption

- 347 -
OrthoBullets2017 Lower Extremity | Knee
 MRI
o obtain MRI if child has normal xrays but is unable to straight leg
raise
Treatment
 Nonoperative
o knee immobilized in extension (brace or cylinder cast) and full
weight bearing
 indications
 intact extensor mechanism (patient able to perform straight
leg raise)
‎VI:19 Patela alta with
 nondisplaced or minimally displaced fractures avulsion fracture of lower
 vertical fracture patterns pole
 early active ROM with hinged knee brace
 early WBAT in full extension
 progress in flexion after 2-3 weeks
 Operative
o ORIF with tension band construct
 indications
 preserve patella whenever possible
 extensor mechanism failure (unable to perform straight leg raise)
 open fractures
 fracture articular displacement >2mm
 displaced patella fracture >3mm
 patella sleeve fractures in children
 techniques
 minifrag lag screw fixation for independent fragments
 tension bands
 0.062 K wires with figure of 8 wire
 longitudinal cannulated screws combined with tension band wires shown to be
biomechanically superior
 circumferential cerclage wiring
 good for comminuted fractures
 interfragmentary screw compression supplemented by cerclage wiring
o partial patellectomy
 indications
 comminuted superior or inferior pole fracture measuring <50% patellar height ONLY if
ORIF is not possible
 techniques
 quadricep or patellar tendon re-attachment
 reattachment close to articular surface prevents patellar tilt
 medial and lateral retinacular repair essential
o total patellectomy
 indications
 reserved for severe and extensive comminution not amenable to salvage
 quadriceps torque reduced by 50%
 medial and lateral retinacular repair essential
- 348 -
By Dr, AbdulRahman AbdulNasser Lower Extremity | Knee
Complications
 Weakness and anterior knee pain
 Symptomatic hardware
o most common
 Loss of reduction (22%)
o increased in osteoporotic bone
 Nonunion (<5%)
o can consider partial patellectomy
 Osteonecrosis (proximal fragment)
o thought to be due to excessive initial fracture displacement
o can observe these, as most spontaneously revascularize by 2 years
 Infection
 Stiffness

longitudinal cannulated screws combined with tension


K wires with figure of 8 wire band wires

circumferential cerclage wiring interfragmentary screw


compression supplemented by total patellectomy
cerclage wiring

- 349 -
OrthoBullets2017 Lower Extremity | Knee

2. Knee Dislocation
Introduction
 Devastating injury resulting from high or low energy
o high-energy
 usually from MVC or fall from height
 commonly a dashboard injury resulting in axial load to flexed knee
o low-energy
 often from athletic injury
 generally has a rotational component
 morbid obesity is a risk-factor
 Pathoanatomy
o associated with significant soft tissue disruption
o 3/4 of ligaments generally disrupted
 Associated injuries
o vascular injury
 5-15% in all dislocations
 40-50% in anterior/posterior dislocations ‎ I:20 Knee recurvatum when held in extension
V
(knee dislocation-clinical instability)
 due to tethering at the popliteal fossa
 proximal - fibrous tunnel at the adductor hiatus
 distal - fibrous tunnel at soleus muscle
o nerve injury
 usually common peroneal nerve injury (25%)
 tibial nerve injury is less common
o fractures
 present in 60%
 tibia and femur most common
 Prognosis
o complications frequent and rarely does knee return to pre-injury state

Classification
 Descriptive
o Kennedy classification based on direction of displacement of the tibia
 anterior (30-50%)
 most common
 due to hyperextension injury
 usually involves tear of PCL
 arterial injury is generally an intimal tear due to traction
 posterior (25%)
 2nd most common
 due to axial load to flexed knee (dashboard injury)
 highest rate of vascular injury (25%) based on Kennedy classification (direction of
dislocation)
 highest rate of complete tear of popliteal artery
 lateral (13%)
 due to varus or valgus force

- 350 -
By Dr, AbdulRahman AbdulNasser Lower Extremity | Knee
 usually involves tears of both ACL and PCL
 highest rate of peroneal nerve injury
 medial (3%)
 varus or valgus force
 usually disrupted PLC and PCL
 rotational (4%)
 posterolateral is most common rotational dislocation
 usually irreducible
 buttonholding of femoral condyle through capsule
 Schenck Classification
o based on pattern of multiligamentous injury of knee dislocation (KD)

Schenck Classification (based on number of ruptured ligaments)


KD I Multiligamentous injury with involvement of ACL or PCL
KD II Injury to ACL and PCL only (2 ligaments)
Injury to ACL, PCL, and PMC or PLC (3 ligaments). KDIIIM (ACL, PCL,
KD III MCL) and KDIIIL (ACL, PCL, PLC, LCL). KDIIIM has highest rate of
vascular injury (31%) based on Schenck classification
KD IV Injury to ACL, PCL, PMC, and PLC (4 ligaments)
KD V Multiligamentous injury with periarticular fracture

Presentation
 Symptoms
o history of trauma and deformity of the knee
o knee pain & instability
 Physical exam
o appearance
 no obvious deformity
 50% spontaneously reduce before arrival to ED (therefore underdiagnosed)
 may present with subtle signs of trauma (swelling, effusion, abrasions)
 obvious deformity
 do not wait for radiographs, reduce immediately, especially if absent pulses
 "dimple sign" - buttonholing of medial femoral condyle through medial capsule
 indicative of an irreducible posterolateral dislocation
 a contraindication to closed reduction due to risks of skin necrosis
o stability
 diagnosis based on instability on exam (radiographs and gross appearance may be normal)
 may see recurvatum when held in extension
 assess ACL, PCL, MCL, LCL, and PLC
o vascular exam
 priority is to rule out vascular injury on exam both before and after reduction
 serial examinations are mandatory
 palpate the dorsalis pedis and posterior tibial pulses
 if pulses are present and normal
 does not indicate absence of arterial injury
- 351 -
OrthoBullets2017 Lower Extremity | Knee
 collateral circulation can mask a complete popliteal artery occlusion
 measure Ankle-Brachial Index (ABI)
 if ABI >0.
 then monitor with serial examination (100% Negative Predictive Value)
 if ABI <0.9
 perform arterial duplex ultrasound or CT angiography
 if arterial injury confirmed then consult vascular surgery
 If pulses are absent or diminished
 confirm that the knee joint is reduced or perform immediate reduction and reassessment
 immediate surgical exploration if pulses are still absent following reduction
 ischemia time >8 hours has amputation rates as high as 86%
 if pulses present after reduction then measure ABI then consider observation vs.
angiography
Imaging
 Radiographs

o may be normal if spontaneous reduction


 look for asymmetric or irregular joint space
 look for avulsion fxs (Segond sign - lateral tibial condyle avulsion fx)
 osteochondral defects
 MRI
o required to evaluate soft tissue injury (ligaments, meniscus) and for surgical planning
o obtain MRI after acute treatment

Treatment
 Initial Treatment
o reduce knee and re-examine vascular status
 considered an orthopedic emergency
 splint in 20-30° flexion
 confirm reduction is held with repeat radiographs in brace/splint
 vascular consult indicated if
 if arterial injury confirmed by arterial duplex ultrasound or CT angiography
 pulses are absent or diminished following reduction
 Nonoperative
o indications : limited and most cases require surgical stabilization
 Operative
o emergent surgical intervention with external fixation
 indications
 vascular repair (takes precedence)
 open fx and open dislocation
 irreducible dislocation
 compartment syndrome
 obese
 multi trauma patient
 technique
 vascular intervention

- 352 -
By Dr, AbdulRahman AbdulNasser Lower Extremity | Knee
 perform external fixation first
 excision of damaged segment and repair with reverse saphenous vein graft
 always perform fasciotomies after vascular repair
o delayed ligamentous reconstruction/repair
 indications
 generally instability will require some kind of ligamentous repair or fixation
 patients can be placed in a knee immobilizer for 6 weeks for initial stabilization
 improved outcomes with early treatment (within 3 weeks)
 technique
 PLC
 early reconstruction before ACL reconstruction
 postoperative
 recommend early mobilization and functional bracing

Complications
 Stiffness (arthrofibrosis)
o is most common complication (38%)
o more common with delayed mobilization
 Laxity and instability (37%)
 Peroneal nerve injury (25%)
o most common in posterolateral dislocations
o poor results with acute, subacute, and delayed (>3 months) nerve exploration
o neurolysis and tendon transfers are the mainstay of treatment
o Dynamic tendon transfer involves transferring the posterior tibial tendon (PTT) to the lateral
cuneiform.
 Vascular compromise
o in addition to vessel damage, claudication, skin changes, and muscle atrophy can occur

- 353 -
OrthoBullets2017 Lower Extremity | Leg

C. Leg

1. Tibial Plateau Fractures


Introduction
 Periarticular injuries of the proximal tibia frequently associated with soft tissue injuries
 Epidemiology
o demographics
 bimodal distribution
 males in 40s (high-energy trauma)
 females in 70s (falls)
o location
 unicondylar vs. bicondylar
 frequency : lateral > bicondylar > medial
 Mechanism
o varus/valgus load with or without axial load
o high energy : frequently associated with soft tissue injuries
o low energy : usually insufficiency fractures
 Associated conditions
o meniscal tears
 lateral meniscal tear
 more common than medial
 associated with Schatzker II fracture pattern
 medial meniscal tear
 most commonly associated with Schatzker IV fractures
o ACL injuries : more common in type V and VI fractures (25%)
o compartment syndrome
o vascular injury
 commonly associated with Schatzker IV fracture-dislocations

Anatomy
 Osteology
o lateral tibial plateau
 convex in shape
 proximal to the medial plateau
o medial tibial plateau
 concave in shape
 distal to the lateral tibial plateau
 Muscles
o anterior compartment musculature : attaches to anterolateral tibia
o pes anserine : attaches to anteromedial tibia
 Biomechanics
o medial tibial plateau bears 60% of knee's load

- 354 -
By Dr, AbdulRahman AbdulNasser Lower Extremity | Leg
Classification
Schatzker Classification
Type I Lateral split fracture
Type II Lateral Split-depressed fracture
Type III Lateral Pure depression fracture
Type IV Medial plateau fracture
Type V Bicondylar fracture
Type VI Metaphyseal-diaphyseal disassociation

Hohl and Moore Classification of proximal tibia fracture-dislocations


Type I Coronal split fracture
Type II Entire condylar fracture
Type III Rim avulsion fracture of lateral plateau
Type IV Rim compression fracture
Type V Four-part fracture
Classification useful for
1) true fracture-dislocations
2) fracture patterns that do not fit into the Schatzker classification (10% of all tibial plateau
fractures)
3) fractures associated with knee instability

Schatzker Classification

Type I

Type II

- 355 -
OrthoBullets2017 Lower Extremity | Leg

Type III Type IV Type V Type VI

Hohl and Moore Classification of proximal tibia fracture-dislocations

Presentation
 History
o high-energy trauma in young patients
o low-energy falls in elderly
 Physical exam
o inspection
 look circumferentially to rule-out an open injury
o palpation
 consider compartment syndrome when compartments are firm and not compressible
o varus/valgus stress testing
 any laxity >10 degrees indicates instability
 often difficult to perform given pain
- 356 -
By Dr, AbdulRahman AbdulNasser Lower Extremity | Leg
o neurovascular exam
 any differences in pulse exam between extremities should be further investigated with anke-
brachial index measurement
Imaging
 Radiographs
o recommended views
 AP, lateral, oblique
 oblique is helpful to determine amount of depression
o optional views
 plateau view
 10 degree caudal tilt
o findings
 posteromedial fracture lines must be recognized
 CT scan
o important to identify articular depression and comminution
o findings
 lipohemarthrosis indicates an occult fracture
 fracture fragment orientation and surgical planning
 MRI
o indications
 not well established
o findings
 useful to determine meniscal and ligamentous pathology

Treatment
 Nonoperative
o hinged knee brace, PWB for 8-12 weeks, and immediate passive ROM
 indications
 minimally displaced split or depressed fractures
 low energy fracture stable to varus/valgus alignment
 nonambulatory patients
 Operative
o temporizing bridging external fixation w/ delayed ORIF
 indications
 significant soft tissue injury
 polytrauma
o external fixation with limited open/percutaneous fixation of articular segment
 indications
 severe open fracture with marked contamination
 highly comminuted fractures where internal fixation not possible
 outcomes
 similar to open reduction, internal fixation
o open reduction, internal fixation
 indications
 articular stepoff > 3mm
 condylar widening > 5mm

- 357 -
OrthoBullets2017 Lower Extremity | Leg
 varus/valgus instability
 all medial plateau fxs
 all bicondylar fxs
 outcomes
 restoration of joint stability is strongest predictor of long term outcomes
 worse results with
 ligamentous instability
 meniscectomy
 alteration of limb mechanical axis > 5 degrees

Techniques
 External fixation (temporary)
o technique
 two 5-mm half-pins in distal femur, two in distal tibia
 axial traction applied to fixator
 fixator is locked in slight flexion
o advantages
 allows soft tissue swelling to decrease before definitive fixation
 decreases rate of infection and wound healing complications
 External fixation with limited internal fixation (definitive)
o technique
 reduce articular surface either percutaneously or with small incisions
 stabilize reduction with lag screws or wires
 must keep wires >14mm from joint
 apply external fixator or hybrid ring fixation
o post-operative care
 begin weight bearing when callus is visible on
radiographs
 usually remain in place 2-4 months
o pros
 minimizes soft tissue insult
 permits knee ROM
o cons
 pin site complications
 Open reduction, internal fixation
‎VI:21 Butress plate
o approach
 lateral incision (most common)
 straight or hockey stick incision anterolaterally from just proximal to joint line to just
lateral to the tibial tubercle
 midline incision (if planning TKA in future)
 can lead to significant soft tissue stripping and should be avoided
 posteromedial incision
 interval between pes anserinus and medial head of gastrocnemius
 dual surgical incisions with dual plate fixation
 indications
 bicondylar tibial plateau fractures
 posterior : can be used for posterior shearing fractures
- 358 -
By Dr, AbdulRahman AbdulNasser Lower Extremity | Leg
o reduction
 restore joint surface with direct or indirect reduction
 fill metaphyseal void with autogenous, allogenic bone graft, or bone graft substitutes
 calcium phosphate cement has high compressive strength for filling metaphyseal void
o internal fixation
 absolute stability constructs should be used to maintain the joint reduction
 screws
 may be used alone for
 simple split fractures
 depression fractures that were elevated percutaneously
 plate fixation
 non-locked plates
 non-locked buttress plates best indicated for simple partial articular fractures in
healthy bone
 locked plates
 advantages
 fixed-angle construct
 less compression of periosteum and soft tissue
o postoperative
 hinged knee brace with early passive ROM
 gentle mechanical compression on repaired osteoarticular segments improves
chondrocyte survival
 NWB or PWB for 8 to 12 weeks

Complications
 Post-traumatic arthritis
o rate increases with
 meniscectomy during surgery
 axial malalignment
 intra-articular infection
 joint instability

2. Proximal Third Tibia Fracture


Introduction
 Fractures of the proximal tibial shaft that are associated with
o high rates of malunion
 valgus
 apex anterior (procurvatum)
o soft tissue compromise
 Epidemiology
o incidence : 5-11% of all tibial shaft fractures
 Pathophysiology
o mechanism
 low energy
 result of torsional injury
 indirect trauma
- 359 -
OrthoBullets2017 Lower Extremity | Leg
 high energy
 direct trauma
 Associated conditions
o compartment syndrome
o soft tissue injury : critical to outcome

Anatomy
 Osteology
o proximal tibia
 triangular
 wide metaphyseal region
 narrow distally
 Muscles
o deforming forces
 patellar tendon
 proximal fragment into extension
 fracture into apex anterior, or procurvatum
 hamstring tendons
 distal fragment into flexion
 pes anserinus
 proximal fragment into varus
 valgus deforming force of the fracture
 anterior compartment musculature
 valgus deforming force of the fracture

Classification
AO Classification - 42
Type A Simple fracture pattern
Type B Wedge fracture pattern
Type C Comminuted fracture pattern

Presentation
 Symptoms
o pain, inability to bear weight
 Physical exam
o inspection and palpation
 contusions
 blisters
 open wounds
 compartments
 palpation
 passive motion of toes
 intracompartmental pressure measurement if indicated
o neurologic
 deep peroneal n.
 superficial peroneal n.

- 360 -
By Dr, AbdulRahman AbdulNasser Lower Extremity | Leg
 sural n.
 tibial n.
 saphenous n.
o pulse
 dorsalis pedis
 posterior tibial : be sure to check contralateral side

Imaging
 Radiographs
o recommended views
 full length AP and lateral views of affected tibia
 AP and lateral views of ipsilateral knee
 AP and lateral views of ipsilateral ankle
 CT
o indications : question of intra-articular fracture extension

Treatment of Closed Tibia Fractures


 Nonoperative
‎VI:23 intramedullary nailing
o closed reduction / cast immobilization
 indications
 closed low energy fractures with acceptable alignment
 < 5 degrees varus-valgus angulation
 < 10 degrees anterior/posterior angulation
 > 50% cortical apposition
 < 1 cm shortening
 < 10 degrees rotational alignment
 technique
 place in long leg cast and convert to functional brace at 4 weeks
 cast in 10 to 20 degrees of flexion
 outcomes
 rotational control is difficult to achieve by closed methods
 Operative
o external fixation
 indications
 fractures with extensive soft-tissue compromise
 polytrauma
 technique
 bi-planar and multiplanar pin fixators are useful
o intramedullary nailing
 indications
 enough proximal bone to accept two locking
screws (5-6 cm)
 outcomes ‎ I:24 percutaneous locking plate
V

 high rates of malunion with improper technique


 most common malunion
 valgus
 apex anterior (procurvatum)

- 361 -
OrthoBullets2017 Lower Extremity | Leg
o percutaneous locking plate
 indications
 inadequate proximal fixation for IM nailing
 best suited for transverse or oblique fractures
 minimal soft-tissue compromise
 technique
 may be used medially or laterally
 better soft tissue coverage laterally makes lateral plating safer
 outcomes
 lateral plating with medial comminution can lead to varus collapse
 long plates may place superficial peroneal nerve at risk

Surgical Technique
 Intramedullary nailing
o approach
 lateral parapatellar
 helps maintain reduction for proximal 1/3 fractures
 requires mobile patella
 medial parapatellar approach may lead to valgus deformity ‎VI:25 suprapatellar approach
 suprapatellar
 facilitates nailing in semiextended position
o starting point
 proximal to the anterior edge of the articular margin
 just medial to the lateral tibial spine
 use of a more lateral starting point may decrease valgus
deformity
 use of a medial starting point may create valgus
deformity
o fracture reduction techniques ‎VI:26 medial starting point may
 blocking (Poller) screws create valgus deformity
 coronal blocking screw
 prevents apex anterior (procurvatum) deformity
 place in posterior half of proximal fragment
 sagittal blocking screw
 prevents valgus deformity
 place on lateral concave side of proximal fragment
 enhance construct stability if not removed
 unicortical plating
 short one-third tubular plate placed anteriorly, anteromedially, or
posteromedially across fracture ‎ I:27 coronal blocking screw
V

 secure both proximally and distally with 2 unicortical screws


 universal distractor
 Schanz pins inserted from medial side, parallel to joint
 pin may additionally be used as blocking screws
o nail insertion
 options
 standard insertion with knee in flexion
- 362 -
By Dr, AbdulRahman AbdulNasser Lower Extremity | Leg
 nail insertion in semiextended position
 may help to prevent apex anterior (procurvatum) deformity
 neutralizes deforming forces of extensor mechanism
o locking screws
 statically lock proximally and distally for rotational stability
 no indication for dynamic locking acutely
 must use at least two proximal locking screws

Complications
 Malunion ‎VI:28 sagittal blocking screw
o incidence : 20-60% rate of malunion following intramedullary nailing (valgus/procurvatum)
o treatment
 revision intramedullary nailing
 osteotomy if fracture has healed
o prevention
 blocking screws
 temporary plating
‎VI:29 unicortical
 universal distractors plating
 nailing in semiextended position

3. Tibia Shaft Fractures


Introduction
 Proximal third-tibia fractures
 Epidemiology : most common long bone fx
o account for 4% of all fx seen in the Medicare population
 Mechanism
o low energy fx pattern
 result of torsional injury
 indirect trauma results in spiral fx
 fibula fx at different level
 Tscherne grade 0 / I soft tissue injury
o high energy fx pattern
 direct forces often result in wedge or short oblique fx and sometimes significant
comminution
 fibula fx at same level
 severe soft tissue injury
 Tscherne II / III
 open fx
 Associated conditions
o soft tissue injury (open wounds) : critical to outcome
o compartment syndrome
o bone loss
o ipsilateral skeletal injury
 extension to the tibial plateau or plafond
 posterior malleolar fracture
 most commonly associated with spiral distal third tibia fracture
- 363 -
OrthoBullets2017 Lower Extremity | Leg
Classification
Gustilo-Anderson Classification of Open Tibia Fxs
Type I Limited periosteal stripping, wound < 1 cm
Type II Mild to moderate periosteal stripping, wound 1-10 cm in length
Type IIIA Significant soft tissue injury (often evidenced by a segmental fracture or comminution),
significant periosteal stripping, no flap required
Type IIIB Significant periosteal stripping and soft tissue injury, flap required due to inadequate soft
tissue coverage (STSG doesn't count). Treat proximal 1/3 fxs with gastrocnemius
rotation flap, middle 1/3 fxs with soleus rotation flap, distal 1/3 fxs with free flap.
Type IIIC Significant soft tissue injury (often evidenced by a segmental fracture or comminution),
vascular injury requiring repair to maintain limb viability
For prognostic reasons, severly comminuted, contaminated barnyard injuries, close range shotgun/high velocity gunshot
injuries, and open fractures presenting over 24 hours from injury have all been later included in the grade III group.

Presentation
 Symptoms
o pain, inability to bear weight, deformity
 Physical exam
o inspection and palpation
 deformity / angulation / malrotation
 contusions
 blisters
 open wounds
 compartments
 palpation
 pain
 passive motion of toes
 intracompartmental pressure measurement if indicated
o neurologic
 deep peroneal n.
 superficial peroneal n.
 sural n.
 tibial n.
 saphenous n.
o pulse
 dorsalis pedis
 posterior tibial : be sure to check contralateral side

Imaging
 Radiographs
o recommended views
 full length AP and lateral views of affected tibia
 AP, lateral and oblique views of ipsilateral knee and ankle
 CT : indications
 intra-articular fracture extension or suspicion of joint involvement
 CT ankle for spiral distal third tibia fracture
 to exclude posterior malleolar fracture
- 364 -
By Dr, AbdulRahman AbdulNasser Lower Extremity | Leg
Treatment of Closed Tibia Fractures
 Nonoperative
o closed reduction / cast immobilization
 indications
 closed low energy fxs with acceptable alignment
 < 5 degrees varus-valgus angulation
 < 10 degrees anterior/posterior angulation
 > 50% cortical apposition
 < 1 cm shortening
 < 10 degrees rotational malalignment
 if displaced perform closed reduction under general anesthesia
 certain patients who may be non-ambulatory (ie. paralyzed), or those unfit for surgery
 technique
 place in long leg cast and convert to functional brace at 4 weeks
 outcomes
 high success rate if acceptable alignment maintained
 risk of shortening with oblique fracture patterns
 risk of varus malunion with midshaft tibia fractures and an intact fibula
 non-union occurs in 1.1% of patients treated with closed reduction
 Operative
o external fixation
 indications
 can be useful for proximal or distal metaphyseal fxs
 complications
 pin tract infections common
 outcomes : higher incidence of malalignment compared to IM nailing
o IM Nailing
 indications
 unacceptable alignment with closed reduction and casting
 soft tissue injury that will not tolerate casting
 segmental fx
 comminuted fx
 ipsilateral limb injury (i.e., floating knee)
 polytrauma
 bilateral tibia fx
 morbid obesity
 contraindications
 pre-existing tibial shaft deformity that may preclude passage of IM nail
 previous TKA or tibial plateau ORIF (not strict contraindication)
 outcomes
 IM nailing leads to (versus external fixation)
 decreased malalignment
 IM nailing leads to (versus closed treatment)
 decrease time to union
 decreased time to weight bearing
 reamed vs. unreamed nails

- 365 -
OrthoBullets2017 Lower Extremity | Leg
 reamed possibly superior to unreamed nails for treatment of closed tibia fxs for
decrease in future bone grafting or implant exchange (SPRINT trial)
 recent studies show no adverse effects of reaming (infection, nonunion)
 reaming with use of a tourniquet is NOT associated with thermal necrosis of the tibial
shaft
o percutaneous locking plate
 indications
 proximal tibia fractures with inadequate proximal fixation from IM nailing
 distal tibia fractures with inadequate
distal fixation from IM nail
 complications
 non-union
 wound infection and dehiscence
 long plates may place superficial peroneal
nerve at risk
 Percutaneous plate shown to have (versus
infrapatellar IMN)
 Equivalent time to union
 Greater radiation exposrure
 Longer surgical duration
 Lower postoperative pain scores
 More difficulty in hardware removal

Treatment of Open Tibia Fractures


 Operative
o antibiotics, I&D
 indications
 all open fractures require an emergent I&D
 timing of I&D
 surgical debridement 6-8 hours after time of injury is preferred
 grossly contaminated wounds are irrigated in emergency department
 antibiotics
 standard abx for open fractures (institution dependent)
 cephalosporin given for 24-48 hours in Grade I,II, and IIIA open fractures
 aminoglycoside added in Grade IIIB injuries : minimal data to support this
 penicillin administered in farm injuries : minimal data to support this
 tetanus prophylaxis
 outcomes
 early antibiotic administration is the most important factor in reducing infection
 emergent and thorough surgical debridement is also an important factor
 must remove all devitalized tissue including cortical bone
o external fixation
 indications
 provisional external fixation an option for open fractures with staged IM nailing or
plating
 falling out of favor in last decade
 indicated in children with open physis
- 366 -
By Dr, AbdulRahman AbdulNasser Lower Extremity | Leg
o IM Nailing
indications
 most open fx can be treated with IM nail within 24 hours
 contraindicated in children with open physis (use flexible nail, plate, or external fixation
instead)
 outcomes for open fxs
 IM nailing vs. external fixation
 no difference with respect to
 infection rate
 union rate
 time to union
 IM nailing superior with respect to
 decreased malalignment
 decreased secondary surgeries
 shorter time to weight bearing
 reamed nails vs. unreamed nails
 reaming does not negatively affect union, infection, or need for additional surgeries in
open tibia fractures
 gapping at the fracture site is greatest risk for non-union
 transverse fx pattern and open fractures also at increased risk for non-union
 rhBMP-2
 prior studies have shown use in open tibial shaft fractures
 accelerate early fracture healing
 decrease rate of hardware failure
 decrease need for subsequent autologous bone-grafting
 decrease need for secondary invasive procedures
 decrease infection rate
 recent studies have not fully supported the above findings and rhBMP-2 remains
highly controversial
o amputation
 indications
 no current scoring system to determine if an amputation should be performed
 relative indications for amputation include
 significant soft tissue trauma
 warm ischemia > 6 hrs
 severe ipsilateral foot trauma
 outcomes
 LEAP study
 most important predictor of eventual amputation is the severity of ipsilateral
extremity soft tissue injury
 most important predictor of infection other than early antibiotic administration is
transfer to definitive trauma center
 study shows no significant difference in functional outcomes between amputation and
salvage
 loss of plantar sensation is not an absolute indication for amputation

- 367 -
OrthoBullets2017 Lower Extremity | Leg
Technique
 IM nailing of shaft fractures
o preparation
 anesthesia : general anesthesia recommended
 positioning
 patient positioned supine on radiolucent table
 bring fluoro in from opposite, non-injured, side
 bump placed under ipsilateral hip
 leave full access to foot and ankle to help judge intraoperative length, rotation, and
alignment of extremity
 tourniquet
 tourniquet placed on proximal thigh
 not typically inflated
 use in patients with vascular injury or significant bleeding associated with extensive soft
tissue injuries
 deflate during reaming or nail insertion (weak data to support this)
o approach
 options include
 medial parapatellar
 most common starting point
 can lead to valgus malalignment when used to treat proximal fractures
 lateral parapatellar
 helps maintain reduction when nailing proximal 1/3 fractures
 requires mobile patella
 patellar tendon splitting
 gives direct access to start point
 can damage patellar tendon or lead to patella baja (minimal data to support this)
 semiextended medial or lateral parapatellar
 used for proximal and distal tibial fractures
 suprapatellar (transquadriceps tendon)
 requires special instruments
 can damage patellofemoral joint
 starting point
 medial parapatellar tendon approach with knee flexed
 incision from inferior pole of patella to just above tibial tubercle
 identify medial edge of patellar tendon, incise
 peel fat pad off back of patellar tendon
 starting guidewire is placed in line with medial aspect of lateral tibial spine on AP
radiograph, just below articular margin on lateral view
 insert starting guide wire, ream
 semiextended lateral or medial parapatellar approach
 skin incision made along medial or lateral border of patella from superior pole of
patella to upper 1/3 of patellar tendon
 knee should be in 5-30 degrees of flexion
 choice to go medial or lateral is based of mobility of patella in either direction
 open retinaculum and joint capsule to level of synovium

- 368 -
By Dr, AbdulRahman AbdulNasser Lower Extremity | Leg
 free retropatellar fat pad from posterior surface of patellar tendon
 identify starting point as mentioned previously
o fracture reduction techniques
 spanning external fixation (ie. traveling traction)
 clamps
 femoral distractor
 small fragment plates/screws
 intra-cortical screws
o reaming
 reamed nails superior to unreamed nails in closed fractures
 be sure tourniquet is released
 advance reamers slowly at high speed
 overream by 1.0-1.5mm to facilitate nail insertion
 confirm guide wire is appropriately placed prior to reaming
o nail insertion
 insert nail in slight external rotation to move distal interlocking screws anteriorly decreasing
risk of NVS injury
 if nail does not pass, remove and ream 0.5-1.0mm more
o locking screws
 statically lock proximal and distally for rotational stability
 no indication for dynamic locking acutely
 number of interlocking screws is controversial
 two proximal and two distal screws in presence of <50% cortical contact
 consider 3 interlock screws in short segment of distal or proximal shaft fracture

Complications
 Knee pain
o >50% anterior knee pain with IM nailing
 occurs with patellar tendon splitting and paratendon approach
 pain relief unpredictable with nail removal
o lateral radiograph is best radiographic views to make sure nail is not too proud proximally
 Malunion
o high incidence of valgus and procurvatum (apex anterior) malalignment in proximal third
fractures
o varus malunion leads to ipsilateral ankle pain and stiffness
o chronic angular deformity is defined by the proximal and distal anatomical/mechanical axis of
each segment
 center of rotation of angulation is intersection of proximal and distal axes
 Nonunion
o definition
 delayed union if union at 6-9 mos.
 nonunion if no healing after 9 mos.
o treatment
 nail dynamization if axially stable
 exchange nailing if not axially stable
 reamed exchange nailing most appropriate for aseptic, diaphyseal tibial nonunions with
less than 30% cortical bone loss.
- 369 -
OrthoBullets2017 Lower Extremity | Leg
 consider revision with plating in metaphyseal nonunions
 posterolateral bone grafting if significant bone loss
 non-invasive techniques (electrical stimulation, US)
 BMP-7 (OP-1) has been shown equivalent to autograft
 often used in cases of recalcitrant non-unions
 compression plating has been shown to have 92-96% union rate after open tibial fractures
initially treated with external fixation
 Malrotation
o most commonly occurs after IM nailing of distal 1/3 fractures
o can assess tibial rotation by obtaining perfect lateral fluoroscopic image of knee, then rotating c-
arm 105-110 degrees to obtain mortise view of ipsilateral ankle
o reduced risk with adjunctive fibular plating
 Compartment syndrome
o incidence 1-9% : can occur in both closed and open tibia shaft fxs
o diagnosis
 high index of clinical suspicion
 pain out of proportion
 pain with passive stretch
 compartment pressure within 30mm Hg of diastolic BP is most sensitive diagnostic test
o treatment
 emergent four compartment fasciotomy
o outcome
 failure to recognize and treat compartment syndrome is most common reason for successful
malpractice litigation against orthopaedic surgeons
o prevention
 increased compartment pressure found with
 traction (calcaneal)
 leg positioning
 Nerve injury
o LISS plate application without opening for distal screw fixation near plate holes 11-13 put
superficial peroneal nerve at risk of injury due to close proximity
o saphenous nerve can be injured during placement of locking screws
o transient peroneal nerve palsy can be seen after closed nailing
 EHL weakness and 1st dorsal webspace decreased sensation
 treated nonoperatively; variable recovery is expected

4. Tibial Plafond Fractures


Introduction
 Also known as pilon fractures
 Epidemiology
o incidence
 account for <10% of lower extremity injuries
 incidence increasing as survival rates after motor vehicle collisions increase
o demographics
 average patient age is 35-40 years
 more common in males than females
- 370 -
By Dr, AbdulRahman AbdulNasser Lower Extremity | Leg
 Pathophysiology
o mechanism
 high energy axial load (motor vehicle accidents, falls from height)
o pathoanatomy
 often characterized by
 articular impaction and comminution
 metaphyseal bone comminution
 soft tissue injury (open or Tscherne II/III closed fractures)
 associated musculoskeletal injuries
 3 fragments typical with intact ankle ligaments
 medial malleolar (deltoid ligament)
 posterolateral/Volkmann fragment (posterior inferior tibiofibular ligament)
 anterolateral/Chaput fragment (anterior inferior tibiofibular ligament)
 Associated conditions : 75% have associated fibula fractures
 Prognosis
o parameters that correlate with a poor clinical outcome and inability to return to work
 lower level of education
 pre-existing medical comorbidities
 male sex
 work-related injuries
 lower income levels

Anatomy
 Osteology
o tibia
 distal tibia forms an inferior quadrilateral surface and pyramid-shaped medial malleolus
 articulates with the talus and fibula laterally via the fibula notch
 Vascular anatomy
o anterior tibial artery
 first branch of popliteal artery
 passes between 2 heads of tibialis posterior and interosseous membrane (IOM)
 lies anterior to IOM between tibialis anterior and EHL
 terminates as dorsalis pedis artery
o posterior tibial artery
 continues in deep posterior compartment of leg
 courses obliquely to pass behind medial malleolus
 terminates by dividing into medial and lateral plantar arteries
o peroneal artery
 main branch takes off 2.5 cm distal to popliteal fossa
 continues in deep posterior compartment between tibialis posterior and FHL
 terminates as calcaneal branches
 Nerves
o tibial nerve (L4-S3)
 crosses over popliteus from the popliteal fossa and splits 2 heads of gastrocnemius
 passes deep to soleus coursing to the posterior aspect of the medial malleolus
 terminates as medial and lateral plantar nerves
 muscular branches supply posterior leg (superficial and deep posterior compartments)
- 371 -
OrthoBullets2017 Lower Extremity | Leg
o common peroneal nerve (L4-S2)
 winds around neck of fibula and runs deep to peroneus longus
 divides into superficial and deep peroneal nerves
o superficial peroneal nerve
 courses along border between lateral and anterior compartments of leg
 supplies muscular branches to peroneus longus and brevis (lateral compartment)
 terminates as medial dorsal and intermediate dorsal cutaneous nerves
o deep peroneal nerve
 courses along anterior surface of IOM
 supplies musculature of anterior compartment and sensation to first web space
o saphenous nerve (L3-L4)
 continuation of femoral nerve of the thigh
 becomes subcutaneous on medial aspect of knee between sartorius and gracilis
 supplies sensation to medial aspect of leg and foot
o sural nerve (S1-S2)
 formed by cutaneous branches of tibial (medial sural cutaneous) and common peroneal
(lateral sural cutaneous) nerves
 lies on lateral aspect of leg and foot

Classification
AO/OTA Classification
43-A Extra-articular
43-B Partial articular
43-C Complete articular
Each category is further subdivided based on amount and degree of comminution

Ruedi and Allgower Classification


Type I Nondisplaced
Type II Simple displacement with incongruous joint
Type III Comminuted articular surface
Each category is further subdivided based on amount and degree of comminution

AO/OTA Classification Ruedi and Allgower Classification

- 372 -
By Dr, AbdulRahman AbdulNasser Lower Extremity | Leg

Ruedi type 1 Ruedi type 2 Ruedi type 3

Presentation
 Symptoms
o ankle pain, inability to bear weight, deformity
 Physical exam
o inspection
 examine soft tissue integrity
 swelling, abrasions, ecchymosis, fracture blisters, open wounds
 examine for associated musculoskeletal injuries
o ROM & stability
 examine stability and alignment of the ankle joint
o neurovascular
 check DP and PT pulses
 look for neurologic compromise
 check for signs of compartment syndrome

Imaging
 Radiographs
o recommended views
 AP, lateral, mortise views of ankle
 full-length tibia/fibula and foot x-rays performed for fracture extension
 CT scan
o delineate articular involvement
o surgical planning
o most useful after ligamentotaxis is provided by a spanning external fixator

Treatment
 Nonoperative
o immobilization
 indications
 stable fracture patterns without articular surface displacement
 critically ill or nonambulatory patients
 significant risk of skin problems (diabetes, vascular disease, neuropathy)
 technique
 long leg cast for 6 weeks followed by fracture brace and ROM exercises
 alternative treatment is with early ROM
 outcomes
- 373 -
OrthoBullets2017 Lower Extremity | Leg
 intra-articular fragments are unlikely to reduce with manipulation of displaced fractures
 loss of reduction is common
 inability to monitor soft tissue injuries is a major disadvantage
 Operative
o temporizing spanning external fixation across ankle joint
 indications
 acute management
 provides stabilization to allow for soft tissue healing
 fractures with significant joint depression or displacement
 leave until swelling resolves (generally 10-14 days)

o ORIF
 indications
 definitive fixation for majority of pilon fractures
 limited or definitive ORIF can be performed acutely with low complications in certain
situations
 outcomes
 ability to drive
 brake travel time returns to normal 6 weeks after weight bearing
o external fixation alone
 indications
 may be indicated in select cases
o intramedullary nailing with percutaneous screw fixation
 alternative to ORIF for fractures with simple intra-articular component (AO/OTA 43 C1/C2)

Techniques
 External fixation
o fixation
 joint-spanning articulated vs. nonspanning hybrid ring
 none have been shown to be superior with respect to ankle stiffness
 2 tibial shaft half pins connected to hindfoot half pins or calcaneal transfixation pin
 with hybrid fixators, thin wires may be placed within joint capsule or within zone of injury
o soft tissues
 maintain soft tissue attachments of fragments
 Chaput fragment - anterior inferior tibiofibular ligament
o pros
 decreased incidence of wound complications and deep infections compared to ORIF
 can combine with limited percutaneous fixation using lag screws
o cons
 pin and wire tract infections
 loss of ankle motion
 injury to neurovascular structures
 anatomic articular reconstruction may not be possible, especially with central depression
 ORIF (AO technique)
o approach
 use of multiple small incisions that can include

- 374 -
By Dr, AbdulRahman AbdulNasser Lower Extremity | Leg

 direct anterior approach to ankle


 anterolateral approach to ankle
 useful with fractures impacted in valgus or with an intact fibula
 puts the deep peroneal nerve at risk during exposure and dissection in the anterior
compartment
 superficial peroneal nerve at risk during superficial dissection in the lateral
compartment
 anteromedial approach to ankle
 medial approach
 posteromedial approach
 posterolateral approach
 lateral approach
 must respect soft tissues (generally >7 cm skin bridge with full thickness skin flaps)
o goals
 anatomic reduction of articular surface
 restore length
 reconstruct metaphyseal shell
 bone graft
 reattach metaphysis to diaphysis
o steps
 reduce and instrument fibula to establish lateral column length (if needed)
 reduce articular surface
 reattach articular block to metaphysis and shaft
o fixation
 may be augmented with external fixation (with or without limited ORIF)
 can use anterolateral, anterior, anteromedial, medial, or posterior plating techniques for the
tibia
 location of plates/screws are fracture and soft-tissue dependent
 ORIF of fibula if needed
 can be with intramedullary screw/wire or plate/screw construct
o pros
 direct anatomic reduction
 rigid fixation
 early motion of ankle
 clinical improvement may occur for up to 2 years
o cons
 high incidence of soft tissue complications and infection without staged ORIF

Complications
 Wound slough (10%)
o free flap for postoperative wound breakdown
 Dehiscence (9-30%)
o wait for soft tissue edema to subside before ORIF (1-2 weeks)
 Infection (5-15%)
 Varus malunion

- 375 -
OrthoBullets2017 Lower Extremity | Leg
 Nonunion
o usually at metaphyseal junction
o treat with bone grafting and plate fixation
o more common with hybrid fixation
 Posttraumatic arthritis
o most commonly begins 1-2 years postinjury
o arthrodesis is not commonly required until many years later
 Chondrolysis
 Stiffness

Collected By : Dr AbdulRahman
AbdulNasser
June 2017

- 376 -
By Dr, AbdulRahman AbdulNasser Lower Extremity | Ankle and Hindfoot

D. Ankle and Hindfoot

1. Ankle Fractures
Introduction
 Injury patterns
o isolated medial malleolus fracture
o isolated lateral malleolus fracture
o bimalleolar and bimalleolar-equivalent fractures
o posterior malleolus fractures
o Bosworth fracture-dislocations
o open ankle fractures
o associated syndesmotic injuries
 isolated syndesmosis injury

Anatomy
 Biomechanics
o deltoid ligament (deep portion)
 primary restraint to anterolateral talar displacement
o fibula
 acts as buttress to prevent lateral displacement of talus

Imaging
 Radiographs
o external rotation stress radiograph
 most appropriate stress radiograph to assess competency of deltoid ligament
 a medial clear space of >5mm with external rotation stress applied to a dorsiflexed ankle
is predictive of deep deltoid disruption
 more sensitive to injury than medial tenderness, ecchymosis, or edema
 gravity stress radiograph is equivalent to manual stress radiograph
 syndesmosis
 decreased tibiofibular overlap
 normal >6 mm on AP view
 normal >1 mm on mortise view
 increased medial clear space
 normal less than or equal to 4 mm
 increased tibiofibular clear space
 normal <6 mm on both AP and mortise views
o radiographic measurements
 talocrural angle
 measured by bisection of line through tibial anatomical axis and another line through the
tips of the malleoli
 shortening of lateral malleoli fractures can lead to increased talocrural angle
 talocrural angle is not 100% reliable for estimating restoration of fibular length
 can also utilize the realignment of the medial fibular prominence with the tibiotalar
joint

- 377 -
OrthoBullets2017 Lower Extremity | Ankle and Hindfoot

tibiofibular overlap medial clear space tibiofibular clear space

Classification
 Lauge-Hansen
o based on foot position and force of applied stress/force
o has been shown to predict the observed (via MRI) ligamentous injury in less than 50% of
operatively treated fractures
Lauge-Hansen Class Sequence
Supination - Adduction (SA) 1. Talofibular sprain or distal fibular avulsion
2. Vertical medial malleolus and impaction of anteromedial distal tibia
Supination - External Rotation (SER) 1. Anterior tibiofibular ligament sprain
2. Lateral short oblique fibula fracture (anteroinferior to
posterosuperior)
3. Posterior tibiofibular ligament rupture or avulsion of posterior
malleolus
4. Medial malleolus transverse fracture or disruption of deltoid
ligament
Pronation - Abduction (PA) 1. Medial malleolus transverse fracture or disruption of deltoid
ligament
2. Anterior tibiofibular ligament sprain
3. Transverse comminuted fracture of the fibula above the level of
the syndesmosis
Pronation - External Rotation (PER) 1. Medial malleolus transverse fracture or disruption of deltoid
ligament
2. Anterior tibiofibular ligament disruption
3. Lateral short oblique or spiral fracture of fibula (anterosuperior to
posteroinferior) above the level of the joint
4. Posterior tibiofibular ligament rupture or avulsion of posterior
malleolus

- 378 -
By Dr, AbdulRahman AbdulNasser Lower Extremity | Ankle and Hindfoot

‎VI:30 Supination - Adduction

‎VI:31 Supination - External Rotation

- 379 -
OrthoBullets2017 Lower Extremity | Ankle and Hindfoot

‎VI:34 Pronation -
Abduction

‎ I:32 Pronation -
V
External Rotation

‎VI:35 Pronation - External Rotation

‎ I:33 Pronation -
V
External Rotation

- 380 -
By Dr, AbdulRahman AbdulNasser Lower Extremity | Ankle and Hindfoot
 Anatomic / Descriptive
o isolated medial malleolar
o isolated lateral malleolar
o bimalleolar
o trimalleolar
o Bosworth fracture-dislocation (posterior dislocation of the fibula behind incisura fibularis)
 Danis-Weber (location of fibular fracture)
o A - infrasyndesmotic (generally not associated with ankle instability)
o B - transsyndesmotic
o C - suprasyndesmotic
 AO / ATA
o 44A - infrasyndesmotic
o 44B - transsyndesmotic
o 44C - suprasyndesmotic

General Treatment
 Nonoperative
o short-leg walking cast/boot
 indications
 isolated nondisplaced medial malleolus fracture or tip avulsions
 isolated lateral malleolus fracture with < 3mm displacement and no talar shift
 posterior malleolar fracture with < 25% joint involvement or < 2mm step-off
 Operative
o open reduction internal fixation
 indications
 any talar displacement
 displaced isolated medial malleolar fracture
 displaced isolated lateral malleolar fracture
 bimalleolar fracture and bimalleolar-equivalent fracture
 posterior malleolar fracture with > 25% or > 2mm step-off
 Bosworth fracture-dislocations
 open fractures
 technique
 goal of treatment is stable anatomic reduction of talus in the ankle mortise
 1 mm shift of talus leads to 42% decrease in tibiotalar contact area
 see fracture patterns below for specific treatment
 outcomes
 overall success rate of 90%
 prolonged recovery expected (2 years to obtain final functional result)
 significant functional impairment often noted
 worse outcomes with: smoking, decreased education, alcohol use, increased age, presence
of medial malleolar fracture
 ORIF superior to closed treatment of bimalleolar fractures
 in Lauge-Hansen supination-adduction fractures, restoration of marginal impaction of the
anteromedial tibial plafond leads to optimal functional results after surgery
 postoperative rehabilitation

- 381 -
OrthoBullets2017 Lower Extremity | Ankle and Hindfoot
 time for proper braking response time (driving) returns to baseline at nine weeks for
operatively treated ankle fractures
 braking travel time is significantly increased until 6 weeks after initiation of weight
bearing in both long bone and periarticular fractures of the lower extremity
Isolated Medial Malleolus Fracture
 Nonoperative
o short leg walking cast or cast boot
 indications
 nondisplaced fracture and tip avulsions
 deep deltoid inserts on posterior colliculus
 symptomatic treatment often appropriate
 Operative
o ORIF
 indications
 any displacement or talar shift
 technique
 lag screw fixation
 lag screw fixation stronger if placed perpendicular to fracture line
 antiglide plate with lag screw
 best for vertical shear fractures
 tension band fixation
 utilizing stainless steel wire
Isolated Lateral Malleolus Fracture
 Nonoperative
o short leg walking cast vs cast boot
 indications
 if intact mortise, no talar shift, and < 3mm displacement
 classically fractures with more than 4-5 mm of medial clear space widening on stress
radiographs have been considered unstable and need to be treated surgically
 recent studies have shown the deep deltoid may be intact with up to 8-10 mm of
widening on stress radiographs
 if the mortise is well reduced, results from operative and non-operative treatment are
similar
 Operative
o ORIF
 indications
 if talar shift or > 3 mm of displacement
 can be treated operatively if also treating an ipsilateral syndesmosis injury
 technique
 open reduction and plating
 plate placement
 lateral
 lag screw fixation with neutralization plating
 bridge plate technique
 posterior
 antiglide technique

- 382 -
By Dr, AbdulRahman AbdulNasser Lower Extremity | Ankle and Hindfoot
 lag screw fixation with neutralization plating
 most common disadvantage of using posterior antiglide plating is peroneal
irritation if the plate is placed too distally
 posterior antiglide plating is biomechanically superior to lateral plate placement
 intramedullary retrograde screw placement
 isolated lag screw fixation
 possible if fibula is a spiral pattern and screws can be placed at least 1 cm apart
 post-operative care
 period of immobilization usually 4-6 weeks after ORIF
 duration of immobilization should be doubled in Diabetic patients
Medial and Lateral (Bimalleolar) Fracture
 Nonoperative
o total contact casting
 indications
 elderly or unable to undergo surgical intervention
 Operative
o ORIF
 indications
 any lateral talar shift
 technique
 fibula
 need to fix with one of the options listed in section above
 medial malleolus
 fixation options
 cancellous lag screws
 bicortical screws
 tension band wiring
 antiglide plate to treat a vertical medial malleolus fracture
 orient screws parallel to joint for vertical medial malleolar fracture (Lauge-Hansen
supination-adduction fracture pattern)
Functional Bimalleolar Fracture (deltoid ligament tear with fibular fracture)
 Operative
o ORIF of lateral malleolus
 indications
 examination has been shown to be largely unreliable in predicting medial injury
 can see significant lateral translation of the talus in this pattern
 technique
 not necessary to repair medial deltoid ligament
 only need to explore medially if you are unable to reduce the mortise
 see isolated fibular fracture techniques above
Posterior Malleolar Fracture
 Nonoperative
o short leg walking cast vs cast boot
 indications
 < 25% of articular surface involved
 evaluation of percentage should be done with CT, as plain radiology is unreliable

- 383 -
OrthoBullets2017 Lower Extremity | Ankle and Hindfoot
 < 2 mm articular stepoff
 syndesmotic stability
 Operative
o ORIF
 indications
 > 25% of articular surface involved
 > 2 mm articular stepoff
 syndesmosis injury
 technique
 approach
 posterolateral approach
 posteromedial approach
 decision of approach will depend on fracture lines and need for fibular fixation
 fixation
 anterior to posterior lag screws to capture fragment (if nondisplaced)
 posterior to anterior lag screw and buttress plate
 antiglide plate
 syndesmosis injury
 stiffness of syndesmosis restored to 70% normal with isolated fixation of posterior
malleolus (versus 40% with isolated syndesmosis fixation)
 stress examination of syndesmosis still required after posterior malleolar fixation
 posteroinferior tibiofibular ligament may remain attached to posterior malleolus and
syndesmotic stability may be restored with isolated posterior malleolar fixation
Bosworth Fracture-Dislocation
 Overview
o rare fracture-dislocation of the ankle where the fibula becomes entrapped behind the tibia and
becomes irreducible
o posterolateral ridge of the distal tibia hinders reduction of the fibula
 Operative
o open reduction and fixation of the fibula in the incisura fibularis
 indicated in most cases
Open Ankle Fracture
 Operative
o emergent operative debridement and ORIF
 indicated if soft tissue conditions allow
 primary closure at the index procedure can be performed in appropriately-selected Gustilo-
Anderson grade I, II, and IIIA open fractures in otherwise healthy patients sustaining low-
energy injuries without gross contamination
o external fixation
 indications
 soft tissue conditions and overall patient characteristics

- 384 -
By Dr, AbdulRahman AbdulNasser Lower Extremity | Ankle and Hindfoot

Associated Syndesmotic Injury Dime sign


Associated Syndesmotic Injury
 Overview
o suspect injury in all ankle fractures
 most common in Weber C fracture patterns
 fixation usually not required when fibula fracture within 4.5 cm of plafond
 up to 25% of tibial shaft fractures will have ankle injury
 Evaluation
o measure clear space 1 cm above joint
 it has also been reported that there is no actual correlation between syndesmotic injury and
tibiofibular clear space or overlap measurements
 lateral stress radiograph has more interobserver reliability than an AP/mortise stress film
o best option is to assess stability intraoperatively with abduction/external rotation stress of
dorsiflexed foot
o instability of the syndesmosis is greatest in the anterior-posterior direction
 Treatment
o operative
 syndesmotic screw fixation
 indications
 widening of medial clear space
 tibiofibular clear space (AP) greater than 5 mm
 tibiofibular overlap (mortise) narrowed
 any postoperative malalignment or widening should be treated with open
debridement, reduction, and fixation
 technique
 length and rotation of fibula must be accurately restored
 outcomes are strongly correlated with anatomic reduction
 "Dime sign"/Shentons line to determine length of fibula
 open reduction required if closed reduction unsuccessful or questionable
 one or two cortical screw(s) 2-4 cm above joint, angled posterior to anterior 20-30
degrees
 lag technique not desired
 maximum dorsiflexion of ankle not required during screw placement (can't
overtighten a properly reduced syndesmosis)
 postoperative
 screws should be maintained in place for at least 8-12 weeks

- 385 -
OrthoBullets2017 Lower Extremity | Ankle and Hindfoot
 must remain non-weight bearing, as screws are not biomechanically strong enough to
withstand forces of ambulation
 controversies
 number of screws
 1 or 2 most commonly reported
 number of cortices
 3 or 4 most commonly reported
 size of screws
 3.5 mm or 4.5 mm screws
 implant material (stainless steel screws, titanium screws,
suture, bioabsorbable materials)
 need for hardware removal
 no difference in outcomes seen with hardware
maintenance (breakage or loosening) or removal at 1
year
 outcome may be worse with maintenance of intact screws
Diabetic Ankle Fractures (with or without Neuropathy)
 Risks
o prolonged healing
o high risk of hardware failure
o high risk of infection
 Enhanced fixation
o multiple quadricortical syndesmotic screws (even in the absence of syndesmotic injury)
o tibiotalar Steinmann pins or hindfoot nailing
o ankle spanning external fixation
o augment with intramedullary fibula K-wires
o stiffer, more rigid fibular plates (instead of 1/3 tubular plates)
 compression plates
 small fragment locking plates
 Delay weightbearing
o maintain non-weightbearing postop for 8-12 weeks (instead of 4-8 weeks in normal patients

Complications
 Wound problems (4-5%)
 Deep infections (1-2%)
o up to 20% in diabetic patients
 largest risk factor for diabetic patients is presence of peripheral neuropathy
 Post-traumatic arthritis
o rare with anatomic reduction and fixation
o corrective osteotomy requires anatomic fibular and mortise correction for optimal outcomes

- 386 -
By Dr, AbdulRahman AbdulNasser Lower Extremity | Ankle and Hindfoot

2. Talar Neck Fractures


Introduction
 Epidemiology
o most common fracture of talus ( 50%)
 Mechanism
o a high-energy injury
o is forced dorsiflexion with axial load
 Associated conditions
o ipsilateral lower extremity fractures common

Anatomy
 Articulation
o inferior surface articulates with posterior facet of calcaneus
o talar head articulates with
 navicular bone
 sustenaculum tali
o lateral process articulates with
 posterior facet of calcaneus
 lateral malleolus of fibula
o posterior process consist of medial and lateral
tubercles separated by groove for FHL
 Blood supply
o talar neck supplied by three sources
 posterior tibial artery
 via artery of tarsal canal (dominant
supply)
 supplies majority of talar body
 deltoid branch of posterior tibial artery
 supplies medial portion of talar body

may be only remaining blood supply with a displaced fracture



 anterior tibial artery
 supplies head and neck
 perforating peroneal artery via artery of tarsal sinus
 supplies head and neck

Classification
Hawkins Classification
Type Description AVN
Hawkins I Nondisplaced 0-13% AVN
Hawkins II Subtalar dislocation 20-50%
Hawkins III Subtalar and tibiotalar dislocation 20-100%
Hawkins IV Subtalar, tibiotalar, 70-100%
and talonavicular dislocation

- 387 -
OrthoBullets2017 Lower Extremity | Ankle and Hindfoot

Imaging
 Radiographs
o recommended views
 AP and lateral
 Canale View
 optimal view of talar neck
 technique is maximum equinus, 15 degrees pronated,
Xray 75 degrees cephalad from horizontal
 CT scan
o best study to determine degree of displacement, comminution and articular congruity
o CT scan also will assess for ipsilateral foot injuries (up to 89% incidence)

Treatment
 Nonoperative
o emergent reduction in ER
 indications
 all cases require emergent closed reduction in ER
o short leg cast for 8-12 weeks (NWB for first 6 weeks)
 indications
 nondisplaced fractures (Hawkins I)
 CT to confirm nondisplaced without articular stepoff
 Operative
o open reduction and internal fixation
 indications

 all displaced fractures (Hawkins II-IV)


 techniques
 extruded talus should be replaced and treated with ORIF
 complications

- 388 -
By Dr, AbdulRahman AbdulNasser Lower Extremity | Ankle and Hindfoot
 post-traumatic arthritis
 mal-union
 non-union
 infection
 wound dehiscence

Techniques
 ORIF
o approach
 two approaches recommended
 visualize medial and lateral neck to assess reduction
 typical areas of comminution are dorsal and medial
 anteromedial
 between tibialis anterior and posterior tibialis
 preserve soft tissue attachments, especially deep deltoid ligament (blood supply)
 medial malleolar osteotomy to preserve deltoid ligament
 anterolateral
 between tibia and fibula proximally, in line with 4th ray
 elevate extensor digitorum brevis and remove debris from subtalar joint
o technique
 anatomic reduction essential
 variety of implants used including mini and small fragment screws, cannulated screws and
mini fragment plates
 medial and lateral lag screws may be used in simple fracture patterns
 consider mini fragment plates in comminuted fractures to buttress against varus collapse
o postoperative : non-weight-bearing for 10-12 weeks

Complications
 Osteonecrosis
o 31% overall (including all subtypes)
o radiographs
 hawkins sign
 subchondral lucency best seen on mortise Xray at 6-8 weeks
 indicates intact vascularity with resorption of subchondral bone
 associated with talar neck comminution and open fractures
 Posttraumatic arthritis
o subtalar arthritis (50%) is the most common complication
o tibiotalar arthritis (33%)
 Varus malunion (25-30%)
o can be prevented by anatomic reduction
o treatment includes medial opening wedge osteotomy of talar neck
o leads to
 decreased subtalar eversion
‎VI:37 hawkins sign
 decreased motion with locked midfoot and hindfoot
 weight bearing on the lateral border of the foot

- 389 -
OrthoBullets2017 Lower Extremity | Ankle and Hindfoot

3. Talus Fracture (other than neck)


Introduction
 Epidemiology
o less than 1% of all fractures
o second most common tarsal fractures after calcaneus fxs
o talar body fractures
 account for 13-23% of talus fractures
o lateral process fractures
 account for 10.4% of talus fractures
o talar head fracture
 least common talus fracture
 Mechanism
o talar body
 injuries often result from high energy trauma, with the hindfoot either in supination or
pronation
o lateral process of talus
 injuries result from forced dorsiflexion, axial loading, and inversion with external rotation
 often seen in snowboarders
 Prognosis
o lateral process injuries have a favorable outcomes with prompt diagnosis and immediate
treatment
Anatomy
 3D Anatomy of talus
 Talus has no muscular or tendinous attachments
 Articulation
o there are 5 articulating surfaces
 seventy percent of the talus is covered by cartilage
 inferior surface articulates with posterior facet of calcaneus
o talar head articulates with
 navicular bone
 sustenaculum tali
o lateral process articulates with
 posterior facet of calcaneus
 lateral malleolus of fibula
 this forms the lateral margin of the talofibular joint
o posterior process consist of medial and lateral tubercle separated by groove for FHL
 Blood supply
o because of limited soft tissue attachments, the talus has a direct extra-osseous blood supply
o sources include
 posterior tibial artery
 via artery of tarsal canal (most important and main supply)
 supplies most of talar body
 via calcaneal braches
 supplies posterior talus

- 390 -
By Dr, AbdulRahman AbdulNasser Lower Extremity | Ankle and Hindfoot
 anterior tibial artery
 supplies head and neck
 perforating peroneal arteries via artery of tarsal sinus
 supplies head and neck
 deltoid artery (located in deep segment of deltoid ligament)
 supplies body
 may be only remaining blood supply with a talar neck fracture

Classification
 Anatomic classification
o Lateral Process Fx
 type 1 fractures do not involved the articular surface
 type 2 fractures involve the subtalar and talofibular joints
 type 3 fractures have comminution
o Posterior Process Fx
 posteromedial tubercle fractures
 result from an avulsion of the posterior talotibial ligament
or posterior deltoid ligament
 posterolateral tubercle fractures
 result from an avulsion of the posterior talofibular
ligament
o Talar Head Fx
o Talar Body Fx

Physical Exam
 Symptoms
o pain ‎VI:38 Lateral Process Fx
 lateral process fractures often misdiagnosed as ankle sprains
 Physical exam
o provocative tests
 pain aggravated by FHL flexion or extension may be found with a posterolateral tubercle
fractures
Imaging
 Radiographs
o recommended views
 AP and lateral
 lateral process fractures may be viewed on AP radiographs
 Canale View
 optimal view of talar neck
 technique
 maximum equinus
 15% pronated
 Xray 75 degrees cephalad from horizontal
‎VI:39 os trigonum
 careful not to mistake os trigonum (present in up to 50%) for fracture
 may be falsely negative in talar lateral process fx

- 391 -
OrthoBullets2017 Lower Extremity | Ankle and Hindfoot
 CT scan
o indicated when suspicion is high and radiographs are negative
 best study for posterior process fx, lateral process fx, and posteromedial process fx
o helpful to determine degree of displacement, comminution, and articular congruity
 MRI
o can be used to confirm diagnosis when radiographs are negative

Lateral Process Fx radiographs Lateral Process Fx CT

Lateral Process Fx Lateral Process Fx


MRI Bone scan Posterior Process Fx radiograph

Talar Body Fx Radiographs Talar Body Fx CT

Treatment
 Nonoperative
o SLC for 6 weeks
 indications
 nondisplaced (< 2mm) lateral process fractures
 nondisplaced (< 2mm) posterior process fractures
 nondisplaced (< 2mm) talar head fractures
 nondisplaced (< 2mm) talar body fractures
 technique : cast molded to support longitudinal arch
 Operative

- 392 -
By Dr, AbdulRahman AbdulNasser Lower Extremity | Ankle and Hindfoot
o ORIF/Kirshner wire Fixation
 indications
 displaced (> 2mm) lateral process fractures
 displaced (> 2mm) talar head fractures
 displaced (> 2mm) talar body fractures
 medial, lateral or posterior malleolar osteotomies may be necessary
 displaced (> 2mm) posteromedial process fractures
 may require osteotomies of posterior or medial malleoli to adequately reduce the
fragments
o fragment excision
 indications
 comminuted lateral process fractures
 comminuted posterior process fractures
 nonunions of posterior process fractures

Technique
 ORIF/Kirshner Wires
o approaches
 lateral approach
 for lateral process fractures
 incision over tarsal sinus, reflect EDB distally
 posteromedial approach
 for medial tubercle of posterior process fracture or for entire posterior process fracture
that has displaced medially
 between FDL and neurovascular bundle
 posterolateral approach
 for lateral tubercle of posterior process fractures
 between peroneal tendons and Achilles tendon (protect sural nerve)
 beware when dissecting medial to FHL tendon (neurovascular bundle lies there)
 combined lateral and medial approach
 required for talar body fractures with more than 2 mm of displacement
 Fragment excisions
o incompetence of the lateral talocalcaneal ligament is expected with excision of a 1 cm fragment
 this is biomechanically tolerated and does not lead to ankle or subtalar joint instability

Complications
 AVN : Hawkins sign (lucency) indications revascularization
o Lack of Hawkins sign with sclerosis is indicative of AVN
 Talonavicular arthritis
o posttraumatic arthritis is common in all of these fractures
o this can be treated with an arthrodesis of the talonavicular joint
 Malunion
 Chronic pain from symptomatic nonunion : may have pain up to 2 years after treatment
 Subtalar arthritis : found in 45% of patients with lateral process fractures, treated either non-
operatively or operatively

- 393 -
OrthoBullets2017 Lower Extremity | Ankle and Hindfoot

4. Subtalar Dislocations
Introduction
 Typically from a high-energy mechanism
o 25% may be open
 lateral dislocations more likely to be open
o 65% to 80% are medial dislocations
o remaining are lateral dislocations
o case reports of anterior or posterior dislocations
 Associated injuries
o associated dislocations
 talonavicular
o associated fractures (up to 44%)
 with medial dislocation
 dorsomedial talar head
 posterior process of talus
 navicular
 with lateral dislocation
 cuboid
 anterior calcaneus
 lateral process of talus
 fibula
‎ I:40 assiciated with posterior process fx of talus
V
Presentation
 Physical exam
o foot will be locked in supination with medial dislocation
o foot will be locked in pronation with lateral dislocation

Imaging
 Radiographs
o medial subtalar dislocation
 talar head will be superior to navicular on lateral image
o lateral subtalar dislocation
 talar head will be colinear or inferior to navicular on lateral image
 CT scan
o perform following reduction
o look for associated injuries or subtalar debris

Treatment
 Nonoperative
o closed reduction and short leg non-weight bearing cast for 4-6 weeks
 indications
 first line of treatment
 60-70% can be reduced by closed methods
 technique
 requires adequate sedation
 typical maneuvers include knee flexion and ankle plantar flexion

- 394 -
By Dr, AbdulRahman AbdulNasser Lower Extremity | Ankle and Hindfoot
 followed by distraction and hindfoot inversion or eversion depending on direction of
dislocation
 perform a post-reduction CT to look for associated injuries
 Operative
o open reduction
 indications
 failure of closed reduction
 up to 32% require open reduction
 medial dislocation reduction blocked by lateral structures including
 peroneal tendons
 extensor digitorum brevis
 talonavicular joint capsule
 lateral dislocation reduction blocked by medial structures including
 posterior tibialis tendon
 flexor hallucis longus
 flexor digitorum longus
 place temporary transarticular pins as needed if joint remains unstable

‎VI:41 lateral dislocation open reduction blocked by posterior tibialis tendon


Complications
 Post-traumatic Arthritis
o long-term follow up of these injuries show degenerative changes
 Subtalar joint most commonly affected with up to 89% of patients demonstrating
radiographic arthrosis (63% symptomatic)

- 395 -
OrthoBullets2017 Lower Extremity | Ankle and Hindfoot

5. Calcaneus Fractures
Introduction
 Epidemiology
o incidence
 most frequent tarsal fracture
 17% open fractures
 Pathophysiology
o mechanism
 traumatic axial loading is the primary mechanism of injury
 fall from height
 motor-vehicle accidents
o pathoanatomy
 intra-articular fractures
 primary fracture line results from oblique shear and leads to the following two primary
fragments
 superomedial fragment (constant fragment)
 includes the sustentaculum tali and is stabilized by strong ligamentous and
capsular attachments
 superolateral fragment
 includes an intra-articular aspect through the posterior facet
 secondary fracture lines
 dictate whether there is joint depression or tongue-type fracture
 extra-articular fractures
 strong contraction of gastrocnemius-soleus with concomitant avulsion at its insertion site
on calcaneus
 more common in osteopenic bone
 anterior process fractures
 inversion and plantar flexion of the foot cause avulsion of the bifurcate ligament
 Associated injuries
o orthopaedic
 extension into the calcaneocuboid joint occurs in 63%
 vertebral injuries in 10%
 contralateral calcaneus in 10%
 Prognosis
o poor with 40% complication rate
 increased due to mechanism (fall from height), smoking, and early surgery
 lateral soft tissue trauma increases the rate of complication

Anatomy
 Osteology
o articular facets
 superolateral fragment contains the articular facets
 superior articular surface contains three facets that articulate with the talus
 posterior facet is the largest and is the major weight bearing surface

- 396 -
By Dr, AbdulRahman AbdulNasser Lower Extremity | Ankle and Hindfoot
 the flexor hallucis longus tendon runs just inferior to it and can be injured with errant
drills/screws that are too long
 middle facet is anteromedial on sustentaculum tali
 anterior facet is often confluent with middle facet
o sinus tarsi
 between the middle and posterior facets lies the interosseous sulcus (calcaneal groove) that
together with the talar sulcus makes up the sinus tarsi
o sustentaculum tali
 projects medially and supports the neck of talus
 FHL passes beneath it
 deltoid and talocalcaneal ligament connect it to the talus
 contained in the anteromedial fragment, which remains "constant" due to medial
talocalcaneal and interosseous ligaments
o bifurcate ligament
 connects the dorsal aspect of the anterior process to the cuboid and navicular

Classification
 Extra-articular (25%)
o avulsion injury of
 anterior process by bifurcate ligament
 sustentaculum tali
 calcaneal tuberosity (Achilles tendon avulsion)

 Intra-articular (75%)
o Essex-Lopresti classification
‎VI:42 anterior process
 the primary fracture line runs obliquely through the posterior facet
forming two fragments
 the secondary fracture line runs in one of two planes
 the axial plane beneath the facet exiting posteriorly in tongue-type fractures
 when the superolateral fragment and posterior facet remain attached to the tuberosity
posteriorly
 behind the posterior facet in joint depression fractures

- 397 -
OrthoBullets2017 Lower Extremity | Ankle and Hindfoot

anterior process avulsion


Achilles tendon avulsion

tongue-type fractures Joint depression fx

- 398 -
By Dr, AbdulRahman AbdulNasser Lower Extremity | Ankle and Hindfoot
Essex-Lopresti Classification
PRIMARY FRACTURE LINE (A, D)
The posterolateral edge of the talus splits the calcaneus obliquely through the posterior facet. The fracture line exits
anterolaterally at the crucial angle or as far distally as the calcaneocuboid joint. Posteriorly, the fracture moves from
plantar medial to dorsal lateral, producing two main fragments: the sustentacular (anteromedial) and tuberosity
(posterolateral) fragments.
The anteromedial fragment is rarely comminuted and remains attached to the talus by the deltoid and interosseous
talocalcaneal ligaments.
The posterolateral fragment usually displaces superolaterally with variable comminution, resulting in incongruity of the
posterior facet as well as heel shortening and widening.
SECONDARY FRACTURE LINE
With continued compressive forces, there is additional comminution, creating a free lateral piece of posterior facet
separate from the tuberosity fragment.
Tongue fracture: (D, E, and F) a secondary fracture line appears beneath the facet and exits posteriorly through the
tuberosity.
Joint depression fracture (A, B and C) a secondary fracture line exits just behind the posterior facet.
Continued axial force causes the sustentacular fragment to slide medially, causing heel shortening and widening. As this
occurs, the tuberosity fragment will rotate into varus. The posterolateral aspect of the talus will force the free lateral piece
of the posterior facet down into the tuberosity fragment, rotating it as much as 90 degrees. This causes lateral wall
blowout, which may extend as far anteriorly as the calcaneocuboid joint. As the lateral edge of the talus collapses further,
there will be additional comminution of the articular surface.
Source : Koval, Kenneth J.; Zuckerman, Joseph D. Handbook of Fractures, 3rd Edition

o Sanders classification
 based on the number of articular fragments seen on the coronal CT image at the widest point
of the posterior facet
Sanders Classification
Type I • Nondisplaced posterior facet (regardless of number of fracture lines)
Type II • One fracture line in the posterior facet (two fragments)
Type III • Two fracture lines in the posterior facet (three fragments)
Type • Comminuted with more than three fracture lines in the posterior facet (four or more
IV fragments)

- 399 -
OrthoBullets2017 Lower Extremity | Ankle and Hindfoot
Presentation
 Symptoms
o pain
 Physical exam
o inspection
 diffuse tenderness to palpation
 ecchymosis and swelling
 shortened, widened, heel with a varus deformity

Imaging
 Radiographs
o recommended views
 required
 AP, lateral, and oblique foot
 optional
 Broden Harris view
 allows visualization of posterior facet
 useful for evaluation of intraoperative reduction of posterior facet
 with ankle in neutral dorsiflexion take x-rays at 40, 30, 20, and 10
degrees of internal rotation
 Harris view
 visualizes tuberosity fragment widening, shortening, and varus positioning
 place the foot in maximal dorsiflexion and angle the x-ray beam 45 degrees
 AP ankle
 demonstrates lateral wall extrusion causing fibular impingement
 findings
 reduced Bohler angle
 increased angle of Gissane
 calcaneal shortening
 varus tuberosity deformity
o measurement
 Bohler angle (normal is 20-40 degrees)
 measured from lateral foot x-ray
 flattening (decreased angle) represents collapse of the posterior facet
 double-density highlights subtalar incongruity
 angle of Gissane (normal is 130-145 degrees)
 an increase represents collapse of posterior facet
 CT
o indications
 gold standard
o views
 30-degree semicoronal
 demonstrates posterior and middle facet displacement
 axial
 demonstrates calcaneocuboid joint involvement
 sagittal : demonstrates tuberosity displacement

- 400 -
By Dr, AbdulRahman AbdulNasser Lower Extremity | Ankle and Hindfoot
 MRI
o indications
 used only to diagnose calcaneal stress fractures in the presence of normal radiographs and/or
uncertain diagnosis

Treatment
 Nonoperative
o cast immobilization with nonweightbearing for 6 weeks
 indications
 calcaneal stress fractures
o cast immobilization with nonweightbearing for 10 to 12 weeks
 indications
 small extra-articular fracture (<1 cm) with intact Achilles tendon and <2 mm
displacement
 Sanders Type I (nondisplaced)
 anterior process fracture involving <25% of calcaneocuboid joint
 comorbidities that preclude good surgical outcome (smoker, diabetes, PVD)
 techniques
 begin early range of motion exercises once swelling allows
 Operative
o closed reduction with percutaneous pinning
 indications
 minimally displaced tongue-type fxs or those with mild shortening
 large extra-articular fractures (>1 cm)
 early reduction prevents skin sloughing and need for subsequent flap coverage
 techniques
 lag screws from posterior superior tuberosity directed inferior and distal
o ORIF
 indications
 displaced tongue-type fractures

- 401 -
OrthoBullets2017 Lower Extremity | Ankle and Hindfoot
 large extra-articular fractures (>1 cm) with detachment of Achilles tendon and/or > 2 mm
displacement
 urgent if skin is compromised
 Sanders Type II and III
 posterior facet displacement >2 to 3 mm, flattening of Bohler angle, or varus
malalignment of the tuberosity
 anterior process fracture with >25% involvement of calcaneocuboid joint
 displaced sustentaculum fractures
 timing
 wait 10-14 days until swelling and blisters resolve and wrinkle sign present 10-14 days
 no benefit to early surgery due to significant soft tissue swelling
 outcomes
 surgical outcome correlates with the number of intra-articular fragments and the quality
of articular reduction
 factors associated with a poor outcome
 age > 50
 obesity
 manual labor
 workers comp
 smokers
 bilateral calcaneal fractures
 multiple trauma
 vasculopathies
 men do worse with surgery than women
 factors associated with most likely need for a secondary subtalar fusion
 male worker's compensation patient who participates in heavy labor work with an
initial Böhler angle less than 0 degrees
o primary subtalar arthrodesis
 indications
 Sanders Type IV
 techniques
 combined with ORIF to restore height

Surgical Techniques
 ORIF with extensile lateral or medial approach
o goals
 restore congruity of subtalar joint
 restore Bohler angle and calcaneal height
 restore width
 correct varus malalignment
o approach
 extensile lateral L-shaped incision is most popular
 provides access to calcaneocuboid and subtalar joints
 high rate of wound complications
 medial approach can also be used
 full-thickness flap is created to maintain soft tissue integrity
o technique
- 402 -
By Dr, AbdulRahman AbdulNasser Lower Extremity | Ankle and Hindfoot
 place a pin in the tuberosity to assist the reduction
 provisional fixation with Kirschner wires
 hold reduction with low profile implants
 bone grafting provides no added benefit
o postoperative care
 bulky posterior U splint
 early supervised subtalar range of motion exercises
 nonweightbearing for 10 weeks
 ORIF with sinus tarsi approach and Essex-Lopresti maneuver
o technique
 manipulate the heel to increase the calcaneal varus deformity
 plantarflex the forefoot
 manipulate the heel to correct the varus deformity with a valgus reduction
 stabilize the reduction with percutaneous K-wires or open fixation as described above

Complications
 Wound complications (10-25%)
o increased risk in smokers, diabetics, and open injuries
 Subtalar arthritis
o increased with nonoperative management
 Lateral impingement with peroneal irritation
 Damaged FHL
o at risk with placement of lateral to medial screws, especially at level of sustentaculum tali
(constant fragment)
 Compartment syndrome (10%)
o results in claw toes
 Malunion
o introduction
 loss of height, widening, and lateral impingement
o physical exam
 limited ankle dorsiflexion
 due to dorsiflexed talus with talar declination angle <20
o classification (see below)
o treatment
 distraction bone block subtalar arthrodesis
 indications
 chronic pain from subtalar joint
 incongruous subtalar joint/post-traumatic DJD
 loss of calcaneal height
 mechanical block to ankle dorsiflexion
 results from posterior talar collapse into the posterior calcaneus
 technique
 goal is to correct
 hindfoot height
 ankle impingement
 subfibular impingement
 subtalar arthritis
- 403 -
OrthoBullets2017 Lower Extremity | Ankle and Hindfoot

Malunion CT Classification & Treatment


Type I • Lateral exostosis with no subtalar arthritis
• Treat with lateral wall resection
Type II • Lateral exostosis with subtalar arthritis
• Treat with lateral wall resection and subtalar fusion
• Lateral exostosis, subtalar arthritis, and varus malunion
Type III
• Treat with lateral wall resection, subtalar fusion, and +/- valgus osteotomy (controversial)

Collected By : Dr AbdulRahman
AbdulNasser
June 2017

- 404 -
By Dr, AbdulRahman AbdulNasser Foot & Ankle Trauma | Ankle and Hindfoot

ORTHO BULLETS

VII. Foot & Ankle


Trauma

- 405 -
OrthoBullets2017 Foot & Ankle Trauma | Ankle Sprains

A. Ankle Sprains

1. High Ankle Sprain & Syndesmosis Injury


Introduction
 Also known as a "high ankle sprain"
 Epidemiology
o incidence
 0.5% of all ankle sprains without fracture
 13% of all ankle fractures
 Pathophysiology
o mechanism of injury
 most commonly associated with external rotation injuries
o pathoanatomy
 external rotation forces the talus to rotate laterally and push the fibula away from tibia
 may lead to
 increased compressive stresses seen by the tibia
 increased likelihood of lateral subluxation of the distal fibula
 incongruence of the ankle joint articulation
 Associated injuries
o osteochondral defects (15% to 25%)
o peroneal tendon injuries (up to 25%)
o fractures
 ankle
 Weber C
 Weber B
 other ‎ II:1 Weber Type B
V Type C
 5th metatarsal base
 anterior process of calcaneus
 lateral or posterior process of talus
o deltoid ligament injury
o loose bodies
 Prognosis
o missed injuries may result in end-stage ankle arthritis
o excellent functional outcomes if syndesmosis is anatomically reduced

Anatomy
 See complete ligament of ankle
 Ligaments
o distal tibiofibular syndesmosis includes
 anterior-inferior tibiofibular ligaments (AITFL)
 originates from anterolateral tubercle of tibia (Chaput's)
 inserts on anterior tubercle of fibula (Wagstaffe's)
 posterior-inferior tibiofibular ligament (PITFL)
 originates from posterior tubercle of tibia (Volkmann's)
 inserts on posterior part of lateral malleolus

- 406 -
By Dr, AbdulRahman AbdulNasser Foot & Ankle Trauma | Ankle Sprains
 strongest component of syndesmosis
 interosseous membrane
 interosseous ligament (IOL)
 distal continuation of the interosseous membrane
 main restraint to proximal migration of the talus
 inferior transverse ligament (ITL)
 Syndesmosis Biomechanics
o function
 maintains integrity between tibia and fibula
 resists axial, rotational, and translational forces
o normal gait
 syndesmosis widens 1mm during gait
o deltoid ligament
 indirectly stabilizes the medial ankle mortise

Presentation
 Symptoms
o anterolateral ankle pain proximal to AITFL
o may have medial sided ankle tenderness/swelling
o difficulty bearing weight
‎VII:2 Syndesmotic ligaments
 lateral ankle sprains are often able to bear weight
 Physical exam
o palpation
 syndesmosis tenderness
 single best predictor for return to play
o provocative tests
 squeeze test (Hopkin's)
 compression of tibia and fibula at midcalf level causes
pain at syndesmosis
 external rotation stress test ‎VII:3 squeeze test
 pain over syndesmosis is elicited with external rotation/dorsiflexion of
the foot with knee and hip flexed to 90 degrees
 Cotton
 widening of the syndesmosis with lateral pull on the fibula
 fibular translation
 anterior and posterior drawer force to the fibula with the tibia stabilized causes increased
translation of the fibula and pain
Imaging
 Radiographs
o recommended views
 AP, lateral, mortise view of ankle
 AP, lateral of entire tibia
 may show fracture of proximal fibula
o optional views
 external rotation stress radiograph
 gravity stress view

- 407 -
OrthoBullets2017 Foot & Ankle Trauma | Ankle Sprains
 will help determine competence of deltoid ligament
 contralateral ankle radiographs
 may help clarify syndesmosis widening versus normal anatomic variant
o findings
 decreased tibiofibular overlap
 normal >6 mm on AP view
 normal >1 mm on mortise view
 increased medial clear space
 normal less than or equal to 4 mm
 increased tibiofibular clear space
 normal <6 mm on both AP and mortise views
 CT
o indications
 clinical suspicion of syndesmotic injury with normal radiographs
 useful post-operatively to assess reduction of syndesmosis after fixation
o sensitivity and specificity
 more sensitive than radiographs for detecting minor degrees of syndesmotic injury
 MRI
o indications
 clinical suspicion of syndesmotic injury with normal radiographs
o sensitivity and specificity
 highly sensitive and specific for detecting syndesmotic injury

external rotation
stress gravity stress view
Mortis &AP radiograph of entire leg radiograph CT

tibiofibular overlap Medial clear space MRI

Treatment of High Ankle Sprains


 Nonoperative

- 408 -
By Dr, AbdulRahman AbdulNasser Foot & Ankle Trauma | Ankle Sprains
o non-weight-bearing CAM boot or cast for 2 to 3 weeks
 indications
 syndesmotic sprain without diastasis or ankle instability
 technique
 delayed weight-bearing until pain free
 physical therapy program using a brace that limits external rotation
 outcomes
 typically display a notoriously prolonged and highly variable recovery period
 recovery may extend to twice that of standard ankle sprain
 Operative
o syndesmosis screw fixation
 indications
 syndesmotic sprain (without fracture) with instability on stress radiographs
 syndesmotic sprain refractory to conservative treatment
 syndesmotic injury with associated fracture that remains unstable after fixation of fracture
 outcomes
 excellent functional outcomes if syndesmosis is accurately reduced
 requires removal
o syndesmosis fixation with suture button
 indications
 same as for screw fixation
 technique
 fiberwire suture with two buttons tensioned around the syndesmosis
 may be performed in addition to a screw ‎ II:4 suture button
V
 outcomes
 early results promising with some showing earlier return to activity when compared to
screw fixation
 does not require removal

Surgical Techniques
 Syndesmotic screw fixation
o technique
 two 3.5 or 4.5 mm syndesmotic screws through 3 or 4 cortices
placed 2-5 cm above the plafond
 screw material
 no difference between stainless-steel and titanium screws
 bioabsorbable screws with similar outcomes
 number of cortices
 no difference between 3 or 4 cortices
 number of screws
 fixation with two screws is preferable
 position of foot during fixation
 a recent study challenges the principle of holding the ankle in maximal dorsiflexion to
avoid overtightening
o postoperative
 typically non-weight-bearing for 6-12 weeks
 may prolong if screw breakage is a concern
- 409 -
OrthoBullets2017 Foot & Ankle Trauma | Ankle Sprains
Complications
 Posttraumatic tibiofibular synostosis
o incidence
 ~10% after Weber C ankle fractures
o treatment
 surgical excision
 reserved for persistent pain that fails to respond to
nonsurgical management
 ossification must be "cold" on bone scintigraphy prior to
removal

2. Low Ankle Sprain


Introduction
 Ankle sprains can be conceptually
broken down into
o high ankle sprain
 syndesmosis injury
 1-10% of all ankle sprains
o low ankle sprain (this topic)
 ATFL and CFL injury
 >90% of all ankle sprains
 Epidemiology
o ankle sprains are the most common
reason for missed athletic
participation
o most common injury in dancers
 Associated injuries include
o osteochondral defects
o peroneal tendon injuries
o subtle cavovarus foot
o deltoid ligament injury (isolated
deltoid ligament injuries are very rare)
o fractures
 5th metatarsal base
 anterior process of calcaneus
 lateral or posterior process of the talus

Anatomy
 Ligamentous anatomy of the ankle
 ATFL
o most commonly involved ligament in low ankle sprains
o mechanism is plantar flexion and inversion
o physical exam shows drawer laxity in plantar flexion
 CFL
o 2nd most common ligament injury in lateral ankle sprains
o mechanism is dorsiflexion and inversion
- 410 -
By Dr, AbdulRahman AbdulNasser Foot & Ankle Trauma | Ankle Sprains
o physical exam shows drawer laxity in dorsiflexion
o subtalarinstability can be difficult to differentiate from posterior ankle instability because the
CFL contributes to both
 PTFL : less commonly involved

Classification
Classification of Low Ankle Sprains
Ligament disruption Ecchymosis and swelling Pain with weight bearing
Grade I none minimal normal
Grade II stretch without tear moderate mild
Grade III complete tear severe severe

Presentation
 Symptoms
o pain with weight bearing
o recurrent instability
o catching or popping sensation may occur following recurrent sprains
 Physical exam
o focal tenderness and swelling over involved ligament(s)
o anterior drawer test
 possible laxity with anterior drawer and eversion/inversion stress testing

Imaging
 Radiographs
o indications for radiographs with an ankle injury include (Ottawa ankle rules)
 inability to bear weight
 medial or lateral malleolus point tenderness
 5MT base tenderness
 navicular tenderness
o radiographic views to obtain
 standard ankle series (weight bearing)
 AP
 lateral
 mortise
‎ II:5 varus stress view
V
 ER rotation stress view
 useful to diagnosis syndesmosis injury in high ankle sprain
 look for asymmetric mortise widening
 medial clear space widening > 4mm
 tibiofibular clear space widening of 6 mm
 varus stress view
 used to diagnose injury to ATFL or CFL
 measures ankle instability by looking at talar tilt and anterior talar translation
 MRI
o indications : consider MRI if pain persists for 8 weeks following sprain
o useful to evaluate
 peroneal tendon pathology
 osteochondral injury
- 411 -
OrthoBullets2017 Foot & Ankle Trauma | Ankle Sprains
Treatment
 Nonoperative
o RICE, elastic wrap to minimize swelling, followed by therapy
 indications : Grade I, II, and III injuries
 technique
 may require short period (approx. 1 week) of weight-bearing immobilization in a walking
boot or walking cast, but early mobilization facilitates a better recovery
 therapy
 once swelling and pain have subsided and patient has full range of motion begin
neuromuscular training with a focus on peroneal muscles strength and proprioception
training
 a functional brace that controls inversion and eversion is typically used during the
strengthening period and used as prophylactic treatment during high risk activities
thereafter
 early functional rehabilitation allows for quickest return to physical activity
 Operative
o anatomic reconstruction vs. tendon transfer with tenodesis
 indications
 Grade I-III that continue to have pain and instability despite extensive nonoperative
management
 Grade I-III with a bony avulsion
 technique (see below)
o arthroscopy
 indications
 recurrent ankle sprains and chronic pain caused by impingement lesions
 anteriorinferior tibiofibular ligament impingement
 posteromedial impingement lesion of ankle
 procedure : debride impinging tissue

Surgical Techniques
 Gould modification of Brostrom anatomic reconstruction
o procedure
 an anatomic shortening and reinsertion of the ATFL and CFL
 reinforced with inferior extensor retinaculum and distal fibular periosteum
o results
 good to excellent results in 90%
 consider arthroscopic evaluation prior to reconstruction for intra-articular evaluation
 Tendon transfer and tenodesis (Watson-Jones, Chrisman-Snook, Colville, Evans)
o procedure
 a nonanatomic reconstruction using a tendon transfer
o technique
 any malalignment must be corrected to achieve success during a lateral ligament
reconstruction
 Coleman block testing used to distinguish between fixed and flexible hindfoot varus
o results
 subtalar stiffness is a common complication

- 412 -
By Dr, AbdulRahman AbdulNasser Foot & Ankle Trauma | Ankle Sprains
Rehabilitation
 Return to play
o depends on, grade of sprain, syndesmosis injury, associated injuries, and compliance with rehab

Classification Time to RTP


Grade I 1-2 weeks
Grade II 1-2 weeks
Grade III few weeks
High ankle (immobilization) several weeks
High ankle (screw fixation) season
 Prevention
o prevention techniques in athletes with prior sprains includes
 semirigid orthosis
 evertor muscle (peroneals) strengthening
 proprioception exercises
 season long prevention program

Complications
 Pain and instability
o up to 50% continue to experience symptoms following and acute ankle sprain
o most common cause of chronic pain is a missed injury, including
 injury to the anterior process of calcaneus
 injury to the lateral or posterior process of the talus
 injury to the base of the 5th metatarsal
 osteochondral lesion
 injuries to the peroneal tendons
 injury to the syndesmosis
 tarsal coalition
 impingement syndromes

Collected By : Dr AbdulRahman
AbdulNasser
June 2017

- 413 -
OrthoBullets2017 Foot & Ankle Trauma | Mid & Forefoot Trauma

B. Mid & Forefoot Trauma

1. Lisfranc Injury (Tarsometatarsal fracture-dislocation)


Introduction
 A condition characterized by disruption between the articulation of the medial cuneiform and base of
the second metatarsal
o unifying factor is disruption of the TMT joint complex
o injuries can range from mild sprains to severe dislocations
o may take form of purely ligamentous injuries or fracture-dislocations
o ligamentous vs. bony injury pattern has treatment implications
 Epidemiology
o incidence
 account for 0.2% of all fractures
o demographics
 more common in the third decade
 more common in males
 Mechanism & Pathoanatomy
o causes include MVAs, falls from height, and athletic injuries
o mechanism is usually caused by indirect rotational forces and axial load through hyperplantar
flexed forefoot
 hyperflexion/compression/abduction moment exerted on forefoot and transmitted to the TMT
articulation
 metatarsals displaced in dorsal/lateral direction
 Associated conditions
o proximal metatarsal fractures or tarsal fractures
 Lisfranc equivalent injuries can present in the form of contiguous proximal metatarsal
fractures or tarsal fractures
 Prognosis
o missed injuries can result in progressive foot deformity, chronic pain and dysfunction
 tarsometatarsal fracture-dislocations are easily missed and diagnosis is critical

Anatomy
 Osteology
o Lisfranc joint complex consists of three articulations including
 tarsometatarsal articulation
 intermetatarsal articulation
 intertarsal articulations
 Ligaments
o Lisfranc ligament
 critical to stabilizing the second metatarsal and maintenance
of the midfoot arch
 An interosseous ligament that goes from medial cuneiform
to base of 2nd metatarsal on plantar surface
 Lisfranc ligament tightens with pronation and abduction of forefoot
o plantar tarsometatarsal ligaments

- 414 -
By Dr, AbdulRahman AbdulNasser Foot & Ankle Trauma | Mid & Forefoot Trauma
 injury of the plantar ligament between the medial cuneiform and the second and third
metatarsals along with the Lisfranc ligament is necessary to give transverse instability.
o dorsal tarsometatarsal ligaments
 dorsal ligaments are weaker and therefore bony displacement with injury is often dorsal
o intermetatarsal ligaments
 between second-fifth metatarsal bases
 no direct ligamentous attachment between first and second metatarsal
 Biomechanics
o Lisfranc joint complex is inherently stable with little motion due to
 stable osseous architecture
 second metatarsal fits in mortise created by medial cuneiform and recessed middle
cuneiform, "keystone configuration"
 ligamentous restraints
 see individual ligaments above
 Columns of the midfoot
o medial column
 includes first tarsometatarsal joint
o middle column
 includes second and third tarsometatarsal joints
o lateral column
 includes fourth and fifth tarsometatarsal joints (most mobile)

Classification
 Multiple classification schemes described
o none proven useful for determining treatment and prognosis

Classification schemes for Lisfranc injuries guide the clinician


in defining the extent and pattern of injury, although they are of
little prognostic value.
Ouenu and Kuss (Fig. 41.7)
This classification is based on commonly observed patterns of
injury. ‎VII:7 Ouenu and Kuss
 Homolateral: All five metatarsals displaced in the same
direction
 Isolated: One or two metatarsals displaced from the others
 Divergent: Displacement of the metatarsals in both the
sagittal and coronal planes

Myerson (Fig. 41.8)


This is based on commonly observed patterns of injury with
regard to treatment.
 Total incongruity: Lateral and dorsoplantar
 Partial incongruity: Medial and lateral
 Divergent: Partial and total
Source : Koval, Kenneth J.; Zuckerman, Joseph D. : Handbook of
Fractures, 3rd Edition
‎VII:6 Myerson

- 415 -
OrthoBullets2017 Foot & Ankle Trauma | Mid & Forefoot Trauma
Physical Exam
 Symptoms
o severe pain
o inability to bear weight
 Physical exam
o inspection & palpation
 medial plantar bruising
 swelling throughout midfoot
 tenderness over tarsometatarsal joint
o motion & stability
 instability test
 grasp metatarsal heads and apply dorsal force to forefoot while other hand palpates the
TMT joints
 dorsal subluxation suggests instability
 if first and second metatarsals can be displaced medially and laterally, global
instability is present and surgery is required
 when plantar ligaments are intact, dorsal subluxation does not occur with stress exam and
injury may be treated nonoperatively
o provocative tests
 may reproduce pain with pronation and abduction of forefoot
o compartment syndrome
 always check for compartment syndrome and take compartment pressures if high suspicion

Imaging
 Radiographs
o recommended views
 AP, lateral, obliques
 stress radiograph
 may be helpful to show instability when non-weight bearing radiographs are normal and
there is high suspicion
 weight-bearing radiographs with comparison view
 may be necessary to confirm diagnosis
o findings
 five critical radiographic signs that indicate presence of midfoot instability
 disruption of the continuity of a line drawn from the medial base of the second metatarsal
to the medial side of the middle cuneiform
 widening of the interval between the first and second ray
 medial side of the base of the fourth metatarsal does not line up with medial side of
cuboid on oblique view
 metatarsal base dorsal subluxation on lateral view
 disruption of the medial column line (line tangential to the medial aspect of the navicular
and the medial cuneiform)
 lateral
 non weight-bearing radiographs may show dorsal displacement of the proximal base of
the first or second metatarsal

- 416 -
By Dr, AbdulRahman AbdulNasser Foot & Ankle Trauma | Mid & Forefoot Trauma
 AP
malalignment of medial margin of the second metatarsal base and the medial edge of the
middle cuneiform diagnostic of Lisfranc injury
 may see bony fragment (fleck sign) in first intermetatarsal space
 represents avulsion of Lisfranc ligament from base of 2nd metatarsal
 diagnostic of Lisfranc injury
 oblique
 malalignment of fourth metatarsal and cuboid
 CT scan
o useful for diagnosis and preoperative planning
 MRI
o can be used to confirm presence of purely ligamentous injury

AP AP Lateral CT

critical radiographic signs that indicate presence of midfoot instability

Treatment
 Nonoperative
o cast immobilization for 8 weeks
 indications
 no displacement on weight-bearing and stress radiographs and no evidence of bony injury
on CT (usually dorsal sprains)
 certain nonoperative candidates
- 417 -
OrthoBullets2017 Foot & Ankle Trauma | Mid & Forefoot Trauma
 nonambulatory patients
 presence of serious vascular disease
 severe peripheral neuropathy
 instability in only the transverse plane
 Operative
o open reduction and rigid internal fixation
 indications
 any evidence of instability (> 2mm shift)
 favored in bony fracture dislocations as opposed to purely ligamentous injuries
 outcomes
 anatomic reduction required for a good result
o primary arthrodesis of the first, second and third tarsometatarsal joints
 indications
 purely ligamentous arch injuries

 delayed treatment
 chronic deformity
 outcomes
 level 1 evidence demonstrates equivalent functional outcomes and decreased rate of
hardware removal or revision surgery compared to primary ORIF
 primary arthodesis is an alternative to ORIF in patients with any evidence of instability

with possible benefits


 medial column tarsometatarsal fusion shown to be superior to combined medial and

lateral column tarsometatarsal arthrodesis


o midfoot arthrodesis
 indications
 destabilization of the midfoot's architecture with progressive arch collapse and forefoot
abduction
 chronic Lisfranc injuries that have led to advanced midfoot arthrosis and have failed
conservative therapy
Technique
 General Surgical Considerations
o equipment
 fluoroscopy and radiolucent table
 small fragment and mini fragment sets (2.0mm-4.0mm screws)
 reduction clamps, k-wires, dental pick, Homan retractors
 small battery powered drill
o timing of surgery
 emergently only if
 the presence of compartment syndrome
 open injury
 irreducible dislocations
 delay operative treatment until soft tissue swelling subsides (up to 2-3 weeks)
 if grossly unstable and in presence of significant soft tissue swelling consider early external
fixation
- 418 -
By Dr, AbdulRahman AbdulNasser Foot & Ankle Trauma | Mid & Forefoot Trauma
 Open reduction and rigid internal fixation
o approach
 single or dual longitudinal incisions can be used based on injury pattern and surgeon
preference
 longitudinal incision made in the web space between first and second rays
 first TMT joint is exposed between the long and short hallux-extensor tendons
o reduction & fixation
 reduce intercuneiform instability first
 fix first through third TMT joints with transarticular screws
 screw fixation is more stable than K-wire fixation
o postoperative care
 early midfoot ROM, protected weight bearing, and hardware removal (k-wires in 6-8 weeks,
screws in 3-6 months
 gradually advance to full weight bearing at 8-10 weeks
 if patient is asymptomatic and screws transfix only first through third TMT joints, they may
be left in place
 preclude return to vigorous athletic activities for 9 to 12 months
 Primary arthrodesis of the first, second and third tarsometatarsal joints
o arthrodesis & fixation
 expose TMT joints and denude all joint surfaces of cartilage
 use cortical screws to fuse joints
o postoperative care
 apply cast or splint for 6 weeks
 progress weight bearing between 6 and 12 weeks in removable boot
 full weight bearing in standard shoes by 12 weeks post-op
 Midfoot arthrodesis
o arthrodesis & fixation
 expose TMT joints and midfoot and remove cartilage from first, second, and third TMT
joints
 add bone graft
 reduce the deformity using windlass mechanism
 variety of definitive fixation constructs exist
o postoperative care
 touch down weight bearing in a cast for 6 weeks
 begin weight bearing at 10 weeks if evidence of healing is noted on radiographs

Complications
 Posttraumatic arthritis
o most common complication
o may cause altered gait and long term disability
o treat advanced midfoot arthrosis with midfoot arthrodesis
 Nonunion
o uncommon
o revision surgery indicated unless patient is elderly and low demand

- 419 -
OrthoBullets2017 Foot & Ankle Trauma | Mid & Forefoot Trauma

2. 5th Metatarsal Base Fracture


Introduction
 Epidemiology
o incidence
 fairly common injury
 Pathophysiology
o mechanism
 depends on zone of injury
 zone 1: hindfoot inversion
 zone 2: forefoot adduction
 zone 3: repetitive microtrauma
 Associated injuries
o midfoot (Lisfranc injury)
o lateral ankle ligamentous complex
o rule out associated foot deformities
 cavus foot or varus hindfoot

Anatomy
 Osteology and Insertions
o divided into tubercle (tuberosity), base, shaft, head and neck
o peroneus brevis and lateral band of plantar fascia insert on base
o peroneus tertius inserts on dorsal metadiaphysis
 Blood supply
o blood supply provided by metaphyseal vessels and diaphyseal nutrient artery
o Zone 2 (Jones fx) represents a vascular watershed area, making these fracture prone to nonunion

Classification

Classification
Class Description
Zone 1  Proximal tubercle (rarely enters 5th tarsometatarsal joint)
(pseudo Jones  Due to long plantar ligament, lateral band of the plantar fascia, or contraction of the
fx) peroneus brevis
 Nonunions uncommon
Zone 2  Metaphyseal-diaphyseal junction
(Jones fx)  Involves the 4th-5th metatarsal articulation
 Vascular watershed area
 Acute injury
 Increased risk of nonunion
Zone 3  Proximal diaphyseal fracture
 Distal to the 4th-5th metatarsal articulation
 Stress fracture in athletes
 Associated with cavovarus foot deformities or sensory neuropathies
 Increased risk of nonunion

- 420 -
By Dr, AbdulRahman AbdulNasser Foot & Ankle Trauma | Mid & Forefoot Trauma
Presentation
 Symptoms
o pain over lateral border of forefoot, especially with
weight bearing
o look for antecedent pain in setting of stress fracture
 Physical Exam
o manual palpation of area of concern
o resisted foot eversion

Imaging
 Radiographs : AP, lateral and oblique foot images
 CT
o not routinely obtained
o consider in setting of delayed healing or nonunion
 MRI : not routinely obtained
o consider in setting of delayed healing or nonunion

Treatment
 Nonoperative
o protected weight bearing in stiff soled shoe, boot or cast
 indications
 Zone 1
 technique
 advance as tolerated by pain
 early return to work but symptoms may persist for up to 6 months
o non weight bearing short leg cast for 6-8 weeks
 indications
 Zone 2 (Jones fx) in recreational athlete
 Zone 3
 technique : advance with signs of radiographic healing
 Operative
 intramedullary screw fixation : indications
 zone 2 (Jones fx) in elite or competitive athletes
 minimizes possibility of nonunion or prolonged restriction from activity
 zone 3 fx with sclerosis/nonunion or in athletic individual

Complications
 Nonunion
o increased risk in Zone 2 (Jones fx) and Zone 3 due to vascular supply
o smaller diameter screws (<4.5mm) associated with delayed union or nonunion
 Failure of fixation : higher failure rate in
 elite athletes
 return to sports prior to radiographic union
 fracture distraction or malreduction due to screw length
 screws that are too long will straighten the curved metatarsal shaft or perforate the medial
cortex

- 421 -
OrthoBullets2017 Foot & Ankle Trauma | Mid & Forefoot Trauma

3. Metatarsal Fractures
Introduction
 Metatarsal fractures are among the most common injuries of the foot
o goals of treatment include:
 maintenance of transverse and longitudinal arch of forefoot
 restore alignment to allow for normal force transmission
across metatarsal heads
 Epidemiology
o 5th metatarsal most commonly fractured in adults
o 1st metatarsal most commonly fractured in children less than 4
years old
o peak incidence between 2nd and 5th decade of life
o 3rd metatarsal fractures rarely occur in isolation
 68% associated with fracture of 2nd or 4th metatarsal
 Mechanism
o direct crush injury
 may have significant associated soft tissue injury
o indirect mechanism (most common)
 occurs with forefoot fixed and hindfoot or leg rotating
 Associated conditions
o Lisfranc injury
 Lisfranc equivalent injuries seen with multiple proximal metatarsal fractures
o stress fracture
 consider metabolic evaluation for fragility fracture
 look for associated foot deformity
 seen at base of 2nd metatarsal in ballet dancers
 may have history of amenorrhea
 Prognosis
o majority of isolated metatarsal fractures heal with conservative management
o malunion may lead to transfer metatarsalgia

Anatomy
 Osteology
o shape and function similar to metacarpals of the hand
o first metatarsal has plantar crista that articulates with sesamoids
 widest and shortest
 bears 30-50% of weight during gait
o second metatarsal is longest
 most common location of stress fracture
 Muscles
o muscular balance between extrinsic and intrinsic muscles
o extrinsics include
 Extensor digitorum longus (EDL)
 Flexor digitorum longus (FDL)

- 422 -
By Dr, AbdulRahman AbdulNasser Foot & Ankle Trauma | Mid & Forefoot Trauma
o intrinsics include
 Interossei
 Lumbricals
o see Layers of the Plantar Foot
 Ligaments
o Metatarsals have dense proximal and distal ligamentous attachments
o 2nd-5th metatarsal have distal intermetatarsal ligaments that maintain length and alignment with
isolated fractures
 implicated in formation of interdigital (Morton's) neuromas
 multiple metatarsal fractures lose the stability of intermetatarsal ligaments leading to
increased displacement
 Blood supply
o dorsal and plantar metatarsal arteries
 Biomechanics
o see Foot and Ankle Biomechanics

Classification
 Classification of metatarsal fractures is descriptive and should include
o location
o fracture pattern
o displacement
o angulation
o articular involvement

Presentation
 History
o look for antecedent pain when suspicious for stress fracture
 Symptoms
o pain, inability to bear weight
 Physical Exam
o inspection
 foot alignment (neutral, cavovarus, planovalgus)
 focal areas or diffuse areas of tenderness
 careful soft tissue evaluation with crush or high-energy injuries
o motion
 evaluate for overlapping or malrotation with motion
o neurovascular
 semmes weinstein monofilament testing if suspicious for peripheral neuropathy

Imaging
 Radiographs
o recommended views
 required
 AP, lateral and oblique views of the foot
 optional
 contralateral foot views
 stress or weight bearing radiographs

- 423 -
OrthoBullets2017 Foot & Ankle Trauma | Mid & Forefoot Trauma
 CT
o not
routinely obtained
o may be of use in periarticular injuries or to rule out Lisfranc injury
 MRI or bone scan
o useful in detection of occult or stress fractures

Treatment
 Nonoperative
o stiff soled shoe or walking boot with weight bearing as tolerated
 indications
 first metatarsal
 non-displaced fractures
 second through fourth (central) metatarsals
 isolated fractures
 non-displaced or minimally displaced fractures
 stress fractures
 second metatarsal most common
 look for metabolic bone disease
 evaluate for cavovarus foot with recurrent stress fractures
 Operative
o percutaneous vs open reduction and fixation
 indications
 open fractures
 first metatarsal
 any displacement
 no intermetatarsal ligament support
 30-50% of weight bearing with gait
 central metatarsals
 sagittal plane deformity more than 10 degrees
 >4mm translation
 multiple fractures
 techniques
 antegrade or retrograde pinning
 lag screws or mini fragment plates in length unstable fracture patterns
 maintain proper length to minimize risk of transfer metatarsalgia
 outcomes
 limited information available in literature

Complications
 Malunion
o may lead to transfer metatarsalgia or plantar keratosis
o treat with osteotomy to correct deformity

- 424 -
By Dr, AbdulRahman AbdulNasser Foot & Ankle Trauma | Mid & Forefoot Trauma

4. Tarsal Navicular Fractures


Introduction
 Navicular fractures can be
o traumatic
 navicular avulsion fractures : mechanism is plantar flexion
 navicular tuberosity fractures
 mechanism is eversion with simultaneous contraction of PTT
 may represent an acute widening/diastasis of an accessory navicular
 navicular body fractures : mechanism is axial loading
o stress fracture
 mechanism of injury is usually due to chronic overuse
 often seen in athletes running on hard surfaces
 also common in baseball players
 considered a high risk injury due to risk of AVN
 most common complications include delayed union and non-union

Anatomy
 Articulations
o navicular bone articulates with
 cuneiforms
 cuboid
 calcaneus
 talus
 Biomechanics
o navicular bone and its articulations play an important role in inversion and eversion
biomechanics and motion
Classification
Sangeorzan Classification of Navicular Body Fractures
(based on plane of fracture and degree of comminution)
Transverse fracture of dorsal fragment that involves < 50% of bone.
Type I
No associated deformity
Oblique fracture, usually from dorsal-lateral to plantar-medial.
Type II
May have forefoot aDDuction deformity.
Central or lateral comminution.
Type IIII
ABDuction deformity.

- 425 -
OrthoBullets2017 Foot & Ankle Trauma | Mid & Forefoot Trauma

Type I Type III


Presentation
 Symptoms
o vague midfoot pain and swelling
 Physical exam
o midfoot swelling
o tenderness to palpation of midfoot
o usually full ROM of ankle and subtalar joint

Imaging
 Radiographs ‎VII:8 CT
o may be difficult to see and are often missed
o oblique 45 degree radiograph
 best to visualize tuberosity fractures
 CT
o more sensitive to identify fracture than radiographs
 MRI
o will show signal intensity on T2 image due to inflammation

Treatment of Stress Fractures


 Nonoperative
o cast immobilization with no weight bearing ‎VII:9 MRI
 indications
 any navicular stress fracture, regardless of type, can be initially treated with cast
immobilization and nonweight bearing for 6-8 weeks with high rates of success
 Operative
o open reduction and internal fixation
 indications
 high level athletes
 nonunion of navicular stress fracture
 failure of cast immobilization and non weight bearing

- 426 -
By Dr, AbdulRahman AbdulNasser Foot & Ankle Trauma | Tendon Injuries
Treatment of Traumatic Fractures
 Nonoperative
o cast immobilization with no weight bearing
 indications
 acute avulsion fractures
 most tuberosity fractures
 minimally displaced Type I and II navicular body fractures
 Operative
o fragment excision
 indications
 avulsion fractures that failed to improve with nonoperative modalities
 tuberosity fractures that went on to symptomatic nonunion
o open reduction and internal fixation
 indications
 avulsion fractures involving > 25% of articular surface
 tuberosity fractures with > 5mm diastasis or large intra-articular fragment
 displaced or intra-articular Type I and II navicular body fractures
 technique
 medial approach
 used for Type I and II navicular body fractures
o ORIF followed by external fixation VS. primary fusion
 indications
 Type III navicular body fractures
 technique
 must maintain lateral column length

C. Tendon Injuries

1. Achilles Tendon Rupture


Introduction
 Acute rupture of the achilles tendon
o often misdiagnosed as an ankle sprain
o may be missed in up to 25%
 Epidemiology
o incidence : 18:100,000 per year
o demographics
 more common in men
 most common in ages 30-40
o risk factors
 episodic athletes, "weekend warrior"
 flouroquinolone antibiotics
 steroid injections
 Mechanism
o usually traumatic injury during a sporting event
o may occur with

- 427 -
OrthoBullets2017 Foot & Ankle Trauma | Tendon Injuries
 sudden forced plantar flexion
 violent dorsiflexion in a plantar flexed foot
 Pathoanatomy
o rupture usually occurs 4-6 cm above the calcaneal insertion in hypovascular region

Anatomy
 Achilles tendon
o largest tendon in body
o formed by the confluence of
 soleus muscle tendon
 medial and lateral gastrocnemius tendons
o blood supply from posterior tibial artery

Presentation
 History
o patient usually reports a "pop"
 Symptoms
o weakness and difficulty walking
o pain in heel
 Physical exam
o inspection
 increased resting ankle dorsiflexion in prone position with knees bent
 calf atrophy may be apparent in chronic cases
o palpation
 palpable gap
o motion
 weakness to ankle plantar flexion
 increased passive dorsiflexion
o provocative test
 Thompson test
 lack of plantar flexion when calf is squeezed

Imaging
 Radiographs
o indications
‎VII:10 Partial tear U/S
 used to rule out other pathology
 Ultrasound
o indications
 may be useful to determine complete vs. partial ruptures
 MRI
o indications
 equivocal physical exam findings
 chronic ruptures
o findings ‎VII:11 Complete tear U/S
 will show acute rupture with retracted tendon edges

Treatment
 Nonoperative
- 428 -
By Dr, AbdulRahman AbdulNasser Foot & Ankle Trauma | Tendon Injuries
o functional bracing/casting in resting equinus
 indications
 acute injuries with surgeon or patient preference for non-
operative management
 sedentary patient
 medically frail patients
 outcomes
 decreased plantar flexion strength compared to operative
management
 new studies show that this may not be true
 increased risk of re-rupture compared to operative management ‎VII:12 MRI showing rupture
 new studies show that this may not be significant achilles tendon

 fewer complications compared to operative treatment


 Operative
o open end-to-end achilles tendon repair
 indications
 acute ruptures (approximately <6 weeks)
 outcomes
 decreased rate of re-rupture compared to non-operative management
 new Level 1 evidence has suggested no difference in re-rupture rates
 increased plantar flexion strength compared to non-operative management
 new Level 1 evidence has suggested no significant difference in plantar flexion
strength
o percutaneous Achilles tendon repair
 indications
 concerns over cosmesis of traditional scar
 outcomes
 higher risk of sural nerve damage
 lesser risk of wound complications/infection compared with open repair
o reconstruction with VY advancement
 indications
 chronic ruptures with defect < 3cm
o flexor hallucis longus transfer +/- VY advancement of gastrocnemius
 indications
 chronic ruptures with defect > 3cm
 requires a functioning tibial nerve

Surgical Techniques
 Functional bracing/casting in resting equinus
o technique
 cast/brace in 20 degrees of plantar flexion
 early functional rehab for those treated without a cast
 End-to-end achilles tendon repair
o approach
 make incision just medial to achilles tendon to avoid sural nerve

- 429 -
OrthoBullets2017 Foot & Ankle Trauma | Tendon Injuries
o technique
 incise paratenon
 expose tendon edges
 repair with heavy non-absorbable suture
o postoperative care
 immobilize in 20° of plantar flexion to decrease
tension on skin and protect tendon repair for 4-6
weeks
 Percutaneous achilles tendon repair ‎VII:13 VY advancement
o technique
 Reconstruction with VY advancement
o technique
 make V cut with apex at musculotendinous junction with limbs divergent to exit the tendon
 V is incised through only the superficial tendinous portion leaving the muscle fibers intact
 Flexor hallucis longus transfer ± VY advancement of gastrocnemius
o technique
 excise degenerative tendon edges
 release FHL tendon at the Knot of Henry and transfer through the calcaneus

Complications
 Re-rupture
o incidence
 higher with non-operative management (~10-40% vs 2%)
 new Level 1 evidence has shown no difference in re-rupture rates
o treatment
 surgical repair
 Wound healing complications
o incidence
 5-10%
o risk factors
 smoking (most common)
 female gender
 steroid use
 open technique (versus percutaneous)
o treatment
 deep infection
 debridement of necrotic/infected Achilles tendon
 culture-specific antibiotics for 6 weeks
 Sural nerve injury
o incidence
 higher when percutaneous approach is used ‎ II:14 Wound healing
V
complications

- 430 -
By Dr, AbdulRahman AbdulNasser Foot & Ankle Trauma | Tendon Injuries

2. Peroneal Tendon Subluxation & Dislocation


Subluxation/Dislocations
 Peroneal tedon dislocation and repetitive subluxation from behind lateral malleolus
 Epidemiology
o most of these injuries occur in young, active patients
 Mechanism
o rapid dorsiflexion of an inverted foot inversion leading to rapid reflexive contraction of the PL
and PB tendons
o rapid contraction can also lead to injury to the superior peroneal retinaculum
 Pathophysiology
o subluxation of the peroneal tendons leads to longitudinal tears over time which usually
involves peroneus brevis at fibular groove
o subfibular impingment secondary to calcaneal malunion
 Associated conditions
o tears of the peroneus brevis and or longus
 longitudinal split tears more common than transverse
o lateral ankle ligament injuries (ATFL, CFL) in up to 75% of patients with SPR injuries

Anatomy
 Muscles & innervation
o peroneus brevis
 innervated by the superficial peroneal nerve, S1
 acts as primary evertor of the foot
 tendinous about 2-4cm proximal to the tip of the fibula
 lies anterior and medial to the peroneus longus at the level of the lateral malleolus
o peroneus longus
 innervated by superficial peroneal nerve, S1
 primarily a plantar flexor and foot and first metatarsal
 can have an ossicle (os peroneum) located within the tendon body
 Space & compartment
o peroneal tendons contained within a common synovial sheath that splits at the level of the
peroneal tubercle
o the sheath is runs in the retromalleolar sulcus on the fibula
 peroneus longus is posterior in the sulcus (longus takes the long way around)

- 431 -
OrthoBullets2017 Foot & Ankle Trauma | Tendon Injuries
peroneus brevis is anterior in the sulcus (brevis is behind the bone)
 deepened by a fibrocartilaginous rim (still only about 5 millimeters deep)
 covered by superior peroneal retinaculum (SPR)
 originates from the posterolateral ridge of the fibula and inserts onto the lateral calcaneus
(peroneal tubercle)
 the inferior aspect of the SPR blends with the inferior peroneal retinaculum
 is the primary restraint the peroneal tendons within the retromalleolar sulcus
o at the level of the peroneal tubercle of the calcaneus
 peroneus longus is inferior
 peroneus brevis is superior
 both tendons covered by inferior peroneal retinaculum

the sheath is runs in


the retromalleolar at the level of the peroneal tubercle of the calcaneus
sulcus on the fibula

Classification
Ogden Classification of Superior Peroneal Retinaculum (SPR) Tears
Grade 1 The SPR is partially elevated off of the fibula allowing for subluxation of both
tendons
Grade 2 The SPR is separated from the cartilofibrous ridge of the lateral malleolus,
allowing the tendons to sublux between the SPR and the cartilofibrous ridge
Grade 3 There is a cortical avulsion of the SPR off of the fibula, allowing the subluxed
tendons to move underneath the cortical fragment
Grade 4 The SPR is torn from the calcaneous, not the fibula

- 432 -
By Dr, AbdulRahman AbdulNasser Foot & Ankle Trauma | Tendon Injuries
Presentation
 History
o patients often report they felt a pop with a dorsiflexion ankle injury
 Symptoms
o clicking, popping and feelings of instability or pain on the lateral aspect of the ankle
 Physical exam
o inspection
 swelling posterior to the lateral malleolus
 tenderness over the tendons
 'pseudotumor' over the peroneal tendons
 voluntary subluxation of the tendons +/- a popping sound
o provocative tests
 apprehension tests
 the sensation of apprehension or subluxation with active dorsiflexion and eversion against
resistance cause subluxation/dislocation and apprehension
 compression test
 pain with passive dorsiflexion and eversion of the ankle

Imaging
 Radiographs
o recommended views
 best recognized on an internal rotation view
o findings
 may see a cortical avulsion off the distal tip of the lateral malleolus
(fleck sign, rim fracture)
 needed to evaluate for varus hindfoot
 MRI
o best evaluated with axial views of a slightly flexed ankle
o can demonstrate anatomic anomalies leading to pathology
 peroneus quartus muscle
 low-lying peroneus brevis muscle belly
‎VII:15 fleck sign
Treatment
 Nonoperative
o short leg cast immobilization and protected weight bearing for 6 weeks
 indications
 all acute injuries in nonprofessional athletes
 technique
 tendons must be reduced at the time of casting
 outcomes
 success rates for nonsurgical management are only marginally better than 50%.
 Operative
o acute repair of superior peroneal retinaculum and deepening of the fibular groove
 indications
 acute tendon dislocations in serious athletes who desire a quick return to a sport or active
lifestyle
 presence of a longitudinal tear
- 433 -
OrthoBullets2017 Foot & Ankle Trauma | Tendon Injuries
o groove-deepening with soft tissue transfer and/or osteotomy
 indications
 chronic/recurrent dislocation
 technique
 less able to reconstruct SPR so treatment focuses on other aspects of peroneal stability
 typically involves groove-deepening in addition to soft tissue transfers or bone block
techniques (osteotomies to further contain the tendons within the sulcus)
 plantaris grafts can act to reinforce the SPR
 hindfoot varus must be corrected prior to any SPR reconstructive procedure

groove-deepening with soft tissue transfer and/or osteotomy


Peroneal Brevis Tears
 Introduction
o presentation and physical exam is often very similar to peroneal tendon dislocation, however
there is no instability of the tendon
 Imaging
o MRI is requried for diagnosis
 Treatment
o nonoperative
 NSAIDs, activity restriction and a walking boot are often the first line of treatment
 failure rate may be as high as 83%
o operative
 core repair and tubularization of the tendon

- 434 -
By Dr, AbdulRahman AbdulNasser Foot & Ankle Trauma | Tendon Injuries
 indications
 simple tears
 debridement of the tendon with tenodesis of distal and proximal ends of the brevis
tendon to the peroneus longus or reconstruction with allograft
 indications
 complex tears with multiple longitudinal tears and significant tendinosis (> 50% of
the tendon involved)

core repair and tubularization of the tendon

3. Anterior Tibialis Tendon Rupture


Introduction
 Partial or complete discontinuity of the tibialis anterior tendon
 Epidemiology
o demographics
 attritional rupture more common in older patients
 strong eccentric contraction more common in younger patients
o body location
 at the level of the ankle joint with varying degrees of retraction of the proximal stump
o risk factors
 older age
 diabetes
 fluoroquinolone use
 local steroid injection
 inflammatory arthritis
 Pathophysiology
o mechanism of injury
 result of either laceration of the tendon or closed rupture
 may also result from strong eccentric contraction in young individual
 Medical conditions & comorbidities
o diabetes
o inflammatory arthritis
o Prognosis : Good with treatment

Anatomy
 Ankle dorsiflexion

- 435 -
OrthoBullets2017 Foot & Ankle Trauma | Tendon Injuries
o primary ankle dorsiflexor (80%)
 tibialis anterior
o secondary ankle dorsiflexors
 extensor hallucis longus
 extensor digitorum longus

Presentation
 History
o acute
 patient reports a 'pop' followed by anterior ankle swelling
o chronic
 patient reports difficulty clearing foot during gait
 Symtpoms:
o acute
 pain
o chronic
 may be painless
 Physical exam
o acute injury
 pain swelling anterior to ankle
 weakness in dorsiflexion of the ankle
 delay in diagnosis is common because of intact ankle dorsiflexion that occurs as a result
of secondary function of the extensor hallucis longus and extensor digitorum longus
muscles
o chronic injury
 inspection and palpation
 swelling may be minimal
 painless mass at the anteromedial aspect of the ankle
 loss of the contour of the tibialis anterior tendon over the ankle (tendon not palpable
during resisted dorsiflexion)
 weakness
 use of the extensor hallucis longus and extensor digitorum communis to dorsiflex the
ankle
 gait
 steppage gait (hip flexed more than normal in swing phase to prevent toes from catching)
 foot slaps down after heel strike

Imaging
 Radiographs
o three views of foot and ankle helpful to exclude any associated osseous injury
 CT : not indicated
 MRI
o helpful to diagnose complete versus partial tear but not to determine if interposition graft is
necessary
Differential
 Lumbar radiculopathy (L4)

- 436 -
By Dr, AbdulRahman AbdulNasser Foot & Ankle Trauma | Tendon Injuries
o can be differentiated from TA rupture by
 intact tendon palpable
 no ankle mass
 may have dermatomal sensory abnormality
 positive lumbar spine MRI
 Common peroneal nerve compression neuropathy
o EDL, EHL also affected
o sensory abormalities
o history of compression to common peroneal nerve

Treatment
 Nonoperative
o ankle-foot orthosis
 indications
 low demand patient
o casting
 indications
 partial ruptures
 Operative
o direct repair
 indications
 acute injury (<6 week) injuries
 should be attempted up to 3 months out
 outcomes
 surgical repair leads to improved AOFAS scores and improved levels of activity
 some residual weakness of dorsiflexion is expected
o reconstruction
 indications
 most often required in chronic (>6 week) old injuries

Technique
 Direct repair
o approach
 open laceration: incorporate laceration
 closed rupture: longitudinal incision centered over palpable defect
o repair technique
 distal end usually accessible through laceration, proximal end may retract ~3cm
 place hemostat in wound under extensor retinaculum and pull tendon into wound
 primary end-to-end non-absorbable suture with Krackow, Bunnell or Kessler technique
 ends oversewn with small monofilament if frayed to create smoother gliding surface
 in cases of avulsion, suture anchors or bone tunnels may be used for reattachment
 Tendon reconstruction
o approach
 curvilinear incision over course of tibialis tendon, may need to be extensile depending needs
of reconstruction
 EHL can be divided through separate small incision and tunneled proximally
o sliding tendon graft

- 437 -
OrthoBullets2017 Foot & Ankle Trauma | Tendon Injuries
 harvest one half width of tibialis anterior tendon proximally and turn down to span gap
 repair can be strengthened by securing tibialis anterior tendon to medial cuneiform or dorsal
navicular distal to extensor retinaculum
o free tendon graft
 interposition of autograft (hamstring, plantaris) or allograft
o EHL tenodesis or EHL transfer
 distal EHL stump tenodesed to EHB
 proximal EHL stump used as tendon graft to repair tibialis anterior insertion
 proximal tibialis anterior placed under tension prior to suturing to proximal EHL stump

Complications
 Failure of reconstruction/repair
 Weakness of dorsiflexion
 Adhesion formation
 Neuroma formation

Collected By : Dr AbdulRahman
AbdulNasser
June 2017

- 438 -
By Dr, AbdulRahman AbdulNasser Pediatric trauma | Tendon Injuries

ORTHO BULLETS

VIII. Pediatric trauma

- 439 -
OrthoBullets2017 Pediatric trauma | Introduction

A. Introduction

1. Pediatric Abuse
Introduction
 Epidemiology
o incidence
 >1 million children are victims of substantiated abuse or neglect in United States each year
 child abuse is the second most common cause of death in children behind accidental injury.
 in child abuse, head injury is the most frequent cause of long term physical morbidity in the
child
o demographics
 astounding 79% of all cases of nonaccidental trauma occur in children younger than 4 years
of age
 50% of fractures in children younger than 1 year of age are attributable to abuse
 the most common cause of femur fractures in the nonambulatory infant is nonaccidental
trauma
o social risk factors
 recent job loss of parent
 children with disabilities (cerebral palsy, premature)
 step children
o 4 Types (can have more than one type present):
 Neglect 78%
 Physical Abuse 18%
 Sexual 9%
 Psychological 8%
‎ III:1 corner fxs
V
 Prognosis
o If unreported, 30-50% chance of repeat abuse and 5-10% chance of death from abuse
 Differential Diagnosis
o true accidental injury
o osteogenesis imperfecta
o metabolic bone disease

Presentation
 History
o injury often inconsistent with history
‎VIII:2 posterior rib fractures
o red flags
 long bone fxs in infant that is not yet walking
 multiple bruises
 multiple fxs in various stages of healing
 corner fxs
 primary spongiosa (metaphyseal)
 high specificity for child abuse
 posterior rib fractures
 bucket handle fractures ‎VIII:3 bucket handle fractures
 same as corner fractures

- 440 -
By Dr, AbdulRahman AbdulNasser Pediatric trauma | Introduction
 avulsed bone fragment is seen en face as a bucket handle
 transphyseal separation of the distal humerus
 Symptoms
o pain related to fractures
 fractures are the second most common lesion in abused children
 frequency of fractures
 humerus > tibia > femur
 diaphyseal fractures 4 times more common than metaphyseal
 Physical exam
o skin lesions
 most common presenting lesion

Imaging
 Radiographs
o recommended views
 AP and lateral of bone or joint of suspicion
 initial evaluation should include skeletal survey
 Bone scan
o alternative or adjunct to the radiographic skeletal survey in selected cases, particularly for
children older than 1 year. Scintigraphy provides increased sensitivity for detecting rib fractures,
subtle shaft fractures, and areas of early periosteal elevation. Not useful in metaphyseal or
cranial fractures. Not indicated after 5 years of age
Treatment
 Nonoperative
o report abuse to appropriate agency
 indications
 Physicians are mandated reporters, and are legally obligated to report suspected child
abuse and neglect.
 Physicians are granted immunity from civil and criminal liability if they report in good
faith, but may be charged with a crime for failure to report
 early involvement of social workers and pediatricians is essential
o hospital admission
 indications
 early multidisciplinary evaluation
 admit infants with fractures to the hospital and consult child protective services
 obtain social service consult
 Operative
o definitive treatment as indicated for particular injury

- 441 -
OrthoBullets2017 Pediatric trauma | Introduction

2. Pediatric Evaluation & Resuscitation


Epidemiology
 Trauma is most common cause of death in children > 1 years old
 Epidemiology
o mortality rate is approximately 20%
o CNS injuries have highest overall morbiditya and mortality
o spine fractures have highest morbidity/mortality among musculoskeletal injuries
 cervical spine injuries more common in children <8-years-old
 due to fact that restraints do not fit young children
 Pathophysiology
o falls and motor vehicle accidents most common

Transport
 Occipital cut-out needed in spine board when transporting children <6 y.o.
o larger head size can flex unstable cervical spine leading to injury during transport
 Help tips
o Broselow tape = estimate medication doses, size of equipment, shock voltage for defibrillator
o ETT = (age/4) + 4 or (age+16)/4 = uncuffed
o BP = 80 + (age x 2)
o Chest tube = 4 x ETT
o Blood volume = 70 x wt (kg) or 75 - 80 mL/kg
 Intraosseous lines commonly needed due to difficulty obtaining venous access
o Children may remain hemodynamically stable even after significant blood loss
 hypovolemic shock may result from inadequate fluid resuscitation
o "triad of death" reflects inadequate resusitation and is characterized by:
 acidosis
 hypothermia
 coagulopathy

ATLS with Children


 Overview
o Should follow ATLS protocol
 Airway
 Breathing
 Circulation
 Disability
 Exposure

Airway
 Smaller airway
o greater risk of airway obstruction with foreign bodies
o small amounts of swelling will result in a relatively greater reduction in airway diameter
 Larger tongue, floppy epiglottis,
 Larger occiput
o flexes the head forward when placed supine on a flat surface.

- 442 -
By Dr, AbdulRahman AbdulNasser Pediatric trauma | Introduction
o to achieve a neutral position, it may be necessary to lift the chin or place a pad under the torso of
the infant (or head cut out)
 Larynx is higher and more anterior
o sits at the level of the C2-C3 vertebrae body in the young child, compared with C6-C7 in the
adult.
 positioning of the larynx makes its visualisation in the paediatric airway more difficult than
in the adult.
Breathing
 Most common cause of cardiorespiratory arrest is hypoventilation
 Ribs positioned more horizontally
o with inspiration the ribs only move up, and not up-and-out, like the adult rib cage.
o limits the capacity to increase tidal volumes
 Diaphragmatic breathing
 Fewer Type 1 fibres in respiratory muscles
o smaller number of fatigue-resistant, Type I fibres in their respiratory muscles
o exhaust more quickly than adults
 Respiratory rate varies with age
o higher oxygen demand = higher respiratory rates

Circulation
 Initial bolus = 20ml/kg NS
 After two boluses = 10ml/kg of PRBC’s
 Blood volume is relatively larger, but absolute volume is smaller
o small volumes of blood will constitute significant blood loss in small children,
 example = 100ml haemorrhage experienced by a 5 kg child represents the loss of
approximately 10% of their total blood volume.
 Systemic vascular resistance is lower
o increases from birth to adulthood
 Hypotension is a late sign
o remain normotensive until they are loosing large intravascular volumes
 25-30% of blood volume before signs of shock
 Smaller vessels / more subcutaneous tissue
o difficult to obtain vascular access due to small veins and increased subcutaneous tissue
 IV access more difficult – consider intraosseus

Disability
 Open sutures, presence of fontanelle
 Thinner cranial bones
o thinner cranial bones of children do not afford as much protection to the brain tissue
 Head relatively larger
o higher centre of gravity = higher incidence of head and neck trauma

Exposure
 Relatively small size
o large head and organs
 Higher BMR and surface area
o greater consumption of oxygen and other metabolites
- 443 -
OrthoBullets2017 Pediatric trauma | Introduction
o higher respiratory and heart rates
o larger surface-area to body-mass ratio results in greater heat loss
 Increased glucose requirements but decreased glycogen stores
o higher metabolic rate
o small glycogen stores

Pediatric Scoring Systems


 Pediatric trauma score (PTS)
o PTS<0=100% mortality
o PTS of 1-4=40% mortality
o PTS of 5-8=7% mortality
 PTS less than or equal to 8 should be sent to designated peds trauma center
 Pediatric Galsgow Coma Scale
o GCS<8 correlates with a higher rate of mortality
 O2 sat at presentation and GCS 72hrs post-injury are both prognostic of long-term neurologic
recovery

Injuries
 Head and neck
o ICP can be elevated by pain
 it is possible to decrease ICP by fracture fixation
o heterotopic ossification is more common following traumatic brain injury
 increase serum alkaline phosphatase heralds onset of HO
 NSAID prophylaxis is indicated in these situations
 Peripheral nerve injuries
o most common in closed fractures
 obtain EMG if no return of function 2-3 months after injury

- 444 -
By Dr, AbdulRahman AbdulNasser Pediatric trauma | Introduction

3. Physeal Considerations
Introduction
 Always look to see if physis is open
 Unique principals in pediatric bone
o elasticity
 more elastic which leads to unique fracture patterns
 buckle fractures
 greenstick fractures
o remodeling potential
 open physes (growth plates) can allow extensive bone
deformity remodeling potential
 occurs more rapidly in plane of joint motion
 sagittal plane in wrist, due to primarily extension/flexion
 occurs more at the most active physes, due to most growth and
potential for remodeling
 most active physes in upper extremity
 proximal humerus
 distal radius
 most active physes in lower extremity
 distal femur
 proximal tibia
 Same principles as adult bone
o intra-articular fractures must be reduced

‎VIII:4 Illustration of blood supply of the physis ‎VIII:5 Perichondrial fibrous ring of La Croix

- 445 -
OrthoBullets2017 Pediatric trauma | Introduction
Physeal Anatomy

Physeal Growth Plate


(letters on left correspond to histology in top right)
B. Reserve zone  Cells store lipids, glycogen, and proteoglycan  Gaucher's
aggregates for later growth and matrix production  diastrophic dysplasia
 Low oxygen tension  Kneist
 pseudoachondroplasia
C. Proliferative  Proliferation of chondrocytes with longitudinal  Achondroplasia
zone growth and stacking of chondrocytes.  Gigantism
 Highest rate of extracellular matrix production  MHE
 Increased oxygen tension in surroundings inhibits
calcification
D. Hypertrophic  Zone of chondrocyte maturation, chondrocyte  SCFE (not renal)
zone hypertrophy, and chondrocyte calcification.  Rickets (provisional
 Three phases occur in the hypertrophic zone calcification zone)
o Maturation zone: preparation of matrix for  Enchondromas
calcification, chondrocyte growth  Mucopolysacharide disease
o Degenerative zone: further preparation of  Schmids
matrix for calcification, further chondrocyte  Fractures most commonly
growth in size (5x) occur through hypertrophic
o Provisional calcification zone: zone
chondrocyte death allows calcium release,
allowing calcification of matrix
 Chondrocyte maturation regulated by local growth
factors (parathyroid related peptides, expression
regulated by Indian hedgehog gene)
 Type X collagen produced by hypertrophic
chondrocytes important for mineralization
E. Primary  Vascular invasion and resorption of transverse  Metaphyseal "corner fracture"
spongiosa septa. in child abuse
(metaphysis)  Osteoblasts align on cartilage bars produced by  Scurvy
physeal expansion.
 Primary spongiosa mineralized to form woven bone
and then remodels to become secondary spongiosa
(below)

Secondary  Internal remodeling (removal of cartilage bars,  Renal SCFE


spongiosa replacement of fiber bone with lamellar bone)
(metaphysis)  External remodeling (funnelization)

Physis Periphery
Groove of  During the first year of life, the zone spreads over  Osteochondroma
Ranvier the adjacent metaphysis to form a fibrous
circumferential ring bridging from the epiphysis to
the diaphysis.
 This ring increases the mechanical strength of the
physis and is responsible for appositional bone
growths
o supplies chondrocytes to periphery
Perichondrial  Dense fibrous tissue that is the primary limiting
fibrous ring of membrane that anchors and supports the physis
La Croix through peripheral stability

 Perichondrial artery
o major source of nutrition to physis

- 446 -
By Dr, AbdulRahman AbdulNasser Pediatric trauma | Introduction
Injury Classification
 Salter-Harris classification
o Type 1- physeal separation
o Type 2- fracture traverses physis and exits metaphysis
 most common type
 Thurston Holland fragment
o Type 3- fracture traverses physis and exits epiphysis
o Type 4- fracture passes through epiphysis, physis, metaphysis
 Thurston Holland fragment
o Type 5- crush injury to physis

SH 2 distal femur SH 3 distal tibia SH 4 middle phalynx

- 447 -
OrthoBullets2017 Pediatric trauma | Shoulder & Humerus Fractures
Treatment
 Closed reduction vs. CRPP vs Open reduction
o depends on injury pattern
o intra-articular fractures must be reduced

Complications
 Growth arrests
o overview
 complete arrest leads to shortening
 see Leg Length Discrepancy
 partial arrest leads to angulation
o treatment
 bar resection with interposition
 indications
 < 50% physeal involvement

 > 2 years or 2cm growth remaining ‎VIII:6 partial arrest leads to angulation
 ipsilateral completion of arrest
 indications
 > 50% physeal involvement
 can combine with contralateral epiphysiodesis and/or ipsilateral lengthening

B. Shoulder & Humerus Fractures

1. Medial Clavicle Physeal Fractures


Introduction
 Suspect in any young individual (< 25 yrs) with a
medial clavicle or sternoclavicular injury
 Usually Salter-Harris Type I or II
 Delay in diagnosis

Anatomy
 Medial clavicle ossification center
o appears during later teenage years
o last physis to close in body (20-25yrs)
 sternoclavicular dislocations in
teenagers/young adults are usually
physeal fracture-dislocations
Imaging
 Radiographs
o difficult to visualize on AP, and radiographs usually unreliable to assess for fracture and degree
of displacement
o obtain serendipity views ( beam at 40 deg cephalic tilt)
 anterior dislocation/fxs - affected clavicle is above contralateral clavicle
 posterior dislocation/fxs - affected clavicle is below contralateral clavicle

- 448 -
By Dr, AbdulRahman AbdulNasser Pediatric trauma | Shoulder & Humerus Fractures
 Axial CT scan
o is study of choice
o can differentiate from sternoclavicular dislocations
o can visualize mediastinal structures and injuries

Treatment
 Nonoperative
o observation
 controversial
 most asymptomatic injuries will remodel and do not require intervention
 anterior displaced physeal fracture
 has good functional results treated nonoperatively
o closed reduction in operating room under anesthesia
 indications
 early posterior displaced physeal fx
 hoarsness
 blunt or direct trauma to subclavian vessels
 thoracic outlet syndrome
 pneumothorax
 technique
 approach : thoracic surgeon available
 reduction
 traction and abduction of arm, while applying direct pressure
 posterior displaced fractures usually require sterile towel clip for manipulation
 convert to open
 if irreducible by closed means, consider open approach
 postreduction
 if stable - obtain CT to document
 if unstable - open reduction with wire/suture from medial clavicle to sternum/medial
epiphysis
 immobilization : figure of 8 harness or sling and swathe x 4 weeks (anterior displaced)
 Operative
o open reduction
 rarely needed
 indications
 unreducible and symptomatic in a patient > 23 yrs old
 instability after reduction

Complications
 Delay in reduction >48h
o reduces success of closed reduction
o because of progressive callus formation in dislocated state
 Late presenting posterior displaced injuries
o do NOT attempt closed reduction because medial clavicle may be adherent to vascular structures
in mediastinum

- 449 -
OrthoBullets2017 Pediatric trauma | Shoulder & Humerus Fractures

2. Distal Clavicle Physeal Fractures


Introduction
 Rare injury accounting for only 5% of clavicle fractures in children
o considered a childhood equivalent to adult AC separation
 Pathoanatomy
o periosteum usually remains intact with injury
o clavicle displaces away from physis and periosteal sleeve,
both of which remain attached to the AC and CC ligaments
Classification
 None

Presentation
 Symptoms
o pain, dysfunction, ecchymosis in older children
 Physical exam
o pseudo-paralysis of the affected ipsilateral extremity may be present in newborns
 reflexes remain intact following isolated clavicle fractures, which can help differentiate from
brachial plexus injuries
Imaging
 Radiographs
o obtain AP and serendipity view to help define injury

Treatment
 Nonoperative
o sling management
 indicated in most cases, especially if periosteum is
intact
 a new clavicle will form within the intact periosteal
sleeve, and the displaced clavicle will typically
reabsorb with time and growth
 Operative
o surgical reduction
 indications (rarely indicated)
 open fractures
 severly displaced fractures in older patients with near closed physis

3. Proximal Humerus Fracture - Pediatric


Introduction
 Physeal and metaphyseal proximal humerus fractures
 Epidemiology
o incidence
 < 5% of fractures in children
 Demographics : most common in adolescents (peak age at 15 years) who get SH-II fractures
o fracture pattern
- 450 -
By Dr, AbdulRahman AbdulNasser Pediatric trauma | Shoulder & Humerus Fractures
 Salter-Harris classification
 SH-I is most frequent in <5 year olds
 SH-II is most frequent in >12 year olds
 Metaphyseal fractures
 typically occur in 5 to 12 year olds
 Pathophysiology
o mechanism of injury
 blunt trauma
 indirect trauma
 overuse injury in throwers (Little Leaguer's shoulder)
 SH-I fracture secondary to overuse
 mild widening of the physis and metaphyseal changes ‎VIII:7 Little
o pathoanatomy Leaguer's shoulder

 proximal fragment (epiphysis) displacement


 abducted and externally rotated because of pull from rotator cuff muscles
 distal fragment (shaft) displacement
 anterior, adducted and shortened because of pull from pectoralis major and deltoid muscle
 Prognosis
o excellent
 abundant remodeling potential of the proximal humerus
 due to range of motion of the shoulder joint

Anatomy
 Radiographic appearance of secondary ossification centers
o proximal humeral epiphysis at 6 mos
o greater tuberosity appears at 1-3 yrs
o lesser tuberosity appears at 4-5 yrs
 Growth
o Proximal humerus physis closes at 14-17 in girls, 16-18 in boys
 80% of humerus growth comes from the proximal physis
 highest proximal:distal ratio difference (femur is second with 30:70 proximal:distal ratio)
 high remodeling potential (most fractures can be treated nonoperatively)

Classification
 Neer-Horowitz Classification

Neer-Horowitz Classification
Type I • Minimally displaced (<5m)
Type II • Displaced < 1/3 of shaft width
Type III • Displaced greater than 1/3 and less than 2/3 of shaft width
Type IV • Displaced greater than 2/3 of shaft width

- 451 -
OrthoBullets2017 Pediatric trauma | Shoulder & Humerus Fractures

Type 2 Type 4
Presentation
 History
o identify any precipitating injury
 Symptoms
o shoulder pain
o dysfunction
o deformity
o ecchymosis
 Physical exam
o inspection of skin
o motion and tenderness of neck, ipsilateral sternoclavicular joint and
elbow ‎VIII:8 Y view
o neurovascular examination
 brachial plexus distribution
 vascular examination of arm

Imaging
 Radiographs
o standard views
 obtain AP, lateral, and scapula Y or axillary views of
shoulder
o as needed views
 hand or elbow for bone age
 contralateral shoulder for comparison views ‎VIII:9 Axillary view
o findings
 stress fractures in athletes
 glenohumeral dislocation (very rare with associated fracture)
 assess maximum angulation of fracture displacement
 identify pathologic fracture if present
 Classify fracture type: newborn, acute fracture, stress fracture, pathologic fracture
 Ultrasound
o ultrasound may be neccessary in newborns before secondary ossification centers are formed

- 452 -
By Dr, AbdulRahman AbdulNasser Pediatric trauma | Shoulder & Humerus Fractures
Treatment
 Nonoperative
o immobilization
 indications
 acceptable alignment for non-operative management
 <10y = any degree of angulation
 10-13y = up to 60° of angulation
 >13y = up to 45° of angulation and 2/3 displacement
 technique
 immobilization modalities
 sling + swathe
 shoulder immobilizer
 coaptation splint
o closed reduction under anesthesia/analgesia and fluoroscopy, without fixation
 indications
 severely displaced (>Neer-Horowitz III or >66%) with >45° angulation and <2y of
growth left
 risk of loss of reduction
 Operative
o open reduction and fracture fixation
 indications
 severely displaced fractures > 13 years old failed closed reduction
 >Neer-Horowitz III (>66% displaced)
 severely angulated fractures in > 9 year old failed closed reduction
 open fractures in any age
 fractures associated with vascular injuries
 intra-articular displacement
 techniques
 closed reduction ± k-wire fixation
 reduction maneuver
 longitudinal traction
 shoulder abduction to 90 degrees
 external rotation
 percutaneous pinning
 two or three lateral threaded pins
 starting point must consider branches of axillary nerve (lateral) and
musculocutaneous nerve (anterior)
 ideally divergent pattern across fracture
 open reduction ± k-wire fixation
 indications
 unacceptable closed reduction maneuver
 blocks to reduction
 long head of biceps tendon (most common)
 joint capsule
 infolded periosteum
 deltoid muscle

- 453 -
OrthoBullets2017 Pediatric trauma | Shoulder & Humerus Fractures
 approach
 deltopectoral interval
 fixation methods
 wire fixation (smooth or threaded)
 cannulated screw
 retrograde flexible nails

Complications
 Loss of reduction
o risk factors
 unstable fractures treated with closed reduction WITHOUT pinning
 Axillary nerve Injuries
o occur in <1% of case due to injury alone
 typically are neuropraxias
 associated with a medially displaced shaft
o higher risk with percutaneous pinning
 avoid lateral pin entry 5-7cm distal to acromion
 Malunion
o varus malalignment, more common in younger patients
 may cause glenohumeral impingement
 Limb-length inequality
o fracture shortening
 <3cm usually well tolerated
o growth arrest
 usually rare
 Hypertrophic scar
o deltopectoral approach with open reduction and fixation
 Pin site infection

4. Humerus Shaft Fracture - Pediatric


Introduction
 Plastic deformation and greenstick fractures of the humeral shaft are rare
 Epidemiology
o incidence
 represent <10% of humerus fractures in children
 Pathophysiology
o mechanism of injury
 typically associated with trauma
o pathomechanics
 neonates
 hyper-extension or rotational injury during birth
 adolescents
 usually direct, high-energy trauma
o pathophysiology
 consider a pathologic process if fracture is a result of a low energy mechanism
 may be associated with child abuse if age <3 and fracture pattern is spiral

- 454 -
By Dr, AbdulRahman AbdulNasser Pediatric trauma | Shoulder & Humerus Fractures
 Associated conditions
o radial nerve palsy
 associated with up to 5% of humeral shaft fractures
 Prognosis
o excellent
 associated with enormous remodeling potential and rarely requires surgical intervention
 up to 30° of angulation is associated with excellent outcomes due to the large range of motion
of the shoulder

Presentation
 Symptoms
o history of traumatic event
o pain
o upper arm deformity
 Physical exam
o inspection
 mid-arm swelling and deformity
 open fractures rare
o palpation
 tenderness to palpation
o motion
 weakness or absence of wrist and digit extension if radial nerve palsy is present
 pseudoparalysis
 irritability or refusal to move upper limb in neonates

Imaging
 Radiographs
o recommended views
 full length AP, lateral views of humerus
 must include joint above and below
o optional views
 orthogonal views of shoulder and elbow
 required to rule out associated injuries
o findings
 typical fracture patterns are transverse and oblique
 examine closely for pathologic lesions

- 455 -
OrthoBullets2017 Pediatric trauma | Shoulder & Humerus Fractures
Treatment
 Nonoperative
o analgesia, immobilization
 indications
 uncomplicated diaphyseal fracture without intra-articular involvement in a child of any
age
 utilized for almost all pediatric humeral shaft fractures
 techniques
 sling and swathe or cuff and collar in young children
 Coaptation splint or hanging arm cast
 Sarmiento functional brace in older children/adolescents
 ROM exercises can be initiated in 2-3 weeks once pain is controlled
 Operative
o open reduction internal fixation
 indications
 open fractures
 multiply injured patient
 ipsilateral forearm fractures
 "floating elbow"
 associated shoulder injury
 techniques
 flexible intramedullary nail fixation
 anterior, anterolateral or posterior approach with plate fixation

Complications
 Radial nerve palsy
o occurs in <5%
 most commonly associated with middle and distal 1/3 fractures
o typically due to a neuropraxia
o spontaneous resolution is expected
o exploration is rarely needed
 if function has not returned in 3-4 months, EMGs are performed and exploration considered
 Malunion
o rarely produces functional deficits, due to the wide range of motion at the shoulder
 up to 30° of angulation is associated with excellent outcomes
 Delayed union
o rare given the capacity to remodel
o may consider ultrasound bone stimulation
 Limb length discrepancy
o commonly occurs, but rarely causes functional deficits
 Physeal growth arrest
o proximal and distal humerus growth plates contributes 80:20 percent to overall humeral length

- 456 -
By Dr, AbdulRahman AbdulNasser Pediatric trauma | Shoulder & Humerus Fractures

5. Distal Humerus Physeal Separation - Pediatric


Introduction
 Also called transphyseal distal humerus fracture
 Epidemiology
o demographics
 typically seen in children under the age of 3
 Mechanism
o can happen during vaginal delivery
 from force of labor or obstetric maneuvers
 especially traumatic delivery and shoulder dystocia
o cesarean section : from excessive traction
o nonaccidental trauma
 rotational force / twisting
o fall on outstretched hand with elbow extended
 Pathoanatomy
o physis is biomechanically the weakest location in distal humerus
 Associated conditions
o abuse or battered child syndrome (up to 50%)
 Prognosis
o often missed diagnosis as very difficult to diagnose
 >50% missed by radiologist

Classification
 Salter Harris classification
o older children (>3y) have Salter Harris II injuries
 metaphyseal piece attached to distal fragment
o younger children (<3y) have Salter Harris I injuries
 pure physeal
o rare cases have intra-articular extension (Salter Harris III or IV)

Presentation
 History
o birthing process (see above)
o fall from height (bed, chair, down stairs)
o another child jumps/falls on younger child's elbow
o suspect nonaccidental trauma if
 unwitnessed injuries
 inconsistent explanations
 history of multiple injuries, burns, bites, bruising
 Physical exam
o inspection
 pseudoparalysis / diminished spontaneous movement
o neurovascular
 rarely neurovascular compromise

- 457 -
OrthoBullets2017 Pediatric trauma | Shoulder & Humerus Fractures
Imaging
 Radiographs
o recommended views
 AP and lateral centered on the elbow
 "baby gram" (radiograph of entire extremity) often miss diagnosis
 stress radiographs may be helpful to clarify the diagnosis
 skeletal survey if child-abuse suspected
o findings
 in infant only sign may be posteromedial displacement of the radial
and ulnar shafts relative to the distal humerus
 forearm not aligned with humeral shaft
 soft tissue swelling, joint effusion (posterior fat pad)
 anterior fat pad may be absent
 if capitellar ossification center is present, will be aligned with radius shaft, making diagnosis
definitive
 Ultrasound
o indications
 uncertain diagnosis
o advantage
 no need for sedation
o findings
 static exam
 detect separation of epiphysis from
metaphysis by noting lack of
‎ III:10 Magnetic resonance imaging
V
cartilage at distal humeral metaphysis demonstrates fracture through the
 dynamic exam humeral physes with posterior
displacement of the cartilaginous
 detect instability of epiphysis relative to metaphysis epiphysis (curved arrow) but intact
 MRI articulation with radius and ulna
(arrow)
o disadvantage : requires sedation
 Elbow arthrography
o indications : uncertain diagnosis
o findings
 visualization of entire distal articular surface and proximal radius
o technique
 posterolateral approach or direct posterior approach
 direct posterior into olecranon fossa recommended in young children to prevent scuffing
of articular cartilage when posterolateral portal is used
 inject equal parts saline:contrast
 bring through range of motion
 if pinning is needed, arthrogram aids visualization of pin starting points on capitellum
 aids assessment of quality of reduction by seeing anterior humeral line intersecting
capitellum
o advantage
 if performed under anesthesia in OR, can perform reduction and stabilization simultaneously
if needed

- 458 -
By Dr, AbdulRahman AbdulNasser Pediatric trauma | Shoulder & Humerus Fractures
Differentials
 Elbow dislocation
o almost never happens in <3 yrs because physis is weaker than bone-ligament interface,
predisposing to physeal fracture rather than ligament disruption/dislocation
 Other fractures
o often misdiagnosed (or delayed diagnosis up to 1 week) as supracondylar, condyle, epicondyle
fractures
Treatment
 Nonoperative
o limited role because most fractures are displaced
o posterior long arm splint then long arm casting x 2-3wk
 indications
 nondisplaced fractures
 late presenting fractures
 treat nonop initially
 deformity will persist/develop, requiring osteotomy in future
 Operative
o closed reduction and pinning
 pinning is necessary to ensure adequate reduction, which may be lost with casting alone once
swelling subsides
 indications
 displaced fractures

Technique
 Closed reduction and pinning
o general anesthesia
o reduction maneuver
 gentle traction (very little force required)
 distal fragment may sometimes be grasped between index finger and thumb and reduced
to humeral shaft
 correction of translation/malrotation
 elbow flexion
o use elbow arthrogram to aid
o parameters
 no cubitus varus
 anterior humeral line should bisect capitellum
 no malrotation
o pinning
 2 or 3 x 0.062inch K wires
 these larger pins help prevent loss of reduction
 from lateral side, retrograde fashion
 divergent
 engage both cortices
 good spread at fracture site
o then perform live fluoroscopy through range of motion
o bend / cut pins, splint the arm
- 459 -
OrthoBullets2017 Pediatric trauma | Elbow Fractures
o postop care
 admit overnight 24h for IV antibiotics, observe for compartment syndrome
 see 1 week postop
 see 3 weeks postop with radiographs and remove pins in office
 allow active ROM at that time

Complications
 Cubitus varus
o up to 70% have this complication
 more common than with supracondylar fractures
o cause
 AVN of medial condyle
 malunion (common because of missed diagnosis, or loss of reduction)
o treatment
 lateral closing wedge osteotomy
 Medial condyle AVN
 Loss of motion
o usually no functional limitation
 Growth disturbance
o progressive cubitus varus
o joint irregularities
o angular deformity
o limb-length discrepancy
o treatment
 observe initially, undertake surgery when >5yo
 larger extremity
 child more cooperative

C. Elbow Fractures

1. Supracondylar Fracture - Pediatric


Introduction
 Epidemiology
o incidence
 extension type most common (95-98%)
 flexion type less common (<5%)
o demographics
 occur most commonly in children aged 5 to 7
M=F
 Pathophysiology
o mechanism of injury
 fall on outstretched hand
 Associated injuries
o neuropraxia
 anterior interosseous nerve neurapraxia (branch of median n.)

- 460 -
By Dr, AbdulRahman AbdulNasser Pediatric trauma | Elbow Fractures
 the most common nerve palsy seen with supracondylar humerus fractures
 radial nerve palsy
 second most common neurapraxia (close second)
 ulnar nerve palsy
 seen with flexion-type injury patterns
 nearly all cases of neurapraxia following supracondylar humerus fractures resolve
spontaneously, and therefore, further diagnostic studies are not indicated in the acute setting
o vascular injury (1%)
 rich collateral circulation can maintain circulation despite vascular injury
o ipsilateral distal radius fractures

Anatomy
 Ossification centers of elbow
o age of ossification/appearance and age of fusion are two independent events that must be
differentiated
 e.g., internal (medial epicondyle) apophysis
 ossifies/appears at age 6 years (table below)
 fuses at age ~ 17 years (is the last to fuse)

Years at ossification Years at fusion


Ossification center
(appear on xray) (1) (appear on xray) (1)
Capitellum 1 12
Radius 4 15
Medial epicondyle 6 17
Trochlea 8 12
Olecranon 10 15
Lateral epicondyle 12 12
(1) +/- one year, varies between boys and girl

- 461 -
OrthoBullets2017 Pediatric trauma | Elbow Fractures
Classification
Gartland Classification
(may be extension or flexion type)
 Nondisplaced, beware of subtle medial comminution leading to cubitus varus
 Anterior periosteum detached from anterior humerus by up to 3cm (but not
Type I torn)
 Treated with cast immobilization x 3-4wks, with radiographs at 1 wk, pull pins
at 3 wks
 Displaced, posterior cortex and posterior periosteal hinge intact
 IIA - no rotational deformity/fragment translation
Type II  IIB - has rotational deformity/fragment translation (high risk of coronal/rotational
malalignment)
 Treated with CRPP
 Completely displaced, no cortical contact but has intact posterior periosteal
Type III hinge
 Treated with CRPP
 Complete periosteal disruption with instability in flexion and extension
Type IV*
 Treated with CRPP
 Collapse of medial column, loss of Baumann angle (leads to varus
Medial comminution* malunion/classic gunstock deformity)
in Type II
 Treated with CRPP
 Shear mechanism, oblique orientation, inherently unstable
Flexion type
 Treated with CRPP
*not a part of original Gartland classification
**diagnosed intraoperatively when capitellum is anterior to AHL with elbow flexion and posterior with extension on
lateral XR

Gartland classification Type 2 AP view

Type 1 lateral view Type 1 AP view Type 2 lateral view

- 462 -
By Dr, AbdulRahman AbdulNasser Pediatric trauma | Elbow Fractures

Type 3 lateral view Type 4 Type 4

Presentation
 Symptoms
o pain
o refusal to move the elbow
 Physical exam
o inspection
 gross deformity
 swelling
 bruising
o motion
 limited active elbow motion
o neurovascular
 nerve exam
 AIN neurapraxia
 unable to flex the interphalangeal joint of his thumb and the distal interphalangeal
joint of his index finger (can't make A-OK sign)
 radial nerve neurapraxia
 inability to extend wrist or digits may be present due to radial nerve injury
neurapraxia
 vascular exam
 vascular insufficiency at presentation is present in 5 -17%
 defined as cold, pale, and pulseless hand
 a warm, pink, pulseless hand does not qualify as vascular insufficiency
 treat with immediate reduction and pinning in OR. Attempted closed reduction in ER first
(see treatment below)

Posterior fat pad sign anterior humeral line Baumann angle

- 463 -
OrthoBullets2017 Pediatric trauma | Elbow Fractures
Imaging
 Radiographs
o recommended views
 AP and lateral x-ray of the elbow
o findings
 posterior fat pad sign
 lucency along the posterior distal humerus and olecranon fossa is highly suggestive of
occult fracture around the elbow
o measurement
 displacement of the anterior humeral line
 anterior humeral line should intersect the middle third of the capitellum
 capitellum moves posteriorly to this reference line in extension type fracture
 alteration of Baumann angle
 Baumann's angle is created by drawing a line parallel to the longitudinal axis of the
humeral shaft and a line along the lateral condylar physis as viewed on the AP image
 normal is 70-75°, but best judge is a comparison of the contralateral side
 deviation of >5° indicates coronal plane deformity and should not be accepted

Treatment of Perfused Hand


 Nonoperative
o long arm posterior splint then long arm casting with less than 90° of elbow flexion
 indications
 Type I (non-displaced) fractures
 Type II fractures that meet the following criteria
 anterior humeral line intersects the anterior half of capitellum
 minimal swelling present
 no medial comminution
 technique
 typically used for 3-4 weeks and maybe followed for additional time in removable long
arm posterior splint
 Operative
o emergent closed reduction and percutanous pinning (CRPP)
 timing
 surgical emergency, take patient from ER directly to OR
 indications ("red flag" warning signs)
 dysvascular hand
 see treatment of pulseless hand below
 neurological defect
 severe elbow swelling
 "brachialis sign"
 ecchymosis, dimpling/puckering, palpable subcutaneous bone fragment
 indicates proximal fragment buttonholed through brachialis
 implies more serious injury, higher likelihood of arterial injury, significant swelling,
more difficult closed redution
 "floating elbow"
 ipsilateral supracondylar humerus and forearm /wrist fractures necessitate immediate
pinning of both fractures to decrease risk of compartment syndrome
- 464 -
By Dr, AbdulRahman AbdulNasser Pediatric trauma | Elbow Fractures
o urgent closed reduction and percutanous pinning (CRPP)
 indications
 type II (esp IIB) and III supracondylar fractures
 flexion type
 type II with De Boeck pattern (medial column collapse)
 timing
 if no concern for vascular injury operate when surgical team available (surgical "urgency"
rather than "emergency")
 splint in 30-40° elbow flexion, admit overnight for observation
o open reduction with percutaneous pinning
 indications
 unacceptable closed reduction
 more frequently required with flexion type fractures (than extension type)
 when vascular exploration
needed
 open fracture
 technique
 a variety of approaches include
anterior, medial or lateral

Treatment of Pulseless Hand


 Operative
o emergent CRPP followed by serial
vascular exams
 indications
 pulseless BUT perfused pink hand on presentation that is successfully reduced without a
gap
 technique
 check vascular status after reduction
 if evidence of good distal perfusion (warm, pink hand with good capillary refill, biphasic
doppler pulses) admit for 48 hours of observation
 arteriography is typically not indicated
o emergent CRPP followed immediately by vascular exploration
 indications
 pulseless white hand (pale, cool, no doppler) that is successfully reduced without a gap
 pulsatile and perfused hand that looses pulses following CRPP
o emergent open reduction and vascular exploration
 indications
 pulseless white OR pink hand that is unable to be reduced or their remains a gap
 gap might represent entrapped vascular structure

- 465 -
OrthoBullets2017 Pediatric trauma | Elbow Fractures

Techniques
 Closed reduction and percutanous pinning (CRPP)
o fixation
 closed reduction (extension-type)
 posteromedial fragments: forearm pronated with hyperflexion
 posterolateral fragments: forearm supinated with hyperflexion
 2 lateral pins
 usually sufficient in most cases
 test stability under fluoroscopy
 technical pearls
 maximize separation of pins at fracture site
 engage both medial & lateral columns proximal to fracture
 engage sufficient bone in proximal & distal segments
 low threshold for 3rd lateral pin if concern about stability with 1st 2 pins
 for difficult cases (type IV free floating segment)
 place 2 parallel lateral pins initially in distal fragment as joysticks
 rotate fluoro (not the patients arm) to obtain lateral image
 after adequate reduction, advance distal pins into proximal fragment
 add a 3rd pin
 3 lateral pins
 biomechanically stronger in bending and torsion than 2-pin constructs
 indications (where 2 lateral pins are insufficient)
 comminution
 type IV (free floating distal fragment)
 no significant difference in stability between three lateral pins and crossed pins
 risk of iatrogenic nerve injury from a medial pin makes three lateral pins the construct
of choice
- 466 -
By Dr, AbdulRahman AbdulNasser Pediatric trauma | Elbow Fractures
 crossed pins
 biomechanically strongest to torsional stress
 higher risk of ulnar nerve injury (3-8%)
 highest risk if placed with elbow in hyperflexion as ulnar nerve subluxates anteriorly
over medial epicondyle in some children
 reduce risk of ulnar nerve injury by
 placing medial pin with elbow in extension
 use small medial incision (rather than percutaneous pinning)
 remove pins postop at 3 weeks
 these techniques reduce complication risk to equal to lateral-only pins

Complications
 Pin migration
o most common complication (~2%)
 Infection
o occurs in 1-2.4%
o typically superficial and treated with oral antibiotics
 Cubitus valgus
o caused by fracture malunion
o can lead to tardy ulnar nerve palsy
 Cubitus varus (gunstock deformity)
o caused by fracture varus malunion, especially in De Boeck medial comminution pattern
o usually a cosmetic issue with little functional limitations
 Recurvatum
o common with non-operative treatment of Type II and Type III fractures
 Nerve palsy from injury
o usually resolve
o extension type fractures
 neuropraxia in 11%
 most commonly AIN (34% of extension-type fracture nerve injuries)
 mechanism = tenting of nerve on fracture, or entrapment in fracture site
o flexion type fractures
 neuropraxia in 17%
 most commonly cause ulnar neuropraxia (91% of flexion-type fracture nerve injuries)
 Vascular Injury
o radial pulse absent on initial presentation in 7-12%
o pulseless hand after closed reduction and pinning (3-4%)
o decision to explore is based on quality of extremity perfusion, rather than absence of pulse
o arteriography is NOT indicated in isolated injuries
 Volkmann ischemic contracture
o rare, but dreaded complication
o result of brachial artery compression with treatment utilizing elbow hyperflexion casting than
true arterial injury
 increase in deep volar forearm compartment pressures and loss of radial pulse with elbow
flexed >90°
o rarely seen with CRPP and postoperative immobilization in less than 90°

- 467 -
OrthoBullets2017 Pediatric trauma | Elbow Fractures
 Postoperative stiffness
o rare after casting or after pinning procedures
 remove pins and allow gentle ROM at 3 weeks postop
o resolves by 6 months
o literature does not support the use of physical therapy

2. Medial Epicondylar Fractures - Pediatric


Introduction
 Fracture of the medial epicondylar apophysis on the posterormedial aspect of the elbow
o medial epicondyle is avulsed via tension created by structures attached to it
 include flexor-pronator mass and MCL
o fracture occurs secondary to excess valgus stress at elbow
 Mechanism
o fall on outstretched arm
 most common
o elbow dislocation
 associated with elbow dislocations in up to 50%
 most spontaneously reduce but fragment may be incarcerated in joint
o traumatic avulsion
 usually occurs in overhead throwing athletes
 Epidemiology
o usually occur in children between the ages of 9 and 14 years

Anatomy
 Common flexor wad muscles of medial epicondyle include
o pronator teres
o flexor carpi radialis
o palmaris longus
o flexor digitorum superficialis
o flexor carpi ulnaris

Presentation
 Symptoms
o medial elbow pain
 Physical exam
o tenderness over medial epicondyle
o valgus instability

Imaging
 Radiographs
o recommended views
 AP and lateral of elbow
 axial view is most accurate as medial epicondyle is located on the posteromedial aspect of the
distal humerus
 especially because fragment displaces anteriorly
 internal oblique views helpful

- 468 -
By Dr, AbdulRahman AbdulNasser Pediatric trauma | Elbow Fractures

 3D CT
o most accurate but radiation dose is 200x that of plain film

Treatment
 Nonoperative
o brief immobilization (1 to 2 weeks) in a long arm cast or splint
 indications
 < 5mm displacement usually treated non-operatively, 5-15 mm remains controversial
 often heal with fibrous union
 fibrous union of the fragment is not associated with significant symptoms or diminished
function
 Operative
o open reduction internal fixation
 indications
 absolute
 displaced fx with entrapment of medial epicondyle fragment in joint
 if medial condyle is involved (articular surface)
 relative
 ulnar nerve dysfunction
 > 5-15mm displacement
 displacement in high level athletes

Techniques
 Open Reduction Internal Fixation
o approach
 medial approach to elbow
 incision is made directly over medial epicondyle
 brachialis / triceps interval, ulnar nerve at risk
 patient supine on table with arm abducted to 90 degrees and externally rotated
o technique
 identify ulnar nerve and protect
 reduce fracture
 use cannulated screw for fixation
 K-wires indicated for smaller fragments or in younger children

Complications
 Nerve injury
o ulnar nerve can become entrapped
o neuropathy with dislocatoin which usually resolves
 Missed incarceration
o missed incarceration of fragment in elbow joint
 Elbow stiffness
o loss of elbow extension, avoid prolonged immobilization
 Non-union

- 469 -
OrthoBullets2017 Pediatric trauma | Elbow Fractures

3. Lateral Condyle Fracture - Pediatric


Introduction
 Fractures involving the lateral condyle of the humerus that have higher risk of nonunion, malunion,
AVN than other pediatric elbow fractures
 Epidemiology
o incidence
 17% of all distal humerus fractures in the pediatric population
 2nd most common elbow fracture (after supracondylar)
o demographics
 typically occurs in patients aged 6 years
o location
 most commonly are Salter-Harris IV fracture patterns of the lateral condyle
 Pathophysiology
o mechanism of injury
 pull-off theory
 avulsion fracture of the lateral condyle that results from the pull of the common extensor
musculature
 push-off theory
 fall onto an outstretched hand causes impaction of the radial head into the lateral condyle
causing fracture
 Prognosis
o outcomes have historically been worse than supracondylar fractures
 articular nature, missed diagnosis, and higher risk of malunion/nonunion

Classification
Milch Classification-controversial
Type I Fracture line is lateral to trochlear groove (less common, elbow is stable as fracture
does NOT enter trochlear groove)
Type II Fracture line into trochlear groove (more common, more unstable)

Milch classification Milch type I Milch type II

- 470 -
By Dr, AbdulRahman AbdulNasser Pediatric trauma | Elbow Fractures
Fracture Displacement Classification-Jakob et al.
Type 1 <2mm, indicating intact cartilaginous hinge Casting
Type 2 >2 mm < 4 displacement, intact articular cartilage on Open/closed reduction
arthrogram and fixation
Type 3 >2-4 mm, articular surface disrupted on arthrogram Open reduction and
fixation

Presentation
 History
o fall onto an outstetched hand
 Symptoms
o lateral elbow pain
o mild swelling
 Physical exam
o inspection
 exam may lack the obvious deformity often seen with supracondylar fractures
 swelling and tenderness are usually limited to the lateral side
o motion
 may have increased pain with resisted wrist extension/flexion
 may feel crepitus at the fracture site

Imaging
 Radiographs
o recommended views
 AP, lateral, and oblique views of elbow
 internal oblique view most accurately shows fracture displacement because fracture is
posterolateral
o optional views
 contralateral elbow for comparison when ossification is not yet complete
 routine elbow stress views are not recommended due to risk of fracture displacement
o findings
 fracture fragment most often lies posterolateral which is best seen on internal oblique views

- 471 -
OrthoBullets2017 Pediatric trauma | Elbow Fractures
 Arthrogram
o indications
 to assess cartilage surface when there is incomplete/absent epiphyseal ossification
 allows dynamic assessment
 CT scan
o indication
 improved ability to assess the fracture pattern in all planes
o findings
 CT has limited ability to evaluate the integrity of articular cartilage
 may require sedation to perform the test
 MRI
o indication
 provides the ability to assess the cartilaginous integrity of the trochlea
o expensive
o require GA/sedation to perform the test
o arthrograms preferred to MRI

Differential
Pediatric Elbow Injury Frequency
Fracture Type % elbow injuries Peak Age Requires OR
Supracondylar fractures 41% 7 majority
Radial Head subluxation 28% 3 rare
Lateral condylar physeal fractures 11% 6 majority
Medial epicondylar apophyseal fracture 8% 11 minority
Radial Head and Neck fractures 5% 10 minority
Elbow dislocations 5% 13 rare
Medial condylar physeal fractures 1% 10 rare

Treatment
 Nonoperative
o long arm casting x 6wks
 indications
 only if < 2 mm displacement (cartilaginous hinge most likely intact) (30-70% are
nondisplaced)
 sub-acute presentation (>4 weeks)
 technique
 cast with elbow at 90 degrees and forearm supination
 weekly follow up and radiographs every 3-7 days x first 3 weeks
 total length of casting 6 weeks
 Operative
o CRPP + 3-6 wks in above elbow cast
 indications
 somewhat controversial, but Weiss et al suggest fractures with < 4 mm of displacement
have intact articular cartilage and can be treated with CRPP
 technique
 closed reduction performed by providing a varus elbow force and pushing the fragment
anteromedial
 divergent pin configuration most stable

- 472 -
By Dr, AbdulRahman AbdulNasser Pediatric trauma | Elbow Fractures
 third pin may be used in transverse plane to prevent fragment derotation
 arthrogram used to confirm joint congruity
o open reduction and fixation + 3-6 wks in above elbow cast
 open reduction (rather than closed) necessary to align joint surface
 indications
 if > 2-4mm of displacement
 any joint incongruity
 fracture non-union
 technique
 interval between the triceps and brachioradialis
 avoid dissection of posterior aspect of lateral condyle (source of vascularization
 implants
 most fractures can be fixed with 2 percutaneous pins (3 if comminuted) in parallel or
divergent fashion
 single screw for large fragments or non-union ± bone grafting
o supracondylar osteotomy
 indications
 deformity correction in late presenting cubitus valgus

Complications
 Stiffness
o most common complication
 Nonunion
o higher rate of nonunion than other elbow fractures
o normal radiographic union of lateral condyle fracture is 6wks
o risk
 nonsurgical management
o mechanism
 constant pull by extensors
 intra-articular (synovial fluid impede fracture healing)
 poor metaphyseal circulation to distal fragment
o prevent nonunion by
 preserving soft tissue attachments to lateral condyle
 stable internal fixation
o treatment
 ORIF + bone grafting
 AVN
o occurs 1-3 years after fracture
o posterior dissection can result in lateral condyle osteonecrosis (may also occur in the trochlea)
 Malunion
o caused from delay in diagnosis and improper treatment
o 20% cubitus varus in nondisplaced/minimally displaced fractures
 traumatic inflammation leads to lateral overgrowth (see spurring below)
o 10% cubitus valgus ± tardy ulnar nerve palsy
 because of lateral physeal arrest as fracture is Salter Harris IV
o fishtail deformity

- 473 -
OrthoBullets2017 Pediatric trauma | Elbow Fractures
 area between medial ossification center and lateral condyle ossification center resorbs or fails
to develop
 does NOT predispose to arthritis
 may predispose to further fracture
o treatment
 supracondylar osteotomy
 Tardy ulnar nerve palsy
o slow, progressive ulnar nerve palsy caused by stretch in cubitus valgus
o usually late finding, presenting many years after initial fracture
 Lateral overgrowth/prominence (spurring)
o up to 50% regardless of treatment, families should be counseled in advance ‎ III:11 Fishtail deformity
V
o lateral periosteal alignment will prevent this from occurring
o spurring is correlated with greater initial fracture displacement
 Growth arrest with or without angular deformity
 Unsatisfactory appearance of surgical scar
 Late elbow presentation or deformity
o cubitus varus most common in nondisplaced and minimally displaced fractures
o cubital valgus less common, but more likely with significant deformities that cause physeal
arrest
o controversy whether to treat subacute fractures (week 3-12) nonoperatively or surgically
o most deformities can be corrected after skeletal maturation with a supracondylar osteotomy

4. Olecranon Fractures - Pediatric


Introduction
 Epidemiology
o incidence
 uncommon fracture in children
 in the US, accounts for <5% of all pediatric fractures
 peak age between 5-10 year old
 Pathophysiology
o mechanism
 fall onto outstretched hand with
 elbow in flexion
 will lead to triceps tensioning causing an olecranon avulsion fracture
 elbow in extension
 can lead to varus/valgus bending forces through the olecranon causing oblique
fracture lines
 direct trauma (least common)
o location
 metaphyseal (most common)
 physeal
 epiphyseal (apophyseal)
 intra-articular
 extra-articular

- 474 -
By Dr, AbdulRahman AbdulNasser Pediatric trauma | Elbow Fractures
 Associated conditions
o osteogenesis imperfecta
 olecranon fractures are highly suspicious for osteogenesis imperfecta

Anatomy
 Ossification centers of elbow
o age of ossification/appearance and age of fusion are two independent events that must be
differentiated
 olecranon apophysis
 ossifies/appears at age 9 years
 fuses at age ~ 15 -17 years

Years at ossification (appear on Years at fusion (appear on


Ossification center
xray) (1) xray) (1)
Capitellum 1 12-14*
Radius 3 14-16
Internal (medial) epicondyle 5 16-18
Trochlea 7 12-14*
Olecranon 9 15-17
External (lateral) epicondyle 11 12-14*
(1) +/- one year, varies between boys and girl.
C-R-I-T-O-E to remember age of ossification.
CTE-R-O-I to remember age of fusion (capitellum, trochlea and external (lateral) epicondyle fuse together at puberty.
Together they fuse to the distal humerus between the ages of 14-16 years old)

 Olecranon ossification
o fusion of the epiphysis to the metaphysis of the olecranon occurs from anterior to posterior
o average age of closure is between the ages of 15-17 years old
o partial closure may be mistaken for olecranon fracture

- 475 -
OrthoBullets2017 Pediatric trauma | Elbow Fractures
Presentation
 History
o acute fall onto outstretched hand or direct elbow trauma
 Symptoms
o pain
o swelling of posterior elbow
o inability to extend elbow
 Physical exam
o inspection
 swelling and deformity
 contusion or abrasion over elbow may be suggestive of direct trauma
o palpation
 crepitus
 defect detected between fracture fragments
 gapping may suggest a disruption in the posterior periosteum, which makes the fracture more
unstable
o movement
 lack of active elbow extension

Imaging
 Radiographs
o recommended views
 AP and lateral xrays that should always be obtained on evaluation
o findings
 fracture configuration (transverse, oblique, longitudinal)
 intra-articular displacement
 associated fracture (radial neck, medial/lateral condyle, distal radius, etc.)

Treatment
 Nonoperative
o NSAIDS, rest, immobilization with avoidance of elbow resistance exercises
 indications
 partial stress fractures
 outcomes
 monitor until there is clinical improvement
 convert to casting if needed
o long arm splint or casting
 indications
 minimally displaced fractures
 integrity of posterior olecranon periosteum maintained
 duration
 3-4 weeks total
 repeat imaging at 7-10 days to ensure no significant displacement
 Operative
o ORIF
 I ndications
 displaced fractures

- 476 -
By Dr, AbdulRahman AbdulNasser Pediatric trauma | Elbow Fractures
 techniques
 tension band wiring
 AO technique with axial K-wires
 congruent articular surface
 consider early range of motion post-operatively
 tension band suturing
 use absorbable sutures (e.g. Number 1 polydioxanone (PDS) suture)
 may combine with oblique cortical lag screw with PDS with metaphyseal fractures
 plate and screws
 considered with comminuted fractures with partially fused ossification centers

Complications
 Nonunion
 Delayed Union
 Compartment syndrome
 Ulnar nerve neurapraxia due to pseudarthrosis with inadequate fixation
 Loss of Reduction
 Elbow stiffness

5. Radial Head and Neck Fractures - Pediatric


Introduction
 In children, fractures of the proximal end of the radius typically involve the physis or radial neck
(metaphysis)
o most cases are Salter-Harris type II fractures
o radial head involvement is rare
 Epidemiology
o demographics
 median age is 9-10 years
 no difference in incidence between sexes
 1-5% of all pediatric elbow fractures
 Pathophysiology
o mechanism
 usually associated with a valgus loading
injury of the elbow
 Associated Conditions
o elbow dislocation
o medial epicondyle fracture ‎ III:12 6 ossification centers around the elbow joint
V

Anatomy
 There are 6 ossification centers around the elbow joint
o age of ossification is variable but occurs in the following order (C-R-I-T-O-E) at an average age
of (years)
 Capitellum (1 yr.)
 Radius (3 yr.)
 Internal or medial epicondyle (5 yr.)
 Trochlea (7 yr.)

- 477 -
OrthoBullets2017 Pediatric trauma | Elbow Fractures
 Olecranon (9 yr.)
 External or lateral epicondyle (11 yr.)
 Ossification center of radial head appears between and 3 and 5 years of age
o may be bipartite
o radial head fuses with radial shaft between ages of 16 and 18 years

Classification

Chambers Classification
Group 1: Primary displacement of radial Valgus Injury
head (most common) A: Salter-Harris I or II
B: Salter-Harris IV
C: metaphyseal

Elbow Dislocation
D: reduction injury
E: dislocation injury
Group 2: Primary displacement of radial neck Monteggia variant
Group 3: Stress injury Osteochondritis dissecans

Group 3: Stress injury

Group1 salter harris type 2 radial head Group2 Monteggia variant

- 478 -
By Dr, AbdulRahman AbdulNasser Pediatric trauma | Elbow Fractures
Presentation
 Symptoms
o elbow pain
o refusal to move
 Physical exam
o inspection
 lateral swelling
o motion
 pain exacerbated by motion, especially supination and
pronation. ‎VIII:13 AP and lateral of the elbow, radial
o must have high suspicion for forearm compartment syndrome head intersect capitellum in both views.
o pain may be referred to the wrist

Imaging
 Radiographs
o recommended views
 AP and lateral of the elbow
 radiocapitellar (Greenspan) view
 oblique lateral performed by placing the arm on the radiographic table with the elbow
flexed 90 degrees and the thumb pointing upward
 The beam is directed 45 degrees proximally
o findings
 nondisplaced fractures may be difficult to visualize
 look for fat pads signs
 a portion of the radial neck is extra-articular and
therefore an effusion and fat pads signs may be
absent.
Treatment
 Nonoperative
o immobilization ± closed reduction
 indications
 most fractures can be treated closed
 if < 30° angulation immobilize without closed reduction
 if >30° angulation perform closed reduction and immobilize if angulation reduced to <
30°
 followup
 begin early ROM at 3-7 days to prevent stiffness
 Operative
o operative percutaneous reduction
 indications
 > 30° of residual angulation
 3-4 mm of translation
 < 45° of pronation and supination
 outcomes
 improved outcomes with younger patients, lesser degrees of angulation, and isolated
radial neck fractures
- 479 -
OrthoBullets2017 Pediatric trauma | Elbow Fractures
o open reduction
 indications
 fracture that cannot be adequately reduced with closed or percutaneous methods
 outcomes
 open reduction has been associated with a greater loss of motion, increased rates of
osteonecrosis and synostosis compared with closed reduction.

K-wire joystick technique Metaizeau technique

Techniques
 Closed reduction
o reduction techniques
 Patterson maneuver
 hold the elbow in extension and apply distal traction with the forearm supinated and pull
the forearm into varus while applying direct pressure over the radial head
 Israeli technique
 pronate the supinated forearm while the elbow is flexed to 90° and direct pressure
stabilizes the radial head
 elastic bandage technique
 tight application of an elastic bandage beginning at the wrist continuing over the forearm
and elbow may lead to spontaneous reduction
 Closed Reduction and Percutaneous Pinning
o reduction technique
 K-wire joystick technique
 Metaizeau technique
 involves retrograde insertion of a pin/nail across the fracture site
 fracture is reduced by rotating the pin/nail
 Open reduction
o approach
 performed with lateral approach (Kocher interval) to radiocapitellar joint
 avoid deep branch of radial nerve
o fixation
 avoid transcapitellar pins
 internal fixation only used for fractures that are grossly unstable

- 480 -
By Dr, AbdulRahman AbdulNasser Pediatric trauma | Elbow Fractures
Complications
 Decreased range of motion
o loss of pronation more common than supination
 Radial head overgrowth
o 20-40% of fractures
o usually does not affect function
 Osteonecrosis
o 10-20% of fractures
o up to 70% of cases occur with open reduction
 Synostosis
o most serious complication
o occurs in cases of open reduction with extensive dissection or delayed treatment

6. Nursemaid's Elbow
Introduction
 Also known as subluxation of radial head
 Epidemiology
o most common in children from 2 to 5 years of age.
 Pathophysiology
o mechanism
 caused by longitudinal traction applied to an extended arm
o pathoanatomy
 caused by subluxation of the radial head and interposition of the annular (orbicular) ligament
into the radiocapitellar joint.
Presentation
 Symptoms
o a child with radial head subluxation tends to hold the elbow in slight flexion and the forearm
pronated.
 Physical Exam
o pain and tenderness localized to the lateral aspect of the elbow.

Imaging
 Radiographs
o recommended views
 not routinely indicated in presence of
classic history and physical examination
o findings
 radiographs are normal

- 481 -
OrthoBullets2017 Pediatric trauma | Elbow Fractures
Differential
Pediatric Elbow Injury Frequency & Treatment
Fracture Type % elbow injuries Peak Age Requires OR
Supracondylar fractures 41% 7 majority
Radial Head subluxation 28% 3 rare
Lateral condylar physeal fractures 11% 6 majority
Medial epicondylar apophyseal
fracture 8% 11 minority
Radial Head and Neck fractures 5% 10 minority
Elbow dislocations 5% 13 rare
Medial condylar physeal fractures 1% 10 rare

Treatment
 Nonoperative
o closed reduction
 indications
 acute cases
 Operative
o open reduction
 indications
 chronic injuries
‎VIII:14 reduction technique: supination > flexion > hyperpronation
Techniques
 Closed reduction of radial head subluxation
o reduction techniques
 reduction is performed by manually supinating the forearm and flexing the elbow past 90
degrees of flexion.
 while holding the arm supinated the elbow is then maximally flexed
 during this maneuver the physician’s thumb applies pressure over the radial head and a
palpable click is often heard with reduction of the radial head.
 alternative technique includes hyperpronation of the forearm while in the flexed position.
o followup
 immobilization is not necessary and the child may immediately resume use of the arm.
 follow up is only needed if the child does not resume normal use of his arm in the following
weeks.
Complications
 Recurrence
o occurs in 5% to 39% of cases, but generally ceases after 5 years of age.

- 482 -
By Dr, AbdulRahman AbdulNasser Pediatric trauma | Elbow Fractures

7. Elbow Dislocation - Pediatric


Introduction
 Epidemiology
o incidence
 3-6% of all pediatric elbow injuries
 high index of suspicion for child abuse
o demographics
 M:F = 3:1
 most common in 10-15 year olds
 very rare in younger children < 3 years old
o locations
 transphyseal fracture more common
 Pathophysiology
o mechanism of injury : fall onto an outstretched hand
o pathoanatomy
 classically thought to be a combination of
 supinated forearm and ‎VIII:15 transphyseal fracture (A,B)
 elbow extended or partially flexed (posterior dislocation) and normal elbow (C,D)

 relatively small coronoid process in children cannot resist proximal and posterior
displacement of ulna
 Associated conditions
o traumatic
 child abuse
 high index of suspicion for child abuse
 avulsion of the medial epicondyle
 is the most common associated fracture
 incarcerated intra-articular bone fragment may block reduction
 fractures of proximal radius, olecranon and coronoid process
 neurovascular injury
 brachial artery and median nerve
 may be stretched over displaced proximal fragment
 ulnar nerve
 at risk with associated medial epicondyle avulsions
 most common neuropathy
o congenital
 dislocation of radial head
‎VIII:16 avulsion of the
Classification medial epicondyle
 Anatomic classification
o based on the position of the proximal radio-ulnar joint in relation to the distal humerus
o includes
 posterior or posterolateral (most common)
 anterior (rare)
 medial
 lateral

- 483 -
OrthoBullets2017 Pediatric trauma | Elbow Fractures
Presentation
 Symptoms
o painful and swollen elbow
o attempts at motion are painful and restricted
 Physical exam
o inspection
 elbow held in flexion
 forearm appears to be shortened from the anterior and posterior view
o palpation
 distal humerus creates a fullness within the antecubital fossa
o essential to perform neurovascular examination

Imaging
 Radiographs
o required views
 AP and lateral radiograph of elbow
 comparison radiographs of the contralateral elbow may be helpful
o findings
 look for fractures of medial epidcondyle, coronoid, proximal radius
 high index of suspicion for transphyseal (distal humerus epiphyseal separation) fractures in
very young children (<3 years old)
Treatment
 Nonoperative
o closed reduction, brief immobilization with early range of motion
 indications
 dislocation that remains stable following reduction
 indicated in the majority of cases
 reduction technique (see below)
 brief immobilization
 immobilization should be minimized to 1- 2 weeks to minimize risk of stiffness
 early therapy
 encourage early active range of motion
 Operative
o open reduction
 indications
 open dislocation
 incarcerated medial epicondyle or coronoid process in the joint
 failure to obtain or maintain an adequate closed reduction
 significant joint instability

Technique
 Closed reduction technique
o closed reduction performed using gradual traction and flexion for posterior dislocations
o post-reduction films should be reviewed to rule out presence of entrapped bone fragment

- 484 -
By Dr, AbdulRahman AbdulNasser Pediatric trauma | Elbow Fractures
 Open reduction
o approach
 depends on reason for blocked reduction
 elbow medial approach
 indicated if medial epicondyle avulsion with incarcerated fragment is blocking
reduction
Complications
 Stiffness
o most common
 due to prolonged immobilization
 Heterotopic ossification
 Neurologic injuries
o usually transient
o ulnar nerve most commonly affected
 Loss of terminal flexion or extension
 Chronic instability (recurrent dislocations)
‎VIII:17 incarcerated medial epicondyle in the joint

- 485 -
OrthoBullets2017 Pediatric trauma | Forearm Fractures

D. Forearm Fractures

1. Both Bone Forearm Fracture - Pediatric


Introduction
 One of the most common pediatric fractures
 Mechanism
o usually occurs from fall from a height

Anatomy
 Normal rotational alignment
o relationship of bicipital tuberosity and radial styloid should be 180 degrees from each other on
the AP radiograph
o relationship of coronoid process and ulnar styloid should be 180 degrees from each other on the
lateral radiograph
Classification
 Greenstick fractures
o are incomplete fractures
o can be described as apex volar or apex dorsal
 Complete fractures
o are categorized the same as adults

Presentation
 Symptoms
o forearm pain and deformity
 Physical exam
o swelling and focal tenderness
o should assess for neurovascular injury
o should rule out compartment syndrome
o open fracture
 can be subtle poke-holes, and can often be missed if not evaluated
by an orthopaedic surgeon
Imaging
 Radiographs
o help to describe apex dorsal vs apex volar injuries
o can help judge forearm rotation deformity based on relationship of bicipital
tuberosity and radial styloid which are 180 degees apart on the AP
view
o ulnar styloid and coronoid are 180 degrees apart on the lateral view

- 486 -
By Dr, AbdulRahman AbdulNasser Pediatric trauma | Forearm Fractures
Treatment
Table of Acceptable Reduction (Tolerances)
Bayonet
Age Angulation (°) Malrotation (°)
Apposition
0-9 years <15 <45 Yes, if <1cm short
≥10y, mid to distal shaft <15 <30 No
≥10y, proximal shaft <10 0 No
Approaching skeletal maturity (<2y No
0 0
growth remaining)

 Nonoperative
o closed reduction and immobilization
 indications
 most pediatric forearm fractures can be treated without surgery
 greenstick injuries
 bayonet apposition ok if <10 years
 followup ‎VIII:18 example of
Bayonet Apposition
 weekly radiographs for first 3-4 weeks to monitor reduction
 casting for 6-12 weeks total
 Short arm cast vs above elbow cast
 short arm for distal 1/3 BBFA
 above elbow immobilization for any fracture proximal to distal 1/3
 Operative
o percutaneous vs open reduction and nancy nailing
 absolute indications
 unacceptable alignment following closed reduction
 angulation >15°, rotation >45° in children <10y
 angulation >10°, rotation >30° in children >10y
 bayonet apposition in children older than 10 years
 both bone forearm fractures in children> 13
 relative indications
 highly displaced fractures
 technique
 allows smaller dissection and advantage of a load-sharing device allowing rapid healing
 fixation of one bone often sufficient stability
 considerations
 shorter surgical time than ORIF
 less blood loss than ORIF
 equal union rates, radial bow and rotation as ORIF
o open reduction and internal fixation
 absolute indications
 unacceptable alignment following closed reduction
 open fractures
 refractures
 angulation >15° and rotation >45° in children <10y
 angulation >10° and rotation >30° in children >10y
- 487 -
OrthoBullets2017 Pediatric trauma | Forearm Fractures
 bayonet apposition in children older than 10 years
 both bone forearm fractures in children> 13
 relative indications
 highly displaced fractures
 technique
 same technique as an adult

Techniques
 Closed Reduction
o steady three point bending of immobilization depending on fracture type
 apex volar fractures (supination injuries)
 may be treated and reduced by forearm pronation
 apex dorsal fractures (pronation injuries)
 may be treated and reduced by forearm supination
o greenstick both bone fractures
 most pediatric greenstick both bone fractures can be temporarily reduced by placing the palm
in the direction of the deformity (pronate arm for supination injury with apex-volar
angulation of fracture)
 Casting
o usually long arm cast x 6-8wks, possible conversion to short arm cast after 4wks depending on
fracture type and healing response
o no increased risk of loss of reduction with short arm vs. long arm casting
o loss of reduction is associated with increasing cast index (sagittal width/coronal width) >0.8

Complications
 Refracture
o occurs in 5-10% following both bone fractures
o is an indication for an ORIF
 Malunion
o loss of pronation and supination is common but mild
 Compartment syndrome
o may occur due to high energy injuries
o may occur due to multiple attempts at reduction and rod passage
 if unsuccessful nail passage after 2-3 attempts, open the fracture site to visualize rod passage

2. Distal Radius Fractures - Pediatric


Introduction
 Epidemiology
o incidence
 common - forearm fractures in total account for approximately 40% of all pediatric long bone
fractures
 distal radius (and ulna) is the most common site of pediatric forearm fractures.
 male > female
o demographics
 most common during metaphyseal growth spurt

- 488 -
By Dr, AbdulRahman AbdulNasser Pediatric trauma | Forearm Fractures
 peak incidence occurring from:
 10-12 years of age in girls
 12-14 years of age in boys
 most common fracture in children under 16 years old
 Pathophysiology
o mechanism
 usually fall on an outstretched hand
‎VIII:20 Salter-Harris I
 often during sports or play
o remodeling
 remodeling greatest closer to physis and in plane of joint
(wrist) motion
 sagittal plane (flexion/extension)

Anatomy
 Distal radius physis
o contributes 75% growth of the radius ‎VIII:21 Salter-Harris II
o contributes 40% of entire upper extremity
o growth at a rate of ~ 5.25mm per year

Classification
 Relation to distal physis
o Physeal considerations
o Salter-Harris I
o Salter-Harris II
o Salter-Harris III
o Salter-Harris IV ‎VIII:22 Salter-
o Salter-Harris V Harris III
 Metaphysis (distal) (62%)
o complete (Distal Radius fracture)
 apex volar (Colles' fracture)
 apex dorsal (Smith's fracture)
o incomplete (Torus/Buckle fracture)
 typically unicortical
 Diaphysis (20%)
o both bone forearm fracture
o isolated radial shaft fracture ‎VIII:23 Buckle
fracture
o isolated ulnar shaft fracture
o plastic deformation
 incomplete fracture with deforming force resulting in shape change of bone without clear
fracture line
 thought to be due to a large number of microfractures resulting from a relatively lower force
over longer time compared to mechanism for complete fractures
o greenstick fracture
 incomplete fracture resulting from failure along tension (convex) side
 typically plastic deformation occurs along compression side

- 489 -
OrthoBullets2017 Pediatric trauma | Forearm Fractures
 Fracture with dislocation / associated injuries
o Monteggia fracture
 ulnar shaft fracture with radiocapitellar dislocation
o Galeazzi fracture
 radius fracture (typically distal 1/3) with associated DRUJ
injury, often dislocation
Presentation
 History
o wide range of mechanism for children, often fall during play or
other activity ‎VIII:24 Galeazzi fracture
o rule out child abuse
 mechanism or history appears inconsistent with injury
 multiple injuries, especially different ages
 child's affect
 grip marks/ecchymosis
 Symptoms
o pain, swelling, and deformity
 Physical exam
o gross deformity may or may not be present ‎VIII:25 Monteggia fracture
o ecchymosis and swelling
o inspect for puncture wounds suggesting open fracture
o although uncommon, compartment syndrome and neurovascular injury should be evaluated for
in all forearm fractures.
Imaging
 Radiographs
o recommended views
 AP and lateral of wrist
 AP and lateral of forearm
 AP and lateral of elbow
o findings
 in addition to fracture must evaluate for associated injuries
 scapholunate joint
 DRUJ
 ulnar styloid
 elbow injuries
 CT
o indications
 useful characterize fracture if intra-articular
 however use sparingly in children given concerns regarding increased longitudinal effects of
radiation

- 490 -
By Dr, AbdulRahman AbdulNasser Pediatric trauma | Forearm Fractures
Treatment
"Classically" Acceptable Angulation for Closed Reduction in Pediatric Forearm Radius Fractures
(controversial with ongoing discussion)
Shaft / Both bone fx Distal radius/ulna
Acceptable
Age Acceptable Angulations Malrotation* Dorsal Angulation
Bayonetting
< 9 yrs < 1 cm 15-20° 45° 30 degrees
> 9 yrs. < 1 cm 10° 30° 20 degrees
 Bayonette apposition, or overlapping, of less than 1 cm, does not block rotation and is acceptable in patients less than 10
years of age.
 General guidelines are that deformities in the plane of joint motion are more acceptable, and distal deformity (closer to distal
physis) more acceptable than mid shaft.
 The radius and ulna function as a single rotational unit. Therefore a final angulation of 10 degrees in the diaphysis can block
20-30 degrees of rotation.
 *Rotational deformities do not remodel and are increasingly being considered as not acceptable.

 Nonoperative
o immobilization in short arm cast for 2-3 weeks without reduction
 indications
 greenstick fracture with < 10 deg of angulation
 torus/buckle fracture
 studies ongoing to treat minimally displaced or torus fractures with pre-fabricated
removable wrist splint, no cast
o closed reduction under conscious sedation followed by casting
 indications
 greenstick fracture with > 10-20 degrees of angulation
 Salter-Harris I with unacceptable alignment
 Salter-Harris II with unacceptable alignment
 technique (see below) : reduction technique determined by fracture pattern
 acceptable criteria (see table above)
 acceptable angulations are controversial in the orthopedic community.
 accepted angulation is defined on a case by case basis depending on
 the age of the patient
 location of the fracture
 type of deformity (angulation, rotation, bayonetting).
 outcomes
 short-arm (SAC) vs long-arm casting (LAC)
 good SAC (proper cast index = sagital/coronal widths) considered equal to LAC for
distal radius fractures
 conservative treatment though often utilizes LAC to reduce impact of variable cast
technique/quality
 no increased risk of loss of reduction with (good) short arm vs. long arm casting
 cast index : loss of reduction is associated with increasing cast index
 follow-up
 all forearm fractures serial radiographs should be taken every 1 to 2 weeks initially to
ensure the reduction is maintained.

- 491 -
OrthoBullets2017 Pediatric trauma | Forearm Fractures
 Operative
o closed reduction and percutaneous pinning (CRPP)
 indications
 unstable patterns with loss of reduction in cast
 Salter-Harris I or II fractures in the setting of NV compromise
 CRPP reduces need for tight casting in setting with increased concern for
compartment syndrome
 any fractures unable to reduce in ED but are successfully reduced under anesthesia in the
OR
o open reduction and internal fixation
 indications
 displaced Salter-Harris III and IV fractures of the distal radial physis/epiphysis unable to
be closed reduced
 irreducible fracture closed
 often periosteum or pronator quadratus block to reduction

Treatment Techniques
 Closed Reduction
o timing
 avoid delayed reduction of greater than 1 week after injury
 for physeal injuries, generally limit to one attempt to reduce growth arrest
o reduction technique
 gentle steady pressure for physeal reduction
 for complete metaphyseal fractures re-create deformity to unlock fragments, then use
periosteal sleeve to aid reduction
 traction can be counter-productive due to thick periosteum
 Casting
o usually consists of a long arm cast (conservative approach) for 6 to 8 weeks with the possibility
of conversion to a short arm cast after 2-4 weeks depending on the type of fracture and healing
response.
 may utilize well molded short arm cast with adequate cast index instead of long arm cast
initially
 CRPP
o approach
 avoid dorsal sensory branch of radial nerve, typically with small incision
o reduction
 maintain closed reduction during pinning
o fixation
 radial styloid pins
 usually 1 or 2 radial styloid pins, entry just proximal to physis preferred
 if stability demands transphyseal pin, smooth wires utilized
 for intra-articular fractures, may pin distal to physis transversely across epiphysis
 dorsal pins
 may also utilize dorsal pin, especially to restore volar tilt
 for DRUJ injuries, or severe fractures unable to stabilize with radial pins alone, pin across
ulna and DRUJ

- 492 -
By Dr, AbdulRahman AbdulNasser Pediatric trauma | Forearm Fractures
o postoperative considerations
 followup in clinic for repeat imaging to assess healing and position
 pin removal typically in clinic once callus formation verified on radiograph
 may consider sedation or removal of pins in OR for children unable to tolerate in clinic
 must immobilize radio-ulnar joints in long arm cast if stabilizing DRUJ
 may supplement with external fixator for severe injuries

Complications
 Casting Thermal Injury
o thermal injury may occur if:
 dipping water temperature is > 24C (75F)
 more than 8 layers of plaster are used
 during cast setting, the arm is placed on a pillow. This decreases the dissipation of heat from
the exothermic reaction
 fiberglass is overwrapped over plaster
 Malunion
o most common complication
 Physeal arrest
o from initial injury or repeated/late reduction attempts
o isolated distal radial physeal arrest can lead to ulnocarpal impaction, TFCC injuries, DRUJ injury
o distal ulnar physis most often to arrest
 Ulnocarpal impaction
o from continued growth of ulna after radial arrest
 TFCC injuries
 Neuropathy
o Median nerve most commonly affected

3. Monteggia Fracture - Pediatric


Introduction
 Definition
o radial head dislocation plus
o proximal ulna fracture or
o plastic deformation of the ulna without obvious fracture
 Epidemiology
o 4 to 10 years of age is peak incidence
 Treatment differs from adult Monteggia fractures

Classification
Bado Classification
Type I Apex anterior proximal ulna fracture with anterior dislocation of the radial head
Type II Apex posterior proximal ulna fracture with posterior dislocation of the radial head
Type III Apex lateral proximal ulna fracture with lateral dislocation of the radial head
Type IV Fractures of both the radius and ulna at the same level with an anterior dislocation of
the radial head (1-11% of cases)

- 493 -
OrthoBullets2017 Pediatric trauma | Forearm Fractures

Bado type I

Bado type II Bado type III Bado type IV


Presentation
 Symptoms
o pain, swelling, and deformity about the forearm and elbow
 Physical exam
o must palpate over radial head with all ulna fractures because spontaneous relocation of radial
head is common
o must examine for plastic deformation of the ulna if there is a presumed isolated radial head
dislocation
 isolated radial head dislocations almost never occur in pediatric patients

Imaging
 Radiographs
o obtain elbow radiographs for all forearm fractures to evaluate for
radial head dislocation
 assess radiocapitellar line on every lateral radiograph of the
elbow
 a line down the radial shaft should pass through the center
of the capitellar ossification center
o obtain forearm radiographs for all radial head dislocations

Treatment
 Nonoperative
o closed reduction of ulna and radial head dislocation and long
arm casting
 indications
 Bado Types I-III with
 radial head is stable following reduction
 length stable ulnar fracture pattern
 reduction technique
 reduction technique uses traction
 radial head will reduce spontaneously with reduction of the ulna and restoration of
ulnar length
- 494 -
By Dr, AbdulRahman AbdulNasser Pediatric trauma | Forearm Fractures
 immobilization
 immobilize in 110° of flexion and full supination for Types I and III to tighten
interosseous membrane and relax biceps tendon
 Operative
o plating of ulna + reduction of radial head ± annular ligament repair/reconstruction
 indications
 Bado Types I-III with
 radial head is not stable following reduction
 ulnar length is not stable (unable to maintain ulnar length)
 acute Bado Type IV
 open fractures
 older patients ≥ 10y
 technique
 annular ligament reconstruction almost never required for acute fractures
 open reduction of radial head through a lateral approach if needed
o ulnar osteotomy and annular ligament reconstruction
 indications
 chronic (>2-3 weeks old) Monteggia fractures where radial head still retains concave
structure
 symptomatic individuals (pain, loss of forearm motion, progressive valgus deformity)
who had delayed treatment or missed diagnosis
 technique
 reduce surgically within 6-12 months postinjury
o ORIF similar to adult treatment
 indications : closed physes

Complications
 Neurovascular
o posterior interosseous nerve neurapraxia (10% of acute injuries)
 almost always spontaneously resolves
 Delayed or missed diagnosis
o common when evaluation not performed by an orthopaedic surgeon
o complication rates and severity increase if diagnosis delayed >2-3 weeks

4. Galeazzi Fracture - Pediatric


Introduction
 Characterized by
o fracture of the distal radius at the distal metaphyseal-diaphyseal junction with concominant
disruption of the distal radioulnar joint (DRUJ)
o disruption of the DRUJ in a pediatric patient can consist of a dislocation or, more commonly, a
displaced ulnar physeal injury
 Epidemiology
o incidence
 relatively rare injury, less frequent than in adults
 often missed injury pattern (up to 41%)
o demographics : peak incidence 9 to 13 years old
- 495 -
OrthoBullets2017 Pediatric trauma | Forearm Fractures
 Pathophysiology
o pathoanatomy
 axial loading in combination with extremes of forearm rotation (pronation or supination)
 pronation produces an apex dorsal radial fracture with the distal ulna displaced dorsally
 supination produces an apex volar radial fracture with the distal ulna displaced volarly
 Associated injuries
o nerve injuries are rare

Anatomy
 DRUJ
o osteology
 possesses poor bony conformity in order to allow some translation with rotatory movements
o ligamentous
 ligament structures are critical in stabilizing the radius as it rotates about the ulna during
pronation and supination
 triangular fibrocartilage complex (TFCC) is a critical component to DRUJ stability
o biomechanics
 the joint is most stable at the extremes of rotation

Presentation
 Symptoms
o wrist and forearm pain
o radial deformity
o limitation of wrist motion
o ulnar head prominence or deformity can sometimes be seen
 Physical exam
o pain with movement or palpation of the wrist
o DRUJ instability may be appreciated by local tenderness and instability to testing of the DRUJ
 compare to contralateral side
o careful examination for nerve injury

Imaging
 Radiographs
o required views
 AP and true lateral radiographs
 true lateral radiograph is essential in determining the direction of displacement
o additional views
 contralateral radiographs often helpful for comparison
o findings
 displaced distal radial shaft fracture
 DRUJ disruption
 may be subtle and radiographs must be scrutinized
 additional signs of DRUJ instability include
 ulnar styloid fracture
 widened DRUJ on posteroanterior view
 greater than or equal to 5mm radial shortening

- 496 -
By Dr, AbdulRahman AbdulNasser Pediatric trauma | Forearm Fractures
Treatment
 Nonoperative
o closed reduction with long arm casting
 indications
 indicated as first line of treatment
 in younger patients (higher likelihood of successful nonoperative treatment than in
adults)
 reduction
 requires anatomic reduction of both the radius fracture and the DRUJ
 immobilization : place in above elbow cast in supination
 Operative
o open reduction internal fixation +/- DRUJ pinning
 indications
 unable to obtain anatomic closed reduction
 irreducible DRUJ due to interposed tendon or periosteum
 technique
 radial fixation can be done with volar plate of flexible IMN (see below)
o ORIF, soft tissue reconstruction of DRUJ and TFCC, +/- corrective osteotomy
 indications
 chronic DRUJ instability (a rare consequence of a missed injury)
o corrective osteotomy with soft tissue reconstruction of DRUJ and TFCC
 indications
 DRUJ subluxation is caused by a radial malunion
 a corrective osteotomy is also required in addition to reconstruction, otherwise a soft
tissue reconstruction of the DRUJ alone will fail
Technique
 ORIF with volar plating, +/- DRUJ pinning
o approach
 dorsal approach to DRUJ to remove interposed material if unable to obtain closed reduction
 volar approach for ORIF(with plate)
o open reduction
 irreducible DRUJ requires an open reduction to remove interposed material
 reduction can be blocked by interposed
 tendon
 ECU most common interposed tendon
 periosteum
o DRUJ stability
 following fixation, test DRUJ
 if unstable, pin ulna to radius in supination
 if unstable with large ulnar styloid fragment, fix ulnar styloid and splint in supination
 ORIF with flexbile intramedullary nailing, +/- DRUJ pinning
o approach
 percutaneous (with IMN) of radius fracture
o open reduction : same as above
o DRUJ stability : same as above

- 497 -
OrthoBullets2017 Pediatric trauma | Hip & Femur Fractures
Complications
 Malunion/nonunion
 Chronic DRUJ instability
o chronic DRUJ instability (a rare consequence of a missed injury)
 Superficial radial nerve plasy
o can be seen with IMN
 Extensor pollicus longus
o can be seen with IMN

E. Hip & Femur Fractures

1. Pelvis Fractures - Pediatric


Introduction
 Epidemiology
o incidence
 uncommon, only 1-2% of all pediatric
fractures
o demographics
 avulsion injuries almost exclusively in
adolescent patients
 Pathophysiology
o apophyseal avulsion
 result of low energy trauma
 avulsion injury occur from the
disruption of tendon origin on the
pelvis during "explosive" type activities (ie. jumping, sprinting)
 ischial avulsion (54%) = hamstrings and adductors
 AIIS avulsion (22%) = rectus femoris
 ASIS avulsion (19%) = sartorius
 pubic symphysis (3%)
 iliac crest (1%) = abdominal muscles
 lesser trochanter = iliopsoas
o pelvic ring
 result of high energy trauma
 often result from pedestrian vs MVA or rear seat passenger MVA
 although rare, can be open
 differences from adult pelvic ring injuries
 higher incidence of lateral compression injuries than adults, who are more commonly AP
compression injuries
 differences from adults
 higher rate of single pelvic ring break (rather than complete ring)
 increased bony plasticity
 cartilage able to absorb more energy
 SI joint/symphysis pubis more elastic
 thick periosteum

- 498 -
By Dr, AbdulRahman AbdulNasser Pediatric trauma | Hip & Femur Fractures
 apparent dislocations (symphyseal, SI) may have periosteal tube that heals like
fracture
 lower rate of hemmorhage secondary to
 smaller vessels, which are more capable of vasoconstriction
 injuries less commonly increase pelvic volume than in adult
o acetabular fractures
 only 1-15% of pelvis fractures
 more common after triradiate closure
 differences from adult
 triradiate cartilage injury can cause growth arrest and lead to deformity
 fractures into triradiate cartilage occur with less force than adult acetabular fractures
 transverse fracture pattern more common than both column
 classified using Letournel
 Associated conditions
o CNS and abdominal visceral injury
 high rate (> 50%) in traumatic pelvic injuries, presumed secondary to higher energy required
to create fracture
o femoral head fractures/dislocations
 associated with acetabular fractures
o GU injury
 increased rate with Torode Type IV fractures
o life threatening hemmorhage
 Prognosis
o complications are rare
o need for operative intervention increases after closure of triradiate cartilage

apophyseal avulsion AIIS & ASIS Ischial apophyseal avulsion

Anatomy
 Pelvis undergoes endochondral ossification (like long bones) at 3 primary ossification centers
o ilium appears at 9 wks
o ischium appears at 16 wks
o pubis appears at 20 wks
 all meet and fuse at 12yr in girls, 14yr in boys
 Acetabular growth
o enlargement is a result of interstitial growth within triradiate cartilage
o concavity is a response to pressure from femoral head
- 499 -
OrthoBullets2017 Pediatric trauma | Hip & Femur Fractures
o depth of acetabulum results from
 interstitial growth in acetabular cartilage
 appositional growth in periphery of cartilage
 periosteal new bone formation at acetabular margin
 Puberty
o 3 secondary ossification centers of the acetabulum appear at 8-9yr and fuse at 17-18yr
 os acetabuli (OA, forms anterior wall)
 acetabular epiphysis (AE, forms superior acetabulum)
 secondary ossification center of ischium (SCI, forms posterior wall)
o other secondary ossification centers (of the pelvis)
 do not confuse with avulsion fractures
 iliac crest
 appears at 13-15y, fuses at 15-17y
 used in Risser sign
 ischial apophysis
 appears at 15-17y, fuses at 19-25y
 anterior inferior iliac spine
 appears at 14y, fuses at 16y
 pubic tubercle
 angle of pubis
 ischial spine
 lateral wing of sacrum

Classification

Tile Classification
Type A • Stable injuries (rotationally & vertically)
Type B • Rotationally unstable
• Vertically stable
Type C • Unstable rotationally & vertically

Type A Type B Type C

Torode/Zieg Classification (pediatric pelvic ring)


Type I • Avulsion injuries
Type II • Fractures of the iliac wing
Type III • Fractures of the ring with no segmental instability
Type IV • Fracture of the ring with segmental instability

- 500 -
By Dr, AbdulRahman AbdulNasser Pediatric trauma | Hip & Femur Fractures
Torode/Zieg Classification (pediatric pelvic ring)

Type I Type II Type III Type IV


Bucholz Classification (pediatric acetabulum)
Shearing • Salter Harris I or II

Blow to pubis/ischial ramus/proximal femur leads to injury at interface of 2


superior arms of triradiate cartilage and metaphyses of ilium.

A triangular medial metaphyseal fragment (Thurston-Holland fragment) is


often seen in SH II injuries.
Crushing/Impaction • Salter Harris V

Difficult to see on initial radiographs. May detect narrowing of triradiate


space. Leads to premature triradiate cartilage closure. The earlier the
closure, the greater the eventual deformity.

Presentation
 History
o pediatric pelvic ring fractures often occur secondary to motor vehicle accidents or when a
pedestrian is struck by a motor vehicle
o pelvic avulsion injuries often occur during sporting activities such as sprinting, jumping or
kicking
 Physical exam
o as in all trauma patients, initial evaluation should include ABC's followed by primary and
secondary surveys
o important to thoroughly complete a rectal/genitourinary evaluation in polytrauma patient

Imaging
 Radiographs
o recommended views
 AP
 Judet views (45 degree internal and external oblique views, to better evaluate the
acetabulum),
 Inlet/Outlet views (35 degree caudal and cranial tilt views, to better evaluate integrity of the
pelvic ring)
o sensitivity
 plain radiographs will miss ~50% of all pediatric pelvic fractures
 CT
o indications
 negative plain films with increased suspicion
 preoperative evaluation

- 501 -
OrthoBullets2017 Pediatric trauma | Hip & Femur Fractures
 MRI
o indications
 occasionally required to detect apophyseal avulsion injuries
 apophyseal avulsion injuries are usually easily detected and adequately imaged with plain
radiographs
Treatment
 Nonoperative
o protected weight bearing followed by therapy
 indications
 pelvic ring
 dislocations of symphysis and SI joint
 potential for periosteal healing
 Type I Avulsion Injuries with < 2 cm displacement
 Type II Iliac Wing Fractures with < 2 cm displacement
 Type III pelvic ring fractures without segmental instability and non-displaced
acetabulum
 acetabulum
 few indications for non-op treatment
 results often poor, especially with comminution, joint incongruity
 if non-op chosen, needs close followup for 1-2yr to detect premature triradiate closure
 technique
 for Type I and II
 protected weight bearing for 2-4 weeks
 stretching and strengthening 4-8 weeks
 return to sport and activity after 8 weeks and asymptomatic
 Type III
 weight bearing as tolerated for 6 weeks
o bedrest
 indications : Type IV pelvic ring with instability AND < 2 cm pelvic ring displacement
 Operative
o ORIF
 principles
 physis sparing where possible
 where not possible, smooth pins across physis (especially triradiate) x 4-6wks with early
removal
 indications
 pelvis
 Type I Avulsion Injuries with > 2-3 cm displacement
 Type II Iliac Wing Fractures with > 2-3 cm displacement
 Type III pelvic ring with displaced acetabular fractures > 2mm
 Type IV pelvic ring with instability and > 2 cm pelvic ring displacement
 acetabulum
 comminuted acetabular fracture when traction does not improve position of fragments
 joint displacement >2mm
 joint incongruity
 joint instability (persistent medial subluxation or posterior subluxation)
- 502 -
By Dr, AbdulRahman AbdulNasser Pediatric trauma | Hip & Femur Fractures
central fracture dislocation
intra-articular fragments
 open fractures
o temporizing external fixation followed by ORIF
 indications
 vertical shear with hemodynamic instability

Complications ‎VIII:26 premature closure of triradiate cartilage


 Death : rare
o most often occur in association with head or visceral injury
 Pelvic fracture-associated hemmorhage : rare
o see above under death
 Physeal cartilage injury
o premature closure of triradiate cartilage/growth arrest (<5%)
 highest risk
 <10yr old at injury
 Bucholz crushing type (SH V)
 diagnosis
 high level of suspicion
 CT scan
 consequences
 progressive acetabular dysplasia with thickening of medial
acetabular wall giving rise to shallow acetabulum (lateral hip
subluxation) ‎VIII:27 physeal bar excision
hypoplastic hemipelvis
 treatment
 early reconstruction with physeal bar excision
 premature triradiate closure can still occur in spite of
bar excision
‎VIII:28 Chiari pelvic osteotomy
 late reconstruction with pelvic osteotomy
o leg length discrepancy
 Malunion/nonunion : rare
o pelvic asymmetry of >1- 2 cm can lead risk of scoliosis, lower back pain, Trendelenberg gait,
sacroiliac joint tenderness
 Neurovascular injury
 Heterotopic ossification

2. Traumatic Hip Dislocation - Pediatric


Introduction
 Epidemiology
o location
 80% are traumatic posterior dislocations
 more common than hip fracture
 Pathophysiology
o mechanism of injury
- 503 -
OrthoBullets2017 Pediatric trauma | Hip & Femur Fractures
 age <10
 may have low energy sporting injury, or trip and fall
 age >10
 mostly high energy (e.g. MVA)
 Associated injuries
o femoral head or neck fracture
o acetabular fractures
 lesser incidence of acetabular fractures compared with adults
 due to cartilaginous acetabulum and ligamentous laxity
 Prognosis
o typically associated with good long-term outcomes when treated promptly
 most have mild or no pain
 most return to high-demand activities

Presentation
 Symptoms
o pain, inability to bear weight
 Physical exam
o posterior dislocation (most common)
 slight flexion, adduction, and internal rotation of the limb
 clinical limb length discrepancy
 if large posterior wall acetabular fracture, can appear shortened without malalignment
o anterior dislocation
 flexion, abduction, and external rotation
o neurovascular exam
 check for sciatic or gluteal nerve palsy (rare)

Imaging
 Radiographs
o recommended views
 ap and lateral
‎VIII:29 post reduction xray of hip
 most can be diagnosed on AP pelvis films dislocation showing medial joint space
 lateral hip radiographs will confirm anterior vs posterior widening due to non concentric
reduction
dislocation
 post reduction films
 post-reduction radiographs are necessary to confirm concentric reduction
o findings
 radiographs must be scrutinized in order to inspect for joint incongruity or nonconcentric
reduction
 CT
o indications
 post-reduction CT scan is utilized to further evaluate for any entrapped
osteochondral fragment
o findings
 inspect for joint incongruity or nonconcentric reduction
 entrapped labrum or capsule can produce a subtle asymmetry ‎VIII:30 osteochondral
 interposed soft-tissue can be difficult to appreciate on CT scan fragment

- 504 -
By Dr, AbdulRahman AbdulNasser Pediatric trauma | Hip & Femur Fractures
 osteochondral fragments can be seen in older children and are easily detected by CT
 a non-concentric reduction requires exploration to remove entrapped labrum, capsule,
osteochondral fragment or ligamentum teres
 MRI
o best for evaluating interposed soft tissue

Treatment
 Nonoperative
o closed reduction under general anesthesia with fluoroscopy
 indications
 urgent attempt at closed reduction is first line treatment
 most are successful reduced with closed means (85%)
 Operative
o open reduction
 indications
 nonconcentric reduction
 intra-articular fragment
 unstable acetabular rim fracture
 irreducible by closed means
 technique
 surgical approach is typically performed in direction of dislocation (most commonly
posterior)
Techniques
 Closed reduction technique
o reduction
 adequate anesthesia or sedation during reduction is mandatory in order to decrease the risk of
displacing the proximal femoral epiphysis
 reduction under fluoroscopy has been recommended to decrease risk of displacement
o post-reduction
 test hip stability before weaning sedation
 obtain post-reduction imaging
 some advocate spica cast in younger children or bracing in older children with 6 weeks
protected weight-bearing on crutches
Complications
 Osteonecrosis
o reported in 3-15%
o less frequent than in adults if there is an absence of an associated femoral neck fracture
o if present, thought to be related to delayed reduction
 Coxa magna
o common radiographic finding (20%)
o not associated with functional limitation
 Redislocation
o rare sequela
o treatment
 prolonged immobilization

- 505 -
OrthoBullets2017 Pediatric trauma | Hip & Femur Fractures
 if recurrent and recalcitrant to immobilization: capsulorrhaphy
 treatment based on age of patient and time elapsed since injury
 Nerve injury
o sciatic or gluteal nerve injury can occur, usually resolves with prompt reduction

3. Proximal Femur Fractures - Pediatric


Introduction
 Overview
o challenging because of the high rates of complications
 AVN
 coxa vara malunion
 Pathophysiology
o mechanism
 high energy trauma (75-80%)
 Type I can occur in newborns after breech delivery
 similar to Salter Harris I injury
 Prognosis
o timing of treatment impact prognosis
 Delbet type I to III are surgical emergencies

Anatomy
 Growth centers of the proximal femur
o proximal femoral epiphysis
 accounts for 13-15% of leg length
 accounts for 30% length of femur
 proximal femoral physis grows 3 mm/yr
 entire lower limb grows 23 mm/yr
o trochanteric apophysis
 traction apophysis
 contributes to femoral neck growth
 disordered growth
 injury to the GT apophysis leads to shortening of the GT and coxa valga
 overgrowth of the GT apophysis leads to coxa vara
 Vascularity
o medial femoral circumflex artery
 main blood supply to the head via the posterosuperior lateral epiphyseal branch and via
posteroinferior retinacular branch
 becomes main blood supply after 4 years after regression of LFCA and artery of ligamentum
teres
o lateral femoral circumflex artery
 regresses in late childhood
o artery of the ligamentum teres
 diminishes after 4 years old
o metaphyseal vessels
 also contribute to blood supply to the head < 3 years old and after 14-17years
 between 3 to 14-17 years, the physis blocks metaphyseal supply

- 506 -
By Dr, AbdulRahman AbdulNasser Pediatric trauma | Hip & Femur Fractures
 after 14-17 years, anastomoses between metaphyseal-epiphyseal vessels develop
 Neurovacular
o superior gluteal nerve (L5, S1, S2)
 gluteus medius and gluteus minimus

Classification

Delbet Classification
Type Description Incidence AVN Nonunion
Type I Transphyseal (IA, without dislocation of <10% 38% (AVN 100%
epiphysis from acetabulum; IB, with in type IB)
dislocation of epiphysis)

Type II Transcervical 40-50% 28%


15%
Type III Cervicotrochanteric (or basicervical) 30-35% 18%
15-20%
Type IV Intertrochanteric 10-20 5% 5%

Type I Type II Type III Type IV

- 507 -
OrthoBullets2017 Pediatric trauma | Hip & Femur Fractures
Imaging
 Radiographs : AP pelvis and cross-table lateral
 CT : for nondisplaced fractures and stress fractures
 MRI : for nondisplaced fractures and stress fractures

Treatment
 Nonoperative
o spica cast in abduction, weekly radiographs for 3wks
 indications
 Type IA, II, III, IV, nondisplaced, <4yrs
 evaluate Type IA fractures for child abuse
 Operative
o emergent ORIF, capsulotomy, or joint aspiration
 indications
 open hip fracture
 vessel injury where large vessel repair is required
 concomitant hip dislocation or significant displacement, especially type I
 may decrease the rate of AVN (supporting data equivocal)
o closed reduction internal fixation (CRIF)/ percutaneous pinning (CRPP)
 indications
 Type II, displaced
 postop spica (abduction and internal rotation) x 6-12wk
 Type III and IV, displaced and older children
o open reduction and internal fixation (ORIF)
 indications
 Type IB
o pediatric hip screw / DHS
 indications
 Type IV

Techniques
 Emergent reduction and capsulotomy
o timing of reduction
 early reduction (<24h) may diminish risk of AVN by restoring
blood flow through kinked vessels
o reduction technique
 radiolucent table for 0-10 years

 fracture table for >10 years ‎VIII:31 pediatric hip screw


o acceptable alignment
 Type II
 accept <2mm cortical translation, <5° of angulation, no malrotation
 Type III and IV
 accept <10° of angulation
o capsulotomy
 may decrease AVN
 aspiration with large bore needle through subadductor/anterior hip approach

- 508 -
By Dr, AbdulRahman AbdulNasser Pediatric trauma | Hip & Femur Fractures
open capsulotomy through anterior incision
 Closed reduction and percutaneous pinning (CRPP)
o reduction technique : see above
o fixation
 smooth or threaded pins / K wires (use 2-3 pins or wires)
 indications
 younger patients
 transphyseal
 recommended when there is little metaphyseal bone available
 cannulated screws
 indications
 short of the physis
 less stable than transphyseal
 for patients <4-6yrs
 transphyseal
 older patients close to skeletal maturity (>12yrs old)
 where crossing the physis is necessary to achieve stable fixation
 it is easier to treat leg length discrepancy from premature physeal closure than
nonunion
 place within 5mm of subchondral bone
 avoid anterolateral quadrant of epiphysis and posterior perforation of femoral neck
 to prevent injury to vasculature
 Closed reduction and internal fixation (CRIF)
o indications
 type IV
 appropriate if immediately available
o implants
 pediatric hip screws
 Open reduction and internal fixation (ORIF)
o approach
 anterolateral (Watson-Jones) for types I, II, III
 lateral (Hardinge) for type IV

Complications
 AVN
o most common complication
‎VIII:32 Avascular necrosis (AVN)
 risks = age + fracture type
 most susceptible age = 3-8 years
 highest for Delbet type I (nearly 100% for Delbet type IB)
o etiology
 kinking/laceration of vessels
 tamponade by intracapsular hematoma
o treatment
 core decompression
 vascularized fibular graft
 Coxa vara (neck-shaft angle <130°) ‎VIII:33 Coxa vara
o 2nd most common complication
- 509 -
OrthoBullets2017 Pediatric trauma | Hip & Femur Fractures
o more common if fracture is treated non-operatively
o more common for types I, II and III
 incidence 25% for type III
o treatment
 young patients (0-3yrs) will remodel
 surgical arrest of trochanteric apophysis
 indication
 coxa vara in <6-8yrs
 only works in younger patient
 subtrochanteric or intertrochanteric valgus osteotomy
 indication
 coxa vara + nonunion
 coxa vara with severe Trendelenburg limp or FAI signs and
symptoms
 for the older patient
 Nonunion
o can occur together with coxa vara (see above)
o etiology
 nonoperative treatment of Type II or III
 occult infection at fracture site
 severe AVN of proximal femur ‎VIII:34 Nonunion
 malreduced fracture
o treatment
 subtrochanteric or intertrochanteric valgus osteotomy
 Coxa valga
o Type IV fractures involving GT in younger patient may have
premature GT apophysis closure, leading to coxa valga
 Physeal arrest
o physeal arrest alone leads to <1.5cm leg length discrepancy
 only in very young children
 proximal femoral physis contributes to 15% of limb length
(3mm/yr)
 Limb length discrepancy
o significant LLD occurs in combined AVN + physeal arrest
o treatment
 shoe lift if projected LLD at skeletal maturity <2cm
 epiphysiodesis of contralateral distal femur ± proximal tibia if projected LLD at skeletal
maturity 2-5cm
 Chondrolysis
o usually associated with AVN
o etiology
 poor vascularity to femoral head cartilage
 persistent hardware penetration of joint
o presents as restricted hip motion, hip pain, radiographic joint space narrowing
 Malreduction
o common with subtrochanteric fractures

- 510 -
By Dr, AbdulRahman AbdulNasser Pediatric trauma | Hip & Femur Fractures
 deforming forces on proximal fragment
 displaced into flexion, abduction, and external rotation
 Infection
o <1% incidence
o after ORIF or CRPP
o treatment
 debridement, maintain fixation until union
o may lead to osteomyelitis, AVN, chondrolysis, premature physeal closure

4. Femoral Shaft Fractures - Pediatric


Introduction
 High suspicion for child abuse required
o abuse must be considered if child is < 5 years
 especially if present in a patient before walking age
o femur fractures are the 2nd most common child abuse associated fracture after humerus fractures
 Epidemiology
o bimodal distribution
 increased rate in toddlers age 2-4 yrs.
 increased again in adolescents
 Mechanism
o correlated with age due to the increasing thickness of the cortical shaft during skeletal growth
and maturity
 falls most common cause in toddlers
 high energy trauma is responsible for second peak in adolescents
 MVC or ped vs vehicle
o fractures after minor trauma can be the result of a pathologic process
 bone tumors, OI, osteopenia, etc.

Classification
 Descriptive classification
o characteristics of the fracture
 transverse
 comminuted
 spiral etc.
o integrity of soft-tissue envelope
 open
 closed fracture
 Stability
o length stable fractures
 are typically transverse or short oblique
o length unstable fractures
 are spiral or comminuted fractures

Presentation
 Symptoms : thigh pain, inability to walk, report of deformity or instability
 Physical exam : gross deformity, shortening, swelling of the thigh

- 511 -
OrthoBullets2017 Pediatric trauma | Hip & Femur Fractures
Imaging
 Radiographs
o AP and lateral of femur
 typically allow complete evaluation of the fracture location, configuration and amount of
displacement
o ipsilateral AP and lateral of knee and hip
 required to rule out associated injuries

Treatment
 Based on age and size of patient and fracture pattern
 Guidelines provided by AAOS

Treatment Guidelines
< 6 months  Any fx pattern  Pavlik harness
 Early spica casting
7m - 5 years  < 2 - 3 cm shortening  Early spica casting

 > 2 - 3 cm shortening  Traction with delayed spica casting


 polytrauma/multiple fx/open fx  ORIF with submuscular bridge plating
 Flexible nails
 External fixator
6 - 11 years  length stable fx (transverse or  Flexible intramedullary nails
oblique fx patterns)
 length unstable fx (comminuted  ORIF with submuscular bridge plating
or spiral)  External fixation
 very proximal or distal fx o polytrauma patients for damage
control
Approaching skeletal  length stable  Flexible intramedullary nails
maturing (>11 years)  patient weighs < 100 lbs
 length unstable  Antegrade IM nail with trochanteric or
 patient weighs > 100 lbs lateral starting point
 length unstable
 ORIF with submuscular bridge plating
 very proximal or distal fx

Surgical Technqiues
 Pavlik harness
o indications
 children up to 6 mos.
o technique
 avoids the need for sedation or anesthesia
 straps can be adjusted to manipulate fracture ‎VIII:35 Spica cast
‎VIII:36 Pavlik harness
o complications
 can compress femoral nerve if excessive hip flexion is used in presence of a swollen thigh
 identified by decreased quadricep function
 Immediate spica casting
o fewer complications than traction + later casting
o indications
 children 7 m - 5 years with < 2 - 3 cm of shortening
 relatively contraindicated with polytrauma, open fractures and shortening > 2-3 cm
- 512 -
By Dr, AbdulRahman AbdulNasser Pediatric trauma | Hip & Femur Fractures
o technique
 applied with reduction under sedation or with GA
 single-leg spica or one-and-one-half spica (to control rotation)
 the exception is distal femoral buckle fracture (stable) only requires long leg cast (not
spica)
 hips flexed 60-90° and approximately 30° of abduction
 knees in 90° of flexion
 MUST limit compression and/or traction thru popliteal fossa
 external rotation is typically needed to correct rotational deformity
 molds along the distal femoral condyles and buttocks help to maintain reduction
 acceptable limits are based on childs age
 goal of reduction should include obtaining < 10° of coronal plane and < 20° of sagittal
plane deformity with no more than 2cm of shortening or 10° of rotational malalignment
 a special car seat is needed for transport
o follow-up
 weekly radiographs to monitor for loss of reduction for first 2 to 3 weeks
 cast wedging can be used to correct deformities
 healing times vary from 4 - 8 weeks based on age
o complications
 compartment syndrome
 decreased with applying smooth contours around popliteal fossa, limiting knee flexion to
< 90° and avoiding excessive traction
 monitored for by observing the child's neurovascular exam and level of comfort
 Traction + delayed spica casting
o indications
 children 7 mos. - 5 yrs. of age with > 2 - 3 cm of shortening
o technique
 placed in distal femur proximal to distal femoral physis
 proximal tibial traction can cause recurvatum due to damage to the tibial tubercle
apophysis
 used for 2-3 weeks to allow early callus formation
 spica casting then applied until fracture healing
o complications
 more complications than immediate spica casting
 Flexible intramedullary nails
o indications
 treatment of choice for most simple, length stable fracture patterns in children 6 - 10 years
 adolescent patient weighing less than 100 lbs with a length stable fracture
o technique
 allows load sharing and quick moblization of the patient
 nail size determined by multiplying width of narrowest portion of femoral canal by 0.4
 the goal is 80% canal fill
 two nails of equal size are inserted retrograde beginning approximately 2 -2.5 cm above the
distal femoral physis
o follow up : time to union is typically 10 - 12 weeks
 removal of the nail can be performed at 1 year

- 513 -
OrthoBullets2017 Pediatric trauma | Hip & Femur Fractures
o complications
 most common complication is pain at insertion site near the knee
 in up to 40% of patients
 recommended that < 25mm of nail protrusion and minimal bend of the nail outside the
femur are present

 increased rate of complications in patients >11-12 years or > 45 kg


 increased rates of malunion and shortening in very proximal and distal fractures, as well as
significantly comminuted fractures
 Submuscular bridge plate fixation
o indications
 comminuted, length unstable fractures
 very proximal (subtrochanteric) or very distal fractures (distal diaphyseal or metadiaphyseal)
o technique
 fracture is provisional reduced with closed or percutaneous techniques
 small proximal + distal incisions and plate is placed between periosteum and vastus lateralis
on the lateral side of the femur
 typically use 12-16 hole 4.5mm narrow LC-DC plate with 3 screws proximal and 3 screws
distal to fracture
 the plate may need to be bent to accomodate the natural bend of the femur
 locking fixation can be used in osteoporotic areas or in very proximal or very distal
fractures with limited area for fixation
 weightbearing is restricted until visible callus formation at an average of 5 weeks
o advantages
 stability allows for early mobility
 preserves blood supply to femoral head
 performed with minimal surgical exposure and soft-tissue dissection
o disadvantages
 steep learning curve
 load bearing implant
 multiple stress risers following removal of hardware
 Antegrade rigid intramedullary nail fixation
o indications
 in patients > 11 years
 length unstable fractures
 fractures in patients weighing > 100 lbs
o technique
- 514 -
By Dr, AbdulRahman AbdulNasser Pediatric trauma | Hip & Femur Fractures
 use greater trochanter or lateral entry nails
 decreased risk of ON
 do not cross distal physis of femur
o advantages
 rigid fixation with interlocking screws control length and rotation even in significantly
unstable fractures
 permits early weightbearing
 decreased risk of angular malunion
o complications
 ON risk is 1-2% with piriformis start in a patient with open proximal physes
 exact risk of ON with greater trochanter and lateral entry nails is unknown
 secondary deformities of the proximal femur can occur after greater trochanteric insertions
 narrowing of the femoral neck
 premature fusion of greater trochanter apophysis
 coxa valga
 hip subluxation
 External fixation
o indications
 damage control orthopaedics in a polytrauma patient
 open fractures
 associated vascular injuries requiring revascularization
 fractures with associated soft tissue concerns
 segmental or significantly comminuted fractures
 multiply injured patient
o technique
 applied laterally
 avoid disruption and scarring of quadriceps
 10 - 16 weeks of fixation is typically needed for solid union to occur
 weightbearing as tolerated can be considered with stiff constructs
o complications
 more complications than internal fixation
 pin tract infections are frequent
 as high as 50% of fixator related complications
 treated with oral antibiotics and pin site care
 higher rates of delayed union, nonunion and malunion
 increased risk of refracture (1.5-21%) after removal of fixator especially with varus malunion

Complications
 Leg-Length Discrepancy
o overgrowth
 0.7 - 2 cm is common in patients between of 2 - 10 years at time of fracture
 typically presents within 2 years of injury
o shortening
 is acceptable if less than 2 - 3 cm because of anticipated overgrowth
 can be symptomatic if greater than 2 - 3 cm
 temporary traction or internal fixation used to prevent persistent shortening
 Osteonecrosis (ON) of femoral head
- 515 -
OrthoBullets2017 Pediatric trauma | Hip & Femur Fractures
o reported with both piriformis and greater trochanter entry nails
o femoral nailing through the piriformis fossa is contraindicated in adolescents with open physes
because of the risk of osteonecrosis of femoral head
o main supply to femoral head is deep branch of the medial femoral circumflex artery
 branches into superior retinacular vessels that supply the femoral head
 vulnerable as it lies near the piriformis fossa
 Nonunion
o higher risk with load bearing devices
 external fixator or submuscular plates
o can occur after flexible intramedullary nailing in patients
 aged over 11 years old
 who weigh >49 kg (>108 lb)
 Malunion
o typical deformity is varus + flexion of the distal fragment
o remodeling is greatest in sagittal plane (ie flexion/extension deformity)
o rotational malalignment does not remodel
 must be corrected at the initial surgery
 rarely symptomatic
 Refracture
o most common after external fixator removal with varus malalignment
o highest risk in transverse and short oblique fractures
 less likelihood of secondary callus formation

5. Distal Femoral Physeal Fractures - Pediatric


Introduction
 Most commonly seen as a Salter-Harris II fracture
 Mechanism
o often a varus or valgus force on the knee
o physis fails under traumatic force before ligaments in children
o disruption occurs thru multiple zones of the growth plate
 Physeal arrest
o high incidence of physeal arrest that often leads to growth disturbance and deformity
 be sure to counsel parents of poor prognosis associated with this fracture pattern
 an increased incidence of complications have been associated with
 Salter-Harris classification type
 fracture displacement
 surgical hardware invading the physis

Anatomy
 Physeal considerations of the knee
o general assumptions
 leg growth continues until
 16 yrs in boys
 14 yrs in girls
o growth contribution
 leg grows 23 mm/year, with most of that coming from the knee (15 mm/yr)

- 516 -
By Dr, AbdulRahman AbdulNasser Pediatric trauma | Hip & Femur Fractures
 proximal femur - 3 mm / yr (1/8 in)
 distal femur - 9 mm / yr (3/8 in)
 proximal tibia - 6 mm / yr (1/4 in)
 distal tibia - 5 mm / yr (3/16 in)

Presentation
 Symptoms
o unable to bear weight
 Physical exam
o pain and swelling
o tenderness along the physis in the presence of a knee effusion
o may see varus or valgus knee instability on exam
Imaging
 MRI or ultrasound
o indications
 diagnositic modality of choice to confirm physeal fracture
 Radiographs
o Standard AP, lateral, and oblique radiographs of the knee should be done as initial evaluation
o indications
 follow up radiographs after 2-3 weeks of casting if physeal injury is likely but not identifiable
on injury films initially
 stress radiographs to look for physis opening if there was suspicion of physeal injury
 have fallen out of favor due to patient discomfort and possible need for sedation in order
to properly stress the knee
Treatment
 Nonoperative
o long leg casting
 indications
 stable nondisplaced fractures
 close clinical followup is mandatory
 Operative
o closed reduction and percutaneous pinning followed by casting
 indications
 displaced Salter-Harris I or II fractures
 displaced fractures successfully reduced with closed methods should still be pinned
(undulating physis makes unstable following reduction)
 technique
 avoid multiple attempts at reduction
 avoid physis with hardware if possible
 if physis must be crossed (SH I and SH II with small Thurston-Holland fragments),
use smooth k-wires
 SH II fracture, if possible, should be fixed with lag screws across the metaphyseal segment
avoiding the physis
 postoperatively follow closely to monitor for deformity
o ORIF
 indications

- 517 -
OrthoBullets2017 Pediatric trauma | Hip & Femur Fractures
 Salter-Harris III and IV in order to anatomically reduce articular surface
 irreducible SHI and SHII fractures
 reduction often blocked by periosteum infolding into fracture site
 techniques
 If anatomic reduction cannot be obtained via closed techniques, incision over the
displaced physis to remove interposed periosteum is necessary.
Complications
 Limb length discrepancy or angular deformity (most common)
o results from physeal disturbance
o correlates with fracture pattern
 36% of SH 1 fractures
 58% in SH 2 fractures
 49% in SH 3 fractures
 64% in SH 4 fractures
o prevent with
 anatomic physeal alignment (critical)
 close follow up following nonoperative or operative treatment
o treatment
 physeal bridge excision
 indication
 deformity is present with a physeal bar of <50% and ≥ 2 years or 2 cm of growth
remaining
 Popliteal artery injury
o rare and more common with anterior displacement of epiphysis
o most common with anterior, or posteriorly, displaced fracture patterns

Collected By : Dr AbdulRahman
AbdulNasser
June 2017

- 518 -
By Dr, AbdulRahman AbdulNasser Pediatric trauma | Knee & Proximal Tibia

F. Knee & Proximal Tibia

1. Tibial Eminence Fracture


Introduction
 A fracture of the bony attachment of the ACL on the tibia
 Epidemiology
o rare injuries
o most common in ages 8-14
 Mechanism
o traumatic
 rapid deceleration or hyperextension of the knee
 same mechanism that would cause ACL tear in adult
 Associated conditions
o occur in 40% of eminence fractures
 meniscal injury
 collateral ligament injury
 capsular damage
 osteochondral fracture

Anatomy
 Osteology
o tibial eminence
 non-articular portion of the tibia between the medial and lateral tibial plateau
 Ligaments
o anterior cruciate ligament
 inserts 10-14 mm behind anterior border of tibia and extends to medial and lateral tibial
eminence
Classification
Modified Meyers and McKeever Classification
Type I Nondisplaced (<3mm)
Type II Minimally displaced with intact posterior hinge
Type III Completely displaced
Type III+ Type III fracture with rotation
Type IV Completely displaced, rotated, comminuted

- 519 -
OrthoBullets2017 Pediatric trauma | Knee & Proximal Tibia
Presentation
 Symptoms
o pain in knee
 Physical exam
o inspection
 immediate knee effusion
o ROM
 often limited secondary to pain
 once pain is controlled, lack of motion may indicate
 meniscal pathology
 displaced/entrapped fracture fragment
 positive anterior drawer

Imaging
 Radiographs
o recommended views
 standard knee radiographs
 CT
o useful for pre-operative planning
 MRI
o better at determining associated ligamentous/meniscal damage than CT or radiographs
Treatment
 Nonoperative
o closed reduction, aspiration of hemarthrosis, immobilization in 0-20° of flexion
 indications
 non-displaced type I and reducible type II fractures
 reduction maneuver = extend the knee to 20° short of full extension to observe for fragment
reduction
 Operative
o ORIF vs. all-arthroscopic fixation
 indications
 Type III or Type II fractures that cannot be reduced
 block to extension

Sugical Techniques
 Arthroscopic fixation
o approach
 standard arthroscopic portals
o technique
 debride fracture
 disengage entrapped meniscus or intermeniscal ligament
 medial meniscus entrapment most common
 reduce fracture
 fracture fixation
 suture fixation
 pros
 avoids physis

- 520 -
By Dr, AbdulRahman AbdulNasser Pediatric trauma | Knee & Proximal Tibia
 cons
 technically demanding
 screw fixation
 pros
 less demanding than suture fixation
 possibly earlier mobilization
 cons
 hardware irritation
 impingement from improperly placed screw
 physeal damage
o post-operative care
 early range of motion
 length of limited weight bearing is controversial
 Open fixation
o same principles as arthroscopic

Complications
 Arthrofibrosis
o more common with surgical reconstruction
 Growth arrest
 ACL laxity
o incidence
 10% of knees managed surgically
 20% of knees managed non-operatively
o often not clinically significant

2. Tibial Tubercle Fracture


Introduction
 A common fracture pattern that occurs in adolescent boys near end of growth
 Epidemiology
o incidence is less than 1% of pediatric fractures
o males >> females
o ages 12 - 15 (approaching skeletal maturity)
 Mechanism
o eccentric quadriceps contraction, such as coming down from a jump with knee flexed
o common in basketball players, football players, and sprinters
 Associated conditions
o monitor for compartment syndrome
o evaluate for extensor lag
 Prognosis
o high rate of return to sports when appropriately treated
o low incidence of leg length discrepancy

Anatomy
 Osteology

- 521 -
OrthoBullets2017 Pediatric trauma | Knee & Proximal Tibia
o proximaltibia has two ossification centers
primary ossification center (proximal tibial physis)
 secondary ossification center (tibial tubercle physis or apophysis)
 insertion of patellar tendon
 physeal closure occurs from posterior to anterior and proximal to distal
 places distal secondary center at greater risk of injury in older children
 Muscles
o extensor mechanism can exert great force at secondary ossification center
 Blood Supply
o recurrent anterior tibial artery can be torn with these injuries

Classification
Ogden Classification (modification of Watson-Jones)
Type I fracture of the secondary ossification center near the insertion of the patellar tendon
Type II fracture propagates proximal between primary and secondary ossification centers
Type III coronal fracture extend posteriorly to cross the primary ossification center
Modifier: A (nondisplaced), B (displaced)
 Newer descriptions have been added to the original system
o Type IV is a fracture through the entire proximal tibial physis
o Type V is a periosteal sleeve avulsion of the extensor mechanism from the secondary ossification
center

Presentation
 Symptoms
o sudden onset of pain
 generally occurs during the initiation of jumping or sprinting
o extensor mechanism deficiency or lag with Type 2 and 3 injuries
o knee swelling
 hemarthrosis with Type 3 injuries
 Physical exam
o inspection & palpation
 swelling at the knee
 tenderness at the tibial tubercle

- 522 -
By Dr, AbdulRahman AbdulNasser Pediatric trauma | Knee & Proximal Tibia
 evaluate for anterior compartment firmness
o ROM & instability
 extensor lag or extensor deficiency in Type 2 or 3 injuries
o neurovascular exam
 monitor for increasing pain suggestive of compartment syndrome

Imaging
 Radiographs
o recommended views
 required
 lateral of the knee
 optional ‎VIII:37 Ogden type IIIB
 internal rotation view will bring the tibial tubercle into profile
 consider contralateral knee views in pediatric fractures
o findings
 widening or hinging open of the apophysis
 fracture line may be seen extending proximally and variable distance posteriorly
 anterior swelling may be the only sign in the setting of a periosteal sleeve avulsion (type 5
injury)
 evaluate for possible patella alta
 CT
o can be useful to evaluate for intra-articular or posterior extension
o arteriogram can be helpful if concern for anterior tibial artery injury
 should not delay intervention in setting of compartment syndrome
 MRI
o generally not indicated
o useful for determining fracture extension in a nondisplaced Type 2 injury

Treatment
 Nonoperative
‎VIII:38 Ogden type IV B
o long leg cast in extension for 4-6 weeks
 indications
 usually Type 1 and 2 injuries
 minimal displacement (< 2 mm)
 acceptable displacement after closed reduction
 Operative
o closed reduction and percutaneous fixation vs open reduction internal fixation
 indications : Type 1, 2, and 4 fractures
o open reduction with arthrotomy and internal fixation
 indications : Type 3 fractures - need to visualize joint surface for perfect reduction
o open reduction and soft tissue repair
 indication
 Type 5 (periosteal sleeve) fractures

Techniques
 Closed reduction and percutaneous fixation
o approach

- 523 -
OrthoBullets2017 Pediatric trauma | Knee & Proximal Tibia
 closed reduction often done under anesthesia
 percutaneous clamping
o technique
 internal fixation with 4.0 cancellous partially threaded screws
 larger screws can cause soft tissue irritation in the long-term
 smooth K wires for younger child (>3y from skeletal maturity)
o postoperative care
 immobilization
 long leg cast or brace for 4-6 weeks
 prolonged immobilization needed in Type 2 and 3 injuries
 non-weight bearing
 rehabilitation
 progressive extensor mechanism strengthening
 return to sports no sooner than 3 months
 pros & cons
 pros
 no open reduction
 excellent healing potential
 cons
 inability to clean fracture site or remove soft tissue interposition
 hardware irritation can necessitate implant removal
 Open reduction and internal fixation
o approach
 midline incision to the fracture site
o technique
 evaluate and clean fracture site
 remove any soft tissue interposition (periosteum)
 anatomic reduction of fracture fragments
 internal fixation with 4.0 cancellous, partially threaded screws
 larger screws can cause soft tissue irritation in the long-term
 smooth K wires for younger child (>3y from skeletal maturity)
o postoperative care
 immobilization
 long leg cast or brace for 4-6 weeks
 prolonged immobilization needed in Type 2 and 3 injuries
 non-weight bearing
 rehabilitation
 progressive extensor mechanism strengthening
 return to sports no sooner than 3 months
 pros & cons
 pros
 anatomic reduction and stable fixation
 excellent healing potential
 may allow for earlier range of motion
 cons
 hardware irritation can necessitate implant removal

- 524 -
By Dr, AbdulRahman AbdulNasser Pediatric trauma | Knee & Proximal Tibia
 Open reduction with arthrotomy and internal fixation
o approach
 midline approach or median parapatellar arthrotomy
 joint surface must be visualized to assure anatomic reduction
 alternatively, arthroscopy can be used to directly assess the articular reduction
o technique
 same as above
 evaluate for meniscal tears and repair or debride as appropriate
 evacuate intraarticular hematoma
 visualize joint surface to achieve anatomic reduction
o postoperative care
 immobilization
 long leg cast for 4-6 weeks
 prolonged immobilization needed in Type 2 and 3 injuries
 non-weight bearing
 rehabilitation
 progressive extensor mechanism strengthening
 return to sports no sooner than 3 months
 pros & cons
 pros
 addresses intraarticular extension and soft tissue injuries
 cons
 arthrotomy may require longer immobilization and/or rehabilitation
 Open reduction and soft tissue repair
o approach
 midline incision to the soft tissue injury site
o technique
 evaluate soft tissue injury
 remove any soft tissue interposition (periosteum)
 heavy suture repair of periosteum back to the secondary ossification center
o postoperative care
 immobilization
 long leg cast for 8-10 weeks
 prolonged immobilization needed due to soft tissue (rather than bone) healing
 rehabilitation
 progressive extensor mechanism strengthening
 return to sports no sooner than 3 months
o pros & cons
 cons
 prolonged healing time due to soft tissue healing

Complications
 Recurvatum deformity
o more common than leg length discrecancy
o growth arrest anteriorly as posterior growth continues leading to decrease in tibial slope
 Compartment syndrome
o related to injury of anterior tibial recurrent artery
- 525 -
OrthoBullets2017 Pediatric trauma | Knee & Proximal Tibia
 Vascular injury
o to popliteal artery as it passes over distal metaphyseal fragment
 Loss of range of motion
 Bursitis
o due to prominence of screws and hardware about the knee

3. Patella Sleeve Fracture


Introduction
 Sleeve fracture occurs between the cartilage "sleeve" and main part of the patella and ossific nucleus
 Epidemiology
o demographics
 occurs most commonly in children 8 to 12 years of age
 when patellar ossification is nearly complete
 Pathophysiology
o mechanism of injury
 indirect injury caused by powerful contraction of the
quadriceps muscle applied to a flexed knee
o pathoanatomy
 disruption can occur distally, laterally, or proximally

Anatomy
 Ossification
o does not begin until 3 to 5 years of age.
o most patellar fractures occur in adolescents when ossification is nearly complete
o incomplete coalescence of a superolaterally located accessory center of ossification results in
bipartite patella (often confused with fracture)
Presentation
 History
o indirect injury
o not associated with direct blow to the knee
 Symptoms
o severe knee pain
 Physical exam
o inspection
 soft-tissue swelling
 a high-riding patella implies that the extensor
mechanism has been disrupted
 hemarthrosis of the knee joint is often present
o palpation
 palpable gap at the lower end of the patella
o motion
 active extension of the knee is difficult; especially
with resistance
 inability to weightbear

- 526 -
By Dr, AbdulRahman AbdulNasser Pediatric trauma | Knee & Proximal Tibia
Imaging
 Radiographs
o recommended views
 AP and lateral of knee
o findings
 small flecks of bone adjacent to inferior pole
 diagnosis may be missed because the distal bony fragment is not readily discernible on
radiographs
 patella alta
 for distal fractures (most common)
 patella baja
 for proximal fractures
 MRI
o indications
 may be useful for diagnosing a sleeve fracture when the diagnosis is not clear from the
clinical and plain radiographic findings
Treatment
 Nonoperative
o cylinder cast for 6 weeks
 indications
 nondisplaced fractures with intact extensor mechanism
 Operative
o open reduction and internal fixation (modified tension band technique)
 indications
 displacement more than 2-3mm
 majority require ORIF
 may be performed with sutures through drill holes

Technique
 Open reduction and internal fixation
o approach
 parapatellar to knee
 approach the inferior pole of the patella through a 7-cm medial parapatellar incision
 make incision over the distal aspect of the approach directly over the inferior pole of the
patella
o repair
 repair of the torn medial and lateral retinaculum along with the use of sutures through the
cartilaginous and osseous portions of the patella often suffice
o fixation
 once anatomic reduction of articular surface achieved, fracture can be stabilized using
modified tension band wiring around two longitudinally placed Kirschner wires
o post-operative care
 place in cast with knee in mild degree of flexion
 remove cast at ~3 weeks and start ROM exercises

- 527 -
OrthoBullets2017 Pediatric trauma | Knee & Proximal Tibia
Complications
 Patella alta
 Extensor lag
 Quadriceps atrophy

4. Proximal Tibia Epiphyseal Fractures - Pediatric


Introduction
 Epidemiology
o incidence : <1% of pediatric fractures
 Pathophysiology
o mechanism of injury
 high energy trauma
 varus/valgus force
 hyperextension
 Associated conditions
o fracture
‎VIII:39 Type III tibial tubercle fracture
 may occur as a part of a Type III tibial tubercle fracture
o vascular injury
 most commone with hyperextension injuries
 tethering of popliteal artery (5%)
o peroneal nerve injury (5%)
o knee ligamentous injury
o compartment syndrome (3%-4%)

Anatomy
 Physeal considerations of the knee
o general assumptions
 leg growth continues until
 16 yrs in boys
 14 yrs in girls
o growth contribution
 leg grows 23 mm/year, with most of that coming from the knee (15 mm/yr)
 proximal femur - 3 mm / yr (1/8 in)
 distal femur - 9 mm / yr (3/8 in)
 proximal tibia - 6 mm / yr (1/4 in)
 distal tibia - 5 mm / yr (3/16 in)

Presentation
 Symptoms
o unable to bear weight
 Physical exam
o inspection
 pain and swelling
 tenderness along the physis in the presence of a knee effusion
o motion
 may see varus or valgus knee instability on exam

- 528 -
By Dr, AbdulRahman AbdulNasser Pediatric trauma | Knee & Proximal Tibia
o neurovascular exam
 physis is at same level of trifurcation of vessels and there is a risk of vascular compromise
with displacement
Imaging
 Radiographs
o recommended views
 AP and lateral
o optional views
 oblique views
 varus/valgus stress views
o findings
 displacement of fracture fragments
 Salter Harris classification
 CT
o indications : assess fracture displacement
o findings : best modality for SH III or IV fractures

Treatment
 Nonoperative
o immobilization in long leg cast
 indications
 non-displaced fracture
 stable Salter-Harris Type I and Type II fractures
 modalities
 traction for fracture reduction
 cast in slight flexion for 6 weeks
 outcomes
 redisplacement is common without internal fixation
 Operative
o anatomic reduction and fixation with percutaneous pinning
 indications
 displaced fractures
 unstable Salter-Harris Type I and Type II fractures
 redisplacement following closed treatment
 modalities
 percutaneous pins parallel to physis
 pins crossing perpendicular to physis if extra-articular fixation needed
 outcomes
 avoid displacement to affect trifurcation
o open reduction internal fixation
 indications
 displaced fractures
 Salter-Harris Type III and Type IV fractures
 modalities
 screw parallel to physis
 cast in slight flexion for 4-6 weeks

- 529 -
OrthoBullets2017 Pediatric trauma | Knee & Proximal Tibia
Complications
 Loss of reduction
 Growth disturbances (25%)
o can lead to limb length discrepancy and/or angular deformities
 Compartment syndrome
 Ligamentous instability

5. Proximal Tibia Metaphyseal Fractures - Pediatric


Introduction
 Proximal tibia metaphyseal fractures are significant for their tendency to develop a late valgus
deformity
o etiology of valgus deformity is unknown
o known as Cozen's fracture or phenomenon
 Epidemiology
o most common in children 3-6 years of age
 Mechanism
o typically low-energy with valgus force across the knee creating incomplete fracture of proximal
tibia
o classic mechanism is child going down slide in the lap of an adult with leg extended
 Prognosis : valgus deformity resolves spontaneously

Classification
 Classification of pediatric proximal tibia metaphyseal fractures is descriptive.
o important radiographic parameters include:
 complete versus incomplete fracture
 majority are incomplete
 displaced or nondisplaced
 presence and location of associated fibula fracture

Presentation
 Symptoms
o pain
o refusal to bear weight
 Physical exam
o valgus deformity
o evaluate carefully for compartment syndrome

Imaging
 Radiographs
o recommended views
 required
 AP and lateral
o findings
 look for incomplete vs complete and presence of a proximal fibula fracture which may
indicate a more unstable fracture pattern

- 530 -
By Dr, AbdulRahman AbdulNasser Pediatric trauma | Knee & Proximal Tibia
Treatment
 Nonoperative
o long leg cast in extension with varus mold (aim for slight overcorrection)
 indications
 nondisplaced fracture
 technique
 casts are maintained for 6-8 weeks with serial radiographs
 weight bearing may be allowed after 2-3 weeks.
o reduction followed by long leg cast in extension with varus mold (aim for slight
overcorrection)
 indications
 displaced fracture
 technique
 requires conscious sedation or general anesthesia
 cast in near full extension (10 degrees flexion)
 varus mold at fracture site
 Operative
o open reduction
 indications
 inability to adequately reduce a displaced fracture
 secondary to soft tissue interposition
 technique
‎ III:40 Valgus deformity
V
 limited medial approach to proximal tibia
 periosteum or tendons of pes anserinus may block reduction
 internal fixation not commonly required

Complications
 Valgus deformity (Cozen phenomenon)
o may be observed for 12-24 months with expectation of spontaneous correction
o parents should be counseled in advance
o etiology
 incomplete reduction
 concomitant injury to proximal tibia physis
 infolded periosteum
 injury to pes anserinus insertion, with loss of proximal tibia physeal tether, leading to
asymmetric physeal growth

o treatment
 if deformity fails to resolve
 medial hemi-epiphysiodesis in skeletally immature patient
 corrective osteotomy in skeletally mature patient
 osteotomies have significant complications
 Limb length discrepancy
o affected tibia is often longer (average 9mm)
o typically does not require intervention however parents should be counseled that this does not
resolve

- 531 -
OrthoBullets2017 Pediatric trauma | Leg & Ankle Fractures

G. Leg & Ankle Fractures

1. Tibia Shaft Fracture - Pediatric


Introduction
 Among the most frequently encountered pediatric fractures. Consists of
o traumatic tibial shaft +/- fibula fractures
 most commonly due to pedestrian vs vehicle (50%)
o Toddler's fracture (see below)
 Epidemiology
o 15% of all pediatric fractures
o 39% of tibia fractures occur in the diaphysis.
o 30% are associated with a fibula fracture.
 usually undisplaced because of strong periosteum
 Prognosis
o healing
 3 to 4 weeks for toddler's fracture
 6 to 8 weeks for other tibial fractures
 Toddler's fracture
o characteristics
 nondisplaced spiral or oblique fracture of tibial shaft only
 fibula remains intact
 also known as childhood accidental spiral tibial (CAST) fractures
o age group
 children< 3 years (walking toddlers)
 unlike child abuse injury, which occurs in children not yet walking
o mechanism
 low energy trauma with rotational component
 involves distal half of tibia
 unlike non-accidental injury, which typically involves proximal half of tibia

Presentation
 Symptoms
o pain
o bruising
o limping or refusal to bear weight
 Physical exam
o warmth, swelling over fracture site
o tender over fracture site
o pain on ankle dorsiflexion
o always have high suspicion for compartment syndrome

Imaging
 Radiographs
o views
 AP and lateral views of the tibia and fibula are required

- 532 -
By Dr, AbdulRahman AbdulNasser Pediatric trauma | Leg & Ankle Fractures
 ipsilateral knee and ankle must be evaluated to rule out concomitant injury
o findings
 Toddler's fracture are nondisplaced spiral tibial shaft fracture
Treatment Traumatic Tibia +/- Fibular fx
 Nonoperative
o closed reduction and long leg casting
 indications
 almost all Toddler's fracture
 most traumatic fractures
 displaced with acceptable reduction
 50% apposition
 < 1 cm of shortening ‎VIII:41 Toddler's fracture
 < 5-10 degrees of angulation in the sagittal and coronal planes
 followup
 follow up xrays in 2 weeks to evaluate for callus in order to confirm diagnosis in
equivocal cases
 serial radiographs are performed to monitor for developing deformity
 Operative
o surgical treatment
 indications (< 5% of tibia shaft fractures)
 unacceptable reduction (see above)
 marked soft tissue injury
 open fractures
 unstable fractures
 compartment syndrome
 neurovascular injury
 multiple long bone fractures
 >1cm shortening
 unacceptable alignment following closed reduction (>10deg angulation)
 techniques include
 external fixation
 plate fixation
 percutaneous pinning
 flexible IM nails

Techniques
 Long Leg Casting
o immobilization is performed with a long leg cast with the knee flexed to provide rotational
control and prevent weight bearing.
 External fixation
o open fractures with extensive soft tissue injury is most common indication
o most common complication is malunion
o nonunion (~2%)
 Plate fixation

- 533 -
OrthoBullets2017 Pediatric trauma | Leg & Ankle Fractures
 Percutaneous pinning
o younger patients
 Flexible or rigid intramedullary rods
o depending on the age of the patient and degree of soft tissue injury
o complications
 nonunion (~10%)
 malunion
 infection

Complications
 Compartment syndrome
o with both open and closed fractures
 Leg-length discrepancy
 Angular deformity
o varus for tibia only fractures
o valgus for tibia-fibula fractures
 Associated physeal injury
o proximal or distal
 Delayed union and nonunion
o usually only after external fixation

2. Ankle Fractures - Pediatric


Introduction
 Pediatric ankle fractures include
o SH type I
 fibular fx
o SH type II
 fibular fx
o SH type III
 tillaux fractures
 medial malleolus fractures
o SH type IV
 triplane fractures
 medial malleolus shear fractures
 Epidemiology
o incidence
 25-40% of all physeal injuries (second most common)
o demographics
 typically occur between 8-15 years-old
 Pathophysiology
o mechanism of injury
 direct trauma
 rotation about a fixed foot and ankle

- 534 -
By Dr, AbdulRahman AbdulNasser Pediatric trauma | Leg & Ankle Fractures
Anatomy
 Physeal considerations
o distal tibial physis closes in predictable pattern
 central to medial
 anterolateral closes last

Classification
 Anatomic classification
o Salter-Harris Classification
 Diaz and Tachdjian classification (patterned off adult Lauge-Hansen classification)
o supination-inversion
o supination-plantar flexion
o supination-external rotation
o pronation/eversion-external rotation

Presentation
 Symptoms
o ankle pain, inability to bear weight
 Physical exam
o inspection : swelling, focal tenderness

Imaging
 Radiographs
o recommended views
 AP, mortise, and lateral
o optional views
 full-length tibia, or proximal tibia, to rule out Maisonneuve-type fracture
o findings
 triplane fractures
 AP or mortise reveals intraarticular component
 lateral reveals posterolateral metaphyseal fragment (Thurston-Holland fragment)
 CT scan : indications
o assess fracture displacement
o assess articular step-off

Treatment
 Nonoperative
o cast immobilization
 indications
 <2mm articular displacement
 Operative
o CRPP vs ORIF
 indications
 >2mm displacement
 intra-articular fractures
 irreducible reduction by closed means
 may have interposed periosteum, tendons, neurovascular structures

- 535 -
OrthoBullets2017 Pediatric trauma | Leg & Ankle Fractures
Techniques
 CRPP vs ORIF
o reduction
 percutaneous manipulation with k-wires may aid reduction
 open reduction may be required if interposed tissue
o fixation
 transepiphyseal fixation best if at all possible
 cannulated screws parallel to physis
 tillaux and triplane fractures
 2 parallel epiphyseal screws
 medial malleolus shear fractures
 transphyseal fixation
 smooth wire fixation typically used

Complications
 Ankle pain and degeneration
o high rate associated with articular step-off >2mm
 Growth arrest
o medial malleolus SH IV have highest rate of growth disturbance of any fracture
o partial arrests can lead to angular deformity
 distal fibular arrest results in valgus
 medial distal tibia arrest results in varus
o complete arrests can result in leg-length discrepancy
 can be addressed with contralateral epiphysiodesis
 Extensor retinacular syndrome
o displaced fracture can lead to foot compartment syndrome
 Rotational deformity

3. Tillaux Fractures
Introduction
 Salter-Harris III fx of the distal tibia epiphysis
o caused by an avulsion of the anterior inferior tibiofibular ligament
 Mechanism
o mechanism of injury is thought to be due to an external rotation force
 Epidemiology
o typically occur within one year of complete distal tibia physeal closure.
 older than triplane fracture age group
 Pathoanatomy
o lack of fracture in the posterior distal tibial metaphysis in the coronal plane distinguishes this
fracture from a triplane injury
o transitional fractures (tillaux and triplane) occur in older children at the end of growth
 variability in fracture pattern due to progression of physeal closure
 a period of time exists when the lateral physis is the only portion not fused
 leads to Tillaux and Triplane fractures
 often associated with external rotation deformity of the ankle/foot

- 536 -
By Dr, AbdulRahman AbdulNasser Pediatric trauma | Leg & Ankle Fractures
Anatomy
 Ossification
o the distal tibial physis closes in the following order
 central (first)
 posterior
 medial
 anterolateral (last)

Imaging
 Radiographs
o SH III fx of the anterolateral distal tibia epiphysis

 CT scan
o delineate the fracture pattern
o determine degree of displacement
o identify intramalleolar or medial fracture variant patterns

Treatment
 Nonoperative
o closed reduction, long leg cast x 4 weeks (control rotation), SLC x 2-3 weeks
 indications
 if < 2 mm of displacement (rare) following closed reduction
 technique
 reduction technique by internally rotating foot
 CT scans sometimes needed to determine residual displacement (confirm < 2mm)
 long leg cast initially to control rotational component of injury
 Operative
o open reduction and internal fixation
 indications
 if >2 mm of displacement remains after reduction attempt
 technique
 closed reduction (by internal rotation) can be attempted under general anesthesia first
 percutaneous screws can be placed if adequate reduction obtained
 visualize joint line to optimize reduction
 intra-epiphyseal screws
 transphyseal screws can also be used as most patients are approaching skeletal
maturity
 arthroscopically-assisted reduction has been described

Complications
 Premature growth arrest
o rare as little physis remaining as closure is already occuring
o decrease risk with anatomic reduction
 Early arthritis
o increase risk with articular displacement

- 537 -
OrthoBullets2017 Pediatric trauma | Leg & Ankle Fractures

4. Triplane Fractures
Introduction
 A complex SH IV fracture pattern with components in all three planes
o triplane fractures may be 2, 3, or 4 part fractures
 epiphysis fractured in sagittal plane (same as tillaux fracture) and therefore is seen on the AP
radiograph
 physis separated in axial plane
 metaphysis fractured in coronal plane and therefore is seen on the lateral radiograph
 Epidemiology
o Occur between ages 10-17 years, mean 13 years
 juvenile ankle physis ossifies in specific order, which leads to transitional fractures such as
triplane and tillaux fractures
 distal tibia physis order of
ossification
 central > medial > lateral
 Mechanism
o most are result of supination-external
rotation similar to tillaux fractures
(lateral triplane)
 medial triplane is a result of
adduction
Classification
 Parts - 2, 3, 4 part
 Lateral triplane (more common) > medial triplane >> intramalleolar triplane (epiphyseal fracture
exits through medial malleolus)
Presentation
 Symptoms
o ankle pain, inability to bear weight
 Physical exam
o swelling, focal tenderness

Imaging
 Radiographs
o AP radiograph shows Salter-Harris III
o lateral radiograph shows Salter-Harris II
 CT scan
o usually required to delineate fracture pattern and access articular congruity
o fracture involvement seen in all 3 planes

Treatment
 Nonoperative
o cast immobilization
 indications
 < 2 mm displacement

- 538 -
By Dr, AbdulRahman AbdulNasser Pediatric trauma | Leg & Ankle Fractures
 Operative
o CRPP vs ORIF
 indications
 > 2 mm displacement
 techniques
 epiphyseal screw placed parallel to physis
 arthroscopic aided reduction can be used

Complications
 Ankle pain and degeneration
o articular step-off >2mm

Collected By : Dr AbdulRahman
AbdulNasser
June 2017

- 539 -

Vous aimerez peut-être aussi